Skip to main content

Full text of "ASVAB for Dummies_"

See other formats


Making Everything Easier! 




Learn to: 



• Score higher to qualify for the military job 
you want 

' Build word knowledge and paragraph 
comprehension 

' Test your skills with practice problems 
for each of the test's nine subtests 

' Take three full-length practice tests plus 
one AFQT 



Rod Powers 

U.S. military expert and military guide 
forAbout.com 



Get More and Do More at Dummies.com 



M 



(fX 



t,V> 



&1 



Start with FREE Cheat Sheets 

Cheat Sheets include 

• Checklists 

• Charts 

• Common Instructions 

• And Other Good Stuff! 



To access the Cheat Sheet created specifically for this book, go to 
www.dummies.com/cheatsheet/asvab 



Get Smart at Dummies.com 

Dummies.com makes your life easier with 1 ,000s 
of answers on everything from removing wallpaper 
to using the latest version of Windows. 

Check out our 
•Videos 

• Illustrated Articles 

• Step-by-Step Instructions 

Plus, each month you can win valuable prizes by entering our 
Dummies.com sweepstakes.* 

Want a weekly dose of Dummies? Sign up for Newsletters on 

• Digital Photography 

• Microsoft Windows & Office 

• Personal Finance & Investing 

• Health &Wellness 

• Computing, iPods & Cell Phones 
•eBay 

• Internet 

• Food, Home & Garden 



Find out "HOW" at Dummies.com 



*Sweepstakes not currently available in all countries; visit Dummies.com for official rules. 




by Rod Powers 




WILEY 

Wiley Publishing, Inc. 



ASVAB For Dummies*, 3rd Edition 

Published by 

Wiley Publishing, Inc. 

Ill River St. 

Hoboken, NJ 07030-5774 

www. wiley . com 

Copyright © 201 1 by Wiley Publishing, Inc., Indianapolis, Indiana 

Published by Wiley Publishing, Inc., Indianapolis, Indiana 

Published simultaneously in Canada 

No part of this publication may be reproduced, stored in a retrieval system or transmitted in any form or by any means, 
electronic, mechanical, photocopying, recording, scanning or otherwise, except as permitted under Sections 107 or 108 of 
the 1976 United States Copyright Act, without either the prior written permission of the Publisher, or authorization through 
payment of the appropriate per-copy fee to the Copyright Clearance Center, 222 Rosewood Drive, Danvers, MA 01923, (978) 
750-8400, fax (978) 646-8600. Requests to the Publisher for permission should be addressed to the Permissions Department, 
John Wiley & Sons, Inc., Ill River Street, Hoboken, NJ 07030, (201) 748-6011, fax (201) 748-6008, or online at http: //www. 
wiley. com/ go /permissions. 

Trademarks: Wiley, the Wiley Publishing logo, For Dummies, the Dummies Man logo, A Reference for the Rest of Us!, The 
Dummies Way, Dummies Daily, The Fun and Easy Way, Dummies.com, Making Everything Easier, and related trade dress 
are trademarks or registered trademarks of John Wiley & Sons, Inc. and/or its affiliates in the United States and other coun- 
tries, and may not be used without written permission. All other trademarks are the property of their respective owners. 
Wiley Publishing, Inc., is not associated with any product or vendor mentioned in this book. 



LIMIT OF LIABILITY/DISCLAIMER OF WARRANTY : THE PUBLISHER AND THE AUTHOR MAKE NO REPRESENTATIONS 
OR WARRANTIES WITH RESPECT TO THE ACCURACY OR COMPLETENESS OF THE CONTENTS OF THIS WORK AND 
SPECIFICALLY DISCLAIM ALL WARRANTIES, INCLUDING WITHOUT LIMITATION WARRANTIES OF FITNESS FOR A 
PARTICULAR PURPOSE. NO WARRANTY MAY BE CREATED OR EXTENDED BY SALES OR PROMOTIONAL MATERIALS. 
THE ADVICE AND STRATEGIES CONTAINED HEREIN MAY NOT BE SUITABLE FOR EVERY SITUATION. THIS WORK IS 
SOLD WITH THE UNDERSTANDING THAT THE PUBLISHER IS NOT ENGAGED IN RENDERING LEGAL, ACCOUNTING, 
OR OTHER PROFESSIONAL SERVICES. IF PROFESSIONAL ASSISTANCE IS REQUIRED, THE SERVICES OF A COMPETENT 
PROFESSIONAL PERSON SHOULD BE SOUGHT. NEITHER THE PUBLISHER NOR THE AUTHOR SHALL BE LIABLE FOR 
DAMAGES ARISING HEREFROM. THE FACT THAT AN ORGANIZATION OR WEBSITE IS REFERRED TO IN THIS WORK 
AS A CITATION AND/OR A POTENTIAL SOURCE OF FURTHER INFORMATION DOES NOT MEAN THAT THE AUTHOR 
OR THE PUBLISHER ENDORSES THE INFORMATION THE ORGANIZATION OR WEBSITE MAY PROVIDE OR RECOM- 
MENDATIONS IT MAY MAKE. FURTHER, READERS SHOULD BE AWARE THAT INTERNET WEBSITES LISTED IN THIS 
WORK MAY HAVE CHANGED OR DISAPPEARED BETWEEN WHEN THIS WORK WAS WRITTEN AND WHEN IT IS READ. 



For general information on our other products and services, please contact our Customer Care Department within the U.S. 
at 877-762-2974, outside the U.S. at 317-572-3993, or fax 317-572-4002. 

For technical support, please visit www.wiley.com/techsupport. 

Wiley also publishes its books in a variety of electronic formats. Some content that appears in print may not be available in 
electronic books. 

Library of Congress Control Number: 2010937828 

ISBN: 978-0-470-63760-9 

Manufactured in the United States of America 

10 987654321 



About the Author 



Rod Powers joined the United States Air Force in 1975 intending to become a spy. He was 
devastated to learn that he should've joined the CIA instead because the military doesn't 
have that particular enlisted job. Regardless, he fell in love with the military and made it 
both a passion and a career, retiring with 23 years of service. Rod spent 11 of those years 
as a first sergeant, helping to solve the problems of the enlisted corps. 

Since his retirement from the military in 1998, Rod has become a world renowned military 
careers expert. Through his highly popular U.S. Military Information Web site on About.com 
(http: //usmilitary. about .com), Rod has advised thousands of troops about all 
aspects of U.S. Armed Forces career information. 

Rod is the proud father of twin girls, both of whom enjoy successful careers in the United 
States Air Force. Rod currently resides in Daytona Beach, Florida, where he attempts to 
prove that there's no such thing as too much sunshine. Even today, Powers tries to run his 
life according to long-lived military ideals and standards, but he gets a bit confused about 
why nobody will obey his orders anymore. 



Dedication 

To Jeanie and Chrissy — because everything is for you. Always. 

Author's Acknowledgments 

I want to thank Leigh Richards for reviewing the manuscript for technical accuracy. I would 
also like to thank Andrew Ellet, Ph.D., math teacher extraordinaire, for editing the math 
chapters and reminding me how to solve for x. 

Special thanks goes out to Tracy Boggier, my Acquisitions Editor; Chad R. Sievers, my won- 
derful Project Editor; and my top-notch Copy Editor, Danielle Voirol, who dotted my i's and 
crossed my t's. A big thanks goes to Autumn Lindsey, who was my right-hand assistant with 
all the artwork and text revisions and reviews. 

Finally, I send more special thanks to the recruiting commands of the United States 
Army, Air Force, Navy, Marine Corps, and Coast Guard for providing invaluable resource 
information. 



Publisher's Acknowledgments 

We're proud of this book; please send us your comments at http : / /dummies . custhelp . com. For other comments, 
please contact our Customer Care Department within the U.S. at 877-762-2974, outside the U.S. at 317-572-3993, or 
fax 317-572-4002. 

Some of the people who helped bring this book to market include the following: 



Acquisitions, Editorial, and Media Development 

Senior Project Editor: Chad R. Sievers 

(Previous Edition: Kristin DeMint) 

Acquisitions Editor: Tracy Boggier 

Senior Copy Editor: Danielle Voirol 

(Previous Edition: Carrie Burchfield) 

Assistant Editor: David Lutton 

Technical Editors: Leigh Richards, 
Andrew Ellet, Ph.D. 

Editorial Manager: Michelle Hacker 

Editorial Assistant: Rachelle Amick 

Art Coordinator: Alicia B. South 

Cover Photos: © iStockphoto.com / mbbirdy 

Cartoons: Rich Tennant (www. theSthwave . com) 



Composition Services 

Project Coordinator: Kristie Rees 

Layout and Graphics: Carrie A. Cesavice, 
Joyce Haughey 

Special Art: Autumn Lindsey 

Proofreader: Susan Hobbs 

Indexer: Sherry Massey 

Special Help 

Kaitlin McGlone 



Publishing and Editorial for Consumer Dummies 

Diane Graves Steele, Vice President and Publisher, Consumer Dummies 

Kristin Ferguson-Wagstaffe, Product Development Director, Consumer Dummies 

Ensley Eikenburg, Associate Publisher, Travel 

Kelly Regan, Editorial Director, Travel 
Publishing for Technology Dummies 

Andy Cummings, Vice President and Publisher, Dummies Technology/General User 
Composition Services 

Debbie Stailey, Director of Composition Services 



• 



Contents at a Glance 



Introduction / 

Part 1: Making Sense of the ASVAB 5 

Chapter 1: Putting the ASVAB Under a Microscope 7 

Chapter 2: Knowing What It Takes to Get Your Dream Job 17 

Chapter 3: Getting Acquainted with Test-Taking and Study Techniques 23 

Part 11: Words to Live Bu: Communication Skills 31 

Chapter 4: Word Knowledge 33 

Chapter 5: Paragraph Comprehension 43 

Chapter 6: All's Well That Tests Well: Communication Practice Questions 55 

Part HI: Making the Most of Math: Arithmetic Skills 63 

Chapter 7: Mathematics Knowledge and Operations 65 

Chapter 8: Reasoning with Arithmetic: Math Word Problems 91 

Chapter 9: Brother, Can You Spare an Equation? Arithmetic Practice Questions 97 

Part IV: The Whole Ball of Facts: Technical Skills 105 

Chapter 10: General Science 107 

Chapter 11: Auto and Shop Information 127 

Chapter 12: Mechanical Comprehension 141 

Chapter 13: Electronics Information 157 

Chapter 14: Assembling Objects 171 

Chapter 15: Facing the Facts: Technical Skills Practice Questions 179 

Part V: Practice ASVAB Exams 189 

Chapter 16: Practice Exam 1 191 

Chapter 17: Practice Exam 1: Answers and Explanations 221 

Chapter 18: Practice Exam 2 233 

Chapter 19: Practice Exam 2: Answers and Explanations 265 

Chapter 20: Practice Exam 3 277 

Chapter 21: Practice Exam 3: Answers and Explanations 309 

Chapter 22: Practice AFQT Exam 321 

Chapter 23: Practice AFQT Exam: Answers and Explanations 337 

Part VI: The Part of Tens 3b3 

Chapter 24: Ten Surefire Ways to Fail the ASVAB 345 

Chapter 25: Ten Tips for Doing Well on the AFQT 349 

Chapter 26: Ten Ways to Boost Your Math and English Skills 353 

Appendix: Matching ASVAB Scores to Military Jobs 357 

Index 375 



Table of Contents 



• 



Introduction 7 

About This Book 1 

Conventions Used in This Book 2 

What You're Not to Read 2 

Foolish Assumptions 2 

How This Book Is Organized 2 

Part I: Making Sense of the ASVAB 3 

Part II: Words to Live By: Communication Skills 3 

Part III: Making the Most of Math: Arithmetic Skills 3 

Part IV: The Whole Ball of Facts: Technical Skills 3 

Part V: Practice ASVAB Exams 3 

Part VI: The Part of Tens 3 

Appendix 3 

Icons Used in This Book 4 

Where to Go from Here 4 

Part 1: Making Sense of the ASVAB 5 

Chapter 1: Putting the ASVAB Under a Microscope 7 

Knowing Which Version You're Taking 7 

Mapping Out the ASVAB Subtests 8 

Deciphering ASVAB Scores 10 

Defining all the scores 10 

Understanding the big four: Your AFQT scores 12 

Do-Over: Retaking the ASVAB 14 

U.S. Army retest policy 15 

U.S. Air Force retest policy 15 

U.S. Navy retest policy 15 

U.S. Marine Corps retest policy 16 

U.S. Coast Guard retest policy 16 

Chapter 2: Knowing What It Takes to Get Your Dream Job 17 

Eyeing How ASVAB Scores Determine Military Training Programs and Jobs 17 

Understanding How Each Branch Computes Line Scores 18 

Line scores and the Army 19 

Line scores and the Navy and Coast Guard 19 

Line scores and the Marine Corps 20 

Line scores and the Air Force 21 

Chapter 3: Getting Acquainted with Test-Taking and Study Techniques 23 

Taking the Test: Paper or Computerized? 23 

Writing on hard copy: The advantages and disadvantages 

of the paper version 24 

Going paperless: The pros and cons of the computerized test 24 

Tackling Multiple-Choice Questions 25 

When You Don't Know an Answer: Guessing Smart 27 

Studying and Practicing for the ASVAB 28 

Making Last-Minute Preparations: 24 Hours and Counting 29 



Table of Contents (/jj 



Part 11: Wonts to Live By: Communication Skills 31 

Chapter 4: Word Knowledge 33 

Grasping the Importance of Word Knowledge 33 

Checking Out the Word Knowledge Question Format 34 

Building Words from Scratch: Strategies to Help You Decipher Word Meanings 35 

From beginning to end: Knowing prefixes and suffixes 35 

Determining the root of the problem 37 

Word families: Finding related words 38 

Ying and Yang: Understanding Synonyms and Antonyms 39 

You Are What You Speak: Improving Your Vocabulary, Improving Yourself 39 

Reading your way to a larger vocabulary 40 

Keeping a list and checking it twice 40 

Crosswords: Making vocabulary fun 41 

Sounding off by sounding it out 41 

Chapter 5: Paragraph Comprehension 43 

The Importance of Paragraph Comprehension for Military Jobs 43 

Eyeing the Physique of the Paragraph Comprehension Subtest 44 

Trying the Four Flavors of Comprehension Questions 44 

Treasure hunt: Finding specific information 45 

Cutting to the chase: Recognizing the main idea 45 

If the shoe fits: Determining word meaning in context 46 

Reading between the lines: Understanding implications 46 

Do You Get My Point? 48 

What's the big idea? Determining the main idea in a paragraph 48 

Extra, extra! Identifying subpoints 49 

Analyzing What You've Read: Guessing at What the Writer Really Means 50 

Faster than a Speeding Turtle: Tips for Slow Readers 51 

Read more, watch less 51 

Become a lean, mean word machine 51 

Build your confidence 51 

Test-Taking Tips for Reading and Gleaning 52 

Chapter 6: All's Well That Tests Well: Communication Practice Questions 55 

Word Knowledge Practice Questions 55 

Paragraph Comprehension Practice Questions 58 

Part HI: Making the Most of Math: Arithmetic Skills 63 

Chapter 7: Mathematics Knowledge and Operations 65 

Just When You Thought You Were Done 
with Vocab: Math Terminology 66 

Operations: What You Do to Numbers 67 

First things first: Following the order of operations 67 

Completing a number sequence 68 

Working on Both Sides of the Line: Fractions 69 

Common denominators: Preparing to add and subtract fractions 70 

Multiplying and reducing fractions 71 

Dividing fractions 72 

Converting improper fractions to mixed numbers . . . and back again 72 

Expressing a fraction in other forms: Decimals and percents 73 

Showing comparisons with ratios 76 



(//// ASVAB For Dummies, 3rd Edition 



A Powerful Shorthand: Writing in Scientific Notation 77 

Getting to the Root of the Problem 77 

Perfect squares 78 

Irrational numbers 78 

Other roots 78 

An Unknown Quantity: Reviewing Algebra 79 

Solving for x 79 

When all things are equal: Keeping an algebra equation balanced 79 

Explaining exponents in algebra 81 

A step back: Factoring algebra expressions to find original numbers 82 

Making alphabet soup: Solving the quadratic equation 83 

All math isn't created equal: Solving inequalities 84 

Looking at Math from a Different Angle: Geometry Review 84 

Outlining angles 85 

Pointing out triangle types 85 

Back to square one: Quadrilaterals 86 

Going around in circles 87 

Filling 'er up: Calculating volume 88 

Test-Taking Techniques for Your Mathematical Journey 88 

Knowing what the question is asking 88 

Figuring out what you're solving for 89 

Solving what you can and guessing the rest 89 

Using the process of elimination 90 

Chapter 8: Reasoning with Arithmetic: Math Word Problems 91 

Tackling the Real World of Word Problems 92 

Reading the entire problem 92 

As plain as the nose on a fly: Figuring out what the question is asking 92 

Digging for the facts 93 

Setting up the problem and working your way to the answer 93 

Reviewing your answer 94 

The Guessing Game: Putting Reason in Your Guessing Strategy 95 

Using the process of elimination 95 

Solving what you can and guessing the rest 95 

Making use of the answer choices 96 

Chapter 9: Brother, Can You Spare an Equation? 

Arithmetic Practice Questions 97 

Arithmetic Reasoning (Math Word Problems) Practice Questions 97 

Math Knowledge Practice Questions 101 

Part IV: The Whole Ball of Facts: Technical Skills 105 

Chapter 10: General Science 107 

There's a Scientific Method to the Madness 107 

Understanding Forms of Measurement 108 

Doing the metric thing 108 

Figuring temperature conversions 109 

Another Day, Another Science: Scientific Disciplines You Should Know 110 

Uncovering Biology, from Big to Small Ill 

Relating to your world through ecology 112 

Categorizing Mother Nature 112 

Perusing the human body systems 114 

Thinking small: A look at cells 115 

Swimming in the gene pool: Genetics 116 



Table of Contents 



IX 



Chemistry: Not Blowing Up the Lab 117 

Understanding the elements, my dear Watson 118 

Sitting down at the periodic table 118 

Getting physical: Changing states 118 

Causing a chemical reaction 119 

Where Few Have Gone Before: Astronomy 119 

Taking a quick glimpse at the sun 119 

Knowing the planets 120 

Shooting for the moons 120 

Watching for meteors, comets, and asteroids 121 

Down to Earth: Rocking Out with Geology and Meteorology 122 

Peeling back the layers of the planet 122 

Outta this world: Checking the atmosphere 122 

Warming up to cold fronts 123 

Classifying clouds 123 

Improving Your Chances on the General Science Subtest 124 

Using common sense to make educated guesses 124 

Getting back to your Latin roots 125 

Chapter 11: Auto and Shop Information 127 

Checking Under the Hood 127 

The engine: Different strokes 128 

Cooling system: Acting cool, staying smooth 129 

Electrical and ignition systems: Starting up 130 

Drive system: Taking it for a spin 130 

Brake system: Pulling out all the stops 131 

Emissions-control systems: In layman's terms, filters 131 

Picking Up the Tools of the Trade 131 

Striking tools 133 

Fastening tools 133 

Cutting tools 134 

Drilling, punching, and gouging tools 135 

Finishing tools 136 

Clamping tools 136 

Measuring tools 136 

Leveling and squaring tools 137 

Sticking Materials Together with Fasteners 137 

Nails 137 

Screws and bolts 138 

Nuts and washers 138 

Rivets 139 

Building a Better Score 139 

Chapter 12: Mechanical Comprehension 141 

Understanding the Forces of the Universe 141 

He hit me first! The basics of action and reaction 142 

Equilibrium: Finding a balance 142 

Under pressure: Spreading out the force 143 

Looking at kinds of forces 143 

You Call That Work?! 146 

Overcoming resistance 146 

Gaining power by working more quickly 146 

Relying on Machines to Help You Work 147 

Using levers to your advantage 147 

Ramping up the inclined plane 148 

Easing your effort: Pulleys and gears 148 



% ASVAB For Dummies, 3rd Edition 



Multiplying your effort: Wheels and axles 152 

Getting a grip on things with vises 153 

Magnifying your force with liquid: Hydraulic jacks 154 

Working Your Way to a Better Test Score 155 

Using your observations and common sense 155 

Using the mathematics of mechanics 156 

Guessing with a mechanical mind 156 

Chapter 13: Electronics Information 157 

Uncovering the Secrets of Electricity 157 

Measuring voltage: Do you have the potential? 158 

Examining the current of the electrical river 159 

Resistance: Slowing the electrical river 159 

Measuring power 161 

Getting around to circuits 161 

Producing electrical effects 162 

Switching Things Up with Alternating and Direct Current 163 

Figuring out frequency 163 

Impedance: Join the resistance! 163 

Rectifying the situation: Going direct 164 

Turning up the old transistor radio 164 

Picture It: Decoding Electrical Circuit Codes 165 

Eyeing Some Electronic Information Test Tips 168 

Memorizing simple principles 169 

Playing the guessing game 169 

Chapter 14: Assembling Objects 171 

Getting the Picture about Assembling Objects 171 

Two Types of Questions for the Price of One 172 

Putting slot A into tab B: Connectors 172 

Solving the jigsaw puzzle: Shapes 174 

Tips for the Assembling Objects Subtest 176 

Comparing one piece or point at a time 176 

Visualizing success: Practicing spatial skills ahead of time 177 

Chapter 15: Facing the Facts: Technical Skills Practice Questions 179 

General Science Practice Questions 179 

Auto & Shop Information Practice Questions 181 

Mechanical Comprehension Practice Questions 183 

Electronics Information Practice Questions 185 

Assembling Objects Practice Questions 187 

Part V: Practice ASVAB Exams 189 

Chapter 16: Practice Exam 1 191 

Chapter 17: Practice Exam 1: Answers and Explanations 221 

Subtest 1: General Science Answers 221 

Subtest 2: Arithmetic Reasoning Answers 221 

Subtest 3: Word Knowledge Answers 224 

Subtest 4: Paragraph Comprehension Answers 225 

Subtest 5: Mathematics Knowledge Answers 226 

Subtest 6: Electronics Information Answers 229 



Table of Contents 



XI 



Subtest 7: Auto & Shop Information Answers 230 

Subtest 8: Mechanical Comprehension Answers 230 

Subtest 9: Assembling Objects Answers 232 

Chapter 18: Practice Exam 2 233 

Chapter 19: Practice Exam 2: Answers and Explanations 265 

Subtest 1: General Science Answers 265 

Subtest 2: Arithmetic Reasoning Answers 266 

Subtest 3: Word Knowledge Answers 268 

Subtest 4: Paragraph Comprehension Answers 268 

Subtest 5: Mathematics Knowledge Answers 270 

Subtest 6: Electronics Information Answers 272 

Subtest 7: Auto & Shop Information Answers 273 

Subtest 8: Mechanical Comprehension Answers 273 

Subtest 9: Assembling Objects Answers 275 

Chapter 20: Practice Exam 3 277 

Chapter 21: Practice Exam 3: Answers and Explanations 309 

Subtest 1: General Science Answers 309 

Subtest 2: Arithmetic Reasoning Answers 309 

Subtest 3: Word Knowledge Answers 313 

Subtest 4: Paragraph Comprehension Answers 313 

Subtest 5: Mathematics Knowledge Answers 315 

Subtest 6: Electronics Information Answers 317 

Subtest 7: Auto & Shop Information Answers 318 

Subtest 8: Mechanical Comprehension Answers 318 

Subtest 9: Assembling Objects Answers 320 

Chapter 22: Practice AFQT Exam 321 

Chapter 23: Practice AFQT Exam: Answers and Explanations 337 

Subtest 1: Arithmetic Reasoning Answers 337 

Subtest 2: Word Knowledge Answers 340 

Subtest 3: Paragraph Comprehension Answers 340 

Subtest 4: Mathematics Knowledge Answers 341 

Part VI: The Part of Tens 343 

Chapter 24: Ten Surefire Ways to Fail the ASVAB 345 

Choosing Not to Study at All 345 

Failing to Realize How Scores Are Used 345 

Studying for Unnecessary Subtests 346 

Losing Focus 346 

Panicking Over Time 346 

Deciding Not to Check the Answers 347 

Making Wild Guesses or Not Guessing at All 347 

Changing Answers 347 

Memorizing the Practice Test Questions 347 

Misunderstanding the Problem 348 



y\\ ASVAB For Dummies, 3rd Edition 



Chapter 25: Ten Tips for Doing Well on the AFQT 349 

As Soon as the Test Starts, Write Down What You're Likely to Forget 349 

Read All the Answer Choices before Deciding 349 

Don't Expect Perfect Word Matches 350 

Read Passages before the Questions 350 

Reread to Find Specific Information 350 

Base Conclusions Only on What You Read 350 

Change Percents to Decimals 350 

Understand Inverses 351 

Remember How Ratios, Rates, and Scales Compare 351 

Make Sure Your Answers Are Reasonable 351 

Chapter 26: Ten Ways to Boost Your Math and English Skills 353 

Practice Doing Math Problems 353 

Put Away Your Calculator 353 

Memorize the Order of Operations 354 

Know Your Geometry Formulas 354 

Keep a Word List 355 

Study Latin and Greek 355 

Use Flashcards 355 

Read More, Watch TV Less 356 

Practice Finding Main and Supporting Points 356 

Use a Study Guide 356 

Appendix: Matching ASVAB Scores to Military Jobs 357 

Index 375 



Introduction 



I 

■ f you're reading this book, there's a good chance that you want to join the United States 
«C military. Perhaps it's been your lifelong dream to drive a tank, fire a machine gun, or 
blow things up (legally). Maybe you've always wanted to learn how to cook for 2,000 people 
at a time. Possibly you were attracted to the military because of education and training 
opportunities, the chance of travel, or huge enlistment bonuses. In any event, by now 
you've discovered that you can't just walk into a recruiter's office and say, "Hey, I'm here. 
Sign me up!" These days, you have to pass the ASVAB. 

The ASVAB (short for Armed Services Vocational Aptitude Battery) is unlike any test you've 
ever taken. It covers standard academic areas, such as math and English, but it also mea- 
sures your knowledge of mechanics, electronics, science, and assembling objects. 

The good news is that you need to do well on some of the subtests but not all of them. The 
order of importance of the subtests depends on your career goals. You find out what you 
need to know to do well on all the subtests and then get the info to determine which sub- 
tests are important to you. I include charts and tables to help you figure out the subtest 
scores that individual military jobs require. In fact, this guide is the first to include this 
information, so you can use ASVAB For Dummies, 3rd Edition, to ace the subtests that make 
up the ASVAB and to determine which subtests are important for your military career goals. 



About This Book 



The paper enlistment version of the ASVAB and the CAT-ASVAB (the computer version) 
have nine subtests, each of which is covered in its own chapter in this book. This book 
shows you what to expect on each subtest, offers strategies for studying each subject area, 
gives you test-taking (and guessing) tips, and provides three full-length sample tests that 
help you determine your strengths and weaknesses. These sample tests also help you pre- 
pare mentally for taking the real test — you can use them to get in the zone. I've thrown in 
an extra test, covering the four most important subtests of the ASVAB that make up the 
AFQT (Armed Forces Qualification Test) score at no extra cost. 

Although much of the material covered on the ASVAB is taught in practically every high 
school in the country, you may have slept through part of the info or performed a major 
brain-dump as soon as the ink was dry on your report card. Therefore, you also get a basic 
review of the relevant subject areas to help refresh your memory, as well as some pointers 
on where to find more information if you need it. 



Contentions Used in This Book 



The following conventions are used throughout the text to help point out important con- 
cepts and to help make the text easier to understand: 

Iv* All Web addresses appear in mono font. Note: Some Web addresses may extend to two 
lines of text. If you use one of these addresses, just type the address exactly as you see 
it, pretending that the line break isn't there. 



ASVAB For Dummies, 3rd Edition 



v* Each new term appears in italics and is closely followed by an easy-to-understand 
definition. 

f Bold text highlights important points and the action parts of numbered steps, as well 
as correct answers. 



What \lou're Not to Read 



This book has a number of sidebars (the shaded gray boxes) sprinkled throughout, as well as 
some paragraphs marked with Technical Stuff icons. They're full of interesting information 
about the ASVAB and the topics you may find on it, but you don't have to read them if you 
don't want to — they don't contain anything you simply must know in order to ace the test. 

If you're taking the ASVAB for the purpose of enlisting in the U.S. military, you may even 
want to skip entire chapters, depending on your career goals. For example, if the military 
careers you're interested in don't require a score on the General Science subtest, you may 
want to skip that chapter and concentrate your study time on chapters that are required 
for your particular job choices. 



Foolish Assumptions 



While writing and revising this book, I made a few assumptions about you — namely, who 
you are and why you picked up this book. I assume the following: 

v* You've come here for test-taking tips and other helpful information. You may be a 
nervous test-taker. 

v* You want to take a few ASVAB practice tests to measure your current knowledge in 
various subject areas in order to help you develop a study plan. 

«*" You want the military job of your dreams, and passing the ASVAB (or certain sections 
of it) is of utmost importance. Or you're in a high school that takes part in the ASVAB 
Career Exploration Program, and you want to know what to expect on the test. 



HoW This Book Is Organized 



There's a method to the madness ... a reason this book is organized the way you see it 
today. Material having to do with words is all grouped together, material having to do with 
math is all grouped together, and so on. This book is not organized to reflect the order in 
which the subtests appear on the actual ASVAB — rather, I organized it in the most logical 
fashion to help you study. 

Part h Making Sense of the ASVAB 

If you have no clue about how the ASVAB is organized or what it covers, turn to Part I. This 
part also tells you how scores are calculated and how the military bigwigs use the scores to 
determine whether you qualify to join the military and which jobs you qualify for. Chapter 3 
also contains some great study and test-taking tips. 



Introduction 



Part 11: Words to Live By: Communication Skills 

If you already know that you need help on a particular language arts-related subtest, turn 
to Part II, where you can pump up your vocabulary and reading comprehension skills. 

Part 111: Making the Most of Math: 
Arithmetic Skills 

Maybe math wasn't your best subject in school. Maybe you love math, but you've forgotten 
all those equations. In either case, this section is all about math. If you know you need to 
bone up on your mathematical skills, flip to Part III. 

Part IV: The Whole Ball of Facts: Technical Skills 

If you're interested in pursuing a military job that emphasizes science and technology and 
you want to make sure that you're going to nail the corresponding sections of the ASVAB, 
turn to Part IV. 

Part V: Practice ASVAB Exams 

This part gives you three different tests to figure out which areas you need to brush up on. 
Each test is a full-length sample test. Take the first one, and after that, check your answers. 
Then you can determine which subtests are a piece of cake and where you need the most 
help. Taking this approach lets you tailor a study plan to your individual needs. This part 
also includes a practice Armed Services Qualification Test (AFQT), made up of the four 
ASVAB subtests that determine whether you get into the military branch of your choice. 

Part VI: The Part of Tens 

This book is a For Dummies book, so it's not complete without a Part of Tens. If you want to 
find out some of the most important information for doing well on the ASVAB and you like 
your info presented in easily digestible lists, turn to Part VI. This part gives you test-taking 
tips and directs you to additional resources if you need them. 



Appendix 

If you want to check out different military jobs and see which subtests you need to do well on, 
turn to the Appendix. It shows you which subtest scores different military careers require. 



ASVAB For Dummies, 3rd Edition 



leans Used in This Book 




*£!§? 




<0Q. 






Throughout this book, you find icons that help you use the material in this book. Here's a 
rundown on what they mean to you: 

This icon alerts you to helpful hints regarding the ASVAB. Tips can help you save time and 
avoid frustration. 



This icon reminds you of important information you should memorize (or at least read 
carefully). 



This icon flags information that may prove hazardous to your plans of conquering the 
ASVAB. Often, this icon accompanies common mistakes or misconceptions people have 
about the ASVAB or questions on the test. 



This icon points out information that is interesting, enlightening, or in-depth but that isn't 
necessary for you to read. 



This icon points out sample test questions that appear in review chapters. 



Where to Go from Here 



You don't have to read this book from cover to cover to score well. I suggest that you begin 
with Chapters 1 and 2. That way, you can get a feel for how the ASVAB is organized (along 
with the most up-to-date changes on the test) and which subtests may be important for the 
military service branch and job of your choice. This plan of attack helps you set up logical 
and effective goals to maximize your study efforts. 

You may want to start by taking one of the practice tests in Part V. By using this method, 
you can discover which subjects are easier and which subjects you need to work on. If you 
choose this technique, you can use the other practice tests to measure your progress after 
reading through and studying the subject chapters. 



No matter where you start, I wish you luck on taking this test, and if you're one of the folks 
who wants to join the military, I hope your journey is successful! 



Parti 



Making Sense of the ASVAB 



The 5 th Wave 



By Rich Tennant 




"Tkose o£ you vrho did vrell m the math portion 
oi the ASVA5-, line up m multiples oi 3 to £ot*m 
columns -which, vrhen divided ky 5, -would give 
you a non-prime number , v 



In this part . . . 



/j n ancient military proverb goes something like this: 
T • "Understand your enemy, and you will avoid getting 
shot in the buttocks." Okay, that's not a real ancient mili- 
tary proverb. (In fact, I just made it up.) The point is that 
understanding how the ASVAB is organized, how it's 
scored, and what those scores mean to you and your 
potential military career can help you study for this nine- 
part test more efficiently. 

Even if you can't control yourself and you want to jump 
right in by reviewing the principles of algebra and memo- 
rizing word lists, chill out and take a few minutes to read 
through Part I. This part gives you an overview of the 
ASVAB, describes what each part of the exam tests, tells 
you when and where to take the test, and fills you in on 
how the scores are calculated. I even throw in some 
proven study techniques and test-taking strategies at 
no extra cost. 



Chapter 1 

Putting the ASVAB under a Microscope 



In This Chapter 

Checking out the different versions of the ASVAB 

Figuring out what each subtest covers 

Computing the Armed Forces Qualification Test (AFQT) score 

Taking the ASVAB again 



T 

m he Armed Services Vocational Aptitude Battery (ASVAB) consists of nine individual 
P tests (ten for Navy applicants who test at a Military Entrance Processing Station, or 
MEPS) that cover subjects ranging from general science principles to vocabulary. Your 
ASVAB test results determine whether you qualify for military service and, if so, which jobs 
you qualify for. The ASVAB isn't an IQ test. The military isn't trying to figure out how smart 
you are. The ASVAB specifically measures your ability to be trained to do a specific job. 

The famous Chinese general Sun Tzu said, "Know your enemy." To develop an effective plan 
of study and score well on the ASVAB, it's important to understand how the ASVAB is orga- 
nized and how the military uses the scores from the subtests. This chapter describes the 
different versions of the ASVAB, the organization of the subtests, how the AFQT score is cal- 
culated, and the various service policies for retaking the ASVAB. 



Knoitiinq Which Version \lou're lakinq 

The ASVAB comes in many flavors, depending on where and why you take it. You'd think 
that after more than 25 years in existence, the test could've been whittled down to a single 
version by now. But don't get too confused about the different versions. Table 1-1 boils 
down the choices. 



£ Part I: Making Sense of the ASVAB 



Table 1-1 



Versions of the ASVAB 



Version 



How You Take It 



Format 



Purpose 



Student 



Armed Forces 
Classification 
Test(AFCT) 



Given to juniors 
and seniors in 
high school; it's 
administered 
through a coop- 
erative program 
between the 
Department of 
Education and 
the Department 
of Defense at 
high schools 
all across the 
United States 



Paper Its primary purpose is to provide a tool for 

guidance counselors to use when recom- 
mending civilian career areas to high 
school students (though it can be used 
for enlistment if taken within two years 
of enlistment). For example, if a student 
scores high in electronics, the counselor 
can recommend electronics career paths. 
If a student is interested in military service, 
the counselor then refers her to the local 
military recruiting offices. 



Enlistment Given through a Usually This version of the ASVAB is used by all the 

military recruiter com- military branches for the purpose of enlist- 

puter, ment qualification and to determine which 

may be military jobs a recruit can successfully be 

paper trained in. 

Computer Given at the dis- Computer These mini-ASVABs aren't qualification 

Adaptive cretion of a mili- tests; they're strictly recruiting tools. The 

Test (CAT) tary recruiter for EST and CAT contain questions similar but 

or Enlistment a quick enlist- not identical to questions on the ASVAB. 

Screening mentqualifica- The tests are used to help estimate an 

Test (EST) tion screening applicant's probability of obtaining qualify- 

ing ASVAB scores. 



Given in-house 
to people 
already in the 
military 



Paper, 
though 
the 

military 
plans to 
replace 
it with a 
comput- 
erized 
version 
soon 



At some point during your military career, 
you may want to retrain for a different job. 
If you need higher ASVAB scores to qualify 
for such retraining, you can take the AFCT. 
Except forthe name of the exam, the AFCT 
is the same as the other versions of the 
ASVAB. 



For people taking the enlistment version of the test, the vast majority of applicants are pro- 
cessed through a MEPS, where they take the computerized format of the ASVAB (called the 
CAT-ASVAB, short for computerized-adaptive testing ASVAB), undergo a medical physical, 
and run through a security screening, many times all in one trip. However, applicants may 
instead choose to take the paper and pencil (P&P) version, which is generally given by non- 
MEPS personnel at numerous Mobile Examination Test (MET) sites located throughout the 
United States. 



Mapping Out the ASVAB Subtests 



The computerized format of the ASVAB contains ten separately timed subtests, with the 
Auto & Shop Information subtest split in two (also, one small subtest is geared to Coding 



Chapter 1: Putting the ASVAB under a Microscope 



Speed for a few Navy jobs; I don't include this subtest in the practice tests in this book 
because very few people test for these jobs). The paper format of the test has nine subtests. 
The two formats differ in the number of questions in each subtest and the amount of time 
you have for each one. Table 1-2 outlines the ASVAB subtests in the order that you take 
them in the enlistment (computerized or paper) and student (paper only) versions of the 
test; you can also see which chapters to turn to when you want to review that content. 



Table 1-2 



The ASVAB Subtests in Order 



Subtest 



Questions/Time 
(CAT-ASVAB) 



Questions/Time 
(Paper Version) 



Content 



Chapter 



General 
Science (GS) 



16 questions, 
8 minutes 



25 questions, 
11 minutes 



General prin- 
ciples of biologi- 
cal and physical 
sciences 



Chapter 10 



Arithmetic 
Reasoning (AR) 



16 questions, 
39 minutes 



30 questions, 
36 minutes 



Simple word 
problems that 
require simple 
calculations 



Chapter 8 



Word 

Knowledge 

(WK) 



16 questions, 
8 minutes 



35 questions, 
11 minutes 



Correct mean- 
ing of a word; 
occasionally 
antonyms (words 
with opposite 
meanings) 



Chapter 4 



Paragraph 

Comprehension 

(PC) 



11 questions, 
22 minutes 



15 questions, 
13 minutes 



Questions based 
on several para- 
graphs (usually 
a few hundred 
words) thatyou 
read 



Chapter 5 



Mathematics 

Knowledge 

(MK) 



16 questions, 
20 minutes 



25 questions, 
24 minutes 



High school 
math, includ- 
ing algebra and 
geometry 



Chapter 7 



Electronics 
Information (El) 



16 questions, 
8 minutes 



20 questions, 
9 minutes 



Electrical princi- 
ples, basic elec- 
tronic circuitry, 
and electronic 
terminology 



Chapter 13 



Auto & Shop 

Information 

(AS) 



1 1 Auto 
Information 
questions, 7 
minutes; 11 Shop 
Information ques- 
tions, 6 minutes 



25 questions, 
11 minutes 



Knowledge of 
automobiles, 
shop terminology, 
and tool use 



Chapter 11 



Mechanical 

Comprehension 

(MC) 



16 questions, 
20 minutes 



25 questions, 
19 minutes 



Basic mechani- 
cal and physical 
principles 



Chapter 12 



Assembling 
Objects (AO) 



16 questions, 
16 minutes 



25 questions, 
15 minutes* 



Spatial 
orientation 



Chapter 14 



*The Assembling Objects subtest isn't part of the student version of the test. 



/ Q Part I: Making Sense of the ASVAB 



Deciphering ASVAB Scores 



The Department of Defense is an official U.S. Government agency, so (of course) it can't 
keep things simple. When you receive your ASVAB score results, you don't see just one 
score; you see several. Figure 1-1 shows an example of an ASVAB score card used by high 
school guidance counselors (for people who take the student version — see "Knowing 
Which Version You're Taking" for details). 



Figure 1-1: 

A sample 

ASVAB 

score card 

used by 

high school 

guidance 

counselors. 



ASVAB Summary Results Sheet 


ASVAB Results 


Perce 

nth 

Grade 
Females 


ntile S 

nth 

Grade 
Males 


:ores 

nth 

Grade 
Students 


11th Grade Standard Score Bands 


nth 

Grade 

Standard 

Score 


Career Exploration Scores 

Verbal Skills 
Math Skills 
Science and Technical Skills 


62 
44 
66 


64 
45 
43 


63 
45 
54 


20 30 40 50 60 70 SO 


55 

46 
51 


















































General Science 
Arithmetic Reasoning 
Word Knowledge 
Paragraph Comprehension 
Mathematics Knowledge 
Electronics Iniormation 
Auto and Shop Information 
Mechanical Comprehension 

Military Entrance Score (AFQT) 39 


56 
36 
75 

44 
49 
77 
68 
76 


43 
34 
74 
56 
56 
52 
35 
48 


49 
35 
75 
50 
53 
65 
51 
62 


















49 
44 
57 
51 
48 
53 
48 
52 














































































































































20 30 40 50 60 70 80 



Figure 1-2 depicts an example of an ASVAB score card used for military enlistment purposes. 

So what do all these different scores actually mean? Check out the following sections to 
find out. 



Defining all the scores 



When you take a test in high school, you usually receive a score that's pretty easy to 
understand — A, B, C, D, or F. (If you do really well, the teacher may even draw a smiley 
face on the top of the page.) If only your ASVAB scores were as easy to understand. 

In the following list, you see how your ASVAB test scores result in several different kinds of 
scores: 



.S&WG. 




i*" Raw score: This score is the total number of points you receive on each subtest of the 
ASVAB. Although you don't see your raw scores on the ASVAB score cards, they're 
used to calculate the other scores. 

You can't use the practice tests in this book (or any other ASVAB study guide) to calcu- 
late your probable ASVAB score. ASVAB scores are calculated by using raw scores, and 
raw scores aren't determined simply from the number of right or wrong answers. On 
the actual ASVAB, harder math questions are worth more points than easier questions. 



Chapter 1: Putting the ASVAB under a Microscope f f 







SAMPLE CAT-ASVAB TEST SCORE REPORT 








Testing Site ID: 521342 














Service: AF 






Testing Session: Date: 2007/02/24 












Starting Time 


15:30 




Applicant: Jane P. Doe 














SNN: 


333-33-3333 




Test Form: 02E 














Test Type: Initial 




Standard Scores: GS 


AR 


WK 


PC 


MK 


EI 


AS 


MC 


AO 


VE 




63 


59 


60 


52 


56 


81 


64 


62 


52 


58 




COMPOSITE SCORES: 






















Army: GT 


CL 


CO 


EL 


FA 


GM 


MM 


OF 


SC 


ST 




118 
Air Force: M 


121 
A 


128 
G 


130 
E 


127 


132 


134 


129 


128 


125 




Figure 1-2: 


91 


76 


83 


96 














A sample 






















ASVAB 


Navy/CG: GT 


EL 


BEE 


ENG 


MEC 


MEC2 


NUC 


OPS 


HM 


ADM 


score card 


117 


259 


234 


120 


185 


173 


235 


225 


177 


114 


used for 

military 

enlistment 


Army: MM 
139 


GT 

122 


EL 

134 
















purposes. 




SAMPLE CAT-ASVAB TEST SCORE REPORT 







*>*!?? 




V Standard scores: The various subtests of the ASVAB are reported on the score cards 
as standard scores. A standard score is calculated by converting your raw score based 
on a standard distribution of scores with a mean of 50 and a standard deviation of 10. 

Don't confuse a standard score with the graded-on-a-curve score you may have seen 
on school tests — where the scores range from 1 to 100 with the majority of students 
scoring between 70 and 100. With standard scores, the majority score is between 30 
and 70. That means that a standard score of 50 is an average score and that a score of 
60 is an above-average score. 

v* Percentile scores: These scores range from 1 to 99. They express how well you did in 
comparison with another group called the norm. On the student version's score card, 
the norm is fellow students in your same grade (except for the AFQT score). 

On the enlistment and student versions' score cards, the AFQT score is presented as a 
percentile with the score normed using the 1997 Profile of American Youth, a national 
probability sample of 18- to 23-year-olds who took the ASVAB in 1997. For example, if 
you receive a percentile score of 72, you can say you scored as well as or better than 
72 out of 100 of the norm group who took the test. (And by the way, this statistic from 
1997 isn't a typo. The ASVAB was last "re-normed" in 2004, and the sample group used 
for the norm was those folks who took the test in 1997.) 

p" Composite scores (line scores): Composite scores are individually computed by each 
service branch. Each branch has its own particular system when compiling various 
standard scores into individual composite scores. These scores are used by the differ- 
ent branches to determine job qualifications. Find out much more about this in 
Chapter 2. 



/ 2 Part l: Making Sense of the ASVAB 




Understanding the big four: \lour AFQT scores 

The ASVAB doesn't have an overall score. When you hear someone say, "I got an 80 on my 
ASVAB," that person is talking about the Armed Forces Qualification Test (AFQT) score, not 
an overall ASVAB score. The AFQT score determines whether you qualify even to enlist in 
the military, and only four of the subtests are used to compute it: 

W Word Knowledge (WK) 
J-" Paragraph Comprehension (PC) 
v* Arithmetic Reasoning (AR) 
J-" Mathematics Knowledge (MK) 

Doing well on some of the other subtests is a personal-choice type of issue. Some of the sub- 
tests are used only to determine the jobs you qualify for. (See Chapter 2 for information on 
how the military uses the individual subtests.) 

Figure out which areas to focus on based on your career goals. If you're not interested in a 
job requiring a score on the Mechanical Comprehension subtest, you don't need to worry 
about doing well on that subtest. So as you're preparing for the ASVAB, remember to plan 
your study time wisely. If you don't need to worry about mechanics, don't bother with that 
chapter in this book. Spend the time on Word Knowledge or Arithmetic Reasoning. 

Calculating the AFQT score 

The military brass (or at least its computers) determines your AFQT score through a very 
particular process: 

1. Add the value of your Word Knowledge score to your Paragraph Comprehension 
score. 

2. Convert the result of Step 1 to a scaled score, ranging from 20 to 62. 

This score is known as your Verbal Expression or VE score. 

3. To get your raw AFQT score, double your VE score and then add your Arithmetic 
Reasoning (AR) score and your Mathematics Knowledge (MK) score to it. 

The basic equation looks like this: 

Raw AFQT Score = 2VE + AR + MK 

4. Convert your raw score to a percentile score, which basically compares your results 
to the results of thousands of other ASVAB test-takers. 

For example, a score of 50 means that you scored better than 50 percent of the individ- 
uals the military is comparing you to. 

Looking at AFQT score requirements for enlistment 

AFQT scores are grouped into five main categories based on the percentile score ranges in 
Table 1-3. Categories III and IV are divided into subgroups because the services sometimes 
use this chart for internal tracking purposes, enlistment limits, and enlistment incentives. 
Based on your scores, the military decides how trainable you may be to perform jobs in the 
service. 



Chapter 1: Putting the ASVAB under a Microscope f A 



Table 1-3 


AFQT Scores and Trainability 


Category 


Percentile Score 


Trainability 


I 


93-100 


Outstanding 


II 


65-92 


Excellent 


IMA 


50-64 


Above average 


NIB 


31-49 


Average 


IV A 


21-30 


Below average 


IV B 


16-20 


Markedly below average 


IV C 


10-15 


Poor 


V 


0-9 


Not trainable 



„^\NG. 




The U.S. Congress has directed that the military can't accept Category V recruits or more 
than 4 percent of recruits from Category IV. If you're in Category IV, you must have a high 
school diploma to be eligible for enlistment. Even so, if you're Category IV, your chances of 
enlistment are small and mostly limited to the Army. 

Depending on whether you have a high school diploma or a GED, the military has different 
AFQT score requirements. Check out Table 1-4. 



Table 1-4 



AFQT Score Requirements 





Minimum AFQT 


Minimum 


Branch 


Score with 


AFQT 


of 


High School 


Score 


Service 


Diploma 


with GED 


U.S. Air 


36 


65 


Force 







Special Circumstances 



In very rare cases, if the applicant possesses spe- 
cial skills (such as speaking a foreign language 
that the Air Force considers critical), the score of 
36 can be waived to 31. The Air Force allows less 
than 1 percent of its enlistees each year to have a 
GED instead of a high school diploma. 



Army 



31 



31 



Atthe time of this writing, the Army has been 
approving more and more waivers for those folks 
with scores as low as 26 (Category IV A). This is 
one of the standards the Army has changed in 
order to meet its recruiting requirements. 



Coast 
Guard 



40 



50 



A waiver is possible if a recruit's ASVAB line 
scores qualify him or her for a specific job and 
the recruit is willing to enlist in that job. Very few 
people (about 5 percent) each year are allowed to 
enlist with a GED. 



Marine 
Corps 



32 



50 



Between 5 and 10 percent of recruits can enlist 
with a GED. 



Navy 



31 



50 



From 5 to 10 percent of recruits can enlist with a 
GED. Those with a GED must also be at least 19 
and show a proven work history. 



f A Part I: Making Sense of the ASVAB 



The Navy has been known to raise its minimum AFQT requirements to 50 for females (just 
to qualify for enlistment) when it receives too many female applicants. Because of the lim- 
ited number of females that it can house on ships, the Navy restricts the number of women 
who can enlist each year. 

Checking out the military's AFQT requirements for special programs 

Achieving the minimum required AFQT score established by an individual branch gets your 
foot in the door, but the higher you score, the better. For example, if you need a medical or 
criminal history waiver in order to enlist, the military personnel who make those decisions 
are more likely to take a chance on you if they think you're a pretty smart cookie than if you 
barely made the minimum qualifying score. 

Individual branches of the military tie many special enlistment programs to minimum AFQT 
scores: 



*&&*« 




p* Army: The Army requires a minimum AFQT score of 50 to qualify for most of its incen- 
tive programs, such as a monetary enlistment bonus, the college-loan repayment pro- 
gram, and the Army College Fund. 

v* Marine Corps: Like the Army, the Marine Corps requires a minimum AFQT score of 50 
for most of its incentive programs, including the Geographic Area of Choice Program, 
the Marine Corps College Fund, and enlistment bonuses. 

J-" Navy: Applicants who want to participate in the Navy College Fund or college loan 
repayment program need to achieve a minimum score of 50. 

Enlistment programs are subject to change without notice based on the current recruiting 
needs of the service. Your recruiter should be able to give you the most up-to-date informa- 
tion. Or visit usmilitary . about . com. 

If you don't know which kind of job you want to do in the military, the ASVAB helps you and 
the military determine your potential ability for different types of jobs. If you're in this situa- 
tion, review all the chapters in this book, brushing up on the basic principles of everything 
from science to electronics, but focus on the four subtests that enable you to qualify for 
enlistment: Word Knowledge, Paragraph Comprehension, Arithmetic Reasoning, and 
Mathematics Knowledge. Following this plan ensures a relatively accurate appraisal of 
your aptitude for various military jobs. 



Oo-Oi/er: Retaking the ASVAB 



An AFQT score of less than 10 is a failing score, but no branch of the service accepts that 
low of a score anyway. Therefore, you can fail to achieve a score high enough to enlist in 
the service branch you want, even if you pass the ASVAB. This means you need to work 
on one (or more) of the four core areas: Mathematics Knowledge, Arithmetic Reasoning, 
Reading Comprehension, and Word Knowledge. Parts II and III of this book are specifically 
designed to help you improve your scores on these four subtests. 

When you're sure that you're ready, you can apply (through your recruiter) to take the 
ASVAB. After you take an initial ASVAB (taking the ASVAB in high school does count for 
retest purposes), you can retake the test after one month. After the first retest, you must 
again wait one month to test again. From that point on, you must wait at least six months 
before taking the ASVAB again. 

You can't retake the ASVAB on a whim or whenever you simply feel like it. Each of the ser- 
vices has its own rules concerning whether it allows a retest, and I explain them in the fol- 
lowing sections. 



Chapter 1: Putting the ASVAB under a Microscope f jy 



*fW* 




ASVAB tests are valid for two years, as long as you aren't in the military. In most cases, after 
you join the military, your ASVAB scores remain valid as long as you're in. In other words, 
except in a few cases, you can use your enlistment ASVAB scores to qualify for retraining 
years later. 



cjABE* 




U.S. Army retest policy 

The Army allows a retest in one of the following instances: 

J-" The applicant's previous ASVAB test has expired. 

i*" The applicant failed to achieve an AFQT score high enough to qualify for enlistment. 

J-" Unusual circumstances occur, such as if an applicant, through no fault of his own, is 
unable to complete the test. 

Army recruiters aren't authorized to have applicants retested for the sole purpose of increas- 
ing aptitude area scores to meet standards prescribed for enlistment options or programs. 



U.S. Air Force retest policy 



For the U.S. Air Force, the intent of retesting is for an applicant to improve the last ASVAB 
scores so the enlistment options increase. Before any retest is administered, the recruiting 
flight chief must interview the applicant in person or by telephone and then give approval 
for the retest. 



Here are a few other policies to remember: 

v* The Air Force doesn't allow retesting for applicants after they've enlisted in the 
Delayed Entry Program (DEP). 

i*" Current policy allows retesting of applicants who aren't holding a job/aptitude area 
reservation and/or who aren't in DEP but already have qualifying test scores. 

v* Retesting is authorized when the applicant's current line scores (mechanical, adminis- 
trative, general, and electronic) limit the ability to match an Air Force skill with his or 
her qualifications. 



U.S. Nai/y retest policy 

The Navy allows retesting of applicants 

Iv* Whose previous ASVAB tests have expired 
v* Who fail to achieve a qualifying AFQT score for enlistment in the Navy 

In most cases, individuals in the Delayed Entry Program (DEP) can't retest. One notable 
exception is the Navy's DEP Enrichment Program. This program provides for the provisional 
DEP enlistment of high school graduates with AFQT scores between 28 and 30. Individuals 
enlisted under the program are enrolled in academic enhancement training, retested with 
the ASVAB, and accessed to active duty, provided they score 31 or higher on the subse- 
quent ASVAB retest. 



/ Part I: Making Sense of the ASVAB 



Tracing the testing trail 



In 1948, Congress made the Department of Defense 
develop a uniform screening test to be used by all the 
services. The Defense Department came up with the 
Armed Forces Qualification Test (AFQT). This test con- 
sisted of 100 multiple-choice questions in areas such as 
math, vocabulary, spatial relations, and mechanical abil- 
ity. The military used this test until the mid-1970s. Each 
branch of the service set its own minimum qualification 
(AFQT) score. 

When the military decides to do something, it often acts 
with the lightning speed of a snail carrying a backpack. 
So in the 1960s, the Department of Defense decided to 
develop a standardized military selection and classifi- 
cation test and to administer it in high schools. That's 
where your old buddy, the ASVAB, came from. The first 
ASVAB testwas given in 1968, but the military didn't use 
it for recruiting purposes for several years. In 1973, the 
draft ended and the nation entered the contemporary 
period in which all military recruits are volunteers. In 



1976, the ASVAB became the official entry test used by 
all services. 

The ASVAB remained unchanged until 1980, when the 
ASVAB underwent its first revision. The subtest areas 
remained the same, but several of the questions were 
updated to keep up with changes in technology. 

In 1993, the computerized version was released for lim- 
ited operational testing, but it didn't begin to see wide- 
scale use until 1996. The questions on the computerized 
version of the ASVAB were identical to the questions 
on the paper version. It wasn't until the end of 2002 that 
the ASVAB finally underwent a major revision. Two sub- 
sets (Coding Speed and Numerical Operations) were 
eliminated and a new subtest (Assembling Objects)was 
added to the computerized version. Also during the 2002 
revision, all the questions were updated, and the order 
of the subtests was changed. The revised ASVAB was 
first rolled out in the computerized format, and the paper 
versions of the test were updated during the next year. 



tf&ER 




U.S. Marine Corps retest policy 

The Marine Corps authorizes a retest if the applicant's previous test is expired. Otherwise, 
recruiters can request a retest if the initial scores don't appear to reflect the applicant's 
true capability, considering the applicant's education, training, and experience. 

For the Marine Corps, the retest can't be requested solely because the applicant's initial test 
scores didn't meet the standards prescribed for enlistment options or programs. 



U.S. Coast Guard retest policy 

For Coast Guard enlistments, six months must have elapsed since an applicant's last test 
before he or she may retest solely for the purpose of raising scores to qualify for a particu- 
lar enlistment option. 

The Coast Guard Recruiting Center may authorize retesting after one calendar month has 
passed from an initial ASVAB test if substantial reason exists to believe the initial test 
scores or subtest scores don't reflect an applicant's education, training, or experience. 



Chapter 2 

Knowing What It Takes to Get 
Your Dream Job 



In This Chapter 

Finding out there's more to life than the AFQT score 

Making sense out of line scores 

Discovering how each military branch uses line scores 



T 

m he Armed Forces Qualification Test (AFQT) portion of the ASVAB is your most impor- 
P tant score because it determines whether you can join the service of your choice. 
However, qualifying to join is only part of the picture. Unless you'd be content to spend 
your military career painting things that don't move, you need to understand how the 
ASVAB relates to various military job opportunities. 

Civilian employers generally use a person's education and experience level when selecting 
candidates for a job position, but in the military, 99 percent of all enlisted jobs are entry- 
level positions. The military doesn't require you to have a college degree in computer sci- 
ence before you're hired to become a computer programmer. You don't even have to have 
any previous computer experience, nor does the military care if you do. You're going to go 
to military school to study how to make computers stand at attention and fly right. 

Sounds like a good deal, right? So what's the catch? Well, believe me — the military spends 
big bucks turning high school graduates into highly trained and skilled aircraft mechanics, 
language specialists, and electronic-doodad repair people. In an average year, the services 
enlist about 274,000 new recruits. Any way you look at it, that's a lot of combat boots! Each 
and every recruit has to be sent to a military school to train for a job. Uncle Sam needs a 
way to determine whether a wet-behind-the-ears high school graduate has the mental apti- 
tude to succeed at that job — preferably before he spends your hard-earned tax dollars. 

Enter the ASVAB. The services combine various ASVAB subtest scores into groupings called 
composite scores or line scores. Through years of trial and error, the individual military ser- 
vices have each determined what minimum composite scores are required to successfully 
complete its various job training programs. In this chapter, you discover how those test 
scores translate into finding the military job of your dreams. 



Eyeing Hou) ASVAB Scores determine Military 
Training Programs and Jobs 

Each service branch has its own system of scores. Recruiters and military job counselors 
use these scores, along with other factors such as job availability, security clearance eligi- 
bility, medical qualifications, and physical strength, to match up potential recruits with 
military jobs. 



/ £ Part I: Making Sense of the ASVAB 



s#*** 




During the initial enlistment process, your service branch determines your military job or 
enlistment program based on established minimum line scores: various combinations of 
scores from individual subtests (see the next section for details). If you get an appropriate 
score in the appropriate areas, you can get the job you want — as long as that job is avail- 
able and you meet other qualification factors. 

For active duty, the Army is the only service that looks at the scores and offers a guaran- 
teed job for all its new enlistees. In other words, every single Army recruit knows what his 
or her job is going to be before signing the enlistment contract. The other active-duty ser- 
vices use a combination of guaranteed jobs or guaranteed aptitude/career areas: 

v* Air Force: About 40 percent of active duty Air Force recruits enlist with a guaranteed 
job. The majority enlists in one of four guaranteed aptitude areas, and during basic 
training, recruits are assigned to a job that falls into that aptitude area. 

i*" Coast Guard: The Coast Guard rarely, if ever, offers a guaranteed job in its active duty 
enlistment contracts. Instead, new Coasties enlist as undesignated seamen and spend 
their first year or so of service doing general work ("Paint that ship!") before finally 
applying for specific job training. 

p" Marine Corps: A vast majority of Marine Corps active duty enlistees are guaranteed 
one of several job fields, such as infantry, avionics, logistics, vehicle maintenance, air- 
craft maintenance, munitions, and so on. Each of these fields is further divided into 
specific subjobs, called Military Occupation Specialties (MOS). Marine recruits usually 
don't find out their actual MOSs until about halfway through basic training. 

j*" Navy: Most Navy recruits enlist with a guaranteed job, but several hundred people 
each year also enlist in a guaranteed career area and then strike (apply) for the specific 
job within a year of graduating boot camp. 

All enlistment contracts for the reserve forces (regardless of branch) contain guarantees for 
a specific job. Why? Because reserve recruiters recruit for vacancies in specific reserve 
units, usually located within 100 miles of where a person lives. 



Understanding HovO Each Branch Computes 
Line Scores 

A line score combines various standard ASVAB scores to see which jobs or training programs 
you qualify for. The standard scores are your scores on the individual ASVAB subtests (with 
Word Knowledge and Paragraph Comprehension combined as a Verbal Expression score): 

J-" General Science (GS) 

v* Arithmetic Reasoning (AR) 

(-" Auto & Shop Information (AS) 

J-" Mathematics Knowledge (MK) 

v* Mechanical Comprehension (MC) 

v* Electronics Information (EI) 

j-" Assembling Objects (AO) 

v* Verbal Expression (VE), the sum of Word Knowledge (WK) and Paragraph 
Comprehension (PC) 



Chapter 2: Knowing What It Takes to Get Your Dream Job 



19 



Each of the military services computes its line scores differently. Some calculations even 
include dummy scores — average scores received by thousands of test takers — for 
Numerical Operations (NO) and Coding Speed (CS), subtests that are no longer part of the 
ASVAB. The following sections outline how each branch comes up with its line scores. 



Line scores and the Army 



To compute line scores for job qualification, the Army combines the various scores into ten 
separate areas by simple addition of the ASVAB standard scores. Table 2-1 shows the line 
scores and the ASVAB subtests that make them up. 



Table 2-1 


The U.S. Army's Ten Line Scores 






Line Score 


Standard Scores Used 


Formula Used 




Clerical (CL) 


Verbal Expression (VE), Arithmetic Reasoning 
(AR), and Mathematics Knowledge (MK) 


VE + AR + MK 




Combat(CO) 


Arithmetic Reasoning (AR), Coding Speed (CS), 
Auto & Shop Information (AS), and Mechanical 
Comprehension (MC) 


AR + CS + AS + 


MC 


Electronics (EL) 


General Science (GS), Arithmetic Reasoning (AR), 
Mathematics Knowledge (MK), and Electronics 
Information (El) 


GS+AR + MK + EI 


Field Artillery 
(FA) 


Arithmetic Reasoning (AR), Coding Speed (CS), 
Mathematics Knowledge (MK), and Mechanical 
Comprehension (MC) 


AR + CS + MK-i 


-MC 


General 

Maintenance 

(GM) 


General Science (GS), Auto & Shop Information 
(AS), Mathematics Knowledge (MK), and 
Electronics Information (El) 


GS+AS + MK + EI 


General 
Technical (GT) 


Verbal Expression (VE) and Arithmetic 
Reasoning (AR) 


VE + AR 




Mechanical 

Maintenance 

(MM) 


Numerical Operations (NO), Auto & Shop 
Information (AS), Mechanical Comprehension 
(MC), and Electronics Information (El) 


NO + AS + MC 


f El 


Operators and 
Food (OF) 


Verbal Expression (VE), Numerical Operations 
(NO), Auto & Shop Information (AS), and 
Mechanical Comprehension (MC) 


VE + N0 + AS + 


MC 


Surveillance and 
Communications 
(SO 


Verbal Expression (VE), Arithmetic Reasoning (AR), 
Auto & Shop Information (AS), and Mechanical 
Comprehension (MC) 


VE + AR + AS + 


MC 


Skilled Technical 
(ST) 


General Science (GS), Verbal Expression (VE), 
Mathematics Knowledge (MK), and Mechanical 
Comprehension (MC) 


GS + VE + MK + 


MC 



Line scores and the Nai/y and Coast Guard 

The Navy and Coast Guard use the standard scores directly from the ASVAB: the individual 
subtest scores and Verbal Expression (VE) score, which is the sum of Word Knowledge (WK) 
and Paragraph Comprehension (PC). 



2Q Part I: Making Sense of the ASVAB 



Although the Navy and Coast Guard don't use their line scores for officially determining 
jobs, the scores provide recruiters, job counselors, and recruits with a snapshot of which 
broad career areas recruits may qualify for. For example, the Navy regulation, which lists 
the qualifications to become an Air Traffic Control Specialist, states that an ASVAB score of 
VE + AR + MK + MC = 210 (or higher) is required for that job. 

Table 2-2 shows the Navy and Coast Guard line scores that show up on the ASVAB score 
sheet. 



Table 2-2 


The U.S. Navy and Coast Guard's Line 


Scores 


Line Score 


Standard Scores Used 


Formula Used 


Engineman (ENG) 


Auto & Shop Information (AS) and 
Mathematics Knowledge (MK) 


AS + MK 


Administrative (ADM) 


Mathematics Knowledge (MK) and Verbal 
Expression (VE) 


MK + VE 


General Technical (GT) 


Arithmetic Reasoning (AR) and Verbal 
Expression (VE) 


AR + VE 


Mechanical 
Maintenance (MEC) 


Arithmetic Reasoning (AR), Auto & 
Shop Information (AS), and Mechanical 
Comprehension (MC) 


AR + AS + MC 


Health (HM) 


General Science (GS), Mathematics 
Knowledge (MK), and Verbal Expression (VE) 


GS + MK + VE 


Mechanical 
Maintenance 2 
(MEC2) 


Assembling Objects (A0), Arithmetic 
Reasoning (AR), and Mechanical 
Comprehension (MC) 


AO + AR + MC 


Electronics (EL) 


Arithmetic Reasoning (AR), Electronics 
Information (El), General Science (GS), and 
Mathematics Knowledge (MK) 


AR + EI + GS + MK 


Nuclear Field (NUC) 


Arithmetic Reasoning (AR), Mechanical 
Comprehension (MC), Mathematics 
Knowledge (MK), and Verbal Expression (VE) 


AR + MC + MK + VE 


Engineering and 
Electronics (BEE) 


Arithmetic Reasoning (AR), General Science 
(GS), and two times Mathematics Knowledge 
(MK) 


AR + GS + 2MK 


Operations (OPS) 


Arithmetic Reasoning (AR) and Mathematics 
Knowledge (MK) 


AR + MK 



Line scores and the Marine Corps 

The Marine Corps computes its three line scores for job qualification by adding scores from 
various ASVAB subtests, as Table 2-3 shows. 



Chapter 2: Knowing What It Takes to Get Your Dream Job J? / 



Table 2-3 


The Marine Corps's Line Scores 




Line Score 


Standard Scores Used 


Formula Used 


Mechanical 
Maintenance (MM) 


General Science (GS), Auto & Shop Information 
(AS), Mathematics Knowledge (MK), and 
Mechanical Comprehension (MC) 


GS + AS + MK + MC 


General Technical 
(GT) 


Verbal Expression (VE) and Arithmetic Reasoning 
(AR) 


VE + AR 


Electronics (EL) 


General Science (GS), Arithmetic Reasoning (AR), 
Mathematics Knowledge (MK), and Electronics 
Information (El) 


GS+AR + MK + EI 



Line scores and the Air Force 



The U.S. Air Force uses standard scores from the ASVAB subtests to derive scaled scores in 
four aptitude areas called MAGE (mechanical, administrative, general, and electronics). The 
Air Force MAGE scores are calculated as percentiles, ranging from to 99, which show your 
relationship to thousands of others who've taken the test. In other words, a percentile score 
of 51 indicates you scored better in this aptitude area than 50 percent of the testers who 
were used to establish the "norm." 

Table 2-4 lays out the four areas, the subtests used, and the formula used to calculate the 
score for each particular area. After calculating the score for a particular area, the test- 
scorer converts that score to a percentile. 



Table 2-4 


The U.S. Air Force's MAGE Scores 




Line Score 


Standard Scores Used 


Formula Used 


Mechanical 


General Science (GS), Mechanical 
Comprehension (MC), and two times Auto & 
Shop Information (AS) 


GS + MC + 2AS 


Administrative 


Numerical Operations (NO), Coding Speed (CS), 
and Verbal Expression (VE) 


NO + CS + VE 


General 


Arithmetic Reasoning (AR) and Verbal 
Expression (VE) 


AR + VE 


Electronics 


General Science (GS), Arithmetic Reasoning (AR), 
Mathematics Knowledge (MK), and Electronics 
Information (El) 


GS+AR + MK + EI 



22 Part l: Makin 9 Sense of the ASVAB 




Score! Speaking the lingo 



When you sit down with your recruiter to discuss your 
ASVAB scores and what you qualify for, you may think 
he suddenly decided to speak in a foreign language. For 
job-qualification purposes, remember three key terms 
and their definitions: 

\^ Standard score: A standard score refers to indi- 
vidual ASVAB subtest scores (that is. Verbal 
Expression, Arithmetic Reasoning, Mathematics 
Knowledge, and so on). 



Line score: A line score combines various standard 
scores that the services use for job qualification 
purposes. 

AFQT score: Calculated from the math and 
English subtests of the ASVAB, the Armed Forces 
Qualification Test (AFQT) score is used by the mili- 
tary to determine overall enlistment qualification. 
Chapter 1 explains exactly how this critical score is 
computed. 



Chapter 3 

Getting Acquainted with Test-Taking 
and Study Techniques 

• ••••••••••••••••••••••••••••••••••••••••••••••••••••••• 

In This Chapter 

^ Choosing your weapon: Pencil or keyboard 

Developing multiple-choice strategies 

Making educated guesses 

Getting some studying and test tips 

Preparing down to the last detail 
• ••••••••••••••••••••••••••••••••••••••••••••••••••••••• 

Wow many times have you heard someone say (or may have even said yourself), "I just 
can't take tests"? Well, of course you can't do well on tests if you keep telling yourself 
that! In basic training, your drill sergeant (hereafter known as "Sir" or "Ma'am") will con- 
vince you that the words "I can't" simply don't exist in the military. If you don't believe me, 
try telling your drill sergeant, "I just can't do push-ups." You will find that with sufficient 
practice (and your drill sergeant will ensure you get a lot of practice), you can do push-ups 
just as well as the next person. (Actually, I don't recommend testing this, for reasons that 
should be obvious.) The truth is that those who do well on tests are those who've figured 
out how to study efficiently and how to use a dash of test-taking psychology. 

This chapter includes information on how to prepare for the test — how you study and how 
and why you should take the practice exams. In addition, you get some inside info, such as 
secrets for guessing when you don't know the answer to a question (although if you study 
for the test, that will never happen, right?). The tips and techniques provided in this chap- 
ter can help you get a jump on the ASVAB and your military career. 

lakinq the Test: Paper or Computerized? 

Many versions of the ASVAB exist (although you probably won't get a choice of which one 
to take), but they primarily boil down to two basic differences: the paper version and the 
computerized version. Each version has advantages and disadvantages, which I discuss in 
the following sections. 

If you're taking the ASVAB as part of the student program in high school, or if you're already 
in the military and are retaking the ASVAB to qualify to retrain into a different job, you'll 
take the paper version. 

If you're taking the ASVAB to enlist in the military, you'll take the enlistment ASVAB. This 
version is available in paper format and via computer. There's a great chance that you'll 



2Z( Part I: Making Sense of the ASVAB 




take the computerized version (CAT-ASVAB), because to save time and money, the recruit- 
ing services often send applicants to the nearest Military Entrance Processing Station 
(MEPS) for testing, medical examination, and enlistment (one-stop shopping). The comput- 
erized version is used exclusively at MEPS. 

If you have your heart set on taking the test in paper format, ask your recruiter whether a 
Mobile Examination Test (MET) site is nearby. Roughly 685 MET sites are located through- 
out the United States (generally located in National Guard Armories). Your recruiter can 
schedule you to take the enlistment paper version at any one of these MET sites, which may 
offer testing sessions anywhere from once a month to several times per week. 



Writing on hard copy: The advantages 
and disadvantages of the paper Version 

Modern technology isn't always better. Taking the pencil-and-paper version of the ASVAB 
can provide you with certain advantages: 

is You can skip questions that you don't know the answer to and come back to them 
later. This option can help when you're racing against the clock and want to get as 
many answers right as possible. You can change an answer on the subtest you're cur- 
rently working on, but you can't change an answer on a subtest after the time for that 
subtest has expired. 

v* You may not make any marks in the exam booklet; however, you may make notes 
on your scratch paper. If you skip a question, you can lightly circle the item number 
on your answer sheet to remind yourself to go back to it. If you don't know the answer 
to a question, you can mentally cross off the answers that seem unlikely or wrong 
to you and then guess based on the remaining answers. Be sure to erase any stray 
marks you make on your answer sheet before time is called for that subtest. 

Killing trees isn't the only disadvantage of the paper-based test. Other drawbacks include 
the following: 

v* Harder questions are randomly intermingled with easier questions. This means you 
can find yourself spending too much time trying to figure out the answer to a question 
that's too hard for you and may miss answering some easier questions at the end of 
the subtest, thereby lowering your overall score. 

v* The paper answer sheets are scored by using an optical mark scanning machine. 

The machine has a conniption when it comes across an incompletely filled-in answer 
circle or stray pencil marks and will often stubbornly refuse to give you credit, even if 
you answered correctly. 

v* Getting your scores may seem like it takes forever. The timeline varies; however, 
your recruiter will have access to your score no later than 72 hours (3 days) after 
you finish the test (not counting days the MEPS doesn't work, such as weekend days 
or holidays). 



Going paperless: The pros and cons 
of the computerized test 

The computerized version of the ASVAB, called computerized-adaptive testing, or CAT- 
ASVAB, contains questions similar to the ones on the paper version, but the questions are 
presented in a different order. The CAT-ASVAB adapts the questions it offers you based on 



Chapter 3: Getting Acquainted with Test-Taking and Study Techniques ^S 



WER 




your level of proficiency (that's why it's called adaptive). Translation: The first test item is 
of average difficulty. If you answer this question correctly, the next question will be more 
difficult. If you answer it incorrectly, the computer will give you an easier question. By con- 
trast, on the paper ASVAB, hard and easy questions are presented randomly. 

The CAT-ASVAB also has significantly fewer questions than the pencil-and-paper version of 
the test, although the questions tend to be a bit harder, which tends to result in the same 
scores (level of knowledge). 

Maybe it's because people today are more comfortable in front of a computer than with a 
pencil, but military recruiters have noted that among applicants who've taken both the 
paper-based and computerized versions of the ASVAB, many applicants tend to score 
slightly higher on the computerized version of the test. 

You don't have to be a computer guru to appreciate the advantages of the computerized 
version of the ASVAB: 

v* It's impossible to record your answer in the wrong space on the answer sheet. 

Questions and possible answers are presented on the screen, and you press the key 
that corresponds to your answer choice before moving on to the next question. Often, 
only the A, B, C, and D keys are activated when you take the test. 

v* The difficulty of the test items presented depends on whether you answered the pre- 
vious question correctly. On the two math subtests of the ASVAB, harder questions 
are worth more points than easier questions, so this method helps maximize your 
AFQT score. 

j*" You get your scores right away. The computer automatically calculates and prints 
your standard scores for each subtest and your line scores for each service branch. 
(For more on line scores, see Chapter 2.) This machine is a pretty smart cookie — it 
also calculates your AFQT percentile score on-the-spot. You usually know whether you 
qualify for military enlistment on the same day you take the test and, if so, which jobs 
you qualify for. 

On the downside, you can't skip questions or change your answers after you enter them 
on the CAT-ASVAB. Instead of being able to go through and immediately answer all the 
questions you're sure of, you have to answer each question as it comes. This can make it 
difficult to judge how much time to spend on a difficult question before guessing and 
moving on. Also, if you have a few minutes at the end of the test, you can't go back and 
make sure you marked the correct answer to each question. 



Tackling Multiple-Choice Questions 



Both the paper-based and the computerized versions of the ASVAB are multiple-choice 
tests. You choose the correct (or most correct) answer from among the available (usually 
four) choices. Here are some tips to keep in mind as you tackle the choices: 

v* Read the directions carefully. Most ASVAB test proctors agree — the majority of 
the time when there's an issue with an applicant's scores, misreading directions is a 
prime offender. Each subtest has a paragraph or two describing what the subtest 
covers and instructions on how to answer the questions. If the directions on Paragraph 
Comprehension subtest informs you that a paragraph applies to questions 3, 4, and 5 
and you misread it as 4, 5, and 6, you're probably going to get at least one of those 
questions wrong. 



2 {J Part I: Making Sense of the ASVAB 



jJ^ 




tfBE/? 




\»NG/ 




e* Make sure you understand the question. If you don't understand the question, you're 
naturally not going to be able to make the best decision when selecting an answer. 
Understanding the question requires attention to three particular points: 

• Take special care to read the questions correctly. Most questions ask some- 
thing like, "Which of the following equals 2 x 3?" But sometimes, a question may 
ask, "Which of the following does not equal 2 x 3?" You can easily skip right 
over the word not when you're reading, assume that the answer is 6, and get 
the question wrong. 

• On the math subtests, be especially careful to read the symbols. When you're 
in a hurry, the + sign and the * sign can look very similar. And blowing right by a 
negative sign or another symbol is just as easy. 

• Make sure you understand the terms being used. When a math problem asks 
you to find the product of two numbers, be sure you know what finding the prod- 
uct means (you have to multiply the two numbers). If you add the two numbers, 
you arrive at the wrong answer. 

v* Take time to review all the answer options. On all the subtests, you almost always 
select the correct answer from only four possible answer options. On the ASVAB, 
you're supposed to choose the answer that is most correct. (Now and then you do the 
opposite and choose the answer that's least correct.) Sometimes several answers are 
reasonably correct for the question at hand, but only one of them is the best answer. If 
you don't stop to read and review all the answers, you may not choose the one that's 
most correct. Or if you review all the answer options, you may realize that you hastily 
decided on an incorrect answer because you misread it. 

Often, a person reads a question, decides on the answer, glances at the answer options, 
chooses the option that agrees with his or her answer, marks it on the answer sheet, 
and then moves on. Although this approach usually works, it can sometimes lead you 
astray. 

v* If you're taking a paper test, mark the answer carefully. A machine scores the paper- 
based ASVAB answer sheets. You have to mark the answer clearly so that the machine 
knows which answer you've selected. This means carefully filling in the space that rep- 
resents the correct letter. You've done this a million times in school, but it's worth 
repeating: Don't use a check mark, don't circle the answer, and don't let your mark 
wander into the next space. If you must erase, make sure all evidence of your prior 
choice is gone; otherwise, the grading machine may credit you with the wrong choice 
or disregard your correct answer and give you no credit at all. 

Incorrectly marking the answer sheet — answering Question 1 1 on the line for 
Question 12, Question 12 on the line for Question 13, and . . . you get the idea — is a 
very real possibility. Be especially careful if you skip a question that you're going to 
return to later. 

Incorrectly marking the answers can cause a real headache. If you fail to get a qualify- 
ing score, the minimum amount of time you must wait before retaking the ASVAB is 
one month. Even then, your journey to military glory through ASVAB torment may not 
be over. If within six months of a previous test, your retest AFQT score increases by 
20 points or more, you'll be required by MEPCOM regulation to take an additional 
ASVAB test, called a confirmation test. (Confirmation tests can be taken only at MEPS 
facilities, by the way.) So if you're not careful, you'll be taking three ASVABs when all 
you really needed to take was one. Sound fun? Chapter 1 discusses how and when you 
can retake the ASVAB. 



Chapter 3: Getting Acquainted with Test-Taking and Study Techniques J/ 7 



When l/ou Don't Know an Answer: 
Guessing Smart 





On the ASVAB, guessing is okay. In fact, it's encouraged. The reason is in the scoring of the 
test. Here's how the point system breaks down: 

u* If you choose the correct answer, you get one point (or more, depending on how the 
question is weighted). 

u* If you don't answer a question, you get nada. 

u* If you guess on a question and get the question wrong, you get nada — no worrying 
about losing points or getting any sort of penalty! 

Because most questions have four possible answers, you generally have a 25 percent 
chance of guessing correctly, which means that you have chances to increase your score by 
guessing. Here are some guessing tips: 

u* There's always at least one answer that isn't even close to the correct answer. By using 
simple deduction, you can often narrow your choices down to two answers or fewer. 

j*" Don't eliminate an answer based on how frequently that answer comes up. For exam- 
ple, if Choice (B) has been the correct answer for the last five questions, don't assume 
that it must be the wrong answer for the question you're on just because that would 
make it six in a row. 

J*" Usually, an answer that has always, all, everyone, never, none, or no one is incorrect. 

v* The longer the answer, the more likely that it's the correct answer. The test-makers have 
to get all those qualifiers in there so you can't find an example to contradict the correct 
answer. If you see phrases like "in many cases" or "frequently," that's a clue that the test- 
makers are trying to make the answer most correct. 

v* If two choices are very similar in meaning, neither of them is probably the correct 
choice. On the other hand, if two answer options contradict each other, one of them is 
usually correct. 

J-" Don't change an answer after you select it. If you have to guess, never, ever go back 
and change the answer, unless you're absolutely, 100 percent, positively convinced 
that you're changing it to the correct answer and you only answered incorrectly 
because you had sweat in your eyes and didn't read the choices properly. 

The United Stated Air Force Senior NCO Academy conducted an in-depth study of sev- 
eral Air Force multiple-choice test results, taken over several years, and found that 
when students changed answers on their answer sheets, they changed from a right 
answer to a wrong answer more than 72 percent of the time! 

In each of the chapters in this book that reviews a particular subtest, you find more hints 
for making educated guesses that are specific to those topics. 

If you guess on more than one question throughout the test, choosing the same answer for 
every guess is a smart way to go. For example, all your guesses could be Choice (B). This 
technique slightly increases your chances of getting more answers correct. However, if you 
can eliminate Choice (B) as a wrong answer, then, by all means, choose a different answer 
option for that question. 



28 Part l: Making Sense of the ASVAB 





Studying and Practicing (or the ASVAB 

The practice tests that come with this book are valuable study aids. Before you begin study- 
ing, take one of the tests. Feel free to take a paper test in the book or pop the CD into your 
computer and go the electronic route. Either way, try to duplicate the testing environment — 
take the entire exam at one time, time yourself, and don't allow interruptions. 

The military has a saying, "Train as you expect to fight." The same is true of the ASVAB. If 
you plan to take the pencil-and-paper version of the ASVAB, concentrate most of your efforts 
on the written practice tests in this book. If you'll be taking the CAT-ASVAB, spend most of 
your time practicing on the included CD. 

Get a sense of how long it takes you to complete each subtest so you know how much 
time you have to spend on educated guessing. After you complete the first practice test, 
check your answers to see where you need improvement. 

When you study for the ASVAB, fall in line with these study habits to make the most of 
your time: 

v* Focus on the subtests that matter to you. If you have no interest in pursuing a career 
in electronics, the Electronics Information subtest is irrelevant to you, so don't spend 
time studying for it. Instead, devote yourself to areas that are important to your future 
career plans. (See Chapter 2 and Appendix A for lists of the subtests that affect your 
acceptance into the job areas you're pursuing.) 

J*" Concentrate on subject areas that need improvement. It's human nature to find your- 
self spending your study time on subject areas that you have an interest in or that 
you're good at. If you're a whiz at fixing cars, don't waste your time studying auto infor- 
mation. You're already going to ace that part of the test, right? On the other hand, if 
you had a hard time in math during your high school years, you need to spend extra 
time brushing up on your arithmetic skills. 

i^Bea loner. You may want to study with a partner now and then so the two of you can 
brainstorm answers and quiz each other, but most of your studying should be done on 
your own. 

J-" Try to reduce distractions. Always study in a well-lit, quiet area away from pets, 
screaming babies, and the TV. 

v* Study in long blocks of time. Studying for an hour or two once or twice a day is much 
more effective than 15 minutes six times a day. 

i*" Keep study breaks short. A few minutes every hour is sufficient. Don't ignore breaks 
completely, however. Studies show that taking short breaks improves how well you're 
able to remember information. 

v* Practice the actual act of test-taking. Practice marking answers correctly on the 
answer key and time yourself to see how long it takes you to answer questions. 

After you do some additional studying, take the second practice exam. Again, try to dupli- 
cate testing conditions. Check your answers. Compare your scores to the scores from your 
first test. Have you improved? If so, continue studying as you have been. If not, reconsider 
how you're studying or whether you're setting aside enough time to study. A school coun- 
selor or teacher can give you additional study pointers. Continue practicing with the next 
few tests on the CD. 



A couple of weeks before the ASVAB, take the next-to-last practice test. Brush up on any of 
those nagging areas that still give you fits. Check to see which areas you need help with and 
spend more time studying those areas. 



Chapter 3: Getting Acquainted with Test-Taking and Study Techniques £y 



cjABE* 




A week before your test date, take the last test. This test helps you calm your nerves before 
taking the ASVAB — how the test works will be fresh in your mind. 

Don't waste time memorizing the practice questions in this guide or any other ASVAB study 
guide. You won't see the same questions on the ASVAB. Use this guide and the sample tests 
for two purposes: 

J*" To determine the subject areas in which you need to improve: Use the tips and tech- 
niques, along with standard study materials (like high-school textbooks), to improve 
your knowledge of that specific subject. 

u* To familiarize yourself with the types of test questions and the way they're pre- 
sented on the test: Getting a good idea of what all the subtests look like will improve 
your test-taking speed. You won't have to spend time trying to figure out how a ques- 
tion looks. You can spend your time answering the question. 



Making Last-Minute Preparations: 
21* Horns and Counting 




You want some good advice? On the night before the test, get some sleep — at least eight 
hours. Don't drink alcohol the night before — headaches and the ASVAB don't work well 
together. And don't pull an all-night cram session. If you don't know the material the night 
before the test, it's too late. Staying up all night only guarantees that you'll do poorly on the 
test, because you'll be too tired in the morning. Here are some other suggestions: 

«*" On the morning of the test, eat a light meal. Anything too heavy will make you drowsy, 
but not eating enough will make it hard for you to concentrate. 

Try to avoid a breakfast high in carbohydrates. Although the carbs will initially make 
you feel energetic, a couple of hours into the test, you may come crashing down. Select 
foods high in protein instead. 

J*" Get exercise the day before and even the morning of the test. Doing so gets your 
blood pumping and helps you remain mentally sharp. 

v* If you're sick, upset, or injured, consider rescheduling the test. Right before the test 
starts, the proctor will ask if there's anything, such as sickness or injury, that may 
affect your test performance. After the test actually starts, it's considered an "official 
test," and you'll have to wait a certain time period before any possibility of a retest. 
See Chapter 1 for details. 

v* Don't bring personal supplies to the test. Your test administrator will provide you 
with pencils and scratch paper. Don't bring calculators, personal CD players, back- 
packs, or a cooler of munchies to the testing site. You won't be allowed to have them 
with you. (But if you wear eyeglasses, bring them. If you wear contacts, bring your 
glasses as a backup.) 

v* Bring a watch to help you keep track of time if you're taking the paper version. The 

computerized version has a clock on the screen. 

J-" Don't drink a lot of liquids just before the test. You don't want to waste valuable test 
time in the bathroom! 

v* Make sure you arrive at the test site with plenty of time to spare. In the military, 
arriving on time means that you're five minutes too late. You should plan to be in your 
seat at least 15 minutes before the scheduled testing time. Unless your recruiter is driv- 
ing you (which is often the case), you may want to do a test run a day or two before 
your testing date to make sure you know where the test is, the availability of parking, 
and how to find the testing room. 



%0 Part I: Making Sense of the ASVAB 



Meeting supply and demand 



The United States Congress sets the authorized size 
(called strength ceilings) for each of the service 
branches. The Army is the largest branch by far. To 
maintain its strength ceiling, the active duty Army has 
to recruit 80,000 new troops each year. Compare this 
number with the 36,000 for active duty Navy, 30,000 
for active duty Air Force, 32,000 for the Marine Corps, 
and 4,000 for the Coast Guard. Because of these 
higher recruiting requirements and because the Army 
bears the brunt of deployments to such places as 
Iraq, Afghanistan, Bosnia, and Kosovo, the Army has 
been forced to lower many of its recruiting standards. 
On the other hand, the Army also offers higher enlist- 
ment bonuses and other incentives than do the other 
branches. 

Congress sets the permissible ranges, but the individual 
services can act within those ranges based on its cur- 
rent recruiting needs. For example. Congress has set 
the maximum allowable enlistment bonus to $40,000. 
But only the Army offers the maximum, and then only 



for a few "hard-to-fill" jobs. The Air Force (and the other 
services) can also offer $40,000 if it feels it needs to, but 
it doesn't. (The maximum bonus currently authorized by 
the Air Force is $16,000 for a six-year enlistment as a 
Linguist. The Army, on the other hand, gives a $40,000 
bonus to an applicant who's "trainable" for the Linguist 
position for a 4-year enlistment.) 

The same is true for ASVAB score categories. Congress 
has said that the maximum number of CAT IVs (poor to 
below-average trainability) that any service can accept 
per year is 4 percent. However, probably due in large 
part to the lackluster civilian job market and the chal- 
lenging economy, all the services have found that 
they're getting more than enough volunteers to fill their 
needs without accepting CAT IV applicants. In fact, even 
among high school diploma-holders, the services often 
get more applicants than they need and often use the 
ASVAB scores to pick and choose who they'll take. The 
higher your AFQT score, the higher your chances that 
the service branch of your choice will take you. 



Part II 



Words to Live By: 
Communication Skills 



The 5 th Wave By Rich Tennant 




"I guessed my vtay to a perfect score on 
the ASVA3-, then I guessed my -way through 
basic training and several military jobs. So - 
■would you like me -to guess your •weight?" 



In this part . . . 



T 

m he ASVAB measures your communication skills 
P through two subtests: Word Knowledge and Paragraph 
Comprehension. Together, these two subtests make up one- 
half of your Armed Forces Qualification Test (AFQT) score, 
which the military uses to predict whether you're teach- 
able. If you don't achieve a qualifying score on the AFQT, 
you're not allowed to play alongside the other soldiers. 

In this part, you discover why it's important to do well on 
the ASVAB vocabulary and reading subtests, and you find 
some useful tools to accomplish the mission. You review 
basic vocabulary and reading skills and find rock-solid 
advice, such as how to find the main idea of a paragraph, 
and see quick tips for defining a vocabulary word based 
on context, roots, prefixes, and suffixes. 



Chapter 4 

Word Knowledge 



In This Chapter 

Being well-spoken in the military 

Seeing some example questions from the ASVAB 

Keeping a word list 

Knowing the difference between synonyms and antonyms 

Improving your overall vocabulary 



T 

m o make it to boot camp, you'd better know how to spell it (along with an army of other 
m words) to score well on the Word Knowledge subtest of the ASVAB. Not only do you have 
to know how to spell to some degree (so you can differentiate among words), but you also you 
need to know what the words on the test mean. Word Knowledge just means vocabulary, which 
means hard words no one uses in ordinary conversation. (Well, not really.) If you're on a military 
base and you're hungry, don't bother looking for a sign that says Chow Hall. Instead, you need 
to find the Enlisted Personnel Dining Facility. If you want to work out after your big lunch, 
forget about the Base Gym. You're looking for the Fitness and Wellness Center. 

So what if you don't know the difference between a carbine and a carbon? Never fear — I'm 
here to give you a helping hand (bestow upon you inestimable guidance and encouragement — 
that's Word Knowledge speak). With the help of this chapter and a little brow-sweat on your 
part, your word-knowledge skills will whip right into shape. And then in Chapter 6, you can 
check out the practice questions to test your word-knowledge skills. 

Grasping the Importance of Word Knowledge 

Word Knowledge isn't part of the ASVAB just because the military likes to use big words. 
It's included because words stand for ideas, and the more words you understand, the more 
ideas you can understand (and the better you can communicate with others). A decent 
vocabulary is essential in the military if you want to get ahead. The military operates on 
paperwork, and whether you're trying to get more supplies (submit necessary logistical req- 
uisitions) or get the assignment you want (via application for personnel career-enhancement 
programs), you need to develop a good vocabulary. The military considers clear communica- 
tion so important that it's taught and graded at all levels of leadership training, including at 
the Army, Navy, and Air Force War Colleges, which are requirements to be promoted to 
General officer equivalent rank (Admiral in the Navy). 

^jftBE/? The Word Knowledge subtest is one of the four most important subtests on the ASVAB 

(along with the Paragraph Comprehension, Mathematics Knowledge, and Arithmetic 
Reasoning). This subtest comprises a significant portion of the AFQT score — the score 
that determines your eligibility for military service. You also need to do well on the Word 
Knowledge subtest to qualify for many military jobs, such as air traffic controller, military 
intelligence, and even firefighting. 




AQ Part II: Words to Live By: Communication Skills 



Table 4-1 shows the military job qualification line scores that are calculated by using your 
Word Knowledge subtest score. 



Table 4-1 


Military 


Li 


ne Scores that Use the Word Knowledge Score 


Branch of Service 






Line Score 


U.S. Army 






Clerical, General Technical, Operators and Food, Surveillance 
and Communications, and Skilled Technical 


U.S. Air Force 






Administrative and General 


U.S. Navy/Coast Guard 




Administrative, General Technical, Health, and Nuclear 


U.S. Marine Corps 






General Technical 



Chapter 2 has more information about military line scores. Check out Appendix A for more 
information on the scores you need to get the job you want. 



Checking Out the Word Knowledge 
Question Format 




t^PLE 




The Word Knowledge portion of the ASVAB measures your vocabulary. The questions 
usually come in one of two flavors: 

Iu* The first type asks for a straight definition. 
i* The second type gives you an underlined word used in the context of a sentence. 

When you're asked for a straight definition, your task is quite simple: Choose the answer 
closest in meaning to the underlined word. Look at the following example: 

Abate most nearly means 

(A) encourage. 

(B) relax. 

(C) obstruct. 

(D) terminate. 

Abate means to suppress or terminate. In this case, the correct answer is Choice (D). 

When you see an underlined word in a sentence, your goal is to choose the answer closest 
in meaning to the underlined word. Remember: Closest in meaning doesn't mean the exact 
same thing. You're looking for similar or related words. For example: 

His house was derelict . 

(A) solid. 

(B) run-down. 

(C) clean. 

(D) inexpensive. 

Here, the answer is Choice (B). 



Chapter 4: Word Knowledge jH 




When you take the Word Knowledge subtest on the paper version of the ASVAB, you have 
1 1 minutes to answer the 35 questions, which means that you have slightly less than 20 sec- 
onds to answer each question. On the computerized version, you luck out. You have 8 min- 
utes to answer only 16 questions (or 30 seconds for each question). Either way, it's plenty 
of time, as long as you stay focused and don't waste time thinking about last night's date 
(sorry, I mean social encounter). 

Keep in mind that although you may know the word in the question, you may not know one 
or more of the words in the multiple-choice answers. If this is the case, use the process of 
elimination to help you narrow down your choices. Eliminate the words that you know 
aren't correct and guess which of the remaining words is most likely correct. 

Building Words from Scratch: Strategies 
to Help l/ou Decipher Word Meanings 

Webster's New World Dictionary lists more than 170,000 primary English words, and who 
knows how many derivatives of those words? Wow! Any way you look at it, that's a lot of 
memorization. Fortunately, you don't need to study all those words. It's possible to deci- 
pher English word meanings, even if you've never heard a particular word before. 

Developing a large vocabulary takes time — often years. However, just because you have a 
limited amount of time to study doesn't mean you should give up hope. Instead, focus on 
the tips throughout this section to help you improve your Word Knowledge score. 

From beqinninq to end: Knowing prefixes 
and suffixes 

Prefixes, roots, and suffixes are the main parts that make up words. Not every word has all 
three, but most have at least one. Prefixes are the parts that come at the front of a word, 
suffixes are the parts that come at the end of a word, androofs are the parts that lie in the 
middle of a word. Think of roots as the base of the word and prefixes and suffixes as word 
parts that are attached to the base. (Check out the section "Determining the root of the 
problem" later in this chapter for more info on — you guessed it — roots.) 

These basic word parts generally have the same meaning in whatever word they're used. 
For instance, pro- means something along the lines of in favor of, forward, or positive, 
whether you use it in the word proton or the word proceed. 

Tables 4-2 and 4-3 list some common prefixes and suffixes. Each list has the word part, its 
meaning, and one word that uses each word part. Write down additional words that use 
each word part to help you memorize the list. 

If you memorize prefixes, suffixes, and roots, you have a better chance of figuring out the 
meaning of an unfamiliar word when you see it on the ASVAB. Figuring out the meaning of 
unfamiliar words is how people with large vocabularies make them even larger. (They look 
up words in the dictionary, too.) 




3 v ' >art "' Words to Live By: Communication Skills 



Table 4-2 




Prefixes 




Prefix 


Meaning 




Sample Word 


a- 


no, not 




atheist 


ab- or abs- 


away, from 




absent 


anti- 


against 




antibody 


bi- 


two 




bilateral 


circum- 


around 




circumnavigate 


com- or con- 


with, together 




conform 


contra- or counter- 


against 




contradict 


de- 


away from 




detour 


deca- 


ten 




decade 


extra- 


outside, beyond 




extracurricular, extraordinary 


fore- 


in front of 




foreman 


geo- 


earth 




geology 


hyper- 


above, over 




hyperactive 


il- 


not 




illogical 


mal- or male- 


evil, bad 




malediction 


multi- 


many 




multiply 


ob- 


blocking 




obscure 


omni- 


all 




omnibus 


out- 


external 




outside 


que-, quer-, or ques- 


ask 




question, query 


re- 


back, again 




return 


semi- 


half 




semisweet 


super- 


over, more 




superior 


tele- 


far 




telephone 


trans- 


across 




transatlantic 


un- 


not 




uninformed 




Table 4-3 




Suffixes 




Suffix 


Meaning 




Original Word: Suffixed Word 


-able or-ible 


capable of 




agree: agreeable 


-age 


act of 




break: breakage 


-al 


relating to 




function: functional 


-ance or-ence 


instance of an action 


perform: performance 


-ation 


action, process 




liberate: liberation 


-en 


made from 




silk: silken 


-ful 


full of 




help: helpful 


-ic 


relating to, like 




alcohol: alcoholic 



Chapter 4: Word Knowledge A 7 



Suffix 


Meaning 


Original Word: Suffixed Word 


-ical 


possessing a quality of 


magic: magical 


-ion 


result of, act of 


legislate: legislation 


-ish 


resembling 


child: childish 


-ism 


belief in 


Buddha: Buddhism 


-ist 


one who characterizes 


elite: elitist 


-ity 


quality of 


specific: specificity 


-less 


not having 


child: childless 


-let 


small one 


book: booklet 


-man 


relating to humans 


gentle: gentleman 


-ment 


actor process of 


establish: establishment 


-ness 


possessing a quality 


good: goodness 


-or, -er 


one who does a thing 


orate: orator 


-ous 


state of 


danger: dangerous 


-y 


quality of 


taste: tasty 




Determining the root of the problem 

Root words are word parts that serve as the base of a word. In English, one root word can be 
changed slightly to perform all sorts of roles — it can act as a noun, a verb, an adjective, or 
an adverb with just a little modification. If you recognize a root, you can generally get an 
idea of what the word means, even if you're not familiar with it. For example, if you know 
what the root word attach means, you can figure out what the word attachment means. If 
you know adherent, you can deduce what adherence means. 

Table 4-4 lists some common roots. Memorize them. When you sit down to take the ASVAB, 
you'll be glad that you did. 

When you see an unfamiliar word, try dropping a couple of letters from the beginning and/ 
or the end of the word to see whether you recognize what's left — the root. If so, you can 
make a good guess about the meaning of the word. 



Table 4-4 






Roots 




Root 


Meaning 






Sample Word 


anthro or anthrop 


relating to h 


umans 




anthropology 


bibl 


relating to b> 


ooks 




bibliography 


brev 


short 






abbreviate 


cede or ceed 


go, yield 






recede 


chrom 


color 






monochrome 


cogn 


know 






cognizant 


corp 


body 






corporate 


diet 


speak 






diction 



(continued) 



3o ^ art "' Words to Live By: Communication Skills 



Table 4-4 (continued) 


Root 


Meaning 


Sample Word 


domin 


rule 


dominate 


flu or flue 


flow 


influx 


form 


shape 


formulate 


fractorfrag 


break 


fragment 


graph 


writing 


biography 


junct 


join 


juncture 


liber 


free 


liberate 


lum or lumen 


light 


illuminate 


oper 


work 


cooperate 


path or pathy 


suffer, feeling 


pathology 


port 


carry 


portable 


press 


squeeze 


repress 


scrib or script 


write 


describe 


sens or sent 


feel 


sentient 


tract 


pull 


traction 


voc or vok 


call 


revoke 




Word families: Finding related Words 

When you see an unfamiliar word on the Word Knowledge section, don't get upset and 
pound on the computer (they make you pay for those things if you break them). You may 
know the word after all . . . just in a different form. Suppose you run across the word benefi- 
cent on the Word Knowledge portion: 

Beneficent most nearly means 

(A) kind. 

(B) beautiful. 

(C) unhappy. 

(D) troubled. 

You sit there in the school-cafeteria chair and begin to sweat. You've never seen the word 
before, and it's all over for you, right? Well, maybe not. Take a closer look. What other word 
starting with the letters benefi do you know? How about the word benefit? A benefit is some- 
thing that helps or aids. It'd be a good bet that the word beneficent is related to helping or 
aiding. So when you look over the possible choices, you can choose the one that has some- 
thing to do with helping. 

But wait! None of the answers state help or aid. Now what? Just use the process of elimina- 
tion. If something is helpful (beneficent), it probably isn't troubled or unhappy. It may be 
beautiful, but more likely, it's kind. So the best answer would be Choice (A). 



Chapter 4: Word Knowledge A Q 





Remember when your high school guidance counselor recommended that you take French 
or Spanish? You should thank her when you score well on this subtest. Why? Because 
knowledge of other languages can help you puzzle out the meaning of many English words. 
For example, if you know that salud means health in Spanish, you may be able to puzzle out 
the meaning of the English word salutary (favorable to or promoting health). Knowing that 
sang means blood in French may help you figure out what the English word sanguine means 
(try to puzzle this one out on your own; then check a dictionary to see how close you are). 

\ling and \lang: Understanding 
Synonyms and Antonyms 

A synonym is a word that has the same meaning as or a very similar meaning to another 
word. Smile and grin are synonyms. They may not mean exactly the same thing, but their 
meanings are very similar. An antonym is a word that has an opposite or nearly opposite 
meaning as another word. Smile and frown are antonyms. 

To help remember the definitions of synonym and antonym, think of a synonym as the same 
(both also start with an s) and an antonym as the enemy. 

The ASVAB may ask you to find the word that most nearly means the same thing as a given 
word, which is a synonym. Or you may be asked to find the word that most nearly means 
the opposite of a given word, which is an antonym. Most of the questions on the Word 
Knowledge subtest ask you to find synonyms, although a few may ask you to find antonyms. 

How can you study and find the synonym of a word (or the antonym, for that matter)? Take 
a look at these suggestions: 

v* Start in the dictionary. Many dictionary entries include the abbreviation syn., which 
means synonym. The words that follow this abbreviation are synonyms of the entry 
word. You may also see the abbreviation ant., which stands for antonym; the word or 
words that follow it mean the opposite of the entry word. 

v* Make a list of synonyms and antonyms of the words you learn. As you study vocabu- 
lary words for the Word Knowledge subtest, add them to your list. 

v* Use the root-word list from Table 4-4 (in the preceding section). Using a dictionary 
and/or thesaurus, come up with a list of synonyms and antonyms for each word listed 
in the Sample Word column. (Of course, not every word has synonyms and antonyms, 
but many do.) 

Many of the ASVAB Word Knowledge questions require you to know a one-word definition 
for another word. There's no better study aid for this concept than a thesaurus, a book of 
synonyms. 

\/ou Are What \lou Speak: Improving \lour 
Vocabulary, Improving l/oursetf 

Having an extensive vocabulary can help you do well on the Word Knowledge subtest. But 
even if you don't have a huge vocabulary, the strategies in this section can help you make 
up for that. 




h Q Part II: Words to Live By: Communication Skills 



*jABBf 




You can acquire vocabulary words in the short term as well as over a long period of time. 
Combining both approaches is best, but if you're pressed for time, focus on short-term 
memorization and test-taking skills. 




Reading your Way to a larger Vocabulary 

In a world of DVDs, video games, and 17 billion channels on TV, the pastime of reading for 
enjoyment is quickly fading. To build your vocabulary, you have to read — it's that simple. 
Studies consistently show that those who read for enjoyment have a much larger vocabu- 
lary than those who dislike reading. You have to see the words in print, not just hear some- 
one say them. Besides, people can read and understand many more words than they could 
ever use in conversation. 

That doesn't mean you have to start with Advanced Astrophysics. In fact, if you don't read 
much, you can start with your daily newspaper, a news magazine, or any type of reading 
material that's just a notch or two above what you ordinarily read. Choose topics that inter- 
est you. If you're interested in the subject matter, you'll enjoy reading more. Plus, you may 
just learn something new! 

When you encounter a word you don't know, try to understand what it means by looking at 
the context in which the word is used. For example, if you read, "The scientist extrapolated 
from the data," and you don't know what extrapolated means, you can try substituting 
words you do know to see whether they'd make sense. For example, the scientist probably 
didn't hide from the data. She probably used the data to make some sort of decision, judg- 
ment, or guess. To confirm your understanding of the word, check your dictionary. Making 
predictions like this can help you remember a definition for the long term. 

You may even consider keeping a running list of terms you come across as you read, along 
with their definitions (see the following section). On the Word Knowledge subtest of the 
ASVAB, you often won't be able to guess what a word means from its context (in many 
cases, there's no context in the test because the words aren't used in sentences). You also 
won't be able to look the word up in the dictionary. But considering context and consulting 
a dictionary are two great ways to discover vocabulary words during your test preparation. 




Keeping a list and checking it twice 

Not long ago, an 11-year-old girl went through the entire dictionary and made a list of all the 
words she didn't know. (The process took several months.) She then studied the list faith- 
fully for a year and went on to win first place in the National Spelling Bee finals. You don't 
have to go to this extent, but even putting in a tenth of her effort can dramatically improve 
your scores on the Word Knowledge subtest. 

One way to improve your vocabulary is to keep a word list. Here's how that list works: 

1. When you hear or read a word that you don't understand, jot it down. 

2. When you have a chance, look up the word in the dictionary and then write the 
meaning on your list. 

3. Use the word in a sentence that you make up. 

Write the sentence down, too. 

4. Use your new word in everyday conversation. 

Finding a way to work the word zenith into a description of last night's basketball game 
requires creativity, but you won't forget what the word means. 



Chapter 4: Word Knowledge A/ 





Arrange your list by related items so that the words are easier to remember. For example, 
list the words having to do with your work on one page, words related to mechanical knowl- 
edge on another page, and so on. 

You can also find Web sites that offer lists of words if you spend a few minutes surfing. Try 
using search phrases such as "vocabulary words" and "SAT words." Here are a few resources: 

v* Free Vocabulary: This site (www. freevocabulary . com) offers a free list of more 
than 5,000 collegiate words, along with brief definitions. 

J*" Dictionary.com: Dictionary.com (www. dictionary, com) includes a great online 
dictionary, thesaurus, and word of the day. 

v* Merriam-Webster online: Merriam-Webster online (www.m-w. com) is another useful 
site with a free online dictionary, thesaurus, and word of the day. 

A ton of books exist to help build your vocabulary. Try Vocabulary For Dummies by Laurie E. 
Rozakis or SAT Vocabulary For Dummies by Suzee Vlk, both published by Wiley. These 
books are great resources designed to help you improve your word knowledge skills. 



Crosswords: Making Vocabulary fun 

Before she passed away, my mom was a walking dictionary. It seemed like she could give 
you a single-word definition for almost any English word, which was quite a feat for a 
woman who only made it to the ninth grade. What was her secret? Early in life, she fell in 
love with crossword puzzles. 

One of the great things about crossword puzzles (other than fun) is that you can find them at 
all levels of difficulty. Start with one that has a difficulty consistent with your word-knowledge 
ability and then work your way up to more difficult puzzles. Before you know it, you'll be a 
lean, mean word machine and have loads of fun in the process. 



Sounding off by sounding it out 

Sometimes you actually know a word because you've heard it in conversation, but you don't 
recognize it when you see it written down. For instance, a student who'd heard the word 
placebo (pronounced "plah-see-bow") knew that it meant an inactive substance, like a sugar 
pill. But when she came across it in writing, she didn't recognize it. She thought it was a 
word pronounced "plah-c/?ee-bow," which she'd never heard before. 

When you see a word on the ASVAB that you don't recognize, try pronouncing it (not out 
loud, please) a couple of different ways. The following pronunciation rules can help you out: 

i*" Sometimes letters are silent, like the b in subtle or the k in knight. A letter at the end 
of a word may be silent, especially if the word is French; for instance, coup is pro- 
nounced coo. 

U* Some sounds have unusual pronunciations in certain contexts. Think of the / in colonel, 
which is pronounced like kernel. 

U* The letter c can sound like s (lice) or k (despicable). 

U* The letter i after a t can form a sound like she. Think of the word initiate. 

v* The letter x at the beginning of a word is generally pronounced like z (Xerox). 



h 2 Part II: Words to Live By: Communication Skills 



v* A vowel at the end of a word can change the pronunciation of letters in the word. The 
word wag has a different g sound than the word wage. 

v* When several vowels are right next to each other, they can be pronounced many 
different ways (consider boo, boa, and bout). Try a couple of different possibilities. 
For instance, if you see the word feint, you may think that it should be pronounced 
feent or fiynt, but it in fact sounds like faint. It means fake or pretend. 



Chapter 5 

Paragraph Comprehension 



In This Chapter 

Knowing what to expect of the Paragraph Comprehension subtest 
Pumping up your comprehension 
Maxing out your reading speed 
Improving your odds at test time 



JM ny other organization would call this section of the ASVAB the Reading Compre- 
T • hension subtest, but the Department of Defense is a stickler for precision. You'll be 
reading paragraphs, darn it, so you're being tested on how well you understand paragraphs! 
Not words, not sentences, not essays, but paragraphs! Don't you just love the military way? 

One thing you get from military boot camp is that comprehending the drill sergeant's 
orders and the information in your instruction manuals is important. The ability to read and 
understand the written directions in your basic training manual can save you and your bud- 
dies hundreds of push-ups. Trust me on this one. The Paragraph Comprehension subtest 
measures your ability to understand what you read and draw conclusions from that material. 
It contains a number of reading passages and questions about those passages. 

After you enlist, you discover that the military runs on paperwork. If you can't read and 
understand a regulation that's buried within a pile of papers, how are you going to obey it? 

The Importance of Paragraph Comprehension 
for Military Jobs 

The Paragraph Comprehension subtest is an important part of your AFQT score, which is 
the most important score because it determines whether a particular branch of service lets 
you join. The score is so important that I plan to keep on repeating it until you're mumbling, 
"The AFQT is the most important score," in your sleep. 

You'd be surprised at how many diverse military jobs require a decent score on the 
Paragraph Comprehension subtest. But think about it for a moment: If the directions in a 
military recipe make you rub your eyes and mumble to yourself, how are you going to cook 
a meal for 2,000 troops? (Assuming you want to become a military cook, that is.) Table 5-1 
shows the military job qualification line scores that are calculated by using your Paragraph 
Comprehension subtest score. 



h h Part II: Words to Live By: Communication Skills 



Table 5-1 


Military Line Scores that Use 
the Paragraph Comprehension Score 


Branch of Service 


Line Score 


U.S. Army 


Clerical, General Technical, Operators and Food, Surveillance 
and Communications, and Skilled Technical 


U.S. Air Force 


Administrative and General 


U.S. Navy and Coast Guard 


Administrative, General Technical, Health, and Nuclear 


U.S. Marine Corps 


General Technical and Clerical 



Chapter 2 has more information about military line scores. See Appendix A for more infor- 
mation on the scores you need to get the job you want. 

Eyeing the Physique of the Paragraph 
Comprehension Subtest 

When you get to the Paragraph Comprehension subtest, you have several passages to read. 
Most passages are only one paragraph long, and rarely are they longer than two para- 
graphs. Each passage contains between 50 and 200 words. (Look at it this way: At least you 
won't be required to read War and Peace?) 

The ASVAB test-makers may ask you to answer only one question about a given reading 
passage, or they may ask you to answer as many as five questions about one passage. 
Unfortunately, this subtest doesn't consist of the most interesting passages you'll ever read. 
(You won't find paragraphs from your favorite spy or romance novel here.) So it's important 
that you set your attention span dial all the way to the maximum setting. If you're taking the 
paper-and-pencil version of the ASVAB, you have 13 minutes to read the passages and answer 
15 questions. On the computer version, you have 22 minutes to answer 11 questions. 

In order to understand what you read — which is what the Paragraph Comprehension sub- 
test is all about — you need to develop several abilities, which I cover later in this chapter: 

v* Finding the main idea or argument that the author is making 
k" Remembering specific details about the reading 
v* Drawing conclusions from what you've read 
J-" Understanding relationships between ideas 
v* Paraphrasing or summarizing what you've read 



Trying the Four Flavors of Comprehension 
Questions 

The Paragraph Comprehension questions on the ASVAB usually take one of four forms: 



I 



U* Finding specific information 
u* Recognizing the main idea 



Chapter 5: Paragraph Comprehension An 



Iv* Determining word meaning in context 
i* Drawing an implication from a stated idea 

Each type of question asks you to perform a different kind of analysis of the reading pas- 
sage. If a passage has more than one question associated with it, chances are each question 
falls under a different category. The following sections spell out the differences among 
these four types of questions. 




\»NG/ 




Treasure hunt: Finding specific information 

This type of Paragraph Comprehension question asks you to pick out (you guessed it) spe- 
cific information from a passage. Sounds easy, right? Take a look at the following passage, 
which clearly states the answer to the question that directly follows it: 

An industry trade association found that more than 13,000 martial-arts schools exist in 
the United States with nearly 6 million active members. Of the 13,000 schools, nearly 
7,000 offered tae kwon do lessons. 

According to this passage, how many people actively participated in martial arts lessons? 

(A) 13,000 

(B) 7,000 

(C) 6 million 

(D) It can't be determined. 

The correct answer is Choice (C). 

At times, the information that a question asks about isn't directly stated in the question, but 
you can infer the information from the text. Remember, in the military, the only easy day 
was yesterday. 

When questions are phrased in the negative, you may be easily confused about what the 
question is asking. (This fact is especially true when the information being sought isn't 
directly stated in the passage.) Misreading a negative question is also easy. Research has 
shown that people often skip over a negative word, such as not, when they read. Be aware 
that questions on the Paragraph Comprehension portion of the ASVAB are frequently stated 
in the negative. When you see a negative word, an alarm should go off in your head to remind 
you to read the question more carefully. 




Cutting to the chase: Recognizing the main idea 

Sometimes the Paragraph Comprehension questions ask you to identify the main point of a 
passage. The main point can be directly stated, or it can be implied. 

If you're not sure what the main point of a paragraph is, reread the first sentence and the 
last sentence. Chances are one of these two sentences contains the main point. (Flip to 
"What's the big idea? Determining the main idea in a paragraph," later in this chapter, for 
more information on identifying main ideas.) 



h Part II: Words to Live By: Communication Skills 



b^PLE 





If the shoe fits: Determining Word meaning 
in context 

Sometimes the Paragraph Comprehension subtest asks you to determine the meaning of a 
word when it's used in a passage. The correct definition that the question is looking for can 
be the most common meaning of the word, or it can be a less well-known meaning of the 
word. In either case, you have to read the passage, make sure you understand how the 
word is being used, and select the answer option that's closest in meaning to the word as 
it's used in the passage. Consider this example: 

In the 18th century, it was common for sailors to be pressed into service in Britain. 
Young men found near seaports could be kidnapped, drugged, or otherwise hauled 
aboard a ship and made to work doing menial chores. They weren't paid for their ser- 
vice, and they were given just enough food to keep them alive. 

In this passage, pressed means 

(A) hired. 

(B) ironed. 

(C) enticed. 

(D) forced. 

The descriptions of the conditions these sailors found themselves in should help you decide 
that they weren't hired or enticed; ironed is one meaning of the word pressed, but it isn't 
correct in this context. The correct answer is Choice (D). Here's another example: 

Since the 1980s, computers have become an indispensable part of American business. 
Computers can be used for thousands of applications from word processing and run- 
ning spreadsheets to keeping one's checkbook updated. 

In this passage, applications means 

(A) functions. 

(B) sizes. 

(C) requests. 

(D) types. 

Try putting the answer choices in this sentence: "Computers can be used for thousands of 
applications." You can see that functions is closest in meaning to applications, although in a 
different context, some of the other answer choices may be correct. The correct answer is 
Choice (A). 



Reading between the tines: 
Understanding implications 



Some Paragraph Comprehension questions ask you to draw an inference from a stated idea. 
This simply means that you may need to draw a conclusion from what you've read. This 
conclusion should always be based on the reading, not your own particular opinions about 
a subject. 



Chapter 5: Paragraph Comprehension h 7 



*jABBf 







The conclusion — which may be called an inference or implication — must be reasonably 
based on what the passage says. You have to use good judgment when deciding which con- 
clusions can be logically drawn from what you've read. Give it a shot: 

Twenty-five percent of all automobile thefts occur when the doors of a car are left 
unlocked. People often forget to lock their doors, find it inconvenient, or tell them- 
selves, "I'll only be a minute." But it only takes a minute for an accomplished car thief 
to steal a car. And thieves are always alert to the opportunities that distracted or 
rushed people present them with. 

To prevent auto theft, it's a person's responsibility to 

(A) leave the doors unlocked. 

(B) never be in a rush. 

(C) prevent the opportunity. 

(D) be willing to perform a citizen's arrest. 

Although the paragraph doesn't state, "To prevent auto theft, it's a person's responsibility 
to prevent the opportunity," this idea is certainly implied. The correct answer is Choice (C). 
There's no implication that people should be willing to (or can) perform a citizen's arrest. 
Leaving the doors unlocked is the opposite of what one should do, and never being in a 
rush is probably impossible. 

An example of an unreasonable conclusion drawn from the passage would be something like 
"if everyone locked their doors, there would be no crime" or "all car thieves should be sen- 
tenced to 30 years in prison." Nothing in this particular passage supports such a conclusion. 

One way to help determine whether you've drawn a reasonable conclusion is to ask your- 
self, "Based on what I've just read, would the author agree with the conclusion I've 
reached?" If the answer is yes, your conclusion is probably reasonable. If the answer is no, 
it's time to think up a new conclusion. 

Check out another example: 

Boiler technicians operate main and auxiliary boilers. They maintain and repair all 
parts, including pressure fittings, valves, pumps, and forced-air blowers. Technicians 
may have to lift or move heavy equipment. They may have to stoop and kneel and 
work in awkward positions. 

According to this job description, a good candidate for this job would be 

(A) a person with joint problems. 

(B) an individual unaccustomed to heavy lifting. 

(C) a person who isn't mechanically minded. 

(D) a person who's physically fit. 

Although the passage doesn't state, "This job requires a physically fit person," the duties 
listed imply that this is so. The correct answer is (D). A person with joint problems may not 
be able to stoop or kneel or work in awkward positions. A person who's unaccustomed to 
heavy lifting may not be able to lift or move the heavy equipment as needed. A person who 
isn't mechanically minded may not have the knowledge necessary to maintain and repair 
boilers and all their parts. This leaves Choice (D) as the answer, and it's true that a person 
who's physically fit would be a good choice for the job. 



h q Part II: Words to Live By: Communication Skills 



Do \lou Get My Paint) 



All good writing has a point. Some writing has more than one point. Points are ideas that 
the writer is trying to convey to the reader. The primary purpose of the writing is known as 
the main point or main idea. Points used to support or clarify the main point are called sub- 
points or supporting points. You should know how to identify main points and subpoints 
when you practice reading. 





What's the biq idea} Determining 
the main idea in a paragraph 

Questions on the Paragraph Comprehension subtest frequently ask you to identify the main 
point of a reading passage. How do you get better at identifying main ideas? Practice. The 
main idea, which is the most important point the author is making, is sometimes stated and 
sometimes implied in a piece of writing. 

Finding a topic sentence 

Often, the author begins or ends a paragraph or passage with the main idea, which is 
located in what's called a topic sentence. A topic sentence, reasonably enough, describes 
the topic that the author's writing about. 

If you're looking for the main idea, start off by checking the first and last sentence of the 
passage. (No, this doesn't mean that you should skip the rest of the passage.) For example, 
suppose you read the following paragraph: 

The local school district is facing a serious budgetary crisis. The state, suffering a reve- 
nue shortfall of more than $600 million, has cut funding to the district by $18.7 million. 
Already, 65 teachers have been laid off, and more layoffs are expected. 

No, the primary theme of this passage isn't "schools in our area suck." The main point of 
this paragraph can be found in the opening sentence, "The local school district is facing a 
serious budgetary crisis." What follows are details regarding the budget crisis. 

Sometimes, a passage builds up to its main idea, and sometimes the main idea is implied 
instead of stated. Consider the following paragraph: 

The farmers' market reopened on the second weekend of May. Amid the asparagus 
and flowers, shoppers chatted about the return of temperatures in the 70s. Across the 
street, children (and their dogs) played Frisbee in the park. Finally, spring has come to 
town. 

In this paragraph, you may think that the farmers' market's reopening is the main point, but 
the other information about the temperature and the kids' playing Frisbee tells you that the 
main idea is something a bit broader than the market's opening. The main idea is stated in 
the last sentence: "Finally, spring has come to town." 

In boot camp, your drill instructor may say, "Some of you better check to see that your 
bunks are properly made." Or he may rip your bunk bed apart and say, "Now make this $%*& 
bunk the right way, you moron!" Both comments mean the same thing. In the first statement, 
the drill instructor implies the meaning; the second statement is a bit more direct. 



Chapter 5: Paragraph Comprehension h Q 





in other Words; Rephrasing passages 

One of the best ways to identify the main point of a paragraph is to put the paragraph into 
your own words (paraphrase it) or to sum up the basic idea of the paragraph (summarize it). 
By quickly doing this when you take the Paragraph Comprehension portion of the ASVAB, 
you can be confident that you're answering the question correctly. In other words (to para- 
phrase), you'll know you understand what the paragraph is talking about. 

You likely won't have time to write down the main point or to jot down your paraphrase or 
summary. Instead, as you're reading, simply try to mentally keep track of what's being said 
by putting it into your own words. 

Look at the following paragraph: 

The local school district is facing a serious budgetary crisis. The state, suffering a reve- 
nue shortfall of more than $600 million, has cut funding to the district by $18.7 million. 
Already, 65 teachers have been laid off, and more layoffs are expected. 

Now close this book and spend a few moments paraphrasing the previous paragraph. Come 
on. Pick up that pencil and get those brain cells firing. When you're done, reopen to this 
page and compare your ideas to the passage. If you wrote something like the following, 
you're right on track: 

The school district has a budget crunch because the state has a budget crunch. The 
state cut funding to the school district. Some teachers have been laid off already. More 
may be laid off soon. 

Now if you wrote something like, "It's finally May, and shoppers and kids-at-play are out and 
about, enjoying the warmer temperatures of spring," then you're not paying attention. Turn 
off the TV and give it another try. 

As you study for the ASVAB, practice paraphrasing reading passages. You can paraphrase 
or summarize any short passage you read — a few sentences or a paragraph or two. Read 
different passages from a book or magazine and then close the pages. Get out a pencil and 
jot down your paraphrases. (Remember, you won't have time to do this on the test itself, 
but the practice helps you mentally prepare for when you take the test.) 

Extra, extra! Identifying subpoints 

If a writer stuck to just one point, the Paragraph Comprehension subtest would be a breeze. 
However, an author usually doesn't just make one point in a piece of writing, so you also 
need to understand the other points the author makes. These details, or subpoints, may 
include facts or statistics, or they may be descriptions that support the main point of the 
passage. Subpoints help you see what the author's saying. For instance, look at this passage 
(from the previous section): 

The local school district is facing a serious budgetary crisis. The state, suffering a reve- 
nue shortfall of more than $600 million, has cut funding to the district by $18.7 million. 
Already, 65 teachers have been laid off, and more layoffs are expected. 

The subpoints help you understand the main point, which is that the school district is facing 
a severe budgetary crisis. The subpoints help you understand why: "The state, suffering a 
revenue shortfall of more than $600 million, has cut funding to the district by $18.7 million." 



j%Q Part II: Words to Live By: Communication Skills 



You can see that the budgetary crisis is part of a larger problem, which is the state's suffer- 
ing a severe revenue shortfall. The subpoints also help you understand what this crisis 
means: "Already, 65 teachers have been laid off, and more layoffs are expected." By using 
these facts and figures, the author helps you grasp not only the main point but also the impli- 
cations of that main point. 



Analyzing What l/au'i/e Read: Guessing at 
What the Writer Ready Means 

The Paragraph Comprehension subtest of the ASVAB also requires you to analyze what 
you've read. Analysis is more than simply picking out the point of text. Analyzing a passage 
requires you to draw conclusions from what you've read and understand relationships 
among the ideas presented in the text. 

By drawing conclusions about the meaning of a passage, you reach new ideas that the 
author implies but doesn't come right out and state. You must analyze the information the 
author presents in order to make inferences from what you've read. For instance, look at 
the following paragraph: 

The local school district is facing a serious budgetary crisis. The state, suffering a reve- 
nue shortfall of more than $600 million, has cut funding to the district by $18.7 million. 
Already, 65 teachers have been laid off, and more layoffs are expected. 

Although the author doesn't say so, you can draw the conclusion that if the state revenue 
shortfall could somehow be corrected — by increasing state sales tax or income tax, for 
example — the local school district's budgetary crisis could be resolved. The $18.7 million 
cut from the school budget could be restored. The author never actually makes this point in 
the paragraph, but by using logic, you can draw this conclusion from the facts presented. 



wer 





Making inferences and drawing conclusions requires you to use your judgment. You don't 
want to read too much into a passage. For example, nothing in the example paragraph sug- 
gests that electing a new governor is necessary or that increasing federal income taxes 
would help the problem. 

Look at the next paragraph: 

The farmers' market reopened on the second weekend of May. Amid the asparagus and 
flowers, shoppers chatted about the return of temperatures in the 70s. Across the street, 
children (and their dogs) played Frisbee in the park. Finally, spring has come to town. 

Suppose you're asked the following question about this paragraph: 

It can be inferred from the passage that 

(A) Frisbee playing in the park doesn't happen in winter. 

(B) the warm weather is unusual for this time of year. 

(C) the shoppers were disappointed in the farmers' market produce. 

(D) rain is imminent. 

If the point of the passage is that spring has come to town and the author uses Frisbee play- 
ing as evidence of the arrival of spring, then it's likely that Frisbee playing doesn't occur in 
the winter but does begin again in spring. The answer is Choice (A). 



Chapter 5: Paragraph Comprehension jy / 



Faster than a Speeding Turtle: 
Tips far SloiV Readers 



WER 




Today's military is much more complex than attending boot camp, learning how to shoot a 
gun, and shipping off to war. After boot camp, you attend intensive classroom training to 
learn your military job. If you can't read well, you're going to have a very hard time. But the 
good news is that it's never too late to work on improving your reading skills. 

For many people, 13 or 22 minutes is enough time to read all the passages, understand the 
questions, and choose the correct answers. But slow readers may have more difficulty get- 
ting all the questions answered before time is up. Don't despair: Take the suggestions in this 
section to help you build your reading speed. Of course, they require work, but you knew 
the mission came with its challenges, right? 




*£5fc> 




Read more, Watch less 



If you're a slow reader, chances are you don't do a lot of reading. If you have plenty of 
time before you're due to take the ASVAB, start reading more — right now. It's in your best 
interest. 

You don't have to pick up A Tale of Two Cities or War and Peace; you can start with the 
newspaper, a biography of a person you admire, or magazines you find at the library. 
(Sorry, but the instruction guide to your favorite video game doesn't count.) You don't need 
to enroll in a speed-reading course. If you devote at least one hour a day simply to reading, 
you'll see your reading comprehension and speed increase within a month or so. 

Several studies have shown that folks who enjoy reading as a pleasurable pastime score 
better on reading comprehension tests than individuals who dislike reading. Sounds obvi- 
ous, right? So why study it? The idea is if you grow to enjoy reading, you'll want to read 
more. You'll become a better reader and thereby score better on reading comprehension 
tests. How do you discover an enjoyment of reading? Simple — choose reading material in 
subject areas that interest you. 




Become a lean, mean Word machine 

People sometimes read slowly because they don't have a large vocabulary and don't under- 
stand everything they read. If you can identify with this situation, improving your vocabu- 
lary is your first step toward increasing your reading comprehension and your reading 
speed. (Chapter 4 gives you info on building your word knowledge. Check it out.) 

Keep a pocket dictionary handy while reading so you can look up words you don't know. If 
you're reading articles on the Internet, keep a window open to one of the online dictionaries 
(such as www. dictionary, com, www.yourdictionary . com, or www.m-w. com) so you 
can quickly find the definition of words you find confusing. Your reading will become more 
enjoyable, and you'll be adding to your vocabulary knowledge to boot. 



Build your confidence 



Another reason people read slowly is that they don't have confidence in themselves. 
They're not convinced that they understand what they're reading, so they read a passage 
several times, trying to make sure they haven't missed anything. But just like people who 



j%£ Part II: Words to Live By: Communication Skills 




*JABE* 




check that the front door is locked 15 times before leaving for vacation and still lie awake at 
night wondering whether they locked the door, reading and rereading a passage doesn't 
give you confidence that you understand the text. You get confidence from proving that you 
understand it. 

How do you prove to yourself that you understand what you're reading? Here are a couple 
of tips: 

u* Get out a textbook or even an encyclopedia (preferably a volume that contains some 
subject matter that interests you) and read one or two paragraphs straight through 
without going back and rereading anything. Then close the book (keeping your place 
marked) and write, in your own words, a brief description of what you've read. Finally, 
turn back to the passage and compare your description to the information on the page. 

u* Play the 20-questions game. Read an article from a magazine, encyclopedia, or text- 
book. Then ask someone to pick out facts from the article and ask you questions. 

v* Create motivation and interest by reading the daily newspaper or news magazines. 
Discuss the news events with your classmates, friends, or co-workers. Stronger interest 
equals greater comprehension. 

Is your written version of the article close in meaning to the original? Are you getting most 
of the 20 questions correct? Do you feel comfortable discussing current events with others? 
If so, you understand what you're reading, and that should build your confidence. If not, 
don't toss the book or magazine aside in frustration or go ballistic on your mom for asking 
you tough questions. Keep working on it, and your comprehension will improve. Do the pre- 
ceding confidence-building drills a few times a day until you feel like you can read any para- 
graph or two and understand the content without having to reread the information. 

The Paragraph Comprehension subtest tests your ability to understand what you read, not 
how quickly you can read it. When you sit down to take this subtest, try to go as quickly as 
you can without sacrificing accuracy. Being methodical in your reading isn't a bad thing as 
long as you're getting the answers right. Just try to read a little faster than normal without 
panicking or missing the point. It's better to read the paragraphs carefully and answer the 
questions correctly on half of the questions and guess on the other half of the questions 
than it is to speed through all the reading and get none of the answers right. 



Test-Taking Tips far Reading and Gleaning 

Although no shortcuts exist for improving your reading comprehension skills (besides prac- 
tice), you can do a few things on test day to make sure that you score as high as possible on 
this part of the ASVAB. 



<$m 




If you're running out of time on this subtest or you're not sure whether you can identify the 
main idea of a passage, take a guess. If you think that's a good piece of advice, check out 
these tried-and-true tactics for test day: 

i*" Read first, ask questions later. Read the passage all the way through before glancing 
at the question and answer options. 

«*" Take it one question at a time. Some passages have more than one question associ- 
ated with them, but look at only one question at a time. 



Chapter 5: Paragraph Comprehension 3^3 



v* Understand each question. What's the question asking you to do? Are you supposed 
to find the main point? Draw a conclusion? Find a word that's nearest in meaning? 
Make sure you know what the question is asking before you choose among the answer 
options. This tip may seem obvious, but when you're in a hurry, you can make mis- 
takes by misunderstanding the questions. 

i*" Read each answer option carefully. Don't just select the first answer that seems right. 
Remember, on the Paragraph Comprehension subtest, one answer is often most right 
and others are almost right. You want to choose the most right answer, not the almost 
right answer. And to do that, you have to read all the answers. 

k" Check your feelings at the door. Answer each question based on the passage, not 
your own opinions or views on the topic. 

v* Don't choose vague answer options. They're incorrect 99.99 times out of 100. (Oh heck, 
call it 100 times out of 100.) If an answer strikes you as not quite true but not totally 
false, that answer is incorrect. Those nasty ASVAB test-makers have put those answers 
in there to throw you off. Don't give them the satisfaction of falling for their trap! 

j-" (Almost) never select never. For the most part, answer options that are absolutes are 
incorrect. Never, always, and related words are often a sign that you should select a dif- 
ferent answer. Words like generally and usually are more likely to be correct. 



Qn Part II: Words to Live By: Communication Skills 



Chapter 6 

All's Well That Tests Well: 
Communication Practice Questions 




IN 



In This Chapter 

Proving your knowledge of word meanings 
Demonstrating that you can retain what you read 



T 

m o help you hone your communication skills a little further, this chapter contains some 
m Word Knowledge and Paragraph Comprehension practice questions to get you rolling. 

Don't sweat. This test is just for practice, and it doesn't count for a score. (Plus, sweating 

makes your answer sheet soggy so it's harder to mark.) 

The communication subtests of the ASVAB are very important because they comprise a sig- 
nificant portion of your AFQT score. (I promised I'd keep pounding this concept into your 
head — and I wouldn't lie to you, ever!) 

On the paper version of the ASVAB (and on the full-length practice tests later in this book), 
you see 35 Word Knowledge questions and 15 questions about Paragraph Comprehension. 
The CAT-ASVAB has 16 Word Knowledge questions and 11 Paragraph Comprehension ques- 
tions. In this chapter, you get 25 total questions just to help you warm up for the practice 
tests later on in this book. 



Word Knowledge Practice Questions 



.WEn 




In the stem of each of the following Word Knowledge practice questions, you see an under- 
lined word. Select the choice that best answers the question in relation to the underlined 
word. 

Pay attention to the wording of each question. Some questions ask you to select the choice 
closest in meaning to the underlined word. Some questions may ask you to select the word 
most opposite in meaning. On other questions, you see the underlined word used in a sen- 
tence. In that case, your task is to select the choice most similar in meaning to the under- 
lined word as it is used in the context of the sentence. 

1. Acclaim most nearly means 

(A) enthusiastic approval. 

(B) religion. 

(C) help. 

(D) program. 

Used as a noun, acclaim means a shout of approval, so the answer is Choice (A). 



/r 



j%Q Part II: Words to Live By: Communication Skills 



2. The college student met with the bursar to discuss tuition payment options. 

(A) planner 

(B) treasurer 

(C) politician 

(D) ghost 

Bursar is similar to the word reimburse. The question gives context clues about tuition pay- 
ment, and that should give you enough clues to select the correct answer, Choice (B). 

3. Estrange most nearly means 

(A) sharp. 

(B) small. 

(C) alienate. 

(D) shiny. 

Estrange means to alienate, Choice (C). Note that estrange is a verb, and the only answer 
choice that's also a verb is Choice (C); the others are adjectives. 

4. Momentous most nearly means 

(A) significant. 

(B) small. 

(C) reality. 

(D) postpone. 

Momentous is an adjective and means very significant, Choice (A). 

5. The mother chastised her child. 

(A) comforted 

(B) carried 

(C) lectured 

(D) supervised 

Chastised means disciplined or punished, so Choice (C) is the most correct choice. Choices 
(A), (B), and (D) are unrelated. 

6. Obtrude most nearly means 

(A) condition. 

(B) absorb. 

(C) prepare. 

(D) impose. 

The correct answer is Choice (D). Obtrude means to intrude or to impose oneself on 
another. The other choices are unrelated. 




Chapter 6: All's Well That Tests Well: Communication Practice Questions jy 7 

7. We often wondered why Daniel lived in such an opulent apartment. 

(A) run-down 

(B) lavish 

(C) far away 

(D) hideous 

Opulent is an adjective that means wealthy, rich, or affluent. Choice (B) is the answer clos- 
est in meaning. The other choices are unrelated or opposite of the meaning. 

8. Now that you've read through it once, it's time to recapitulate the Word Knowledge chapter. 

(A) discuss 

(B) summarize 

(C) test 

(D) reread 

Recapitulate is a verb that means to summarize. It's also the longer version of the word 
recap. The correct answer is Choice (B). Choice (A) is somewhat close, but Choice (B) is the 
closest in meaning. 

9. Clemency most nearly means 

(A) mercy. 

(B) force. 

(C) imprisonment. 

(D) compliment. 

Clemency means forgiveness or leniency in punishing a person. Choice (A) is the correct 
answer. The other choices are unrelated. 

Knowing prefixes can be useful when determining the definitions of many words. For exam- 
ple, you may have heard the word inclement used to describe stormy, severe weather. If you 
know that the prefix in- can mean not, you can conclude that clement is likely to be mild or 
gentle, traits related to mercy. (See Chapter 4 for more info on using prefixes and word fami- 
lies to guess what words mean.) 

10. This year the Paris fashion industry has decided to eschew short skirts and high heels. 

(A) favor 

(B) manufacture 

(C) shun 

(D) sell 

Eschew is a verb that means to avoid or keep away from. Choice (C) is the correct answer, 
and the other answers are unrelated. 



/r 



Gq Part II: Words to Live By: Communication Skills 



11. Pollute most nearly means 

(A) eliminate. 

(B) contaminate. 

(C) clean. 

(D) confuse. 

Pollute means to contaminate, Choice (B). 

12. Latent most nearly means 

(A) hidden 

(B) dull 

(C) pretentious 

(D) active 

Latent means present but not visible or noticeable, so Choice (A) is the correct answer. 
Latent can also mean dormant, but none of the answer choices relate to that definition. 

13. Paul sent all of his friends a salutary message on the Internet. 

(A) beneficial 

(B) profane 

(C) funny 

(D) interesting 

Salutary is an adjective meaning beneficial, so Choice (A) is correct. If you took Spanish in 
high school, you may remember that a related word, salud, relates to health and well-being, 
making (A) a good guess. 

Paragraph Comprehension Practice Questions 

The last half of the questions in this chapter is designed to present you with an opportunity 
to practice your Paragraph Comprehension skills. Read each short paragraph, followed by 
one or more questions regarding information contained in that passage. Make sure to read 
the paragraph carefully before selecting the choice that most correctly answers the question. 

Passage one 

Mrs. Berry's was my first coup de main. The house was at the top of a high hill with more 
steps to climb to reach the porch which spanned a plain but scrupulously neat living room. 
The floor was freshly scrubbed with white sand, there was a deal table also scrubbed to 
snowy whiteness and a few splint bottomed chairs scrubbed likewise. All this I noticed 
standing on the threshold of the front door which stood wide open from habit, one could 
see, rather than with any notion of inviting wayfarers to enter. I knocked on the floor with 
the point of my umbrella and after some minutes a comely little black woman appeared in 
the doorway just opposite and stood with hands crossed in front of her waiting to learn the 
cause of the intrusion. 



Chapter 6: All's Well That Tests Well: Communication Practice Questions %Q 

14. When the narrator arrives at the house, she 

(A) knocks on the door. 

(B) knocks on the floor. 

(C) opens the door. 

(D) rings the doorbell. 

The correct answer is Choice (B) — she knocks on the floor using her umbrella. If you 
missed this one, read the passage more carefully. 

15. The woman's crossed hands imply that she's 

(A) bothered. 

(B) excited. 

(C) afraid. 

(D) bored. 

The correct answer is Choice (A). The passage states that the woman is waiting to see "the 
cause of the intrusion," which indicates she is bothered or annoyed. 

Passage Mo 

Some people argue that baking is an art, but Chef Debra Dearhorn says that baking is a sci- 
ence. She says that if you follow a recipe carefully, assembling the ingredients accurately, 
cooking at the specified temperature for the specified period of time, your cookies will 
always turn out right. Chef Dearborn says the best baking is like the best experiment — 
anyone can duplicate it. 

16. In this passage, the word assembling most nearly means 

(A) measuring. 

(B) putting together. 

(C) buying. 

(D) storing. 

Although measuring is something you do when baking, it doesn't most nearly mean the 
same thing as assembling. Putting together does. Therefore, Choice (B) is the correct answer. 

17. According to the passage, a person who's all thumbs in the kitchen 

(A) should get out of the kitchen. 

(B) is an artist. 

(C) isn't following the recipe carefully. 

(D) is Chef Dearborn. 

The passage states that if you follow a recipe carefully, "your cookies will always turn out 
right." The correct answer is Choice (C). 



/r 



qQ Part II: Words to Live By: Communication Skills 



Passage three 

At dinner-time tonight I was feverish to do three things at once: write out my day's Journal, 
eat my food, and read the Journal of Marie Bashkirtseff. Did all three — but unfortunately not 
at once, so that when I was occupied with one I would surreptitiously cast a glance side- 
ways at the other — and repined. 

18. Which of the following was the author NOT planning to do? 

(A) go shopping 

(B) eat food 

(C) write a journal 

(D) read a journal 

The passage's first sentence tells you everything you need to know — it mentions writing 
and reading journals and eating food. Shopping isn't mentioned, so the correct answer is 
Choice (A). 

Passage four 

To motivate your people, give them tasks that challenge them. Get to know your people and 
their capabilities, so you can tell just how far to push each one. Give them as much respon- 
sibility as they can handle and then let them do the work without looking over their shoul- 
ders and nagging them. When they succeed, praise them. When they fall short, give them 
credit for what they've done and coach or counsel them on how to do better next time. 

19. According to the above paragraph, if your subordinates fail to adequately perform their 
tasks, you should 

(A) punish them. 

(B) praise them. 

(C) counsel them. 

(D) both B and C. 

If you didn't read the passage, praising someone who didn't measure up may seem like a 
bad idea. However, the last sentence states you should give your subordinates credit for 
the parts of the task they performed correctly and counsel them how to do better the next 
time. Although that sentence doesn't use the word praise, you can infer that giving someone 
credit means the same thing. The correct answer is Choice (D). 

20. After assigning responsibility for the tasks at hand to your subordinates, you should 

(A) supervise them closely to ensure the tasks are performed correctly. 

(B) let them do the work on their own. 

(C) check their progress at the end of each day. 

(D) schedule sufficient work-breaks to avoid job burnout. 

Choices (C) and (D) sound like good ideas, but they aren't suggestions discussed in the para- 
graph. Remember to avoid the trap of answering based on your personal feelings. Choice (A) 
is the opposite of what the passage suggests — the writer says to "let [employees] do the 
work without looking over their shoulders." Choice (B) is the correct answer. 



Chapter 6: All's Well That Tests Well: Communication Practice Questions Q J 

Passage fitfe 

Approximately 15,000 years ago the first Native Americans may have appeared in Colorado. 
The earliest inhabitants were hunters and nomadic foragers on the plains, as well as the 
western plateau. Agricultural settlements began appearing along river valleys in the eastern 
part of Colorado from approximately 5,000 B.C. as people learned farming techniques from 
the Mississippi River Native Americans. 

21. The first Native Americans in Colorado were 

(A) farmers. 

(B) traders. 

(C) hunters and gatherers. 

(D) originally from the Mississippi River region. 

The second sentence states that the original inhabitants "were hunters and nomadic forag- 
ers," and because none of the other answer options include hunters, you can deduce that 
nomadic foragers means gatherers. The correct answer is Choice (C). 

Passage six 

Organizational leaders influence several hundred to several thousand people. They do this 
indirectly, generally through more levels of subordinates than do direct leaders. The addi- 
tional levels of subordinates can make it more difficult for them to see results. Organizational 
leaders have staffs to help them lead their people and manage their organizations' resources. 
They establish policies and the organizational climate that support their subordinate leaders. 

22. Organizational leaders provide 

(A) direct leadership. 

(B) general policies. 

(C) organizational budgets. 

(D) daily work schedules. 

The passage mentions direct leaders but only to contrast them with organizational leaders, 
so (A) is wrong. According to the passage, organizational leaders "establish policies and the 
organizational climate that support their subordinate leaders." The correct answer is 
Choice (B). 

23. In order to become more efficient, organizational leaders make significant use of 

(A) computer technology. 

(B) rules and regulations. 

(C) efficiency and management reports. 

(D) staffs. 

Organizational leaders have staffs to help them efficiently lead their subordinates and 
manage the organization. Therefore, Choice (D) is the correct answer. 



/r 



^2 ^ art "' Words to Live By: Communication Skills 



Passage set/en 

His name is Frank Clarke, but his real name isn't really as real as the one the children gave 
him — The Toyman — because he's always making the kids things, such as kites and tops, 
sleds and boats, jokes and happiness and laughter. His face is as brown as saddle leather, 
with a touch of apple red in it from the sun. His face is creased, too, because he laughs and 
jokes so much. Sometimes when The Toyman appears to be solemn you want to laugh most, 
for he's only pretending to be solemn. And, best of all, if you hurt yourself or if your pet 
doggie hurts himself, The Toyman knows how to fix it to make it all well again. 

24. Frank Clarke's face could best be described as 

(A) rugged. 

(B) pink and smooth. 

(C) fair. 

(D) feminine. 

According to the passage, Frank's face is "brown as saddle leather," and he has wrinkles 
from laughing often. Choice (A) is the correct answer. 

25. Clarke received his nickname because he was always 

(A) fixing toys. 

(B) making toys for the children. 

(C) telling stories about toys. 

(D) playing with toys. 

The first sentence in the passage explains why the children gave Frank the nickname of The 
Toyman. Frank knows how to fix things, but that's not how he got his name. The correct 
answer is Choice (B). 



Part III 



Making the Most of Math 
Arithmetic Skills 



The 5 th Wave 



By Rich Tennant 




"The iwage is getting clearer now., I can almost seeit...Yes,' there 
it is. The answer is 3ab*x7d?--VjflL-i-u3£ 1 'f*± lco.O$)l% + Q6Q" 



In this part . . . 



i\J\ any military careers require a solid understanding 
/r lof math principles. Even though the military will 
spring for a calculator, you're expected to know how to 
add and subtract before you hit boot camp. And you can't 
use a calculator on the ASVAB. 

Part III gives you a chance to brush up on your numbers 
knowledge. It includes all kinds of information that can 
help you do well on the two math-related subtests that 
the ASVAB throws at you: Arithmetic Reasoning and 
Mathematics Knowledge. I also give you a ton of tips on 
everything from how to guess whether you're running out 
of time to what to do if you forget how to solve a quadratic 
equation. 



Chapter 7 

Mathematics Knowledge 
and Operations 



In This Chapter 

Getting more terminology under your belt 
Revisiting high school: Algebra and geometry review 
Performing calculations without the calculator 
Perfecting your way to a higher score 



*J*BE* 




XM lbert Einstein once said, "Do not worry about your problems with mathematics. 
V \ I assure you mine are far greater." The good professor obviously never faced an 
upcoming ASVAB exam! Okay, just kidding. You don't have to be a mathematical theoreti- 
cian to score well on the Mathematics Knowledge subtest. This subtest asks questions 
about basic high school mathematics. No college or graduate degrees needed. 

On the paper version of the ASVAB, the Mathematics Knowledge subtest consists of 
25 questions, and you have 24 minutes to complete the subtest. The CAT-ASVAB offers 
16 questions in 20 minutes. You don't necessarily have to rush through each calculation, 
but the pace you need to set doesn't exactly give you time to daydream. You have to focus 
and concentrate to solve each problem quickly and accurately. And no calculators allowed! 

Most of the time, the Mathematics Knowledge subtest contains only one or two questions 
testing each specific mathematical concept. For example, one question may ask you to mul- 
tiply fractions, the next may ask you to solve a mathematical inequality, and the question 
after that may ask you to find the value of an exponent. (If you're freaked out by the last 
sentence, calm down. I cover these concepts in this chapter.) 

All this variety forces you to shift your mental gears quickly to deal with different concepts. 
You can look at this situation from two perspectives. These mental gymnastics can be diffi- 
cult and frustrating, especially if you know everything about solving for x but nothing about 
finding a square root. But variety can also be the spice of life. If you don't know how to 
solve a specific type of problem, any oversight may only cause you to get one or two ques- 
tions wrong. 

To qualify for certain jobs in the military, you have to score well on the Mathematics 
Knowledge subtest. You also have to do well on this subtest, which is part of the AFQT dis- 
cussed in Chapter 1, in order to enlist. Turn to the Appendix A to find out more about the 
subtest scores needed for specific military jobs. 

In this chapter, I go over the basic arithmetic, algebra, and geometry you need to know. 
This info also comes in handy when solving word problems on the Arithmetic Reasoning 
subtest, which I cover in Chapter 8. 



00 Part III: Making the Most of Math: Arithmetic Skills 



Just When l/ou Thought \lou Were bone 
With Vocab: Math Terminology 

Math has its own vocabulary. In order to understand what each problem on the 
Mathematical Knowledge subtest asks, you need to understand certain mathematical terms: 

u* Integer: An integer is any positive or negative whole number or zero. The ASVAB often 
requires you to work with integers, such as -6, 0, or 27. 

u* Factors: Factors are integers (whole numbers) that can be divided evenly into another 
integer. To factor a number, you simply determine the numbers that you can divide 
into it. For example, 8 can be divided by the numbers 2 and 4 (in addition to 1 and 8), 
so 2 and 4 are factors of 8. 

Numbers may be either composite or prime, depending on how many factors they have: 

• Composite number: A composite number is a whole number that can be divided 
evenly by itself and by 1, as well as by one or more other whole numbers; in other 
words, it has more than two factors. Examples of composite numbers are 6 
(whose factors are 1, 2, 3, and 6), 9 (whose factors are 1, 3, and 9), and 12 (whose 
factors are 1, 2, 3, 4, 6, and 12). 

• Prime number: A prime number is a whole number that can be divided evenly by 
itself and by 1 but not by any other number, which means that it has exactly two 
factors. Examples of prime numbers are 2 (whose factors are 1 and 2), 5 (whose 
factors are 1 and 5), and 11 (whose factors are 1 and 11). 

u* Exponent: An exponent is a shorthand method of indicating repeated multiplication. 
For example, 15 x 15 can also be expressed as 15 2 , which is also known as "15 squared" 
or "15 to the second power." The small number written slightly above and to the right 
of a number is the exponent, and it indicates the number of times you multiply the 
number it accompanies by itself. Note that 15 2 (15 x 15, which equals 225) isn't the 
same as 15 x 2 (which equals 30). 

To express 15 x 15 x 15 using this shorthand method, simply write it as 15 3 , which is 
also called "15 cubed" or "15 to the third power." Again, 15 3 (which equals 3,375) isn't 
the same as 15x3 (which equals 45). 

u* Base: A base is a number that's used as a factor at least two times — it's a number 
raised to an exponent. For instance, the term 4 3 (which can be written 4x4x4, and in 
which 4 is a factor three times) has a base of 4. 

j*" Square root: The square root of a number is the number that, when multiplied by itself 
(squared), equals the original number. For example, the square root of 36 is 6. If you 
square 6, or multiply it by itself, you produce 36. (Check out "Getting to the Root of the 
Problem," later in this chapter.) 

J-" Factorial: A factorial is represented by an exclamation point (!). You calculate a facto- 
rial by finding the product of (multiplying) a whole number and all the whole numbers 
less than it down to 1. So 6 factorial (6!) is 6x5x4x3x2x1 = 720. 



*J|lBBf 




A factorial helps you determine permutations — all the different possible ways an event 
may turn out. For example, if you want to know how many different ways six runners 
could finish a race (permutation), you would solve for 6! — 6x5x4x3x2x1. 

i*" Reciprocal: A reciprocal is the number by which another number can be multiplied to 
produce 1; if you have a fraction, its reciprocal is that fraction turned upside down. For 

example, the reciprocal of 3 is ^. If you multiply 3 times ^, you get 1. The reciprocal of 

i o 1 9 Q 

-r is y (which is the same thing as 6); -~ x 6 = 1. The reciprocal of ^ is ^. Get the idea? 



Chapter 7: Mathematics Knowledge and Operations ^ 7 




V Rounding: Rounding is limiting a number to a few (or no) decimal places. You perform 
rounding operations all the time — often without even thinking about it. If you have a 
$1.97 in change in your pocket, you may say, "I have about two dollars." The rounding 
process simplifies mathematical operations. 

Often, numbers are rounded to the nearest tenth. The ASVAB may ask you to do this. 
For any number that ends in 5 and over, round up; for any number under 5, round down. 
For example, 1.55 can be rounded up to 1.6, and 1.34 can be rounded down to 1.3. 

Many math problems require rounding — especially when you're doing all this 
without a calculator. For example, pi (n) represents a number approximately equal to 
3.141592653589793238462643383 (and on and on and on). However, in mathematical 
operations, it's common to round n to 3.14. 



Operations: What \lou Do to Numbers 

When you toss numbers together (mathematically speaking), you perform an operation. When 
you add or multiply, you perform a basic operation. But because math functions according 
to yin-yang-like principles, each of these basic operations also has an opposite operation 
called an inverse operation. Thus, the inverse of addition is subtraction, and the inverse of 
multiplication is division. And of course, the inverse of subtraction is — you got it — addi- 
tion. The inverse of division is multiplication. 



^\NG/ 




Don't confuse opposite with inverse. When you're doing mathematical operations, such as 
adding and multiplying, the inverse operation is the opposite operation. But when you're 
talking numbers, opposite and inverse don't mean the same thing. The opposite of a posi- 
tive number is a negative number, so the opposite of x is -x But the inverse of a number is 
its reciprocal — that number turned on its head! The inverse of x is — . The inverse of -^ is y 
(or just 5). 



First things first: following the order of operations 

Operations must be performed in a certain order. For example, when you have parentheses 
in a math problem, the calculation in the parentheses must be done before any calculations 
outside of the parentheses. In the equation 2 x (16 + 5) = ?, you first add 16 to 5 to arrive at 
21, and then you multiply by 2 to come up with a total of 42. You get a different (and wrong) 
answer if you simply calculate from left to right: 2x16 = 32, and 32 + 5 = 37. And you better 
believe that both results will be choices on the test! 



Naming the answers 



The result of each operation — addition, subtraction, j*" When you subtract, all that remains is a difference. 
multiplication, or division — goes by a different name: 



v When you add two numbers together, you arrive at 



a sum. 



v* When you multiply, you come up with a product. 
w When you divide, you're left with a quotient. 



0$ Part III: Making the Most of Math: Arithmetic Skills 



cjftBEtf 




To figure out which mathematical operation you should perform first, second, third, and so 
on, follow these rules, otherwise known as the order of operations: 

1. Parentheses take precedence. 

You should do everything contained in parentheses first. In cases where parentheses 
are contained within parentheses, do the innermost parentheses first. 

Note: If you're dealing with a fraction, treat the top as though it were in parentheses 
and the bottom as though it were in parentheses, even if the parentheses aren't written 

in the original state. Suppose you have the problem: ^ — ^ = ? Add the numbers below 
the fraction bar before dividing. The answer is # - 1. (For more on fractions, see later 

section "Working on Both Sides of the Line: Fractions.") The square root sign (V) is also 
a grouping symbol, so you solve for whatever's under the top bar of the square root 
sign before doing any other operation in the problem. 

2. Exponents come next. 

Remember that the exponent goes with the number or variable that it's closest to. If 
it's closest to a parenthesis, then you already should've performed the calculation 
inside the parentheses in Step 1. For example, (5 x 2) 2 = 10 2 = 100. The square root sign 
(V) is also treated as an exponent, so you take the square root during this step. 

3. Multiplication and division are next. 

Do these operations in left-to-right order (just like you read). 

4. Addition and subtraction are last. 

Perform these operations from left to right as well. 

Check out the following example for a little practice with order of operations: 

(15 * 5) x 3 + (18 - 7) = ? 
Do the work in parentheses: 

3x3 + 11 =? 

No exponents are present, so division and multiplication come next (in this problem, only 
multiplication is needed): 

9+ 11 =? 

Finally, do the addition and subtraction (in this problem, only addition is needed). Your 
final answer is 20. 



Completing a number sequence 

The Arithmetic Reasoning (AR) subtest often includes questions that test your ability to 
name what comes next in a sequence of numbers. Generally, these problems are the only 
AR questions that aren't word problems (which I cover in Chapter 8). However, sequence 
questions do test your ability to do arithmetic and to reason, because you have to deter- 
mine how the numbers relate to each other. And to do this, you must be able to perform 
mathematical operations quickly. 



Chapter 7: Mathematics Knowledge and Operations Q Q 




Suppose you have a sequence of numbers that looks like this: 1, 4, 7, 10, ? 

Each new number is reached by adding 3 to the previous number: 1+3 = 4, 4 + 3 = 7, and so 
on. So the next number in the sequence is 10 + 3, or 13. But of course, the questions on the 
ASVAB aren't quite this simple. More likely, you'll see something like this: 2, 4, 16, 256, ? 

In this case, each number is being multiplied by itself, so 2x2 = 4, 4x4= 16, and so on. The 
next number in the sequence is 256 x 256, which equals 65,536 — the correct answer. You 
may also see sequences like this: 1, 2, 3, 6, 12, ? 

In this sequence, the numbers are being added together: 1+2 = 3, and 1 + 2 + 3 = 6. The next 
number is 1 + 2 + 3 + 6 = 12. So the next number would be 24. 

Finding the pattern 

To answer sequence questions correctly, you need to figure out the pattern as quickly as 
possible. Some people, blessed with superior sequencing genes, can figure out patterns 
instinctively. The rest of the population has to rely on a more difficult, manual effort. 

Finding a pattern in a sequence of numbers requires you to think about how numbers work. 
For instance, seeing the number 256 after 2, 4, 16 should alert you that multiplication is the 
operation, because 256 is so much larger than the other numbers. On the other hand, 
because the values in 1, 2, 3, 6, 12 don't increase by much, you can guess that the pattern 
requires addition. 

beatinq With more than one operation in a sequence 

Don't forget that more than one operation can occur in a sequence. For example, a sequence 
may be "add 1, subtract 1, add 2, subtract 2." That would look something like this: 2, 3, 2, 4, ? 

Because the numbers in the sequence both increase and decrease as the sequence contin- 
ues, you should suspect that something tricky is going on. 

Make sure to use your scratch paper! Jot down notes while you're trying to find the pattern 
in a sequence. Writing your work down helps you keep track of which operations you've 
tried. 



Working on Both Sides of the Line: Fractions 

I don't know why, but it seems almost all math textbooks explain fractions in terms of pies. 
(I think most mathematicians must have a sweet tooth.) But I like pizza, so I'm going to use 
pizza instead. If a whole number is a pizza, a fraction is a slice of pizza. A fraction also illus- 

o 

trates the slice's relationship to the whole pizza. For example, consider the fraction ^. If 

you accuse your cousin of eating -^ of the pizza when he comes over for movie night, you're 

saying that the pizza is divided into five equal-sized slices — fifths — and your cousin ate 
three of those five slices. Can anyone say pig? 

The number above the fraction bar — the three slices your cousin ate — is called the 
numerator. The number written below the fraction bar — the total number of slices the 
pizza is divided into — is called the denominator. 




0^B 




YQ Part III: Making the Most of Math: Arithmetic Skills 

Common denominators: Preparing to 
add and subtract fractions 

To add and subtract fractions, the fractions must have the same denominator (bottom 
number), which is called a common denominator. If the fractions don't have a common 
denominator, you have to find one. There are two basic methods to use. Sound fun? 
Read on. 

Method one 

Finding a common denominator can be easy, or it can be as hard as picking off all the 

o o 

anchovies. Suppose you want to add 4 and ^. Getting a common denominator is easy here, 

and you use this process whenever you can evenly divide one denominator by another. 
Follow these steps: 

1. Divide the larger denominator by the smaller denominator. 

If there's a remainder, then you can't use this method, and you have to use method two 
(see the next section). In this case, 10 can be divided evenly by 5. The quotient 
(answer) that results is 2. 

2. Take the fraction with the smaller denominator I ^ J; multiply both the numerator 

(top number) and the denominator (bottom number) by the answer that resulted in 
Step 1. 

Multiply 3 by 2, and the result is 6 — that's your new numerator. Multiply 5 by 2, and 
the result is 10 — that's your new denominator. 

3. Replace the numerator and denominator with numbers from Step 2. 

You can also express ^ as -^r. (If you cut the pizza into 10 slices instead of 5 and your 
cousin eats 6 slices instead of 3, he's eaten exactly the same amount of pizza.) 

After you've found a common denominator, you add the two fractions by simply adding the 
numerators together: A. + -^ = ^-. Think of it this way: If your cousin eats ^r of the pizza 

(which is just another way of saying -^) and you eat ^ of the pizza, together you've eaten 
q o 10 

^ of the pizza. 

Method Mo 

O 1 

Suppose your cousin eats ^ of one pizza and your sister eats ^ of another pizza (one that 
was cut into 6 slices instead of 5), and you want to know how much pizza has been eaten. In 

o 1 

this case, you need to add ^ and -±-. 

5 6 

Adding these fractions is a bit more difficult because you can't divide either denominator 
by the other. You have to find a common denominator that both 5 and 6 divide into evenly. 
Here's how: 

1. Multiply the denominator of the first fraction by the denominator of the second 
fraction. 

In the example case, 5 x 6 = 30. The common denominator for both fractions is 30. 

2. Express the first fraction in terms of the new common denominator. 

5 "30 



Chapter 7: Mathematics Knowledge and Operations 7 / 



0tSEH 




3. Multiply the numerator by the number that you multiplied by to result in the new 
denominator. 

To convert the denominator (5) to 30, you multiply by 6, so multiply the numerator (3) 

O I Q 

by 6. The result is 18. Therefore, the fraction 4 can be expressed as ^4. 

o 30 

When you're trying to find the common denominator for a fraction, always multiply the 
numerator and the denominator by the same number. Otherwise, you change the value 
of the fraction. 

With this example, you multiply the numerator and the denominator by 6, discovering 

O 1 Q 

that 4 is the same thing as ^4. But if you were to multiply only the denominator by 6, 

3 o 

you'd have a new number — 4 and t4 don't have the same value. 

4. Next, express the second fraction in terms of the new common denominator. 

I- JL 
6 30 

5. Multiply the numerator of the second fraction by the number you used to result in 
the denominator. 

To get 30, you have to multiply 6 by 5. Multiply the numerator by the same number: 

You find that 1 x 5 = 5, so the fraction j, can be expressed as -£r. 

b 60 

After all that work, you can finally add the fractions: J^ + J^r = ^. Now pause and take a bite 
of pizza. 

Finding common denominators for three or more fractions 

If you have more than two fractions with different denominators, you have to find a common 

12 3 

denominator that all the denominators divide into. Suppose you need to add 4; + 4 + 4. 

Zoo 

A simple way to find a common denominator is to take the largest denominator (in this 
case 5) and multiply it by whole numbers, starting with 1, 2, 3, 4, and so on until you find a 
denominator that the other denominators also divide into evenly. 

If you multiply 5 by 2, you get 10, but 3 doesn't divide evenly into 10. So keep going: 5x3 = 
15, 5 x 4 = 20, and so on until you find a number that 2, 3, and 5 can divide into evenly. In 
this case, 30 is the first number you can find that 2, 3, and 5 can divide into evenly, so 30 is 
your common denominator. 



0»Uft 




Multiplying and reducing fractions 



Multiplying fractions is easy. You just multiply the numerators and then multiply the 

?. You multiply 1x3x3 = 9 (the 



1 3 3 
denominators. So look at the following equation: ± x 4 x 4 



numerators) and then 2 x 4 x 5 = 40 (the denominators) to result in -^. 

Occasionally, when you multiply fractions, you end up with an extremely large fraction that 
can be simplified or reduced. To express a fraction in its lowest terms means to put it in 
such a way that you can't evenly divide the numerator and the denominator by the same 
number (other than 1). 



A number that you can divide into both the numerator and the denominator is called a 
common factor. If you have the fraction y^-, both the numerator (6) and the denominator 

(10) can be divided by the same number, 2. If you do the division, 6 + 2 = 3 and 10 + 2 = 5, 



72 Part lll: Making the Most of Math: Arithmetic Skills 




you find that ^ can be expressed in the simpler terms of y. You can't reduce (simplify) y 
any further; the only other number that both the numerator and denominator can be 
divided by is 1, so the result would be the same, y. 

Remember, you can't use a calculator on the ASVAB, so multiplying large numbers can take 
extra steps and valuable time. You can make your work easier by canceling out common 

factors before multiplying. For example, suppose you have the problem 4r x t$-- Multiplying 

the numerators (20 x 14 = 280), then multiplying the denominators (21 x 25 = 525), and 

finally reducing the fraction I ^50. 

rate multiplication/division problems. But you can save time if a numerator and denomina- 
tor have common factors. Here, the numerator of the first fraction (20) and the denominator 
of the second (25) have a common factor of 5, so you can divide both of those numbers by 

5: Your problem becomes yrX ^. The numerator of the second fraction (14) and the 

denominator of the first fraction (21) are both divisible by 7, so you can cancel out a 7: 

Divide 14 and 21 by 7. This changes the equation to y x -f = ^-, a much simpler math problem. 



-°-j may require you to write out three or more sepa- 



Sjt&Et 




Dividing fractions 



Dividing fractions is simple if you remember this rule: Dividing a fraction by a number is the 
same as multiplying it by the inverse (reciprocal) of that number. Of course, there are 
always exceptions. You can't use this operation on zero. Zero has no inverse. No one knows 
why — it just is. 

The inverse of a number is obtained by reversing the number. That means that if you want 

to divide a fraction by 5, you simply multiply the fraction by the inverse of 5, which is i. 

5 

You can understand this process more easily if you remember that 5 is the same thing as y. 

In other words, 5 divided by 1 equals 5 (that is, 5 * 1 = 5). And the inverse of y is y. To come 
up with the inverse of a number, simply stand the number on its head. 

So to divide a fraction, use the inverse of the number that follows the division symbol (+) 

and substitute a multiplication symbol (x) for the division symbol. Therefore, y ■*■ 2 is 

11 
expressed as y x y, and you already know how to multiply fractions. (If not, check out the 

"Multiplying and reducing fractions" section earlier in the chapter.) 1x1 = 1 and 3x2 = 6, 

so the product of y x y = ^. Therefore, y + 2 = y. 



Converting improper fractions to mixed 
numbers . . . and back again 

If you have a fraction with a numerator larger than its denominator, you have an improper 

■7 

fraction. For example, y is an improper fraction. To put an improper fraction into simpler 

7 
(proper) terms, you can change y into a mixed number (a number that includes a whole 

number and a fraction). Simply divide the numerator by the denominator: 7 divided by 3 gives 
you a quotient of 2 with a remainder of 1. There's something left over because 3 doesn't 
divide evenly into 7. The remainder becomes a numerator over the original denominator, 

1 7 1 

so y is left over. Therefore, y is the same as 2y. 



Chapter 7: Mathematics Knowledge and Operations 7 A 



j^BE/f 




If you want to multiply or divide a mixed number, you need to convert it into a fraction — 
an improper fraction. To make the change, convert the whole number into a fraction and 
add it to the fraction you already have. Here's how: 

1 . Multiply the whole number by the denominator (bottom number) of the existing 

fraction to arrive at a new numerator. 

o 
Suppose you have 7lr. Multiply 7 by 3: 7x3 = 21 

2. Place this new numerator over the existing denominator. 

21 
3 

3. Add that fraction to the original fraction to get the final answer. 

21, 2_23 
3 3 3 

Check out the "Adding and subtracting fractions" section earlier in this chapter for the 
complete scoop on adding fractions. 

Or if you want to get technical, you can look at the whole process this way, too: 
n 2 7x3 + 2 23 



0RHt 




Expressing a fraction in other forms: 
Decimals and percents 

A fraction can also be expressed as a decimal and as a percent. Here's how to convert 
between forms: 

u* To change a fraction into a decimal: Divide the numerator (top number) by the 

denominator (bottom number). Given that handy explanation, ^ (or 3 + 5) converted 
into decimal form is 0.6. 

Some fractions convert to repeating decimals — a decimal in which one digit is repeated 
infinitely. For instance, ^ is the same as 0.66666... (with the sixes never stopping). 

Repeating decimals are often rounded to the nearest hundredth; therefore, ^ rounds to 

0.67. (Remember: The first space to the right of the decimal is the tenths place, the 
second space is the hundredths place, and the third is the thousandths, and so on.) 

u* To make a decimal into a percent: Move the decimal point two spaces to the right and 
add a percent sign. For example, 0.6 becomes 60%. 

See the following sections for more thorough discussions of decimals and percents. 




Adding and subtracting decimals 

To add and subtract decimals, put the numbers in a column and line up the decimal points. 
Then add or subtract as if the decimals were whole numbers, keeping the decimal point in 
the same position in your answer. 



1.4583 
Here are two examples: +0.55 

2.0083 



1.4583 
-0.55 
0.9083 



You can add zeros to the end of a decimal if performing the calculations this way is easier 
for you. So 0.1 can be 0.100 without changing its value. In the preceding problems, 0.55 can 
be 0.5500 to help you line up the decimal points and perform the operation. 



7 (l Part III: Making the Most of Math: Arithmetic Skills 



&m 




&*$! 




Multiplying decimals 

Multiplying a decimal is like multiplying a regular, everyday whole number, except that you 
have to place the decimal point in the correct position after you reach an answer. 

1. Multiply as though you were multiplying whole numbers, without the decimal 
points. 

Suppose you're multiplying 3.77 x 2.8. In this example, 377 x 28 = 10556. 

2. Count and add the number of decimal places (to the right of the decimal point) in 
the numbers being multiplied. 

If one of the numbers you're multiplying is 3.77, you have two decimal places. If the 
other number you're multiplying is 2.8, you have one more decimal place, so the total 
number of decimal places in your answer will be three. 

If you're multiplying a number that has only zeros to the right of the decimal point, 
then those decimals don't count. For instance, 3 can also be expressed as 3.0, but you 
wouldn't count the as a decimal place. All the zeros to the right of the decimal point 
don't count unless a number other than zero comes after them. For instance, 3.000007 
has six decimal spaces, 3.0070 has three decimal spaces, and 3.000 has none, at least 
not for the purpose of multiplying. 

3. In the answer, move the decimal point back to the left the number of places you 
counted in Step 2. 

This time, zeros do count. You counted three total decimal places in 3.77 and 2.8, so 
you move the decimal point in 10566 back to the left three places. The resulting prod- 
uct is 10.556. 

If your answer doesn't include enough numbers for the decimal spaces you need, then 
add as many zeros as necessary to the left of the answer. Suppose your answer is 50, 
and you have to move the decimal point to the left three spaces. There aren't three 
spaces in 50, so you add a zero to the left to make 050 and put the decimal point in its 
proper position: 0.050 is your answer. 

Here's another example: 0.04 x 0.25. Multiply the decimals as if they were whole numbers: 
4 x 25 = 100. Count and add the number of decimal places in the original two numbers; there 
are four. Then put the decimal point in the correct place in the answer. For 100, count from 
right to left four places, and put the decimal point there: 0.0100, or 0.01. Here's the method 
behind the madness: 



100 100 



100 1 

10,000 100 



= 0.01 for 0.0100) 



$mn 




bitfidinq decimals 

Decimals are divided according to slightly different rules, depending on whether both num- 
bers in the problem are decimals. 

Dividing decimals by whole numbers 

Here's how to divide a decimal by a whole number: 

1. Move the decimal point over to the right until the decimal is a whole number, count- 
ing the number of decimal places. 

For example, if you want to find 1.25 * 4, change 1.25 to 125 by moving the decimal two 
decimal places to the right. Remember how many places you moved the decimal — 
you need that info later. 

2. Perform the division operation on the whole number. 

125*4 = 31.25 



Chapter 7: Mathematics Knowledge and Operations (5 




3. In your answer, move the decimal point to the left the number of places you moved 
it in Step 1. 

To make up for moving the decimal point two places to the right when you made 1.25 
into a whole number, move the decimal point two places to the left in 31.25. Your 
answer is 0.3125. 

bioidinq decimals by decimals 

Here's how to divide a decimal by another decimal in which there are the same number of 
places after the decimal point: 

1. Make the divisor (the decimal going into the other number) into a whole number: Move 
the decimal point all the way to the right, counting the number of places you move it. 

Suppose you want to divide 0.15 by 0.25 (that is, 0.15 * 0.25). Move the decimal point in 
0.25 two places to the right: It then becomes 25. 

2. Move the decimal in the dividend (the number being divided) the same number of 
decimal places. 

Move the decimal point in 0.15 two places: 0.15 becomes 15. 

3. Divide. 

When you divide 15 by 25, the result is 0.60. You don't need to move any more deci- 
mals around — 0.60 is your final answer. 

If the dividend is a longer decimal than the divisor, you follow the same steps, but you have 
to do an extra step at the end. So if your problem is 0.125 * 0.50, first move the decimal 
point in the divisor (0.5) one place to the right so that you have the whole number 5. Then 
move the decimal point in the dividend one place to come up with 1.25. 

Now the problem looks like this: 1.25 * 5. Convert the first number (1.25) to a whole number by 
moving the decimal point two places to the right. Now perform the division operation on the 
whole numbers: 125 + 5 = 25. Move the decimal point two places to the left (to make up for moving 
it two places to the right when you converted 1.25 to a whole number). The answer is 0.25. 

When the divisor is a longer decimal than the dividend, such as 0.50 + 0.125, move the decimal 
point in the divisor all the way to the right, in this case making 0.125 into 125, counting places. 
Then move the decimal the same number of places in the dividend, adding zeros as needed: 
0.5 then becomes 500. Then 500 * 125 = 4, which is the correct answer (0.50 * 0.125 = 4). 

Playing With percents 

A percent is a fraction based on one-hundredths. Five percent (5%) is the same as yj^y or 

0.05. The ASVAB often asks you to calculate "10% off" or "an increase of 15%" on the 
Arithmetic Reasoning subtest. You need to be able to convert percents to fractions or 
decimals to answer these questions correctly. 

To add, subtract, multiply, or divide using percents, change the percent to a fraction or a 
decimal. Here are some helpful hints for figuring percents: 

u* Remember, a percent is just hundredths, so 3% is y^r or 0.03, 22% is -^- or 0.22, and 

i i r\ 1U0 1UU 

110% is |^ or 1.10. 

f To convert a percent to a decimal, just drop the percent sign and move the decimal 
point two places to the left, adding zeros as needed. 

p" The decimal point always starts to the right of a whole number, so 60 is the same thing 
as 60.0. Moving the decimal point two spaces to the left leaves you with 0.6. 

After you do the conversion, follow the rules outlined in the earlier sections for performing 
specific operations on fractions or decimals. 



Y0 Part III: Making the Most of Math: Arithmetic Skills 



Showing comparisons With ratios 

A ratio shows a relationship between two things. For example, if Margaret invested in her 
tattoo parlor at a 2:1 (or 2 to 1) ratio to her business partner Julie, then Margaret put in $2 
for every $1 that Julie put in. You can express a ratio as a fraction, so 2:1 is the same as y. 

Or suppose you fill up your brand-new, shiny SUV, and you want to compute your gas 
mileage — miles per gallon. You drive for 240 miles and then refill the tank with 15 gallons 
of gas, so the ratio of miles to gallons is 240:15. You can compute your gas mileage by divid- 
ing the number of miles by the number of gallons: 240 miles + 15 gallons. You're getting 
16 miles per gallon. Time for a tune up! 

In this section, I cover some uses for ratios, including scale drawings and rates such as speed. 



Navigating scale drawings 

Scale, particularly when used on the ASVAB, relates to scale drawings. For example, a map 
drawn to scale may have a 1-inch drawing of a road that represents 1 mile of physical road 
in the real world. The Arithmetic Reasoning portion of the ASVAB often asks you to calcu- 
late a problem based on scale, which can be represented as a standard ratio (1 inch:l mile) 



or a fraction 



1 inch \ 
1 mile f 




On a map with a scale of 1 inch to 1 mile, the ratio of the scale is represented as 1:1. But ques- 
tions are never this easy on the ASVAB. You're more likely to see something like, "If a map 
has a scale of 1 inch to every 4 miles. . . ." That scale is expressed as the ratio 1:4, or i 

Try your hand with the following common scale problem: 

If the scale on a road map is 1 inch = 250 miles, how many inches would represent 1,250 miles? 

The problem wants you to determine how many inches on the map represent 1,250 miles if 
1 inch is equal to 250 miles. You know that 1 inch = 250 miles, and you also know that some 
unknown number of inches, which you can call x, equals 1,250 miles. The problem can be 



expressed as two ratios set equal to each other, known as a proportion; 
you have to do is solve for x: 

1 _ x 
250 



1 x 

250"U50 



Now all 



1 vl OKC\ 

250 Xi ' ZDU " 1,250 



1,250 
x x 1,250 



1,250 
250 



x 



*»££? 




So if 1 inch is equal to 250 miles, then 5 inches would be equal to 1,250 miles. If this problem 
causes you to scratch your head, check out the upcoming section "An Unknown Quantity: 
Reviewing Algebra" for info on solving for x. 

Almost every military job makes use of scales, which is why scale-related questions are so 
common on the ASVAB. Whether you're reading maps at Mountain Warfare School or orga- 
nizing trash pickup around the base, you need to use and interpret scales frequently. 

Remembering important rates 

A rate is a fixed quantity — a 5% interest rate, for example. It can mean the speed at which 
one works (John reads at the rate of one page per minute). Or it can mean an amount of 
money paid based on another amount (life insurance may be purchased at a rate of $1 per 
$100 of coverage). A rate is often a speed, something per a unit of time. 



cjftBEtf 





Chapter 7: Mathematics Knowledge and Operations 7 7 



Word problems often ask you to solve problems that involve speed or simple interest rates. 
Here are two rate formulas you should commit to memory: 

j*" Simple interest: / = Prt, where / represents the amount of interest, P is the principal 
(the initial amount invested), r is the interest rate, and t is the length of time the money 
is invested. 

v* Distance: d - rt, where d represents the distance traveled, r is the rate (speed) of travel, 
and t is the amount of time traveled. 

In a rate, you can generally think of the word per as a division sign. For instance, suppose 
someone drives 141 miles in 3 hours, and you have to find the average speed. You want the 
rate of speed in miles per hour, so you take miles (distance) divided by hours (time): 141 
miles + 3 hours = 47 miles/hour. Using algebra, you can rearrange the distance formula to 
say the same thing: d + t = r. 



A Powerful Shorthand: Writing 
in Scientific Notation 



Scientific notation is a compact format for writing very large or very small numbers. 
Although it's most often used in scientific fields, you may find a question or two on the 
Mathematics Knowledge subtest of the ASVAB, asking you to covert a number to or from 
scientific notation. 

Scientific notation separates a number into two parts: a number between 1 and 10 and a 
power of ten (such as 10 7 , 10 21 , or 10~ 18 ; see the earlier section "Just When You Thought You 
Were Done with Vocab: Math Terminology" for info on powers and exponents). Therefore, 
1.25 x 10 4 means 1.25 x 10 to the fourth power, or 12,500; 5.79 x 10" 8 means 5.79 * 10 to 
the eighth power, or 0.0000000579. The exponent tells you how many places to move the 
decimal point and whether to move it left or right (depending on whether it's positive or 
negative). 



Getting to the Root of the Problem 



cjftBE/? 




A square root is the factor of a number that, when multiplied by itself, produces the number. 
Take the number 36, for example. One of the factors of 36 is 6. If you multiply 6 by itself (6 x 6), 
you come up with 36, so 6 is the square root of 36. The number 36 has other factors, such as 
18. But if you multiply 18 by itself (18 x 18), you get 324, not 36. So 18 isn't the square root of 36. 

All whole numbers are grouped into one of two camps when it comes to roots: 

u* Perfect squares: Only a few whole numbers, called perfect squares, have exact square 
roots. For example, the square root of 25 is 5. 

v* Irrational numbers: Other whole numbers have square roots that are decimals that go 
on forever and have no pattern that repeats (nonrepeating, nonterminating decimals), 
so they're called irrational numbers. The square root of 30 is 5.4772255 with no end to 
the decimal places, so the square root of 30 is an irrational number. 

The sign for a square root is called the radical sign. It looks like this: V. Here's how you use 
it: J36 means "the square root of 36" — in other words, 6. 



7$ Part III: Making the Most of Math: Arithmetic Skills 

Perfect squares 

Square roots can be difficult to find at times without a calculator, but because you can't use 
a calculator during the test, you're going to have to use your mind and some guessing meth- 
ods. To find the square root of a number without a calculator, make an educated guess and 
then verify your results. 

To use the educated-guess method (see the next section), you have to know the square 
roots of a few perfect squares. One good way to do this is to memorize the squares of the 
square roots 1 through 12: 

/"" 1 is the square root of 1 (1 x 1 = 1) 
v* 2 is the square root of 4 (2 x 2 = 4) 
p" 3 is the square root of 9 (3 x 3 = 9) 

V 4 is the square root of 16 (4 x 4 = 16) 
i* 5 is the square root of 25 (5 x 5 = 25) 
e* 6 is the square root of 36 (6 x 6 = 36) 
e" 7 is the square root of 49 (7 x 7 = 49) 

V 8 is the square root of 64 (8 x 8 = 64) 
W 9 is the square root of 81 (9 x 9 = 81) 

V 10 is the square root of 100 (10 x 10 = 100) 

V 11 is the square root of 121 (11 x 11 = 121) 

V 12 is the square root of 144 (12 x 12 = 144) 

Irrational numbers 

When the ASVAB asks you to figure square roots of numbers that aren't perfect squares, the 
task gets a bit more difficult. In this case, the ASVAB usually asks you to find the square 
root to the nearest tenth. 

Suppose you run across this problem: 

J54 



Think about what you know: You know that the square root of 49 is 7, and 54 is slightly 
greater than 49. You also know that the square root of 64 is 8, and 54 is slightly less than 64. 
So if the number 54 is somewhere between 49 and 64, the square root of 54 is somewhere 
between 7 and 8. Because 54 is closer to 49 than to 64, the square root will be closer to 7 
than to 8, so you can try 7.3 as the square root of 54. 

Multiply 7.3 by itself: 7.3 x 7.3 = 53.29, which is very close to 54. Now try multiplying 7.4 by 
itself to see if it's any closer to 54: 7.4 x 7.4 = 54.76, which isn't as close to 54 as 53.29. 
Therefore, 7.3 is the square root of 54 to the nearest tenth without going over. 



Other roots 



The wonderful world of math is also home to concepts like cube roots, fourth roots, fifth 
roots, and so on. A root is a factor of a number that when cubed (multiplied by itself 



Chapter 7: Mathematics Knowledge and Operations 7Q 



three times), taken to the fourth power (multiplied by itself four times), and so on produces 
the original number. A couple of examples seem to be in order: 

v* The cube root of 27 is 3. If you cube 3 (also known as raising it to the third power or 
multiplying 3x3x3), the product is 27. 

v* The fourth root of 16 is the number that, when multiplied by itself four times, equals 
16. Any guesses? Drumroll, please: 2 is the fourth root of 16 because 2x2x2x2 = 16. 



An Unknown Quantity: Rei/ieu/iny Algebra 

Some people freak out just hearing the word algebra. But algebra is just a way to put prob- 
lems into mathematical language using the simplest mathematical terms possible. In fact, 
it's almost impossible to solve most word problems without some use of algebra. 



**M* 




The letters in an algebra problem are called variables. In an algebra equation, if the same 
letter is used more than once, it stands for the same number. In 3x + 2x = 10, the first x will 
never be a different number from the second x. In this case, x = 2 (both times). 



^PLE 





Solving for x 



In algebra, you often hear about "solving for x" or "solving for the unknown," but what's the 
unknown? The unknown is the answer you want find. Check out this example: 

Rod's mom has worked up a powerful thirst solving a ton of math problems and asked Rod 
to run to the corner store and get her one of those super-duper gigantic nuclear soft drinks. 
If a regular-sized soft drink costs $0.50 and the super-duper gigantic nuclear size costs three 
times the cost of the regular size, how much will Rod have to spend? 

You can express this problem in terms of x, with x being the cost of the super-duper sized 
drink: x equals 3 (the price difference) x 50 cents. Written a bit more formally, the equation 
looks like this: x = 3(0.50) or 3(0.50) = x. 

What if you don't know how much the regular sized soft drink costs? You can express this 
missing piece of information in an equation as well: x (how much it will cost to buy a super- 
duper size) equals 3 (the cost increase) times p (the price of one regular sized drink). Once 
again, written a bit more formally, the equation looks like this: x = 2>p or x - 3 • p. 

You can remove the multiplication symbol in algebraic expressions when using a combina- 
tion of letters and numbers. Therefore, the equation x = 3 x p can also be written x - 3p. The 
multiplication symbol is implied. It's also common to use parentheses or the dot multiplica- 
tion symbol, •, to indicate multiplication. Most people avoid using the x because it looks so 
close to the letter x. 



When alt things are equal: Keeping 
an alqebra equation balanced 

Algebra problems are equations, which means that the quantities on both sides of the equal 
sign are equal — they're the same. For instance, 2 = 2, 1 + 1 = 2, and 3 - 1 = 2. In all these cases, 
the quantities are the same on both sides of the equal sign. To solve an algebra equation, 
you find out what the variable equals by getting it by itself on one side of the equal sign. So 
if x = 2, then x is 2 because the equal sign says so. 



80 




Part III: Making the Most of Math: Arithmetic Skills 
&£* 

You can perform any calculation on either side of an equation as long as you do it to both 
sides of the equation. That keeps the equation equal. 

You can also combine like terms — terms that have matching variables — when operating 
on algebraic expressions: 3x + 3x = (3 + 3)x = 6x, However, 3x + 3y doesn't equal 6xy, nor 
does x 2 + x 3 = x 5 (see the section "Explaining exponents in algebra," later in this chapter, to 
find out more about algebra involving exponents). 

Solving one-step equations involving addition and subtraction 

If x + 1 = 2, then x must be 1, because only 1 added to 1 is 2. So far, so simple, so good. But 
what if the equation is a little more complicated? 

x + 47,432 = 50,000 

To solve the problem, you need to isolate x on one side of the equal sign. To get that job 
done, move any other numbers on the x side of the equal sign to the other side of the 
equal sign. 

By looking at the x side of the equation, you can see that it's an addition problem. To move 
the number on the x side to the opposite side, you have to perform the inverse operation. 
The inverse operation of addition is subtraction. (For a full rundown on inverse operations, 
check out the earlier section "Operations: What You Do to Numbers.") So to move 47,432 
from the x side to the non-x side of the equation, simply subtract that number from both 
sides: 

x + 47,432 - 47,432 = 50,000 - 47,432 

Performing these operations removes the 47,432 from the x side of the equation (47,432 - 
47,432 = 0, so that side of the equation is x + or simply x~) and gives you 2,568 on the non-x 
side of the equation (50,000 - 47,432 = 2,568). You're left with the final answer: x = 2,568 

To double-check that this answer is correct, you can plug your answer into the original 
problem: 

x + 47,432 = 50,000 
2,568 + 47,432 = 50,000 



0tm 




If you plug the answer in and it doesn't work, you've made an error in your calculations. 
Start again; remember that you're trying to isolate x on one side of the equation. 

Multiplying and dividing 

In multiplication and division, if the two terms being operated on (on either side of the 
equal sign) are both positive numbers or both negative numbers, the answer will be a posi- 
tive number. If one number is negative and the other is positive, the answer will be negative. 

To solve the problem -6x = 36 (don't forget, 6x is the same thing as 6 x x), you need to iso- 
late x. So perform an inverse operation; the inverse operation of multiplication is division. 
Division in algebra is usually represented with a fraction bar: 



-6x 
-6 



36 
-6 
-6 



The answer is a negative number because the two terms, 36 and -6, have different signs. 



Chapter 7: Mathematics Knowledge and Operations £ / 



Solving multistep equations 

Not all algebra problems have one-step solutions. (That would be too easy, and you 
wouldn't sweat nearly as much.) Solving algebra problems on the ASVAB often requires you 
to perform several steps. 

An example of a multistep equation is one in which x shows up on both sides of the equal 
sign. Then you have to get rid of x on one side of the equation by moving x terms from one 
side to the other. You do this by performing the inverse operation. 

Suppose you want to solve this equation: 3x + 3 = 9 + x. To remove the x from one side of 
the equation, perform the inverse operation. The right side of the equation adds an x, so 
subtract x from both sides of the equation: 

3x + 3-x = 9+x-x 
3x+3-lx = 9 + 
2x + 3 = 9 



&£* 




To get the x term, 2x, by itself, subtract 3 from each side of the equation: 

2x +3-3 = 9-3 
2x=6 

Divide both sides of the equation by 2 to isolate x: 

2x_6 
2 2 
x = 3 

When you have a variable by itself, such as x, it's always equal to 1 times that variable (or 
one of that variable), like lx, even if the 1 isn't written out. In fact, any number is equal to 1 
times itself, so you could also say 2 = 2x1. 



.tfABE* 




Explaining exponents in alqebva 

Exponents are an easy way to show that a number is to be multiplied by itself a certain 
number of times. For example, 5 2 is the same as 5 x 5, and y 3 is the same as yxyxy. The 
number or variable that's multiplied by itself is called the base, and the number or variable 
showing how many times it is to be multiplied by itself is called the exponent or power. 

Here are important rules when working with exponents in algebra: 

j-" Any base raised to the power of one equals itself: x 1 = x 

f Any base raised to the zero power (except 0) equals 1: x° = 1 

v* To multiply terms with the same base, add the exponents: x 2 (x 3 ) = x 2+3 = x 5 

So if x = 2, then 2 2 = 4 and 2 3 = 8, and 4(8) = 32. That's the same as 2 5 = 32, which 
equals 2x2x2x2x2. 

v* To divide terms with the same base, subtract the exponents: x 5 * x 2 = x 5-2 = x 3 

J-" If a base has a negative exponent, it's equal to its reciprocal (inverse) with a positive 

-i 1 
exponent: x = -v 

x 

e* When a product has an exponent, each factor is raised to that power: (xy) 3 = x 3 y 3 



£2 Part " l: Making tne Most of Math: Arithmetic Skills 

A step back: Factoring algebra expressions 
to find original numbers 

Now and then, the ASVAB gives you a product (the answer to a multiplication problem), 
and you have to find the original numbers that were multiplied together to produce that 
product. This process is called factoring. You use factors when you combine like terms and 
add fractions. 

Putting out the greatest common factor 

Your task may be to pull out the greatest common factor from two or more terms. Take, for 
example, this product: Axy + 2x 2 . To factor this product, follow these steps: 

1. Find the greatest common factor — the highest number that evenly divides all the 
terms in the expression. 

Look at both the constants (numbers) and variables. In this case, the highest number 
that divides into 4 and 2 is 2. And the highest variable that divides into both xy and x 2 
is x. Take what you know to this point, and you can see that the greatest common 
factor is 2x. 

2. Divide both terms in the expression by the greatest common factor. 

When you divide Axy and 2x 2 by 2x, the resulting terms are 2y + x. 

3. Multiply the entire expression (from Step 2) by the greatest common factor (from 
Step 1) to set the expression equal to its original value. 

Doing so produces 2x(2y + x). 

Factoring a three-term equation (x 2 + bx + c) 

Time to try something a little more complicated: factoring a trinomial (an expression with 
three terms). Suppose you start with x 2 -\2x + 20. Follow these steps: 

1. Find the factors of the first term of the trinomial. 

The factors of the first term, x 2 , are x and x (x ■ x = x 2 ). Put those factors (x and x) on 
the left side of two sets of parentheses: (x )(x ). 

2. Determine whether the parentheses will contain positive or negative signs. 

You can see that the last term in the trinomial (+20) has a plus sign. That means the 
signs in the parentheses must be either both plus signs or both minus signs. (Why? 
Because two positive numbers multiplied equals a positive number, and two negative 
numbers multiplied equals a positive number, but a negative number times positive 
number equals a negative number.) 

Because the second term (-12x) is a negative number, both of the factors must be neg- 
ative: (x - )(x - ). 

3. Find the two numbers that go into the right sides of the parentheses. 

This part can be tricky. The factors of the third term, when added or subtracted 
together, must equal the second term of the trinomial. 

In this example, the third term is 20 and the second term is -12x. You need to find the 
factors of 20 (the third term) that add to give you -12. The two factors you want are -2 
and -10, because -2 x -10 = 20 (the third term) and -2 + -10 = -12 (the second term). 
Plug in these numbers: (x - 2)(x - 10) 

Thus, the factors of x 2 - 12x + 20 are (x - 2) and (x - 10). 



Chapter 7: Mathematics Knowledge and Operations Oj 



cjABE* 




«>*5!? 




Making alphabet soup: Solving 
quadratic equations 

So what's a quadratic equation? Sounds a little scary, huh? The Mathematics Knowledge 
subtest may ask you to solve one of these equations, but have no fear. You've come to the 
right place. 

A quadratic equation is an equation that includes the square of a variable. The exponent in 
these equations is never higher than 2 (because it would then no longer be the square of an 
unknown but a cube or something else). Here are some examples of quadratic equations: 

u* x 2 - Ax = -A 
(-" 2x 2 = x + 6 
V x 2 = 36 

Simple quadratic equations (those that consist of just one squared term and a number) can 
be solved by using the square root rule: 

If x 2 = k, then x = ±*Jk, as long as k isn't a negative number. 

Remember to include the ± sign, which indicates the answer is a positive or negative 
number. Take the following simple quadratic equation: ly 2 = 28. 

First get rid of the pesky 7 by dividing both sides by 7: y 2 = 4. Using the square root rule, 
take the square root of both sides of the equation. You know that ^jy 2 = y and >/4 = ±2, so 

y = ±2. 

When you're solving a complex quadratic equation, you put all the terms on one side of the 
equal sign, making the equation equal zero. In other words, get the quadratic equation into 
this form: ax 2 + bx + c = 0, where a, b, and c are numbers and x is unknown. Take a look at 
the following equation: x 2 - 2x = 15. You can convert this equation to standard form by sub- 
tracting 15 from both sides of the equation: x 2 - 2x - 15 = 0. 

The most efficient way to solve most quadratic equations is by factoring the equation and 
then setting each separate factor equal to zero. See the section "A step back: Factoring 
algebra expressions to find original numbers" earlier in this chapter for info on factoring. 

Look at the factored equation: 

x 2 -2x-15 = 
(x-5)(x + 3) = 

For the left side of the equation to equal zero, one of the quantities in parentheses has to 
equal zero (because zero times any number equals zero). That means you can split the 
equation in two, setting each factor equal to zero: 



x-5 = 
x = 5 



or x + 3 = 

x = -3 



The solution forx 2 - 2x- 15 is x = 5 or -3. 



$A Part III: Making the Most of Math: Arithmetic Skills 



Alt math isn't created equal: Solving inequalities 

Some algebra problems state that two quantities aren't equal to each other; thus, they're 
inequalities. In an inequality, the first number is either greater than or less than the second. 

Just like with equations, the solution to an inequality is a value that makes the inequality 
true. For the most part, you solve inequalities the same as you would solve a normal equa- 
tion. There are some facts of inequality life you need to keep in mind, however. Short and 
sweet, here they are: 

v* Negative numbers are less than zero. 

j-" Zero is less than positive numbers but greater than negative numbers. 

J-" Positive numbers are greater than zero. 

A regular algebraic equation includes the equal sign (=), because the very basis of the equa- 
tion is that one side of the equation must equal the other. Quite the opposite is true with 
inequalities, and they have their own special symbols, used to express the differences: 

V ^ means does not equal in the way that 3 does not equal 4, or 3 ^ 4. 

J-" > means greater than in the way that 4 is greater than 3, or 4 > 3. 

v* < means less than in the way the 3 is less than 4, or 3 < 4. 

v* < means less than or equal to in the way that x may be less than or equal to 4, or x < 4. 

v* > means greater than or equal to in the way that x may be greater than or equal to 3, 
orx> 3. 

To solve an inequality, you follow the same rules as you would for solving any other equa- 
tion. For example, check out this inequality: 3 + x > 4 

To solve it, simply isolate x by subtracting 3 from both sides of the equation: 

3+x-3>4-3 
x>l 



*J*BE* 




Therefore, 1 or any number greater than 1 makes this inequality true. 

The only special rule for inequalities takes effect when you multiply or divide both sides of 
the inequality by a negative number. In that case, the inequality sign is reversed. So if you 
multiply both sides of the inequality 3 < 4 by -4, your answer is -12 > -16. And if you divide 
both sides of -2x < 14 by -2, your answer is x > -7. 



Looking at Math from a different Anqle: 
Geometry ReVieu) 

Geometry is the branch of mathematics that makes grown adults cry — end of discussion. 
What? You want a more specific explanation of geometry than that? Okay, geometry is the 
branch of mathematics concerned with measuring things and defining the properties of and 
relationships among shapes, lines, points, angles, and other such objects. Hey, don't blame 
me; you asked for it. 



vtfBE/? 




Chapter 7: Mathematics Knowledge and Operations o5 



Before you read any further, you should note a few things: 

f Arcs of a circle and angles are measured in degrees and (not very often) in minutes or 
even seconds; 1 degree equals 60 minutes, and 1 minute equals 60 seconds. 

v A circle has 360 degrees (360°). Any arc that isn't a complete circle measures less 
than 360°. 

j"* A quadrilateral (a shape with four sides, such as a square or rectangle) has angles that 
add up to 360°. 



Outlining angles 



Angles are formed when two lines intersect (cross) at a point. Angles are measured in 
degrees. The greater the number of degrees, the wider the angle is: 

\^ A straight line is 180°. 

*>* A right angle is exactly 90°. 

V An acute angle is more than 0° but less than 90°. 

(<«* An obtuse angle is more than 90° but less than 180°. 

v Complementary angles are two angles that equal 90° when added together. 

J"* Supplementary angles are two angles that equal 180° when added together. 

Take a look at the different types of angles in Figure 7-1. 



Straight Line 



90° 



Right Angle 



Acute Angle 



Figure 7-1: 

Types of 
angles. Obtuse Angle 




Complementary 
Angles 



Supplementary 
Angles 



Pointing out triangle types 



A triangle consists of three straight sides whose three angles always add up to 180°. 
Triangles can be classified according to the relationship among their angles or the relation- 
ship among their sides: 

c" Isosceles triangle: An isosceles triangle has two equal sides; the angles opposite the 
equal sides are also equal. 

V Equilateral triangle: An equilateral triangle has three equal sides; each of the angles 
measures 60°. 



§f(J Part III: Making the Most of Math: Arithmetic Skills 



SjABEft 




J*" Right triangle: A right triangle has one right angle (90°); therefore, the remaining two 
angles are complementary (add up to 90°). The side opposite the right angle is called 
the hypotenuse, which is the longest side of a right triangle. The other two sides are 
called legs. 

The Pythagorean theorem states that if you know the lengths of two sides of a right tri- 
angle, you can determine the length of the third side using the formula a 2 + b 2 = c 2 , 
where a and b represent the legs and c is the hypotenuse. 

Check out Figure 7-2 to see what these triangles look like. 



Figure 7-2: 

Isosceles, 

equilateral, 

and right 

triangles. 




Isosceles Triangle 

If sides a and care equal, 
then angles 1 and 2 are equal. 




Equilateral Triangle 

Sides a, b, care equal. 
Angles 1, 2,3 are equal. 



hypotenuse 




Right Triangle 
a 2 +b 2 =c 2 



«*BEff 




You can find the perimeter — the distance around a shape — of a triangle by adding 
together the lengths of the three sides. The area — the space within a shape — of a triangle 
is one-half the product of the base (the bottom or the length) and the height (the tallest 
point of the triangle), or A = -^bh. 



Back to square one: Quadrilaterals 

Quadrilaterals — shapes with four sides — all contain angles totaling 360°. Many types of 
quadrilaterals exist (see Figure 7-3): 

v* Squares have four sides of equal length, and all the angles are right angles. 

u* Rectangles have all (four) right angles. 

v* Parallelograms have opposite sides that are parallel, and their opposite sides and 
angles are equal. The angles don't have to be right angles. 

k" Rhombuses have four sides of equal length, but the angles don't have to be right angles. 

u* Trapezoids have exactly two sides that are parallel. 









Figure 7-3: 

Types of 
quadri- 
laterals. 


J 

1 


L 

r 









Rectangle 



Parallelogram 



Rhombus 



Trapezoid 



WER 




To determine the perimeter of a quadrilateral, simply add the lengths of all the sides. You can 
write the formula for the perimeter of a rectangle as P = 2(1 + ui) or P =21 + 2w, where / is length 
and w is width. And to figure the area of a rectangle, including squares, multiply length times 
width: A = Iw. 



Chapter 7: Mathematics Knowledge and Operations £ 7 



men 




Going around in circles 



A circle is formed when the points of a closed line are all located an equal distance from its 
center. A circle always has 360°. Here are some key circle terms (see Figure 7-4, which 
shows the parts of a circle): 



Figure 7-4: 

The parts of 
a circle. 



u* Circumference (C): The closed line of a circle ■ 
circle — is called its circumference. 



that is, the distance around the 



is* Radius (r): The radius of a circle is the measurement from the center of the circle to 
any point on the circumference of the circle. 

v* Diameter (d): The diameter of the circle is measured as a line passing through the 
center of the circle, from a point on one side of the circle all the way to a point on the 
other side of the circle. 

The diameter of a circle is always twice as long as the radius of a circle: d = 2r. 



..^c^erence^ 




jcJABE/? 




Navigating the circumference 

To measure the circumference of a circle, use the number pi (71). Although n is a lengthy 
number, it's generally rounded to 3.14 or 4p If you round n so you can solve a problem, the 

equal sign isn't used because the answer isn't equal to the actual length. The approximation 
symbol («) is used. 

The formula for circumference is circumference = n x diameter, or C = nd. Because the 
radius of a circle is half its diameter, you can also use the radius to determine the circum- 
ference of a circle. Here's the formula: C = 2nr. 

Suppose that you know that the pie you just baked has a diameter of 9 inches. You can 
determine its circumference by using the circumference formula: 

C= nd 
C=3.14x9 
C= 28.26 inches 



j^5£* Mapping out the area 




Determining the area of a circle also requires the use of 71. Area 
cle's radius, or A = nr 2 . 



n x the square of the cir- 



$£[ Part III: Making the Most of Math: Arithmetic Skills 



To determine the area of a 9-inch-diameter pie, multiply n by the square of 4.5. Why 4.5 and 
not 9? Remember, the radius is always half the diameter, and the diameter is 9 inches. 

A = nr 2 

^-3.14x4.5 2 
A « 3.14 x (4.5x4.5) 
A» 3.14x20.25 
A ~ 63.585 inches 



Fitting 'er up: Calculating Volume 

Volume is the space a solid (three-dimensional) shape takes up. You can think of volume as 
how much a shape would hold if you poured water into it. Volume is measured in cubic 
units. The formula for finding volume depends on the object: 

v* Boxes: For rectangular objects, you multiply length x width (depth) x height. This is 
possible because the length, width, and height of a rectangle are consistent throughout 
the whole shape. The formula looks like this: V = Iwh. 

For a box that measures 5 feet long, 6 feet deep, and 2 feet tall, you simply multiply 
5 x 6 x 2 to arrive at a volume of 60 cubic feet, or 60 ft. 3 

v* Cylinders: A cylinder has two circles for its bases. The volume equals n x the radius 
squared x height, or V = n^h. 

Basically, you're multiplying the area of the cylinder's circular base (A = nr 2 ) times the 
height (h) of the cylinder. For a cylinder that has a radius of 2 inches and a height of 10 
inches, here's the deal: Multiply the value of n (approximately 3.14) times 4 (which is 
the radius squared) times 10, or 3.14 x 4 x 10 = 125.6 cubic inches. 



Test-Taking Techniques far \lour 
Mathematical Journey 



As with most of the other subtests on the ASVAB, guessing on the Mathematics Knowledge 
subtest doesn't count against you. So scribble in an answer, any answer, on your answer sheet 
because, if you don't, your chances of getting that answer right are zero. But if you take a shot 
at it, your chances increase to 25%, or 1 in 4. In the following sections, you find some tips that 
can help you improve those odds, even when you don't know how to solve the problem. 

If you're not confident in your math skills, you may want to invest some extra study time. 
Check out Algebra For Dummies by Mary Jane Sterling, Geometry For Dummies by Mark 
Ryan, and SAT II Math For Dummies by Scott Hatch — all published by Wiley. 



Knowing what the question is asking 

The Mathematics Knowledge subtest presents the questions as straightforward math prob- 
lems, not word problems, so knowing what the question is asking you to do is relatively 
easy. However, reading each question carefully, paying particular attention to plus (+) and 
minus (-) signs (which can really change the answer) is still important. Finally, make sure 
you do all the calculations needed to produce the correct answer. Check out this example: 



Chapter 7: Mathematics Knowledge and Operations ft Q 






Find the value of V81 2 . 

(A) 9 

(B) 18 

(C) 81 

(D) 6,561 

If you're in a hurry, you may put 9 down as an answer because you remember that the 
square root of 81 is 9. Or in a rush, you could multiply 9 (the square root of 81) by 2 instead 
of squaring it, as the exponent indicates you should. Or you may just multiply 81 by 81 to 
get 6,561 without remembering that you also need to then find the square root, which gives 
you the correct answer, Choice (C). So make sure you perform all the operations needed 
(and that you perform the correct operations) to find the right answer. Here, noticing that 
you're both squaring 81 and taking the square root of 81 2 should make it easy for you to rec- 
ognize that the answer is actually just 81, without having to work out the multiplication. 



Figuring out what you're solving for 

Right out of the gate, read the question carefully. Some questions can seem out of your 
league at first glance, but if you look at them again, a light may go on in your brain. Suppose 
you get this question: 

2 1 
Solve for s:s = F x-^. 
d z 

(A)2l 
(B) 2 
(C)I 

At first glance, you may think, "Oh, no! Solve for an unknown, s. I don't remember how to do 
that!" But if you look at the question again, you may see that you're not solving for s at all. 

2 1 9 

You're simply multiplying a fraction. So you take £■ times -^ and arrive at -^-, but you should 

, " d Z 10 

reduce that fraction to get ^-- The correct answer is Choice (C). 

D 

Solving what you can and guessing the rest 

Sometimes a problem requires multiple operations for you to arrive at the correct answer. 
If you don't know how to do all of the operations, don't give up. You can still narrow down 
your choices by doing what you can. 



Suppose this question confronts you: 
What's the value of (0.03) 3 ? 



(A) 0.0027 

(B) 0.06 

(C) 0.000027 

(D) 0.0009 



(}Q Part III: Making the Most of Math: Arithmetic Skills 



Say you don't remember how to multiply decimals. All isn't lost! If you remember how to 
use exponents, you know that you have to multiply 0.03 x 0.03 x 0.03. So if you simplify the 
problem and just multiply 3x3x3, without worrying about those pesky zeros, your answer 
will have a 27 in it. With this pearl of wisdom in mind, you can see that Choice (B), which 
adds 0.03 to 0.03, is wrong. It also means that Choice (D), which multiplies 0.03 and 0.03, is 
wrong. Now you have two possible answers, and you've improved your chances of guessing 
the right one to 50 percent! Multiply 3 x 3 x 3 to get 27, and don't forget to put the decimal 
points back in. You have six places to make up, so move the decimal from 27 six places to 
the left to get 0.000027. The correct answer is Choice (C). 





Using the process of elimination 

Another method for when you run into questions and draw a total blank is to plug the possi- 
ble answers into the equation and see which one works. Say the following problem is staring 
you right in the eyes: 

Solve for x: x - 5 = 32 

(A) x = 5 

(B) x = 32 

(C) x = -32 

(D) x = 37 

If you're totally stumped and can't think of any possible way of approaching this problem, 
simply plugging in each of the four answers to see which one is correct is your best bet. 

v* Answer (A): 5 - 5 = 32, which you know is wrong 
v* Answer (B): 32 - 5 = 32, which is wrong 
v* Answer (Q: -32 - 5 = 32, which is wrong 
v* Answer (D): 37 - 5 = 32, which is correct 

Don't forget that plugging in all the answers is time-consuming, so save this procedure until 
you've answered all the problems you can answer. If you're taking the computer version, 
you can't skip a question, so remember to budget your time wisely. If you don't have much 
time, just make a guess and move on. You may be able to solve the next question easily. 



Chapter 8 

Reasoning with Arithmetic: 
Math Word Problems 



In This Chapter 

Solving life's little (math) problems 
Multiplying your chances for a better score 



Wow many miles per gallon does your brand-new SUV get? How long does it take to go 
over the river and through the woods to Grandmother's house? How much wood 
could a woodchuck chuck? These are examples of everyday questions that can be answered 
by arithmetic reasoning. (Okay, maybe the woodchuck situation doesn't happen every day.) 

The rest of the world calls this type of question math word problems. The ASVAB calls them 
Arithmetic Reasoning. No matter what they're called, these problems help you apply mathe- 
matical principles to the real world (at least the real world according to the people who 
think up word problems). Your job is to read a word problem, determine what the question 
asks, and select the correct answer. 

Arithmetic Reasoning is an important part of the Armed Forces Qualification Test (AFQT) 
score, which is used to determine your general qualification for enlistment in all the service 
branches (see Chapter 1 for more information). Also, certain military jobs require that you 
score well on this subtest (see Appendix A). 

The test administrator will supply you with scratch paper and a trusty number two pencil, 
but one thing he or she won't give you (or even let you bring) is a calculator. You can 
use your paper and lead to clarify the data, write formulas, and mathematically solve the 
problem. You can even use them to draw pretty pictures to help you understand the prob- 
lem. Don't get too artistic, though — you have only 36 minutes to answer 30 questions if 
you're taking the paper version and 39 minutes to answer 16 questions if you're taking the 
CAT-ASVAB. 

To do well on the Arithmetic Reasoning subtest, you have to remember that there are two 
parts: arithmetic and reasoning. You usually have to use both of these skills for each prob- 
lem. The arithmetic part comes in when you have to perform mathematical operations such 
as addition, subtraction, multiplication, and division. The reasoning comes in when you 
figure out which numbers to use in your calculations. In other words, Arithmetic Reasoning 
tests how you apply your ability to perform calculations to real-life problems. If you slept 
through high school math, don't worry. This chapter helps you decipher these math prob- 
lems, focusing on the reasoning part. For additional info on the arithmetic, flip to Chapter 7. 



^2 Part lll: Makin 9 the Most of Math: Arithmetic Skills 



Tackling the Real World of Word Problems 

Test-takers often waste a lot of time reading and rereading word problems as if the answer 
might reveal itself to them by some miracle; however, correctly solving math word prob- 
lems requires you to perform a series of organized steps: 

1. Read the problem completely. 

2. Figure out what the question is asking. 

3. Dig out the relevant facts. 

4. Set up one or more equations to arrive at a solution and then solve the problem. 

5. Review your answer. 

I cover these steps in detail throughout this section. 

Reading the entire problem 

The first step in solving a word problem is reading the entire problem to discover what it's 
all about. Try forming a picture about the problem in your mind or — better yet — draw a 
sketch of the problem on your scratch paper. Ask yourself whether you've ever seen a prob- 
lem like this before. If so, what's similar about it, and what did you do to solve it in the past? 



«,^P L E 





As plain as the nose on a fly: Figuring 
out What the question is asking 

The second and most important step in solving a word problem is to determine exactly 
what the question is asking. Sometimes the question is asked directly. At other times, iden- 
tifying the actual question may be a little more difficult. Suppose you're asked the following 
question: 

What's the volume of a cardboard box measuring 12 inches long by 14 inches wide by 
10 inches tall? 

(A) 52 cubic inches 

(B) 88 cubic inches 

(C) 120 cubic inches 

(D) 1,680 cubic inches 

The problem directly asks you to determine the volume of a cardboard box. Recall from 
your high school algebra and geometry classes that the volume of a rectangular container is 
length x width x height, or V = Iwh. So 12 x 14 x 10 = 1,680. The correct answer is Choice (D). 

Now take a look at the next example: 

How many cubic inches of sand can a cardboard box measuring 12 inches long by 14 inches 
wide by 10 inches tall contain? 

(A) 52 cubic inches 

(B) 88 cubic inches 

(C) 120 cubic inches 

(D) 1,680 cubic inches 



Chapter 8: Reasoning with Arithmetic: Math Word Problems Q A 




This is the same problem, but the question you need to answer isn't as directly stated. 
Therefore, you have to use clues embedded in the problem to figure out what the actual 
question is. Would figuring out the perimeter of the box help you with this question? Nope. 
Would figuring out the area of one side of the box help you? Nope — you're not painting the 
box; you're filling it. The question wants you to determine the volume of the container. 

Clue words can be a big help when trying to figure out which question is being asked. Look 
for the following clue words: 

u* Addition: Sum, total, in all, perimeter, increased by, combined, added 

v* Division: Share, distribute, ratio, quotient, average, per, out of, percent 

u* Equals: Is, was, are, were, amounts to 

u* Multiplication: Product, total, area, cubic, times, multiplied by, of 

u* Subtraction: Difference, how much more, exceed, less than, fewer than, decreased 



Digging for the facts 




After you figure out which question you're answering in the first place, the next step is to 
figure out which data is necessary to solve the problem and which data is extra. Start by 
identifying all the information and variables in the problem and listing them on your scratch 
paper. Make sure you attach units of measurement contained in the problem (miles, feet, 
inches, gallons, quarts, and so on). After you've made a list of the facts, try to eliminate 
those facts that aren't relevant to the question. Look at the following example: 

To raise money for the school yearbook project, Tom sold 15 candy bars, Becky sold 
12 candy bars, Debbie sold 17 candy bars, and Jane sold the most at 50. How many 
candy bars were sold by the girls? 

The list of facts may look something like this: 

Tom =15 bars 
Becky =12 bars 
Debbie =17 bars 
Jane = 50 bars 
? = total sold by the girls 

Because the question is the total number of candy bars sold by the girls, the number of bars 
sold by Tom isn't relevant to the problem and can be scratched off the list. Just add the 
remaining bars from your list. The answer is 79. 




Setting up the problem and Working 
your Way to the answer 

You need to decide how the problem can be solved and then use your math skills to arrive 
at a solution. For instance, a question may ask the following: 

Joan just turned 37. For 12 years, she's dreamed of traveling to Key West to become a beach 
bum. To finance this dream, she needs to save a total of $15,000. How much does Joan need 
to save each year if she wants to become a beach bum by her 40th birthday? 

Write down, in mathematical terms, what the question is asking you to determine. Because 
the question is asking how much money Joan needs to save per year to reach $15,000, you 



(fA Part III: Making the Most of Math: Arithmetic Skills 



can say y (years Joan has to save) x m (money she needs to save each year) = $15,000. Or to 
put it more mathematically, 

ym = $15,000 

You don't know the value of m (yet) — that's the unknown you're asked to find. But you can 
find out the value of y — the number of years Joan has to save. If she's 37 and wants to be a 
beach bum by the time she's 40, she has 3 years to save. So now the formula looks like this: 

3m = 15,000 

To isolate the unknown on one side of the equation, you simply divide each side by 3, so 
3m * 3 = 15,000 * 3. (If you don't remember how to isolate unknowns, flip to Chapter 7.) 
Therefore, your answer is 

m = 5,000 

Joan needs to save $5,000 each year for 3 years to reach her goal of $15,000 by the time 
she's 40. You may be tempted to include the 12 years Joan has been dreaming of this trip 
in your formula. This number was put into the problem as a distraction. It has no bearing on 
solving the problem. 




RerieWinq yow answer 



Before marking your answer sheet or punching in that choice on the computer, review your 
answer to make sure it makes sense. Review by asking yourself the following questions: 

v* Does your solution seem probable? Use your common sense. If you determine that a 
12- x 16-foot roof is covered in only 12 shingles, you've probably made a mistake in 
your calculations. 

J-" Does it answer the question asked? Reread the problem. For example, if a question 
asks you to calculate the number of trees remaining after 10 percent of the total was 
cut down, the correct answer wouldn't be 10 percent of the trees but rather the 90 per- 
cent still standing. 

v* Are you sure? Double-check your answer. Those tricky test-makers often supply false 
answers, which are very, very close to the correct answer. 

J*" Is your answer expressed using the same units of measurement as used in the prob- 
lem? A question may ask how many cubic feet of concrete is required to cover a drive- 
way. Your answer in cubic yards would have to be converted to cubic feet so you can 
select the correct answer choice. 



.flXNG. 




Although you may have been taught in school to round 5 or above up and below 5 down, 
rounding real-world problems requires a different mindset. For example, if someone needs 
2.2 cans of paint for a particular job, she really needs 3 cans of paint to make sure she has 
enough, even though you'd generally round down. And if someone gets a 15-minute break 
for every 4 hours of work but works only 7 hours, he'd get only one break, even though 7 
divided by 4 equals 1.75, which is generally rounded up to 2. 



You may find that the solution you arrived at doesn't fit the facts presented in the problem. 
If this is the case, back up and go through the steps again until you arrive at an answer that 
seems probable. 



Chapter 8: Reasoning with Arithmetic: Math Word Problems Qjy 



The Guessing Game: Putting Reason 
in l/aur Guessing Strategy 




Guessing wrong on any of the ASVAB subtests doesn't count against you. If you don't guess, 
your chances of getting that answer right are zero, but if you take a shot at it, your chances 
increase to 25%, or 1 in 4. Eliminate two wrong answers, and you have a 50-50 shot. 

If you're taking the paper version of the ASVAB, you can always skip the tough questions 
and come back to them after you've finished the easier ones. If you're taking the computer- 
ized version of the ASVAB, the software won't let you skip questions, so you need to make 
your guess right then and there. 

If you're taking the paper version of the test and elect to skip questions until later, make 
sure you mark the next answer in the correct space on the answer sheet. Otherwise, you 
may wind up wearing out the eraser on your pencil when you discover your error at the end 
of the test. Or even worse, you may not notice the error and wind up getting several 
answers wrong because you mismarked your answer sheet. 




Using the process of elimination 

Guessing doesn't always mean "pick an answer, any answer." You can increase your chances 
of picking the right answer by eliminating answers that can't be right. To eliminate some 
obvious wrong answers, you can do the following: 

v* Make sure the answer is realistic in relation to the question asked. For example, if a 
question asks you how much water would be required to fill a child's wading pool, 
17,000 gallons isn't a realistic answer. You can save time by eliminating this potential 
answer choice immediately. 

v* Pay attention to units of measurement. If a question asks how many feet of rope you'll 
need, answer choices listed in inches or cubic feet are probably incorrect. 

v* Consider easier answer choices first. Remember, you're not allowed to use a calcula- 
tor on the ASVAB, so math answers that you'd arrive at by using complicated formulas 
are probably not correct. 




Solving What you can and guessing the rest 

Sometimes you may know how to solve part of a problem but not all of it. If you don't know 
how to do all the calculations — or don't have time for them — don't give up. You can still 
narrow down your choices by doing what you can. Here's how partially solving problems 
can help: 

u* When adding mixed numbers (a whole number and a fraction), add the whole-number 
parts first; then immediately eliminate answer choices that are too low. Or when 
adding lengths, add full feet first and cross off choices that are too small, even before 
considering the inches. 

v* Multiply just the last digits and cross off all answers that don't end in the right num- 
bers (assuming the answers aren't rounded). 



(ffi Part III: Making the Most of Math: Arithmetic Skills 





Making use of the ansuter choices 

If you're stuck on a particular problem, sometimes plugging possible answers into an equa- 
tion can help you find the right answer. Here's how using the answer choices can improve 
your guessing: 

v* Plug in each remaining answer choice until you get the right answer. Plugging in all 
the answer choices is time-consuming, so make sure you eliminate obviously wrong 
choices first. 

v* Estimate and plug in numbers that involve easy mental calculations. For instance, if 
Choice (A) is 9 and Choice (B) is 12, plug in 10 and solve the equation in your head. 
Think about whether the right answer has to be higher or lower than 10, and choose 
from there. 

v* Using a little logic, do calculations with an obviously wrong answer choice. 

Sometimes a wrong answer choice — especially one that differs drastically from the 
other answers — represents an intermediate step in the calculations, so you can use it 
to solve the problem. For instance, take this example: 

A security guard walks the equivalent of six city blocks when he makes a circuit around the 
building. If he walks at a pace of eight city blocks every 30 minutes, how long will it take him 
to complete a circuit around the building, assuming he doesn't run into any thieves? 

(A) 20.00 minutes 

(B) 3.75 minutes 

(C) 22.50 minutes 

(D) 24.00 minutes 

Choice (B) is obviously way too low to be the right answer, but it would be a logical guess 
for the security guard's rate for a single lap. Multiply 3.75 minutes/block by 6 blocks, and 
you probably have a good candidate for the right answer — 22.50 minutes, Choice (C). 



Chapter 9 

Brother, Can You Spare an Equation? 
Arithmetic Practice Questions 



In This Chapter 

Putting your mathematical reasoning to the test 
Practicing your arithmetic skills 



1 

m t's time to put those calculators, pocket computers, and abacuses away — the actual 
«C ASVAB doesn't allow you to use anything except paper, pencil, and a winning personality 
to solve the math problems. You should use the old-fashioned way on these practice ques- 
tions, too. Tell you what: Instead of a number two pencil, you can use a number three or 
four instead. Heck, you can even use a pen. See? I'm easy. No pressure here! 

You may be tired of hearing this by now (if you've taken any of the tests in this book), but 
the math subtests of the ASVAB are very important because they're used in calculating 
your AFQT score, which makes or breaks you as far as your eligibility to join the military. 
Additionally, every technical-oriented job in the military requires good to excellent math 
scores. Chapter 1 explains the AFQT in more detail. 

On the paper version of the ASVAB (and on the full-length practice tests later in this book), 
you see 30 Arithmetic Reasoning questions and 25 Mathematics Knowledge questions. On 
the CAT-ASVAB, you get 16 questions on each math subtest. In this chapter, you only have 
to sweat through 25 total questions. 



Arithmetic Reasoning (Math Word Problems ) 
Practice Questions 

Arithmetic Reasoning questions are math problems expressed in a story format. Your goal 
is to determine what the question is asking by picking out the relevant factors needed to 
solve the problem, set up mathematical equations as needed, and arrive at the correct solu- 
tion. Sounds easy, right? Check out Chapter 8 for more help with Arithmetic Reasoning. 

1. If apples are on sale at 15 for $3, what's the cost of each apple? 

(A) 50 cents 

(B) 25 cents 

(C) 20 cents 

(D) 30 cents 

Divide $3 by 15. The answer is $0.20, so the correct answer is Choice (C). 



(f$ Part III: Making the Most of Math: Arithmetic Skills 



2. A noncommissioned officer challenged her platoon of 11 enlisted women to beat her record 
of performing a 26-mile training run in 4 hours. If all the enlisted women match her record, 
how many miles will they have run? 

(A) 71.5 miles 

(B) 6.5 miles 

(C) 286 miles 

(D) 312 miles 

Multiply 26 x 11. The other information in the question is irrelevant — it's there to throw 
you off. The correct answer is Choice (C). You can immediately eliminate Choice (B) 
because it isn't a reasonable answer. Identifying unreasonable answers (through the pro- 
cess of elimination) can help you choose the correct answer choice faster. 

3. Margaret gets her hair cut and colored at an expensive salon in town. She's expected to 
leave a 15% tip for services. If a haircut is $45 and a color treatment is $150, how much of a 
tip should Margaret leave? 

(A) $22.50 

(B) $29.25 

(C) $20.00 

(D) $224.25 

Add 45 and 150 to get the cost of the services ($45 + $150 = $195); then multiply the answer 
by 0.15 (15%) to find the tip. The question asks for the amount of the tip, so the correct 
answer is Choice (B), $29.25. You can immediately eliminate Choice (D), because the 
amount is far too high to make sense. 

4. A bag of sand holds 1 cubic foot of sand. How many bags of sand are needed to fill a square 
sandbox measuring 5 feet long and 1 foot high? 

(A) 25 bags 

(B) 5 bags 

(C) 10 bags 

(D) 15 bags 

To find the volume of the sandbox, you take length times width times height (V = Iwti). Don't 
forget that the measurements are for a square sandbox, so you can assume that if the box is 
5 feet long, then it's also 5 feet wide. So 5 x 5 x 1 is 25 cubic feet. Each bag holds 1 cubic foot 
of sand, and 25 + 1 = 25. Choice (A) is the correct answer. If you were thinking answer 
Choice (B) sounded good, remember that the answer should make sense. Five cubic feet of 
sand would not fill a very large sandbox, would it? 

5. The day Samantha arrived at boot camp, the temperature reached a high of 90 degrees in 
the shade and a low of -20 at night in the barracks. What was the average temperature for 
the day? 

(A) 35 degrees 

(B) 45 degrees 

(C) 55 degrees 

(D) 62 degrees 

Add the two temperatures: 90 + (-20) = 70. Divide 70 by 2 to reach the average temperature, 35. 
Choice (A) is the correct answer. 



Chapter 9: Brother, Can You Spare an Equation? Arithmetic Practice Questions Q Q 

6. Farmer Beth has received an offer to sell her 320-acre farm for $3,000 per acre. She agrees to 
give the buyer $96,000 worth of land. What fraction of Farmer Beth's land is the buyer getting? 



(A) 


1 
4 


(B) 


1 

10 


(C) 


1 
5 


(D) 


2 
3 



The buyer's price, $96,000, divided by $3,000 (price per acre) equals 32 acres. Thirty-two 

acres divided by 320 acres (total of the farm) equals 10%, or y^, of the land. The correct 
answer is Choice (B). 

7. A large wall map is drawn so that 1 inch equals 3 miles. On the map, the distance from 

Kansas City to Denver is 192-4 inches. How far is the round trip from Kansas City to Denver 
in miles? 

(A) 192^ miles 

(B) 577^ miles 

(C) 385 miles 

(D) 1,155 miles 

Multiply 192.5 x 3 to get the distance in miles, and then double the answer to account for 
both legs of the trip. Choice (D) is the correct answer. Note: A quick approach here involves 
rounding. The distance is about 200 inches, or 400 inches round trip. Multiply that by 3, and 
you get 1,200 miles. The only choice that comes close is (D). 

8. Margaret and Julie can sell their tattoo parlor for $150,000. They plan to divide the proceeds 
according to the ratio of the money they each invested in the business. Margaret put in the 
most money at a 3:2 ratio to Julie's contribution. How much money should Julie get from 
the sale? 

(A) $50,000 

(B) $30,000 

(C) $60,000 

(D) $90,000 

According to the ratio, Margaret should get -^ of the money and Julie should get ^ of the 

money. You calculate the fractions by adding both sides of the ratio together (3 + 2 = 5) to 
determine the denominator — Margaret gets 3 parts of the total, and Julie gets 2 parts, so 
there are 5 total parts. Each side of the ratio then becomes a numerator. Multiply the total 
amount of money by the fraction representing Julie's share: Multiply $150,000 by 2, and 
then divide the answer by 5 to determine Julie's share of the money. The correct answer is 
Choice (C). 



/ 00 Part lll: Makin 9 the Most of Math: Arithmetic Skills 



9. What is the fifth number in the series 4, 8, 16, 32? 

(A) 48 

(B) 64 

(C) 96 
(D)8 

The pattern is to double each number: 4 + 4 = 8; 8 + 8 = 16; 16 + 16 = 32; so 32 + 32 = 64. The 
correct answer is Choice (B). 

10. In the military, i of an enlisted person's time is spent sleeping and eating, -^ is spent 

1 9 

standing at attention, ± is spent staying fit, and £ is spent working. The rest of the time is 
b ' 5 

spent at the enlisted person's own discretion. How many hours per day does this discre- 
tionary time amount to? 

(A) 6.0 hours 

(B) 1.6 hours 

(C) 2.4 hours 

(D) 3.2 hours 

Calculate this answer by first assigning a common denominator of 60 to all the fractions and 

adjusting the numerators accordingly: ±£, -£r, ^, and £±. Add the fractions to find out how 

60 60 60 60 

much time is allotted to all of these tasks. The total time is ^, which leaves ^ or ^ of the 

bU bU 1U 

day to the enlisted person's discretion. One-tenth of 24 hours is 2.4 hours. Therefore, 

Choice (C) is the correct answer. 

11. A designer sells a square yard of carpet for $15.00. The same carpet can be purchased at 
the carpet outlet store for $12.50. As a percentage, how much more expensive is the design- 
er's carpet? 

(A) The designer's carpet costs about 17% more than the outlet-store carpet. 

(B) The designer's carpet costs about 20% more than the outlet-store carpet. 

(C) The designer's carpet costs about 25% more than the outlet-store carpet. 

(D) The designer's carpet costs about 12% more than the outlet-store carpet. 

You want the cost of the designer's carpet in terms of the outlet-store carpet, so divide the 
difference in costs by the lower price: $15.00 - $12.50 = $2.50, and $2.50 * $12.50 = 0.20 = 20%. 
The correct answer is Choice (B) — the designer's carpet is 20% more expensive. 

12. Terry got a haircut for $32.50, a hair color for $112.20, and a manicure for $17.25. How much 
total money did she spend at the salon? 

(A) $167.45 

(B) $144.70 

(C) $161.95 

(D) $156.95 

Simply add the amounts together: $32.50 + $112.20 + $17.25 = $161.95. Choice (C) is the cor- 
rect answer. 



Chapter 9: Brother, Can You Spare an Equation? Arithmetic Practice Questions / Q / 

13. Mailing the first ounce of a letter costs $0.39, and it costs $0.24 to mail each additional 
ounce. How much does it cost to mail a 5-ounce letter? 

(A) $1.85 

(B) $1.16 

(C) $1.45 

(D) $1.35 

The first ounce costs $0.39. The next 4 ounces cost $0.24 each. Multiply $0.24 x 4 and then 
add $0.39 to determine how much mailing a 5-ounce letter costs: $0.24 x 4 = $0.96, and $0.96 + 
$0.39 = $1.35, the cost of mailing a 5-ounce letter. Choice (D) is the correct answer. 



Math Knowledge Practice Questions 



The remaining practice questions are straightforward math. You won't have to wonder how 
Terry got so much money to spend at the salon in the first place (like in the previous sec- 
tion). Remember, these questions are designed for high school level and below. You won't 
be solving equations to calculate the orbit of Mars around the sun here. 



14. Which of the following fractions is the largest? 

2 
3 



(A) 
(B) 
(C) 
(D) 



n 

16 

3 

4 



To arrive at the answer, find a common denominator that all the denominators divide into 
evenly. In this case, the common denominator is 48. Next, convert all fractions to 48ths. In 

O 1 c oo 

the case of Choice (A), multiply ^ x |§ to reach ^. Perform the same type of calculation for 

all the other fractions, figuring out what number times the denominator gives you 48, and 
then multiplying each numerator by that number; then compare numerators. The largest 

on oo oc 

numerator is the largest fraction. The other fractions are equal to ^, ^, and ^. Choice (D) 
. ., . 4o 4o 4o 

is the correct answer. 

15. What's the product of ^36 and >/49? 

(A) 1,764 

(B) 42 

(C) 13 
(D)6 

The square root of 36 is 6, and the square root of 49 is 7. The product of those two numbers 
(6 x 7) is 42. The correct answer is Choice (B). 



1 02 Part lll: Makin 9 the Most of Math: Arithmetic Skills 



16. Solve for x: 2x - 3 = x + 7. 

(A) 10 

(B) 6 

(C) 21 

(D) -10 

Isolate the x's on the left side of the equation by subtracting x from both sides: 2x - 3 - x = 
x + 7 - x, or x - 3 = 7. Continue to perform operations to isolate x: Add 3 to both sides of the 
equation to get rid of the -3 on the left: x - 3 + 3 = 7 + 3, or x = 10. The correct answer is 
Choice (A). 

17. A circle has a radius of 15 feet. What's most nearly its circumference? 

(A) 30 feet 

(B) 225 feet 

(C) 94 feet 

(D) 150 feet 

The circumference of a circle is n times the diameter; the diameter equals two times the 
radius. Therefore, 3.14 x 30 feet = 94.2 feet. The correct answer is Choice (C). 

18. At 3:00 p.m., the angle between the hands of the clock is 

(A) 90 degrees 

(B) 180 degrees 

(C) 120 degrees 

(D) 360 degrees 

At 3:00 p.m., one hand is on the 12, and the other is on the 3. This creates a right angle — a 
90-degree angle. The correct answer is Choice (A). 

19. 2 3 x2 4 = 

(A) 16 

(B) 108 

(C) 128 

(D) 148 

2 3 x 2 4 = 2 7 = 2x2x2x2x2x2x2 = 128. Choice (C) is the correct answer. 

20. Express 403,000,000,000,000 in scientific notation. 

(A) 4.03 x 10 14 

(B) 4.03 x lO" 14 

(C) 4.03 x 10 

(D) 0.43 x 10 

The correct way to write the number in scientific notation is 4.03 x 10 14 . When the exponent 
is positive, the decimal point moves said number of places to the right. When the exponent 



Chapter 9: Brother, Can You Spare an Equation? Arithmetic Practice Questions / Qj 

is negative, the decimal point moves said places to the left. The exponent in Choice (A) is a 
positive 14, which means you move the decimal point 14 places to the right. Choice (A) is 
the correct answer. 

21. Simplify Sx 2 - 3x + Axy - 9x 2 -5x- 20xy. 

(A) 5x 2 + 9xy 

(B) 8x-9x 2 

(C) -x 2 -8x-\6y 

(D) Sx + 9x 2 

8X 2 - 3x + Axy - 9x 2 -5x- 20xy = (Sx 2 - 9x t ) + (-3a: - 5x) + (Axy - 20xy) = -x 2 -8x- I6xy. The 
correct answer is Choice (C). 

22. What's the prime factorization of 90? 

(A) 2x3x5 

(B) 2 x 3 2 x 5 

(C) 2 2 x 3 2 

(D) 2 x 3 x 5 2 

When you figure which prime numbers you need to multiply together to get the original 
number, you're using prime factorization. Here, 90 = 9xl0 = 3x3x2x5 = 2x3 2 x5. The 
correct answer is Choice (B). If you don't know how to solve this problem, you can guess by 
finding the value of each answer choice. Choice (A) is 30, (B) is 90, (C) is 36, and (D) is 150. 



23 I x 2 
£6. 4 X 3 




(A)| 




0)1 




(C)I 




CD) I 




4 X 3" 


2 
12 



jr. The correct answer is Choice (C). 
o 

24. A baker has s pounds of sugar to use in baking. After she uses 50 pounds to make donuts, 
how much sugar does she have left? 

(A) s + 50 

(B) 50 -s 

(C) s - 50 

(D) s * 50 

The variable s stands for the amount of sugar the baker had before she made the donuts. 
Taking away 50 pounds, the amount of sugar used, gives you s - 50. Choice (C) is the 
correct answer. 



/ OU Part " l: Makin 9 the Most of Math: Arithmetic Skills 



25. Six pizzas are pepperoni, seven are hamburger, four are cheese, and three are "with every- 
thing." What's the probability that a randomly selected pizza is pepperoni? 



(A) 


1 
2 


(B) 


2 
5 


(C) 


3 

10 


(D) 


2 
3 



There are 20 total pizzas (6 + 7 + 4 + 3). The probability that the one chosen is a pepperoni 
pizza is -^r, or ^. The correct answer is Choice (C). 



Part IV 

The Whole Ball of Facts 
Technical Skills 



The 5 th Wave 



By Rich Tennant 



©PL^Ef-^KT 




"Sure I vJorK on diesels. "bring her around and vte'll 
pui her on the lift.* 



In this part . . . 



70 get into the military, you have to know how to read 
and how to add. But to qualify for certain military 
jobs, you also have to understand how the world works. 
Several ASVAB subtests test your knowledge of science 
and mechanics. Don't worry — I've got you covered. 

Part IV helps you review principles of science, auto and 
shop information, mechanical comprehension, principles 
of electronics, and spatial relationships. If you're not inter- 
ested in jobs that require this type of background (for a 
list of jobs that do, see Appendix A), you don't need to 
score high on these tests. But if you've set your heart on 
working on Humvees or F-18 Super Hornets, study this 
part to get you started. 



Chapter 10 

General Science 



In This Chapter 

Figuring out the scientific method 

Grasping measurements 

Studying scientific disciplines 

Using scientific strategies to improve your score 




2M s you study for this subtest, you may feel overwhelmed by facts and figures. General 
V \ Science requires a lot of straight-up memorization. You're presented with questions 
about facts you probably learned in high school in various science classes, such as health, 
Earth science, biology, and chemistry. If you don't know that Earth is the third planet from 
the sun, then all the other science knowledge you have won't help you one bit when the 
question asks, "What is the third planet from the sun?" 

Instead of trying to remember nine million individual facts, spend some time reviewing the 
general principles behind the facts. Think about how the facts relate to each other. Looking 
at the big picture is an effective learning technique. 

You have 11 minutes to answer 25 questions on the paper version of the General Science 
subtest, or you have 8 minutes to answer 16 General Science questions on the CAT-ASVAB. 
That comes out to about 26 or 30 seconds per question, so there's no time to dilly-dally. For 
the most part, you either know the answer or you don't. If you don't know the answer, you 
can always guess (check out Chapter 3 for tips on guessing on the ASVAB). 

You can relax this time around. The General Science subtest has no bearing on your Armed 
Forces Qualification Test (AFQT) score. On the other hand, your score on this subtest is 
used to calculate some of the military composite scores that are used for job qualification 
purposes (see Appendix A for more information). 

Take some time to review the facts in this chapter as a mini science lesson. If the job you 
want requires a good score on this subtest, dedicate yourself to the information in this 
chapter to boost your General Science score. You may also want to seek additional study 
time in these references to boost your science knowledge: Chemistry For Dummies by 
John T. Moore, Biology For Dummies by Donna Rae Siegfried, Astronomy For Dummies by 
Stephen P. Maran, Weather For Dummies by John D. Cox, and Physics I For Dummies and 
Physics II For Dummies by Steven Holzner, Ph.D. (all from Wiley Publishing). 

There's a Scientific Method to the Madness 

Scientists are pretty skeptical. They don't necessarily believe anything said by anyone else 
unless it's been shown to be true (time after time after time) using a process called the sci- 
entific method. Scientists know that personal and cultural biases may influence perceptions 
and interpretations of data, so they've derived a standard set of procedures and criteria to 



108 



Part IV: The Whole Ball of Facts: Technical Skills 



minimize those influences when developing a theory. Because the scientific method is prev- 
alent in all fields of science, you can expect to see a few questions about the process on the 
General Science subtest. 

Here are the usual steps to solving a problem using the scientific method: 

1. Observe some aspect of the universe. 

2. Make an educated guess (hypothesis) about why this is happening. 

3. Make predictions based on the hypothesis. 

4. Experiment and observe to test the predictions. 

If the results don't match the predictions, modify the hypothesis. 

5. Keep repeating Steps 3 and 4 until there are no discrepancies in experimentation 
and observation in relation to the hypothesis. 

When developing and testing a hypothesis, scientists are guided by a principle known as 
Ockham's razor (sometimes spelled Occam's razor). This rule states, "When given two 
equally valid explanations for a phenomenon, one should embrace the less complicated for- 
mulation." In other words, all things being equal, the simplest solution tends to be the best 
one. At this point, if a hypothesis holds up to repeated testing, it becomes a theory. When a 
theory is supported consistently over time, scientists may then consider it as a law, fact, or 
principle. 



Understanding Forms of Measurement 

Because science is based on developing objective facts — evidence and results that are 
measurable and experiments that can be reproduced — measurements are an important 
part of science. And because this subtest is all about science, you can expect to run into a 
few questions about measuring scientifically on the ASVAB. 



Doing the metric thing 




The metric system (or SI, the International System of Units) is based on a decimal system of 
multiples (and fractions) of ten. Scientists almost always use the metric system for precise 
measurement. No, they don't use it just to make the ASVAB harder for you; they use this 
system so a standard exists among scientists around the world. In fact, the majority of 
countries around the globe use the metric system — the United States is in its own world 
when it comes to the Imperial (non-metric) system. 

Here are some units of measurement you need to know for the General Science subtest of 
the ASVAB: 

v* The meter (m) is the unit of length. 

v* The liter (L) is the unit of volume. 

(-" The gram (g) is the unit of mass (similar to weight). 

You can attach prefixes to these base units to indicate units that are larger or smaller. 
Check out Table 10-1 for metric prefixes and Table 10-2 for some abbreviations of common 
metric measurements. 



Chapter 10: General Science / QQ 



Table 10-1 




Metric Prefixes 




Prefix 


Symbol 




What It Means 


milli- 


m 




One-thousandth (0.001) 


centi- 


c 




One-hundredth (0.01) 


deci- 


d 




One-tenth (0.1) 


deca- 


da 




10 


hecto- 


h 




100 


kilo- 


k 




1,000 


mega- 


M 




1,000,000 




Table 10-2 


Common Metric Units and Their 


Abbreviations 


Length 


Liquid Volume 


Mass 


millimeter (mm) 


milliliter 


mL) 


milligram (mg) 


centimeter (cm) 


centiliter 


(cL) 


centigram (eg) 


meter (m) 


liter (L) 




gram(g) 


kilometer (km) 


kiloliter (kL) 


kilogram (kg) 



&£$* 




Figuring temperature conversions 

When you think of temperature, you may think of the Fahrenheit and Celsius scales, which 
measure temperatures in degrees. Scientists actually use three different scales to report 
temperature: 

v* Fahrenheit (°F): This scale is more common in the United States. On the Fahrenheit 
scale, water freezes at 32°F and boils at 212°F. 

i*" Celsius or Centigrade CQ : This scale is the metric standard worldwide. On the Celsius 
scale, the freezing point for water is 0°C, and the boiling point for water is 100°C. 

j-" Kelvin (K): Scientists have theorized that the coldest anything can get is -273.15°C. 
They believe that at this temperature, molecular motion would stop. That's pretty darn 
cold! This temperature, often called absolute zero, is assigned to be on the Kelvin 
scale (with the units the same size as degrees on the Celsius scale). On this scale, the 
freezing point of water is 273.15 K, and the boiling point is 373.15 K. 

The word degrees isn't used when stating temperature in kelvins. Scientists who work 
with thermodynamics, such as physicists and astronomers, measure temperature 
using kelvins. For instance, the surface temperature of planets is always stated in 
kelvins. 

An ASVAB question may ask you to convert temperatures from one scale to another, so 
here are some formulas to commit to memory (C stands for the temperature in degrees 
Celsius, and F is the temperature in degrees Fahrenheit): 

u* To convert from Celsius to Fahrenheit, use this formula: 



F 



'C + 32 



J JO Part IV: The Whole Ball of Facts: Technical Skills 





v* To convert from Fahrenheit to Celsius, use the following formula: 

C = |(F-32) 
J-" To get temperatures in the kelvin scale, add 273.15 degrees to the Celsius temperature: 

K=C+ 273.15 

To go from kelvins to degrees Celsius, do the opposite: Subtract 273.15 from the kelvin 
temperature. Then you can convert the Celsius temperature to Fahrenheit if you like. 

Here's a quick temperature conversion system that may be easier to remember (Note: This 
process only works with Celsius and Fahrenheit): 

1. Add 40 to the temperature you want to convert. 

2. Multiply this sum by ^ if converting from Fahrenheit to Celsius or ^ if converting 
from Celsius to Fahrenheit. 

3. Subtract the 40 you added at the beginning to yield the result. 

9 5 
An easy way to remember whether to use -=- or ^ in the conversion is to associate the /in 

5 9 

Fahrenheit with Fraction (^ is a proper fraction); similarly, ■= can be Converted to a mixed 

number (1-jh — c is for Convert and Celsius. 

Another Day, Another Science: Scientific 
Disciplines \lou Should Know 

Science is divided into areas of study called disciplines, and most of these disciplines have 
subdisciplines. When you take the ASVAB, the General Science subtest may ask you some 
definitions of these disciplines. I couldn't possibly list all the scientific disciplines, but 
here's a handy list for you to start looking over. 

First, here are some popular Earth and space sciences (see the later sections "Where Few 
Have Gone Before: Astronomy" and "Down to Earth: Rocking Out with Geology and 
Meteorology" for more info on these disciplines): 

v* Astronomy: Astronomers (not to be confused with astrologists) study outer space. 
They get their jollies examining the existence, locations, orbits, energy, and composi- 
tions of planets and other celestial matter. 

J-" Geology: Is it a real diamond or just a piece of glass? A geologist can tell you. These 
scientists study the dynamics and physical history of the Earth, the rocks of which it's 
composed, and the physical, chemical, and biological changes that the Earth has 
undergone or is undergoing. 

j*" Meteorology: You know that person who gets on the TV each day and tells you 

whether your planned outing to the beach is going to be ruined by rain? Meteorologists 
study the weather and attempt to predict it. 

v* Paleontology: Paleontologists study prehistoric life, including dinosaurs. How cool is 
that? The science involves the examination of fossils, including those of plants, ani- 
mals, and other organisms. 

Biologists love everything to do with living organisms and life sciences. There are more sub- 
disciplines of biology than you can shake a stick at. And yes, some biologists study sticks. 



Chapter 10: General Science / / / 



Other biologists specialize in fish, trees, snakes, insects . . . you get the picture. Here are 
some subdisciplines of biology (for further info, check out the next section): 

v* Agriculture: An agriculturalist studies farming. This discipline includes studying 
methods of cultivating soil, producing crops, and managing livestock. 

v* Botany: A botanist studies plant life. This includes everything from flowers to the moss 
that grows on the north side of the tree. 

j-" Ecology: Ecologists do more than just warn people that they're destroying the ozone 
layer. They study all aspects of the environment and how organisms (such as people) 
interact with it. 

v* Entomology: Entomologists like bugs. Specifically, they like insects (bugs with six 
legs). This position isn't to be confused with an arachnologist, who studies spiders and 
other critters with eight legs. 

J-" Genetics: Geneticists study heredity, especially the aspect that deals with inherited 
characteristics, such as eye color. (For details, see the later section "Swimming in the 
gene pool: Genetics.") 

j-" Ichthyology: This discipline is the branch of zoology (the study of animals) dealing 
with fish. 



Here are a couple of social sciences: 

j-" Archeology: For an archeologist, the older, the better. Archeologists study past human 
life and culture. The job requires recovery and examination of material evidence, such 
as graves, tools, pottery, and buildings. 

J-" Genealogy: If you want to find out where your great, great, great, great, great grandfa- 
ther was born and what he did for a living, ask a genealogist. These specialists study 
ancestry and family history. 

Another large discipline is chemistry, in which people mix things together to see what 
happens. These scientists study the structure, properties, composition, and reactions of 
matter. I discuss chemistry later in "Chemistry: Not Blowing Up the Lab." 

Lastly, don't forget physics. Physics involves the study of matter and its movement. This 
includes concepts such as energy, force, and motion. In short, physics is concerned with 
the study of the universe's behavior and, in general, how things work in nature. Mechanics, 
which plays a big role in the ASVAB's Mechanical Comprehension subtest (see Chapter 12), 
is a major topic in physics. 

If the ASVAB only asked questions like "What does a chemist do?" the test would be a piece 
of organic matter (cake). Unfortunately, it's not that easy. The ASVAB writers expect you to 
know a little more than just the definitions of various scientific disciplines. The following 
sections detail a few of the main branches of science you see on the ASVAB. 



Uncovering Biology, from Big to Small 

It would be impossible to cover all the areas of biology in this book, and I'm not going to try. 
Luckily, the General Science subtest of the ASVAB measures your knowledge of scientific 
disciplines at the average high school level. You remember studying about the Animal 
Kingdom and the human body and cell structures in high school, right? If not, the following 
sections can serve as a short refresher course. 



112 



Part IV: The Whole Ball of Facts: Technical Skills 



*!*!§? 




Relating to your World through ecology 

Ecology is the study of the environment — more specifically, the relationship between 
organisms and the world around them. All plants and animals are part of an ecosystem (a 
community including living things and their environment). Like the economy, an ecosystem 
includes producers (which make their own food) and consumers (which eat other things). An 
ecosystem also has decomposers, such as bacteria, which break down dead plants, animals, 
and the waste of all organisms. 

Animals can't produce their own food, so they're consumers, which are classified in three 
categories: 

i*" Carnivores eat only meat. A few examples include lions, tigers, polar bears, snakes, 
crocodiles, hawks, and eagles. 

u* Herbivores eat only plants. Cows, moose, giraffes, and elk are herbivores. 

v* Omnivores eat both plants and other animals. People are omnivores, and so are pigs, 
mice, raccoons, chickens, crows, and foxes. 



Conditions in the world either encourage or prevent the establishment of individual ecosys- 
tems. For plants (producers) to grow, adequate sunlight, good soil, moderate temperatures, 
and water must be part of the environment. If plants aren't around, plant-eating consumers 
can't be sustained, which means predators (who eat other animals) can't be sustained, 
either. For consumers, mates are as essential as a food supply. Diseases and enemies can 
prevent an animal from establishing itself in an ecosystem. 

Human actions, such as wasting natural resources and polluting the air, water, or soil, can 
disrupt an entire ecosystem. 



cJftBEtf 




Categorizing Mother Nature 



A long time ago, scientists looked at the world, noticed the hundreds of thousands of plants 
and animals around them, and decided that all these organisms (living things) needed to be 
labeled and grouped. To effectively study and discuss plants, animals, and other living crea- 
tures, all scientists needed to use the same names. Thus, a system of scientific classification 
was developed. 

The most common classification system was created by Swedish botanist Carl Linnaeus, 
who published ten editions of his works from 1753-1758. Scientists often refer to this 
system as taxonomy. Not only does taxonomy provide official names for every plant and 
animal, but it also helps scientists understand how living creatures are related to one 
another. Modern day taxonomy has its roots in the Linnaean taxonomic system. 

No one is privy to the actual questions asked on the ASVAB (test materials are considered 
"controlled items" and are locked up in safes when not in actual use). In this category, ques- 
tions can range anywhere from "How many kingdoms are there?" to "What's the genus for 
Canis familiarisT 

Counting douin the classification system 

The scientific classification system notes the relationships and similarities among organ- 
isms. It consists of seven main levels: 



is* Kingdom: A kingdom is the broadest level, so it contains the most kinds of organisms. 
The relationship between organisms in a kingdom is extremely loose, so members 
share only a few key characteristics. 



Chapter 10: General Science J/j 



v* Phylum: Phylum (plural phyla) is the next major taxonomic group. Within the king- 
doms, organisms are divided into phyla by general characteristics. For example, in the 
Animal Kingdom, animals with backbones (vertebrates) are placed in a separate 
phylum from animals without backbones. 

J*" Class: Organisms in a phylum are divided into classes. In the Animal Kingdom, for 
example, birds, mammals, and fish all go in their own classes. Among plants, all flower- 
ing plants comprise the angiosperm class, and all conifers, such as pines and spruces, 
comprise the conifer class. 

i*" Order: Scientific groupings don't follow hard and fast rules, so when you get to the 
order of a living thing, there's disagreement about where it belongs. You may find that 
different scientific organizations group creatures in different orders or families. 

v* Family: Families further divide organisms of the same class by similar characteristics. 
Sometimes, not all scientific organizations agree to the exact family an organism 
should be classified in. 

v* Genus: Two or more species that share unique body structures or other characteris- 
tics are closely related enough to be placed in a single genus. A genus may include only 
a single species if no other organism has characteristics similar enough for it to be con- 
sidered the same genus. 

v* Species: A species is the most specific level, so it contains the fewest types of organisms. 
Organisms of the same species have very similar characteristics. 

To get a better idea of how the scientific classification system works, consider how a lion is 
classified: 



&£&> 




Kingdom Animalia: This kingdom includes all animals. 

Phylum Chordata: All vertebrate animals belong to the phylum Chordata. 

Class Mammalia: All mammals belong to this class. 

Order Carnivora: All mammals that eat meat belong to the order Carnivora. 

Family Felidae: The family Felidae includes all cats. 

Genus Panthera: This genus includes all the roaring cats, such as lions, tigers, jaguars, 
and leopards. 

Species leo: This is just a lion. 

Humans belong to the kingdom Animalia, the phylum Chordata, the class Mammalia, the 
order Primata, the family Hominidae, the genus Homo, and the species sapiens. You know, 
just in case you were wondering. 

Visiting the kingdoms 

Not every scientist agrees (scientists rarely agree on any subject), but in general, most lab- 
coated individuals settle on five as the number of kingdoms. Check out the kinds of organ- 
isms that comprise the five kingdoms: 

v* Animals: This is one of the two largest kingdoms, and it includes many-celled organ- 
isms that, unlike plants, don't have cell walls, chlorophyll, or the capacity to use light 
to make energy (photosynthesis). Members of this kingdom can move. The Animal 
Kingdom includes more than 1,000,000 species. 

v* Plants: Plants are also one of the two largest kingdoms. This kingdom includes organ- 
isms that can't move, don't have obvious nervous or sensory systems (the Venus fly- 
trap is one exception), and possess cell walls made of cellulose. More than 250,000 
species belong to the Plant Kingdom. 



1H 



Part IV: The Whole Ball of Facts: Technical Skills 



*££&> 




v* Monerans: This kingdom includes bacteria and cyanobacteria (blue-green algae) — 
one-celled organisms that don't have a nucleus (see the later section "Thinking small: 
Looking at cells"). More than 10,000 species have been discovered and classified in the 
Monera Kingdom. 

j-" Protists: Protists include one-celled organisms that do have a nucleus, such as the pro- 
tozoan, which you may remember from biology class. This kingdom consists of more 
than 250,000 species. 

v* Fungi: Examples of common fungi are mushrooms and yeast. Fungi don't photosynthe- 
size (use light to create energy) like plants, but they do have cell walls made of a car- 
bohydrate called chitin. More than 100,000 species belong to the Fungi Kingdom. 

Thirty-three phyla make up the Animal Kingdom, and 12 main phyla comprise the Plant 
Kingdom. Monerans consist of two phyla; protists have seven phyla, and fungi are made up 
of four phyla. Numerous classes, orders, families, genera, and species fall under each 
phylum. 

Just name it: Showing off your genius about the species 

Each organism is given a scientific name that consists of two words (usually derived from 
Latin) — the genus and the species of the organism. The genus is the first word, and the 
species is the second. Thus, Homo sapiens refers to humans. Canis familiaris is the family 
dog, and Canis lupus is the family wolf. Because wolves and dogs share many similarities, 
they share the same genus (no, no, not the same genes, the same genus). 

When writing a scientific name, the genus name is capitalized, and the species name is all 
lowercase. Both names are italicized. 



Perusing the human body systems 

Your body consists of major systems that work together to keep you alive. (And staying 
alive is a good thing, so be sure to thank your circulatory system and all the rest!) These 
systems include the ones listed in Table 10-3. 



Table 10-3 



Five Major Human Body Systems 



System 



Components 



What the System Does 



Central nervous 
system 



Brain, spinal cord, 
and nerves 



Receives, processes, and responds to all physi- 
cal stimuli; for example, if you burn your hand on 
the stove, this system prompts you to remove your 
hand from the stove 



Circulatory system 



Digestive system 



Heart, blood, and Delivers oxygenated blood from the heart to the 

blood vessels rest of the body and returns the blood to the heart 

to be oxygenated again 



Mouth, esopha- 
gus, stomach, 
small and large 
intestines, rectum, 
and anus 



Breaks down food into smaller substances that 
the body can absorb and process into energy and 
eliminates the resulting waste 



Chapter 10: General Science J/j 



System 



Respiratory 
system 



Components 



What the System Does 



Musculoskeletal Bones, joints. Bones supportthe body's muscles and organs; 

system voluntary and joints allow bones to move; voluntary muscles work 

involuntary in pairs to move joints; involuntary muscles, which 

muscles you can't control, are found in organs such as the 

heart 



Nose, nasal cavity, 
trachea, lungs, 
and blood 



Inhales air, uses the oxygen in the airto release 
energy, and exhales the carbon dioxide that results 
from this process 



Thinking small: A took at celts 

Living things are made up of cells that share certain characteristics. Cells come in different 
sizes and shapes, depending on what they do. In the human body, a muscle cell looks very 
different from a brain cell. (Has all this talk of cells caused your brain cells to hurt yet?) 
Cells combine to create tissues, which form structures like bones and skin. 



Looking at cett structure 

A cell has three main parts ■ 



the nucleus, the cytoplasm, and the cell membrane: 



*■£«&, 




0WER 




f Nucleus: The nucleus controls cellular activity. It's like the brains behind the cell, and 
it holds the cell's genetic material, such as DNA. 

Bacteria are prokaryotes, which means their cells don't have nuclei. Their genetic mate- 
rial floats in the cytoplasm instead of being held inside a membrane (nuclear envelope). 

v Cytoplasm: The cytoplasm is a gel-like substance, composed mostly of water, that's 
inside the cell membrane and outside the nucleus. Cytoplasm contains many chemicals 
that carry out the life processes in the cell. 

v^ Cell membrane (plasma membrane): This thin membrane holds the cell together, 
protecting the nucleus and cytoplasm. 

See Figure 10-1 for a description of other cell structures. 

Plant cells differ from animal cells in a number of ways: 

j"* Plant cells have a firm cell wall that supports and protects the cell. Animal cells don't 
have such a structure. 

j*"* Plant cells have larger vacuoles (storage areas) than those found in animal cells. 

i** Unlike animal cells, many plant cells contain chloroplasts, which contain chlorophyll, 
a chemical that helps plants create food with the help of sunlight. 

v Animal cells contain centrioles (cylindrical structures involved in cell division). Most 
plant and fungi cells don't. 

W Animal cells have lysosomes (sacs of enzymes), which aren't found in plant cells. 



116 



Part IV: The Whole Ball of Facts: Technical Skills 



Plant Cell 



1. Chloroplast: Contains chlorophyll, 

which produces food 

2. Cell wall: Protects the cell 

3. Nucleus: The "brain" of the cell 

4. Chromatin: Thin fibers 

containing genes 

5. Nucleoplasm: Protoplasm (living 

material) in the nucleus 

6. Ribosome: Combines amino acids 

into proteins 

7. Cytoplasm: The cell 's factory 

8. Mitochondria: Produce the energy 

for cellular activity 

9. Cell membrane: Contains the cellular 

material within it 

10. Vacuole: Storage area 




Animal Cell 





1. Nucleus 




2. Chromatin 




3. Nucleoplasm 




4. Ribosome 


Figure 10-1: 


5. Cytoplasm 

6. Mitochondria 


Basic 


7. Cell membrane 


structures 


8. Vacuole 


of plant and 




animal cells. 






Profiting from ce(( processes 

Cells perform various processes in order to function at an optimum level. Here are a few of 
these processes: 

u* Metabolism: Chemical processes within a cell that are necessary for life to be 
maintained 

v* Osmosis: Movement of water through the cell membrane 

j-" Phagocytosis: Acquisition of particles of material from outside of the cell; it's accom- 
plished by surrounding the particles and passing them through the cell membrane 

v* Photosynthesis: Conversion of carbon dioxide and water into glucose and oxygen (in 
plants); in other words, sunlight is used to create energy 

J*" Cellular respiration: Process in which food is broken down, producing energy 



Swimming in the gene pool: Genetics 

Someday you're going to find yourself acting like your mother or father. Whether you like it 
or not, it happens because parents pass their traits on to their offspring. Understanding 
genetics — how traits are physically passed from parents to offspring and what happens 
when the process goes wrong — helps scientists pinpoint the causes of diseases and disor- 
ders and can help them develop treatments and cures. 



Chapter 10: General Science / / 7 



cjABE* 




*££& 




In human genetics, a healthy person contains 23 pairs of chromosomes (the structure that 
contains the genes). The mother and the father each supply one chromosome per pair. 
Genes contained in the chromosomes determine many characteristics of the resulting child. 

Copying genes 

When body cells multiply to produce tissues and organs (and eventually a complete living 
thing), they reproduce their genetic material. Most cells reproduce by mitosis, in which the 
nucleus of a cell divides, forming two cells and two identical sets of chromosomes. 

However, sex cells (eggs and sperm) reproduce differently. Through meiosis, each cell 
divides into four cells, each containing only half the number of chromosomes as a nonsex 
cell. This process takes place so that the sex cells of one person (with 23 chromosomes) 
can hook up with the sex cells of another person (with 23 chromosomes) to produce 46 
chromosomes, or 23 pairs. Otherwise, way too many chromosomes would be floating 
around. 

Sometimes cells don't copy themselves and divide perfectly, and a genetic mistake is made. 
This frequently results in a fetus who doesn't live or in a fetus with a genetic disease or 
disorder. For example, Down Syndrome is the result of a fetus's having 47 instead of 46 
chromosomes. 

determining your gender With Wo tittle tetters 

The genes on one pair of chromosomes, called the sex chromosomes, determine whether a 
child will be male or female. In females, the two sex chromosomes are alike, and they're 
labeled XX. In males, the chromosomes are different and are labeled XY. 



*** The child always receives an X chromosome from the mother (who only has XX chromo- 
somes). The father (who has XY chromosomes) can contribute either an X or a Y chromo- 
some, so Papa actually determines the sex of the child. 

Knowing Which genes get passed doWn the f amity tine 

Many characteristics that you possess (from the way your nose turns up at the end to the 
color of your eyes) are determined by a pair of genes (or multiple pairs of genes). These 
two genes may be alike, or they may not. 

Some genes are dominant, and some genes are recessive. If you have two unalike genes, the 
characteristic that they produce comes from the dominant gene; the gene that doesn't over- 
shadow the other is called the recessive gene. If each parent has two unalike genes, both 
parents will have the dominant trait, but they can have a child with the recessive trait — 
because each parent contributes a gene to the offspring, each parent may contribute a 
recessive gene to the child. Whew! 



Chemistry: Not Blowing Up the Lab 



SpRER 




Chemists study matter, and everything that has mass and takes up space — including your 
old Chevy that's up on blocks and the mosquito buzzing around the room — is matter. All 
matter is made up of basic substances (building blocks) called elements. 

Mass isn't the same thing as weight. Weight has to do with the force that gravity exerts on 
mass. If you were in a gravity-free zone, you wouldn't weigh anything, but you'd still be 
there, so you'd still exist and have mass. 

Those mad scientists in the movies always seem to be chemists, but chemistry shouldn't 
drive you crazy. Here's a straightforward review of the chemistry you need to know for the 
General Science subtest. 



118 



Part IV: The Whole Ball of Facts: Technical Skills 



WER 




Understanding the elements, my dear Watson 

The atom is the smallest part of an element that still retains the characteristics of that ele- 
ment. Every atom has particles — pieces of matter that are very, very small. Electrons are 
negatively charged particles that float around the atom's nucleus, or core, which is made up 
of neutrons (particles with no charge) and protons (positively charged particles). 

Each element has its own atomic number that's equal to the number of protons. If an atom 
has one proton in its nucleus, it has the atomic number 1. Hydrogen is the only element 
with just one proton in its nucleus. Magnesium, which has 12 protons in its nucleus, is given 
the atomic number 12. 

Atoms can combine with each other to form molecules. If those atoms are of two or more 
different elements, the molecule is called a compound. A compound can have very different 
properties from the elements that make it up. For example, table salt, which is mostly harm- 
less, consists of two lethal elements — sodium and chlorine. But when combined, these ele- 
ments make a compound that people ingest every day. 

Sitting down at the periodic table 

The periodic table (also known as the table of elements) classifies all elements, because sci- 
entists love to classify things. Elements are listed according to their atomic numbers 
(number of protons) and are arranged into families of similar elements. 

The periodic table lists the atomic number, the abbreviation for each element, and its 
atomic weight, which is the average mass of one atom of the element. Looking at Figure 10-2, 
you can see that copper (Cu, atomic number 29) has an atomic weight of 63.546, which 
means that copper is much, much heavier than helium (He, atomic number 2), which has an 
atomic weight of 4.0026. 

You don't have to memorize these charts to do well on the ASVAB, but you should know the 
atomic numbers for common elements such as hydrogen (1), helium (2), carbon (6), nitro- 
gen (7), oxygen (8), sodium (11), iron (26), copper (29), gold (79), mercury (80), lead (82), 
uranium (92), and plutonium (94). 



Getting physical: Changing states 

Particles of matter are always in motion. How much kinetic energy (motion energy) a parti- 
cle has determines whether the matter is a solid, liquid, or gas in its normal state. Gas parti- 
cles move around very quickly, liquid particles move more slowly, and solid particles move 
much more slowly than either of the other two. 

When heat or cold is applied to matter, the kinetic energy of the matter changes; therefore, 
the nature of the substance can change. Heat applied to water changes the water from a 
liquid to a gas (steam), and cold applied to water changes it from a liquid to a solid (ice). 
When physical changes occur, the molecule itself remains the same. For example, water is 
still made of hydrogen and oxygen, no matter which state it's in. 



Chapter 10: General Science / / V 



Figure 10-2: 

The periodic 
table. 







IA 





































1 


2 




1 


H 

1 [vein i«i'ii 
1.00797 


IIA 




















IIIA 


IVA 


VA 


VIA 


VIIA 


He 

Helium 
4.0026 




3 


4 


5 


6 


7 


8 


9 


10 




2 


Li 

Lithium 
6.939 


Be 

Betyllium 
9.0122 




















B 

10.811 


C 

Carbon 
12.01115 


N 

Nitrogen 
14.0067 




Oxynen 
15.9994 


F 

Fluorine 
18.9984 


Ne 

20.183 




11 


12 


13 


14 


15 


16 


17 


18 




? 


Na 


Mff 












Vlll 








Al 


Si 


P 


S 


CI 


Ar 






Sodium 
22.9898 


24.312 


IIIB 


IVB 


VB VIB 


VIIB 




__^v___ 




IB 


IIB 


Alumrium 
26.9815 


Silicon 

28.086 


Phosphorus 
30.9738 


32.064 


Chlorine 
35.453 


Aryon 
39.948 


(-I 


f 




> 


LJ 


19 


20 


21 


22 


23 


24 


25 


26 


27 


28 


29 


30 


31 


32 


33 


34 


35 


36 


w 
ft. 


/] 


K 


Ca 


Sr 


Ti 


V 


Cr 


Mn 


Fe 


Co 


Ni 


Cll 


Zn 


Ga 


Ge 


As 


Se 


Br 


Kr 




Potassium 


Calcium 


Scandium 


Titanium 


Vanadium 


l ]in iniium 


Hai^'aiH'S! 


Iron 


Cobalt 


Nickel 


Copper 


Zinc 


Gallium 


r.ermaniun 


Arsenic 


Selenium 


Bromine 


Krypton 




39.102 


40.08 


44.956 


47.90 


50.942 


51.996 


54.9380 


55.847 


58.9332 


58.71 


63.546 


65.37 


69.72 


72.59 


74.9216 


78.96 


79.904 


83.80 




37 


38 


39 


40 


41 


42 


43 


44 


45 


46 


47 


48 


49 


50 


51 


52 


53 


54 




5 


Rb 


Sr 


Y 


Zr 


Nb 


Mo 


Tc 


Ru 


Rh 


Pd 


At? 


Cri 


In 


Sn 


Sb 


Te 


I 


Xe 




Rubidium 


Sti"' Hiliinn 


Yttrium 


ZiiV' niiiiiii 


Niobium 


McJybdaium 


Technetiun Ruthenium 


Rhodium 


Palladium 




Cadmium 


Indium 


Tin 


Ai it in kit 


Tellurium 


Iodine 


Xenon 






85.47 


87.62 


88.905 


91.22 


92.906 


95.94 


(99) 


101.07 


102.905 


106.4 


107.868 


112.40 


114.82 


118.69 


121.75 


127.60 


126.9044 


131.30 




55 


56 


57 


72 


73 


74 


75 


76 


77 


78 


79 


80 


81 


82 


83 


84 


85 


86 




6 


Cs 


Ba 


La 


Hf 


Ta 


W 


Re 


Os 


Ir 


Pt 


Au 


He 


Tl 


Pb 


Bi 


Po 


At 


Rn 




Cesium 


Barium 


Lanthanum 


Hafnium 


Tantalum 


Tungsten 


Rhenium 


Osmium 


Iridium 


Platiium 


Gold 


Mercury 


Thallium 


Lead 


Bismudi 


I'l ] lllilllll 


Astatine 


Radon 






132.905 


137.34 


138.91 


179.49 


180.948 


183.85 


186.2 


190.2 


192.2 


195.09 


196.967 


200.59 


204.37 


207.19 


208.980 


(210) 


(210) 


(222) 




87 


88 


89 


104 


105 


106 


107 


108 


109 






















7 


Fr 


Ra 


Ac 


Rf 


Db 


Sp 


Bh 


Hs 


Mt 






















Francium 


Radium 


Actinium 


v:l:.':-il"nLui 


Dubnium 


•■■:■.: H i:--.,i ..; 


Bohrium 


Hassium 


Mi.'iiuiit.un 
























(223) 


(226) 


(227) 


(261) 


(262) 


(266) 


(264) 


(269) 


(268) 























58 


59 


60 


61 


62 


63 


64 


65 


66 


67 


68 


69 


70 


71 


nthanide Series 


Ce 

Cerium 
140.12 


Pr 

140.907 


Nd 

Neodvmium 
144.24 


Pm 

I'll 'iiHlkiun 
(145) 


Sm 

150.35 


Eu 

II , II c J| lil. LL LI 

151.96 


Gd 

1 J.;, i lini.iin 
157.25 


Tb 

Terbium 
158.924 


Jvsprosium 
162.50 


Ho 

Holmium 
164.930 


Er 

Erbium 
167.26 


Tm 

Thulium 
168.934 


Yb 

Vlli 1 1 nun 
173.04 


Lu 

Lutetium 

174.97 




90 


91 


92 


93 


94 


95 


96 


97 


98 


99 


100 


101 


102 


103 


Actinide Series 


Th 

Thorium 
232.038 


Pa 

Protactiniun 
(231) 


U 

Uranium 
238.03 


Np 

Neptunium 
(237) 


Pu 

Plutonium 
(242) 


Am 

Americium 
(243) 


Cm 

(247) 


Bk 

i'< ij iiiin 

(247) 


Cf 

Californium 
(251) 


Es 

l.iiisUiiii'.im 
(254) 


Fm 

Fermium 
(257) 


Md No 

VlendefeviLin Nobelium 
(258) (259) 


Lr 

Liwtviviun 
(260) 



Causing a chemical reaction 



Unlike physical changes, chemical reactions create new molecules. For example, when iron 
rusts, a chemical change occurs. The rust isn't the same molecule as the iron. 

In a chemical reaction, two kinds of substances are present: 



I 



u* Reactants: The elements or molecules involved in the reaction 

v* Products: The elements or molecules that result from the chemical reaction 



Where feu) Hatfe Gone Before: Astronomy 

Earth's solar system consists of the sun and a number of smaller bodies (such as planets, 
the planets' moons, and asteroids) that the sun's mass holds in orbit. The sun's mass cre- 
ates gravity, and this gravity controls the movements of the smaller bodies. 



Taking a quick glimpse at the sun 

The sun is the largest and most important object in the solar system. It contains 99.8 per- 
cent of the solar system's mass (quantity of matter). The sun provides most of the heat, 
light, and other energy that makes life possible. 



/ 20 Part IV: The Whole BaM of Facts: Technical Skills 



$££&* 




Is Pluto really a planet? 



Pluto was referred to as the ninth planet since its dis- 
covery in the 1930s. But in August 2006, the International 
Astronomical Union (IAU) established a new definition 
for the word planet. Pluto has so many unusual features 
that it was reclassified as a dwarf planet. For example, 
it travels around the sun in an elongated oval path much 
differentfrom the nearly circular orbits of the other plan- 
ets. And unlike the other outer planets, Pluto is small and 

1 
solid and contains only— -of the mass of Earth. 



Under the new standards, Pluto is no longer consid- 
ered a planet but instead is classified as a dwarf planet. 
According to the planet definition, the solar system con- 
sists of eight planets and three dwarf planets. The defi- 
nition doesn't apply outside the solar system and doesn't 
include provisions for extra-solar planets. The definition 
was a controversial one; it has been both criticized and 
supported by different astronomers. 



The sun's outer layers are hot and stormy. The hot gases and electrically charged particles 
in those layers continually stream into space and often burst out in solar eruptions. This 
flow of gases and particles forms the solar wind, which bathes everything in the solar system. 

The sun is much larger than Earth. The distance from the sun's center to its surface (the 
sun's radius) is about 109 times the radius of Earth. Some of the streams of gas rising from 
the solar surface are even larger than the Earth's diameter. 



Knowing the planets 



A planet is a nonluminous celestial body larger than an asteroid or comet, illuminated by 
light from a star that the planet revolves around. The solar system consists of eight known 
planets: Mercury, Venus, Earth, Mars, Jupiter, Saturn, Uranus, and Neptune. Pluto is no 
longer classified as a planet by most scientists. (See the sidebar "Is Pluto really a planet?" 
for details.) 

The Earth revolves around the sun in an oval-shaped pattern called an ellipse. Every 365)4 
days, the Earth completes its orbit around the sun and starts again. The Earth rotates 
(spins) on its axis, completing a rotation every 24 hours, but because of the tilt of the Earth, 
hours of daylight and darkness aren't equal, except for on two days a year. 

The inner four planets consist chiefly of iron and rock. They're known as the terrestrial 
(earthlike) planets because they're somewhat similar in size and composition. The outer 
planets are giant worlds with thick, gaseous outer layers. Almost all their mass consists of 
hydrogen and helium, giving them compositions more like that of the sun than of Earth. 
Beneath their outer layers, the giant planets have no known solid surfaces. The pressure 
of their thick atmospheres turns their insides liquid, though they may have rocky cores. 

Rings of dust, rock, and ice chunks encircle all the giant planets. Saturn's rings are the most 
familiar, but thin rings also surround Jupiter, Uranus, and Neptune. 



Shooting for the moons 



Moons (sometimes called satellites) orbit all the planets except Mercury and Venus. The 
moon you refer to as the moon revolves around the Earth. It makes a complete revolution 
every 27M days. When the moon moves into the Earth's shadow, a lunar eclipse results — the 



Chapter 10: General Science j 2 1 



Earth is positioned between the sun and the moon. When the Earth moves into the moon's 
shadow, a solar eclipse results — the moon is positioned between the Earth and the sun. 

The inner planets have few moons. The giant planets probably have more small moons not 
yet discovered. See Table 10-4 for a lineup of the planets and their moons. Although Pluto 
is no longer officially considered a planet, you never know what those rascally ASVAB test- 
writers will ask, so I've included Pluto in the table. 



Table 10-4 




The Number of Moons 


per Planet in Earth's Solar System 


Planet 






Number of Moons 


Mercury 









Venus 









Earth 1 


Mars 






2 tiny satellites 


Jupiter 






63 


Saturn 






61 


Uranus 






27 


Neptune 






13 


Pluto (dwarf planet) 




3 



*£3?fc 




Jupiter's four largest moons are known as the Galilean satellites because the Italian astrono- 
mer Galileo Galilei discovered them in 1610 with one of the first telescopes. The largest 
Galilean satellite — and the largest satellite in the solar system — is Ganymede, which is 
even bigger than Mercury and Pluto. The largest of Saturn's moons, Titan, has an atmo- 
sphere thicker than Earth's and a diameter larger than that of Mercury or Pluto. Pluto's larg- 
est moon, Charon, is more than half the size of Pluto. 



sftJTl/*. 




Watching for meteors, comets, and asteroids 

A meteor is a rock from space that hits Earth's atmosphere and glows as it heats up, result- 
ing in a brief streak of light. It's often called a shooting star. When a meteor enters the 
Earth's atmosphere, it usually burns up (and that's a good thing). If a meteor actually 
strikes the Earth, it's called a meteorite. 

Comets are snowballs composed mainly of ice and rock. When a comet approaches the sun, 
some of the ice in its nucleus (center) turns into gas. The gas shoots out of the sunlit side of 
the comet. The solar wind then carries the gas outward, forming it into a long tail. 
Astronomers divide comets into two main types: 

I i*" Long-period comets, which take 200 years or more to orbit the sun. 
v* Short-period comets, which complete their orbits in fewer than 200 years. 

The most famous of all comets, Halley's Comet — also referred to as Comet Halley after 
Edmond Halley — is a comet that can be seen every 75 to 76 years, making it a short-period 
comet. Halley is the only short-period comet that is visible to the naked eye and will return 
within a human lifetime. Its many appearances over the centuries have had a notable effect 
on human history. Halley's Comet last appeared in the inner solar system in 1986 and will 
next appear in mid-2061. 



122 



Part IV: The Whole Ball of Facts: Technical Skills 



Asteroids are sometimes called minor planets because they're small bodies that orbit the 
sun. Some have elliptical orbits that pass inside the orbit of Earth or even that of Mercury. 
Others travel on a circular path among the outer planets. Most asteroids circle the sun in a 
region called the asteroid belt, between the orbits of Mars and Jupiter. The belt contains 
more than 200 asteroids larger than 60 miles (100 kilometers) in diameter. Scientists esti- 
mate that more than 750,000 asteroids with diameters larger than % mile (1 kilometer) exist 
in the belt. There are millions of smaller asteroids, and astronomers have even found sev- 
eral large asteroids with smaller asteroids orbiting them. 



Dou/n to Earth: Rocking Out With 
Geology and Meteorology 



The study of the physical makeup of the Earth is often called Earth science. Geology describes 
the Earth's physical appearance, and meteorology explains the Earth's atmosphere. 



Peeling back the layers of the planet 

The Earth is like an onion in that it consists of several layers. The crust is the Earth's sur- 
face, and it varies in depth from a few miles to 30 miles. The mantle (including the mantle 
and an upper mantle) is the solid rock below the crust, and it makes up most of the mass of 
the Earth. The core (including the inner and outer cores) is the Earth's fiery center, with a 
temperature estimated to reach as hot as 4,300 degrees Celsius (to see what that is in 
Fahrenheit, use the conversion equations in "Figuring temperature conversions" earlier in 
this chapter). The mantle accounts for about two-thirds of the Earth's mass. 

Sometimes cracks in the Earth's crust, called faults, appear. When the land shifts along 
these faults, earthquakes result. Molten rock trapped between the crust and the mantle is 
called magma. Magma collects in pockets called magma chambers and forms volcanoes. 
When volcanoes erupt, the magma is spewed out as lava. 



Outta this World: Checking the atmosphere 

The atmosphere contains many layers of air surrounding the Earth's surface. Starting with 
the layer closest to the Earth and extending outward, Table 10-5 names those layers. 



Table 10-5 




Layers of Earth's Atmosphere 


Layer Name 


Location 


Details 


Troposphere 


Extends 
about8 
miles above 
the Earth 


This layer is where the jet stream is located and where almost 
all weather changes occur. 


Stratosphere 


Extends 
about 30 
miles 


A major reported cause of ozone depletion is the presence of 
chlorofluorocarbons (CFCs) in the Earth's stratosphere. CFCs 
undergo a series of chain reactions, which ultimately leads to 
the destruction of the ozone layer. 



Chapter 10: General Science / £j 



Layer Name 



Location 



Details 



Mesosphere 



Extends 
about 50 
miles 



Millions of meteors burn up daily in the mesosphere as a result 
of collisions with the gas particles contained there. 



Ionosphere Extends This layer reflects most radio waves, making it important to 

about 70 communications. Note: Scientists disagree among themselves 

miles as to whether the ionosphere is a separate atmospheric layer 

or whether it's part of the thermosphere. 

Thermosphere Extends The International Space Station has a stable orbitwithin 

about 350 the upper part of the thermosphere, between 208 and 285 

miles miles. 

Exosphere Extends It's only from the exosphere that atmospheric gases, atoms, 

about and molecules can escape into outer space. No boundary 

40,000 miles exists between the exosphere and space; therefore, exosphere 
is sometimes used synonymously with outer space. 



Warming up to cold fronts 



Temperature affects air density (how closely packed the air molecules are). When the sun 
shines, land and water absorb its warmth. Land warms up more quickly than water, so air 
over land is warmer than air over water during most of the day. At night, the air over land 
cools more quickly than air over water. The angle of the sun also affects air density (the sun 
shines directly over the Equator but not the poles). 

Cold air is denser than warm air. Because it's denser, cold air has high pressure, compared 
to warm air's low pressure. (A barometer measures atmospheric pressure.) Air moves from 
areas of high pressure to areas of low pressure, creating wind. 

Air masses have certain characteristics depending on where they form: 

Ii^ If an air mass forms over land, it's dry, and if it forms over water, it's wet. 
i* Air masses formed in Earth's northern and southern regions are cold, and those 
formed at the Equator are warm. 

When two different air masses meet, they don't mix. They form a boundary called a front. 
When cold air meets warm air, a cold front develops. The warm air may be pushed up to 
form clouds, causing heavy rain. When a warm air mass meets a cold air mass, a warm front 
develops. The warm air passes over the cold air, forming a different kind of cloud, which 
causes light rain. 



Classifying clouds 



Clouds are made of small droplets of water or bits of ice that are spread out from each 
other. Rain (or snow) falls when the drops get too big and heavy to stay in the cloud. Clouds 
have three main types, and the ASVAB may ask you a question or two about their character- 
istics, which are detailed in Table 10-6. 



12 It 



Part IV: The Whole Ball of Facts: Technical Skills 



Table 10-6 


Types 


of Clouds 


Cloud Type 


Description 


What It Forecasts 


Cirrus 


Thin, wispy, high clouds 


Generally indicate rain or snow 


Cumulus 


White, puffy pillows, often flat- 
bottomed with rounded tops 


Common during fair weather, but when 
they gather, they cause heavy rains 


Stratus 


Broad, flat, and low-hanging 
(gray blanket) 


If close to the ground, they may produce 
drizzle 



Additionally, a prefix or suffix is frequently given to the cloud name to indicate which level 
of the atmosphere it's in or whether it's producing precipitation (rain, sleet, snow, and the 
like): 

p" Cirro- is the prefix given to high clouds (base above 20,000 feet). 

k" Alto- is the prefix given to mid-level clouds (base between 6,000 and 20,000 feet). 

v* Nimbo- added to the beginning of a cloud name or -nimbus added to the end means the 
cloud is producing precipitation. 

Therefore, a cirrocumulus cloud is a white, puffy, flat-bottomed, rounded-topped cloud at 
high altitude. Altostratus clouds are gray, broad, flat clouds at mid altitude. 



Improving \lour Chances on the 
General Science Subtest 



Even if you study hard for the General Science subtest, chances are you may come across at 
least a couple of questions that you can't answer. That's the nature of this subtest — it 
pretty much asks you to know all there is to know about the universe. However, you can use 
several strategies to improve your chances of selecting the correct answer. 



WER 




Using common sense to make educated guesses 

If you don't know the answer to a question right off the bat, don't panic. You can often elimi- 
nate a few incorrect choices simply by using common sense. Even if you can't determine the 
answer, keep in mind that this subtest doesn't penalize you for guessing, so guessing makes 
sense — you have a 25 percent chance of guessing the right answer even if you can't elimi- 
nate any obviously wrong answers. If you can eliminate just one wrong answer, you improve 
your chances to 33 percent. 

Most people don't have to rush to finish the General Science subtest, but then again, you 
don't have much leisure time to stop and think about all the questions at length, either. So 
if you don't know the answer to a question right away, do your best to eliminate wrong 
answers quickly, mark your best guess, and move along. (For help on making these eliminat- 
ing decisions, check out Chapter 3.) 



Chapter 10: General Science J 25 



^PLE 




^PLE 




Try the process of elimination on the following question: 

The knee joint is known as a 

(A) pivot joint. 

(B) fixed joint. 

(C) ball-and-socket joint. 

(D) hinge joint. 

Looking at the choices, you can eliminate Choice (B), fixed joint, because your knee isn't 
fixed, or not moveable (or if it is, it shouldn't be). Your skull is an example of a fixed joint, 
but that's irrelevant to this question. Is your knee a pivot joint? If you think of something 
that pivots, you think of it moving in a circular or at least a semi-circular manner. Your knee 
doesn't do that either; therefore, you can safely eliminate Choice (A). A ball-and-socket joint 
is one that permits limited movement in any direction (your shoulder joint is a ball-and- 
socket joint). Your knee doesn't do that, so you can strike off Choice (C) and choose 
Choice (D), hinge joint, as the most likely answer. Your knee moves like a door on a hinge. 

Now suppose you have a question like this: 

The most common gas found in Earth's atmosphere is 

(A) oxygen. 

(B) nitrogen. 

(C) calcium. 

(D) helium. 

Eliminate Choice (C) because calcium isn't a gas. You can also cross out Choice (D) because 
if helium were the most common gas, everyone would be talking in squeaky voices (you 
know, like after sucking helium from a balloon). Eliminating these two answers leaves you 
with just two choices, and if you simply guessed, you'd have a 50 percent chance of being 
right. Unfortunately, most people would guess that oxygen is the most common gas in 
Earth's atmosphere, but they'd be wrong. Nitrogen — Choice (B) — tops the list, making up 
78 percent of the atmosphere. 




Getting back to your Latin roots 

Just when you thought vocabulary study was over, leave it to me to bring it up again. Many 
scientific words come from Latin or Greek. If you know the meaning of the Latin or Greek 
word, you can often figure out the meaning of the scientific word. Often, a Latin or Greek 
root word is used to create a longer, more specific word. (For common word roots, see 
Chapter 4.) 

For example, the Latin root homo means both human being and same. So Homo sapiens 
refers to members of the human species, but homogeneous means "of the same kind." So if 
you were to run across the word homologous on the General Science subtest, you'd know 
that it has something to do with humans or with things that are the same. 



126 



Part IV: The Whole Ball of Facts: Technical Skills 



^PLE 




Take a look at the following example question: 

Which of the following instruments might an oceanographer be expected to use? 

(A) aspirator 

(B) hydrophone 

(C) calorimeter 

(D) centrifuge 

Even if you don't have a clue about what any of these instruments do, if you know that 
hydro relates to water, you've significantly increased your chances of getting the right 
answer, Choice (B). 



Chapter 11 

Auto and Shop Information 



In This Chapter 

Looking under the hood of vehicles 
Knowing the tools of the trade 
Checking out the many uses of fasteners 
Driving up your test score 



tftBEfl 




r 

•^ver wonder why automobile mechanics and carpenters charge you about a billion dol- 
•Wlars an hour when you need to hire their services? Because if the jobs were easy, every- 
one would do them. 

Fortunately, to do well on the Auto & Shop Information (AS) subtest of the ASVAB, you don't 
have to get your hands greasy or chance hitting your thumb with a hammer. The questions 
on this subtest are pretty basic. Automotive questions usually ask about basic automotive 
systems and malfunctions. The shop questions generally ask you to identify a tool or fas- 
tener or the purpose of such. 

The Auto & Shop Information subtest consists of 25 questions on the paper and pencil ver- 
sion. Happily, the ASVAB gurus give you 26.4 seconds to answer each question (11 minutes 
total). About half of the questions measure your basic knowledge of automotive principles 
and half query you about shop tools and basic shop principles. On the CAT-ASVAB, you 
have 7 minutes to answer 1 1 questions on Auto Information and 6 minutes to answer 1 1 
questions on Shop Information. Your scores on these subtests are combined to give you a 
single Auto & Shop score. 

The military uses the Auto & Shop Information subtest only to determine your qualifications 
for certain jobs. It's not used in the calculation of your AFQT score. Turn to Appendix A at 
the back of this book to find the jobs that require a good score on this subtest. If you don't 
need to do well on this subtest to qualify for the kind of job you want, you may be better off 
studying for a different part of the ASVAB. 



Checking under the Hood 



Contrary to what you may think, an automobile is much more than the mechanical monster 
you park in your driveway each night. It's actually a complex machine that has undergone 
more than a century of evolution. Henry Ford would probably have a stroke if he could see 
what his simple horseless carriage evolved into. 

The modern car is divided into several primary and secondary systems. I cover these sys- 
tems in the next few sections. 



128 



Part IV: The Whole Ball of Facts: Technical Skills 



The engine: Different strokes 



How does an engine work? You turn the key, and if it doesn't start, you call your mechanic 
or your dad, right? Well, not quite. The internal combustion engine burns a mixture of gas 
and air. Burning the gas and air (the fuel mixture) makes it expand quickly (explode). The 
pressure from this explosion is transferred (via additional systems) to the wheels to make 
the car move. 

The movement is brought about by a cycle, which your car's engine repeats a zillion and 
one times. Here are the four strokes that make up a cycle (Figure 1 1-1 illustrates how this 
process works): 

1. Intake: The intake valve opens as the connecting rod pulls the piston down, drawing 
the gas/air mix into the cylinder. 

2. Compression: The valves are closed. The connecting rod pushes the piston up, com- 
pressing gas/air mix. 

3. Power: The spark plug ignites the gas/air mix, forcing the piston down. That pushes 
down on the connecting rod, turning the crankshaft; the crankshaft turns the flywheel, 
which keeps the engine going. 

4. Exhaust: The exhaust valve opens as the connecting rod moves the piston back up, 
pushing out the exploded gases. The valves are timed, of course, using push rods 
attached to the camshaft. 



Push Rod 



Spark Plug 



Figure 11-1: 

A four- 
stroke 
engine. 




ust Gases 
Exhaust Valve 



Connecting Rod 



Crankshaft 



Flywheel 



&S&* 




Chapter 11: Auto and Shop Information / £y 



Generally cars have an even number of cylinders — four, six, or eight. These cylinders are 
arranged in a row or rows, which are called inline (one row) or K(two rows), depending on 
the arrangement. 

Most people refer to their engines as four-cycle engines. This isn't really true. It is a four- 
stroke, one-cycle engine. The intake stroke, compression stroke, power stroke, and exhaust 
stroke are one engine cycle. When the fourth stroke is completed, the cycle begins again. 
Automobile engines do this very fast. When the tachometer (an instrument measuring revo- 
lutions per minute [rpm]) on your dashboard shows 4,800 rpm, for example, that means the 
engine is performing 4,800 of these cycles every minute. 

In order for the cycle to happen at all, fuel must be properly mixed with air and transported 
within the cylinder at the proper time. Various components perform this function. 
Depending on how old a car is, it may use a carburetor or fuel injectors: 



V Carburetors: Carburetors are used on most older cars (pre-1990) to mix the fuel and 
air mechanically. As air moves quickly through the carburetor, it creates a vacuum, 
which draws more and more fuel into the mixture. 

«*" Fuel injectors: Fuel injectors have replaced carburetors on newer cars to perform the 
air/fuel mixture function. (Actually, fuel injectors have been around since the late 
1950s, but they weren't widely introduced until the late 80s and early 90s.) The fuel 
injector acts as the fuel-dispensing nozzle. It injects liquid fuel directly into the engine's 
air stream. In almost all cases, this requires an external pump. 

A doodad called the EFI computer (electronic fuel injection computer) determines the 
amount of fuel entering the engine. The EFI computer receives information from the 
sensors in the fuel, air, and exhaust system, and from that information, it determines 
how much fuel the engine needs to operate at optimum levels. 



A throttle is mechanically connected to the carburetor or electronically connected to the EFI 
computer. Advancing (opening) the throttle causes more fuel to be transferred to the carbu- 
retor or the fuel injectors. The accelerator (the gas pedal) is connected to the throttle by 
mechanical linkages. The harder you push on the gas pedal, the farther the throttle is 
advanced (opened). Thus, more fuel is transported to the carburetor or fuel injectors. 



Cooling system: Acting coot, staying smooth 

Because of the high temperature at which the fuel burns, the engine has a cooling system 
(otherwise, the engine would melt). In this system, water jackets surround the parts that 
reach the highest temperatures. A water pump circulates water through the jackets. While 
the water circulates, it absorbs heat from the engine and then passes through the radiator, 
where outside air cools the water. 



The water in the system is usually mixed with coolant (antifreeze), which raises the boiling 
point of the water (which keeps the water from boiling away) and lowers its freezing point 
(which keeps the system from freezing up during cold weather). 

In addition, the engine parts must be lubricated to prevent them from breaking down, which 
occurs if the metal parts are allowed to rub against each other. An oil pump circulates oil 
through the engine; oil flows through the crankshaft and connecting rods, lubricating as it 
goes. Lubrication reduces friction, which in turn reduces heat. 



130 



Part IV: The Whole Ball of Facts: Technical Skills 



Electrical and ignition systems: Starting up 

Your car requires more than just gasoline to operate. It also needs a supply of electricity. In 
the old days, automotive electrical systems operated on 6 volts. Shortly after World War II, as 
electrical accessories became more prevalent in automobiles, 12 volts became the standard. 

An electric motor powered by the battery starts the engine when you turn the key. This 
motor is called a starter (for obvious reasons). A gizmo called an alternator sends an electric 
current back to the battery to keep the battery charged and also powers the other elec- 
tronic gadgets on your car when the engine is running. 

The ignition system supplies a high-voltage current to the spark plugs to ignite the fuel mix- 
ture in the cylinders. (See the section titled "The engine: Different strokes," earlier in this 
chapter.) The system takes the 12-volt current from the battery, steps it up to about 20,000 
volts, and then sends the current to the spark plugs. 

In older cars, this increase of voltage is accomplished by means of a device called a coil, which 
uses electromagnetic induction to step up the voltage. The current then passes through an 
electrical/mechanical switching device called a distributor. A rotating shaft and a switch within 
the distributor, called breaker points, routes the current through wires to the spark plugs. A 
condenser absorbs excess current and protects the breaker points from damage by the high- 
voltage surge. The distributor and other devices control the timing of the spark-plug discharges. 

In the 1970s, the electronic ignition systems were introduced. In modern ignition systems, 
the distributor, coil, points, and condenser have been replaced by solid-state electronics 
controlled by a computer. A computer controls the ignition system and adjusts it to provide 
maximum efficiency in a variety of driving conditions. 

Dri(/e system: Taking it for a spin 

Having a working engine is all fine and dandy, but the power of the engine still has to be trans- 
ferred to the wheels to make them move. This is the job of the drive system. Cars have drive 
systems that run on axles. The axle is the shaft on which the wheels revolve. The universal 
joint allows the axle to move up and down without breaking the drive shaft. The drive shaft is 
the connecting component that carries torque and transmits rotation. Gears on the axle allow 
the vehicle to make turns. Axle shafts turn the wheels. The wheels on vehicles turn in three 
different ways: 

v* Rear-wheel drive: The rear wheels push the car. The drive shaft extends from the 
transmission to the rear axle. 

v* Front-wheel drive: The front wheels pull the car. The drive shaft extends from the 
transmission to the front axle. 

v* All-wheel drive (four-wheel drive): All wheels push and pull the car at the same time. 
The drive shaft extends from the transmission to both axles. 

Cars also have transmissions. The transmission changes the speed of the engine in relation 
to the speed of the rear wheels (in rear-wheel drive), the front wheels (in front-wheel drive), 
or all the wheels (in four-wheel or all-wheel drive). Vehicles have two types of transmis- 
sions: automatic or manual (stick shift). 

The transmission consists of gears in several combinations so that the amount of torque 
used can vary according to needs. When the terrain is difficult (as in snow), the wheels 
need more torque (the force that produces rotation) in order to move. The transmission 
increases torque as needed. In an automatic transmission, this variation is done automatically 
by the torque converter. In a manual transmission, the driver shifts the gears by hand. The 
clutch is used to facilitate this process by disconnecting the engine from the drive shaft. It's 



Chapter 11: Auto and Shop Information / jj 



necessary to temporarily disconnect the engine in order to change to a different gear 
(torque). The clutch also allows the engine to run when the car isn't moving. 



*£S&> 




Brake system: Putting out alt the stops 

When a vehicle is in motion, you apply brakes to stop that car from moving (a long way 
from the time when Fred and Barney stopped their car by dragging their heels). Each wheel 
has a brake that applies friction to the wheel to stop its rotation. 

A brake system consists of a master cylinder that has brake lines (filled with brake fluid) 
running from it. The brake pedal applies pressure to the master cylinder, which sends pres- 
sure (and brake fluid) through the lines. What happens next depends on the type of brakes: 

v* Drum brakes: In a drum brake, the lines are connected to a hydraulic cylinder on each 
wheel. This cylinder contains pistons that move outward and force two brake shoes 
against the metal drum that rotates the wheel. 

v* Disc brakes: In a disc-brake system, the master cylinder forces a caliper, containing a 
piston, with brake shoes on each side, to squeeze against a rotating disc in each wheel, 
thus stopping your car by using fluid and releasing hot air. 

Most modern cars use both drum brakes and disc brakes. Drum brakes are usually installed 
on the rear wheels, and disc brakes are generally installed on the front wheels. A drum 
brake system usually consists of a rotating drum with shoes that expand to rub the inside of 
a drum. This differs from the disc brake, which uses pads that pinch a rotating disc. 



Emissions-control systems: 
In layman's terms, fitters 



Think of the engine as a giant cigarette and the emissions-control system as a filter. The 
exhaust from automobiles emits pollutants, including carbon monoxide. These pollutants 
are a result of the combustion process (or they're partially combusted fuel or unburned 
fuel). To prevent these pollutants from poisoning the atmosphere, manufacturers place 
emissions-control systems on cars. These systems include the following: 

v* Positive-crankcase ventilation: An old method (still in use) that forces unburned or 
partially burned fuel back into the cylinder so the fuel can be burned 

v* Air-injection system: System that forces air into the engine's exhaust system to burn 
unburned or partially burned fuel before the fuel comes out the exhaust pipe 

J*" Catalytic converter: Oxidizes hydrocarbons and carbon monoxide into water vapor 
and carbon dioxide (the same thing people exhale); this system doesn't control other 
types of pollutants such as nitrogen oxides 

v* Exhaust-gas-recirculation system: Helps control nitrogen-oxide emissions by forcing 
some of the gases back into the cylinders 



Picking Up the Tools of the Trade 



You've probably heard the phrase "the right tool for the right job." This comment is what 
Dad used to yell at you when you'd use a Phillips screwdriver to punch holes in oil cans 
(thereby getting oil on your shirt). The ASVAB folks also believe in using the right tool for 
the right job, and many of the questions on the Auto & Shop Information subtest ask you to 
identify the best tool for certain tasks. 



132 



Part IV: The Whole Ball of Facts: Technical Skills 



Tools are easiest to understand when you classify them by their function, so the following 
sections are divided by function. See Figure 11-2 for an illustration of the various types of 
tools covered. 



Sliding Calipers 



' i ' i ' i ' i ' i ' i ' i ' i ' i ' i ' i ' i ' i ' i ' i ' 



Scale Locking Device 



^ 



Ratchet Handle 



1 ) 



O 



o 





6-point 12-point 
Socket Socket 



O 



Socket 
Wrench 



Deep 
Socket 



C-Clamp 



Calipers 




c 




Butt Chisel 



Mortising Chisel 



Cold Chisel 



Vise Grip Pliers 




Handscrew Vise 



Flat 
Washer 



Split Lock 
Washer 



Shake-proof 
Washer 



Figure 11-2: 

Various 

tools you 

need to 

know for the 

ASVAB. 




Wing Nut 



Cap Nut 



Adjustable 
Pipe Wrench 



Chapter 11: Auto and Shop Information j jj 



Striking toots 



Striking tools apply driving force to an object. (Watch your fingers!) These tools include 
hammers, sledges, and mallets. Here's a brief explanation of the three: 

v* Hammer: A hammer is generally made of metal or plastic and consists of a handle, a 
head, a face (the part of the hammer that touches the nail or other fastener), a claw (to 
pull nails), and a wedge that attaches the head to the handle. The face of a hammer 
may be made of steel, brass, or lead. 

p" Mallet: A mallet is generally made of metal or plastic but may be made of wood, 
rubber, or rawhide. It's used to strike another tool or to strike a surface without dam- 
aging it. A mallet doesn't have a claw like its friend, the hammer. 

p" Sledge: A sledge is generally made of metal. People use it to drive bolts and chisels and 
to break rock. A sledge doesn't have a claw, either. 



Fastening toots 



Fastening tools apply fasteners, such as screws, to objects. (For more info on fasteners, 
check out "Sticking Materials Together with Fasteners," later in this chapter.) Numerous 
tools make up the fastening category: 

p" Stapler: A stapler is a fastening tool. Heavy-duty staplers can staple roofing felt to a 
roof, for instance. 

v* Wrenches: Wrenches turn nuts and bolts. The bolt or nut fits between the jaws of the 
wrench, and the wrench turns the bolt. Some wrenches have adjustable jaws. Not only 
can wrenches be used to turn nuts and bolts, but they may also be used to keep nuts 
and bolts stationary. 

• Open-end wrenches: These wrenches have open jaws. 

• Box wrenches: Box wrenches are closed. Some wrenches have open-end jaws on 
one end and a box wrench on the other. 

• Socket wrenches: Socket wrenches have box-type sockets of varying sizes that 
can be attached to a handle, which in turn can be attached to an extension. 

Note: Socket, box, and open-ended wrenches come in set, standard sizes — either 
in inches or in millimeters. They're not interchangeable. (Selecting the wrong 
socket wrench is how mechanics learn to use cuss words.) 

• Torque wrenches: These wrenches apply additional leverage to a fastener. A 
torque wrench looks much like a socket wrench but has additional internal mech- 
anisms designed to measure and limit the amount of torque (force) being applied. 

• Pipe wrenches: Pipe wrenches have serrated jaws and grip round objects. 

v* Screwdrivers: A screwdriver, in the shop world, turns screws. (In the civilian world, 
it's a yummy drink!) Some special screwdrivers have different blades to fit different 
types of screws: 

• Standard screwdriver: A standard screwdriver has a flat blade at one end of the 
shank (the other end of the shank goes into a handle). 

• Phillips screwdriver: Phillips screwdrivers have a blade that is shaped like a 
cross; this blade fits into a cross-shaped Phillips screw head. 

• Allen wrench: An Allen wrench fits hexagonal screw heads. Nobody knows why 
this tool is called an Allen wrench instead of an Allen screwdriver; after all, it's 
used on hexagonal screws. That's just one of the mysteries of the shop world. 



734 



Part IV: The Whole Ball of Facts: Technical Skills 



$££&> 




The Allen wrench, which was designed in 1943, gets its name from the Allen 
Manufacturing Company of Hartford, Connecticut. 

• Offset screwdriver: Offset screwdrivers have the shank set at an angle to the 
blade to allow the tool to be used in cramped spaces. Offset screwdrivers can 
have a standard blade, Phillips blade, or any number of other blades. 

k" Pliers: Pliers can be used to fasten and unfasten fasteners, hold objects, and cut mate- 
rial. When you squeeze the handles, the jaws of the pliers come together. 

• Long-nosed or needle-nosed: Long-nosed pliers, also called needle-nosed pliers, 
have tapered jaws that can hold small objects or fit into small spaces. 

• Curved-nose: These pliers have curved jaws. 

• Slip-joint: These pliers can be adjusted so the handles lock in a certain position. 

• Wrench or vise-grip: Wrench pliers, or vise-grip pliers, have serrated jaws that 
clamp onto and hold objects of all shapes. 

• Cutting: These pliers are used to cut wire. 



Cutting toots 



Cutting tools use sharp blades to cut through metal, wood, or other materials. Cutting tools 
have teeth. The number of teeth per inch (or points per inch) gives an indication of the type 
of work the saw can do. Because of the way points and teeth are counted, a saw always has 
one more point per inch than tooth per inch. A saw with fewer teeth is used for rough work, 
like cutting wood to size. A saw with more teeth cuts more finely and is used for more deli- 
cate work, like sawing joints and lightweight pieces of wood. Check out Table 11-1 for a 
breakdown of the different cutting tools that may be covered on the ASVAB. 



Table 11-1 


Cutting Tools 


Cutting Tool 


Description/Function 


Bolt cutters 


Heavy-duty shears that produce enough force when the han- 
dles are closed to slice through metal bolts or rods 


Circle snips 


Used to cut curves 


Crosscut saw 


A type of handsaw that cuts against the grain of the wood; the 
shape of the teeth and the angle in which they're set are the 
main differences in this type of saw 


Coping saw 


A type of handsaw that's used to cut curved lines or shapes 


Hacksaw 


A type of handsaw that's used to cut metal; a hacksaw has 
an adjustable frame that holds thin blades of varying length in 
place; a handle is set in one end 


Pipe cutters and tube cutters 


Used to score and cut metal pipes and tubes 


Ripsaw 


A type of handsaw that cuts with the grain of the wood; the 
shape of the teeth and the angle in which they're set are the 
main differences in this type of saws 


Snips and shears 


Snips and shears have two cutting blades that scissor together 
when the handles close; the blades can be curved or straight 



Chapter 11: Auto and Shop Information J 35 




^\NG 




Drilling, punching, and gouging toots 

No, this section isn't about hand-to-hand combat training from basic training. Masters in the 
art of shop often make holes in the material they're working with in order to build that per- 
fect birdhouse (or whatever else they're working on). These holes can be made with a vari- 
ety of tools, which I cover in the following sections. 

Oritts and bits 

Twist drills use drill bits, which are round pieces of steel shaped in a spiral, to create holes. 
Drill bits are attached to a drill (usually a power drill but sometimes a hand drill operated 
by manually turning a crank). The point of the drill bit is sharpened, and the shank is 
smooth and fits into the drill. 

A countersink is a drill bit that enlarges just the surface of a hole so that a screw head can 
be accommodated. A countersink is used to allow the top of the fastener to be set exactly 
even with the material to which it's attached. Without a countersink, the fastener slightly 
protrudes from the material to which it's been attached. 

Auger bits bore larger holes. They're shaped differently from drill bits. They have a long 
deep spiral flute for easy chip removal. They're also much larger. Auger bits are most com- 
monly used with a brace for drilling holes in wood. Their length varies from 7 to 10 inches. 

Punches 

Punches have a sharp end that's placed against the material to be punctured; the other end 
is struck with a hammer. A center punch is used to mark where a drilled hole is to be 
placed; this keeps the drill bit in position and prevents the drill from jumping to another 
part of the material. 

Using a Phillips screwdriver as a punch is bad form in the shop world because hitting the 
handle of a screwdriver with a hammer can damage it (and then you'll get talked about in 
serious shop circles). 

Chisels 

Chisels are made of steel and have a sharp cutting edge. They're used to chip or cut metal 
or wood: 



-«JNG. 




p* Metal-cutting chisels: Chisels that cut metal are usually struck with a mallet to make 
the cut. These chisels have different shapes depending on how they'll be used; cold 
chisels are flat, and they're used for cutting metals without using heating torches or 
forges, whereas round chisels make circular cuts. 

v* Wood-cutting chisels: Some wood chisels, called socket chisels, are also struck with a 
mallet. Other wood chisels require only the pressure of your hands. 

Wood chisels also come in different shapes, depending on what they're used for. A butt 
chisel has a short blade and is used for in-close work. A mortising chisel has a narrow 
blade made for chiseling out the narrow mortises in joints. A framing chisel has a 
heavy, strong blade meant for rough work. 

Because you use chisels with other tools and the pressure of your hands, there's a little bit 
of a risk involved with this tool. One slip and these instruments can easily cut large chunks 
out of your skin, so be careful. 



136 



Part IV: The Whole Ball of Facts: Technical Skills 



Finishing toots 



Filing and finishing shop tools are used to sharpen the blades of other tools and to smooth 
the edges of cut metal objects. Files come in a range of fineness, and the blades can be cut 
in different patterns. Files also come in different shapes to finish different kinds of objects. 
Here are the different kinds of files: 

i*" Single-cut: Single-cut files are used for finishing work and sharpening blades. 

v* Double-cut: Double-cut files are used for rough work. 

«*" Flat files and half-round: These files are for general purposes. 

v* Square and round: These files fit square and round openings. 

Planes are a type of finishing tool used to prepare wood for final finishing and to fit doors 
and trim. Planes consist of a handle to push with, a knob to guide with, a frame, a sole, and 
a mouth (where the blade is). Bench planes are used to smooth surfaces. Longer planes give 
a more uniform surface by shaving off a portion of the wood. 



Clamping tools 



A clamping tool is a device used to hold or fasten objects securely so they won't move while 
you're working on them. There are several types of clamping tools available for many differ- 
ent purposes: 

j-" Pliers: Pliers (discussed in the "Fastening tools" section earlier in the chapter) can be 
used to hold objects while you're working on them. 

v* Vises: Vises hold material while it's being sawed, drilled, or glued. Here are some differ- 
ent types of vises: 

• Bench vise: A bench vise has large, rough jaws that keep the material from 
slipping. 

• Pipe vise: Pipe vises hold round trim or pipes. 

• Handscrew vise: A handscrew vise has two hard, wooden jaws connected by two 
long screws. The screws are tightened to bring the jaws of the handscrew vise 
together. 

v* Clamps: Clamps are used when a vise won't work. Vises generally attach to a work- 
bench, while clamps generally connect only to the items being worked with. C-clamps 
consist of a stationary frame and a screw that moves back and forth to open and shut 
the clamp. 



Measuring toots 



As any shop enthusiast will tell you, the golden rule of shop is to "measure twice and cut 
once." It's frustrating to cut a piece of material only to find it's just a little bit too short 
to fit in the place you intended. Using measuring tools helps you avoid this embarrassing 
situation. 



Tape rules, rigid steel rules, steel (or fiberglass) tape rules, and folding rules are all used to 
measure material. Calipers are also used for very exact and small measurements. Calipers 
can be used with a rule to measure diameter; the legs of a set of calipers curve in to measure 
outside curves and curve out to measure inside curves. Slide calipers have the rule built in. 



Chapter 11: Auto and Shop Information J j7 



Depth gauges measure the depth of holes. Thickness gauges measure the thickness of small 
openings. Thread gauges measure the number of threads per inch in threaded fasteners. 
Wire gauges measure the thickness of wire. 



Leveling and squaring toots 



A square is used to check the trueness (accuracy) of an angle. Because most squares have 
a rule, they can also be used for measuring (see the preceding section). Squares have two 
arms, called the blade and the tongue, that meet at a right angle. A square can be set against 
any angle that is supposed to be a 90-degree angle. If a gap exists between the square and 
the material, the material isn't true — that is, it's not at the specified angle. A sliding T-bevel 
has an adjustable blade so that different angles can be checked. 

Levels show whether a surface is true. A basic level has one or more small tubes filled with 
a liquid (like alcohol) and an air bubble. If the level is placed on a surface and the bubble 
remains exactly in the center of the tube, the surface is level. (This method can't be used to 
see if your recruiter is on the level. I tried it. Recruiters simply won't hold still long enough.) 

A plumb bob is a heavy weight that's suspended from a line. It indicates vertical trueness. 



Sticking Materials Together With Fasteners 

Although wood and metal (and other materials) can be held together with glue, straps, duct 
tape, and other brilliant fastening methods, people usually fasten these types of materials 
with nails, screws, bolts, and rivets. These fasteners offer more strength and stability than the 
white glue that you used to fasten painted macaroni noodles onto construction paper in the 
first grade. 



*£5fe 




Mails 



Nails are used to hold pieces of wood together. The nail head is flat, and the shank is usu- 
ally round. Nail length is designated by the penny system, which is abbreviated with a d. A 
ten-penny nail is a lOd nail. Length and thickness generally correspond. Nails that are larger 
than 20-penny are called spikes and are measured in inches. 



x " The penny system is used in the United States. Penny size indicates the nail's length. The 
higher the penny size, the longer the nail. The penny system is derived from the price of 100 
nails in the 15th century in England. 

Other type of nails include the following: 

v* Brads and finishing nails: They have heads that are made to fit flush with or slightly 
below the surface of the wood. 

i*" Common nails: These nails are the most commonly used nails. (How about that for a 
truly difficult vocab word?) 

v* Double-headed nails: These have two heads, one lower than the other, and a point on 
the other end. The nail is driven to the lower head but can be pulled out of the material 
because of the remaining higher head. These nails are used for temporary construction 
that will be taken apart. 



138 



Part IV: The Whole Ball of Facts: Technical Skills 



The magic of ABS 



In the modern world of cars, most vehicles are equipped 
with an antilock brake system (ABS). The ABS is a four- 
wheel system (usually) that prevents the wheels from 
locking up. The system does this by automatically 
adjusting the brake pressure during an emergency stop. 
This enables the driver to maintain steering control 
and to stop in the shortest possible distance under most 
conditions. 

The theory behind ABS is simple. If your car isn't 
equipped with ABS and you have to stop quickly, your 



wheels simply stop turning when you hit the brakes. 
If your tires don't have much traction on the road, 
your car may continue forward in a skid, even though 
the wheels are locked. You don't stop as quickly as 
you would with ABS, and you won't be able to steer. 
However, with ABS, your wheels are slowed to a stop 
as quickly as possible, without locking up, which gives 
the driver much better control during an emergency stop 
situation. 



Screws and bolts 



Unlike nails, you can easily take screws and bolts out of the wood without causing addi- 
tional damage to the wood (unless, of course, the threads are stripped). These fasteners 
also hold more tightly than nails. Screws have flat heads, round heads, or oval heads; and in 
addition to this classification, they also have standard heads (for standard slotted screw- 
drivers) or Phillips heads (with cross-shape slots). Screw sizes are based on length and the 
diameter of the unthreaded part of the screw. 

Here's the lowdown on these types of fasteners: 

v* Wood screws: Wood screws are used to fasten wood. (Hmm, ingenious!) 

j*" Lag screws: Lag screws have square- or hexagon-shaped heads. 

j-" Bolts: Bolts don't thread into wood. They have flat ends (as opposed to the pointed 
ends of screws). They're held in place by a nut (which is what actually screws into the 
threads) and washer. The body of the bolt may have few threads or many. 

v* Machine screws: Machine screws are used to fasten metal parts. Machine screws are 
sometimes used with nuts. They come in various lengths and widths and have a wide 
variety of heads. 



Nuts and Washers 



Nuts can be square or hexagonal. Cap nuts are rounded and smooth; stop nuts prevent the 
screw or bolt from coming loose. Wing nuts have flanges on each side so they can be tight- 
ened by hand. 

Washers prevent damage to the surface of material by preventing the bolt head from dig- 
ging into the material. They also help keep the bolt (or screw) in place. Flat washers, a 
simple ring of flat metal, are the most common type of washer. Shake-proof washers have 
teeth to prevent them from skipping, while split-lock washers have two ends that dig into the 
nut and the material to keep the screw from slipping out. 



Chapter 11: Auto and Shop Information / $<) 



Rii/ets 



Rivets are commonly used to fasten metal parts together, especially when a weld is insuffi- 
cient. Standard rivets are driven using a bucking bar. Rivets come in a wide variety of 
lengths, diameters, and head shapes. The rivet material should match the material being 
fastened. Pop rivets can be driven when only one side of a joint is accessible. 



Building a Better Score 





If you haven't picked up auto and shop knowledge by this point in your life and want to do 
well on this subtest, one thing you can do is get an automotive manual and take your car 
apart (hoping that you can get it back together again). Then get a woodworking book and 
build some furniture for your mom. (Even if you mess it up, Mom always likes gifts from the 
heart.) 

Or you can check out your local community college, which may be a more practical solu- 
tion. Many community colleges offer basic Auto and Shop classes. You may also want to 
take a gander at the following books, all published by Wiley Publishing: 

K" Auto Repair For Dummies by Deanna Sclar 

u* Woodworking For Dummies by Jeff Strong 

v* Home Improvement All-in-One For Dummies by Roy Barnhart, James Carey, Morris 
Carey, Gene Hamilton, Katie Hamilton, Donald R. Prestly, and Jeff Strong 

On this subtest, you usually either know the answer or you say, "Huh?" However, some 
questions you run into can be answered by using the common sense approach. For exam- 
ple, say you run into a question on the ASVAB that reads something like the following: 

When attaching two pieces of wood together, the most secure bond would be formed by 
using: 

(A) wood screws 

(B) nails 

(C) wood glue 

(D) both A and C 

If you think about it, screws have threads, which are likely to "grab" wood more securely 
than a nail would. Glue would likely strengthen that bond even more. It's obvious that the 
common sense answer would be Choice (D). 

Try a variation of the same question: 

The best fastening method to use when attaching pieces of wood together when time is of 
the essence would be 

(A) wood screws 

(B) nails 

(C) wood glue 

(D) both A and C 



no 



Part IV: The Whole Ball of Facts: Technical Skills 



Making the grade: Octane ratings 



Octane ratings measure gasoline's ability to resist 
engine knock, a rattling or pinging sound that results 
from premature ignition of the compressed fuel-air mix- 
ture in one or more cylinders. Most gas stations offer 
three octane grades: regular (usually 87 octane), mid- 
grade (usually 89 octane) and premium (usually 92 or 
93). By federal law, the ratings must be posted on bright 
yellow stickers on each gasoline pump. 

The octane rating correlates to how much the gasoline 
can be compressed before it ignites spontaneously. 
When gasoline ignites this way, instead of by the spark 
of a spark plug, the engine begins knocking. That's not 
a good thing because early ignition can cause engine 
damage overtime. 

But don't be fooled — that doesn't mean using higher 
octane gas is better. In most cases, using a higher 



octane gasoline than your owner's manual recom- 
mends offers absolutely no benefit. It won't make your 
car perform better, go faster, get better mileage, or 
run cleaner. The only time you may need to switch to 
a higher octane level is if your car engine knocks when 
you use the recommended fuel. This happens to a small 
percentage of cars. Buying higher octane gasoline is a 
waste of money, too. Premium gas costs 1 5 to 20 cents 
or more per gallon more than regular. That can add up to 
hundreds of dollars a year in unnecessary costs. 

How can you tell if you're using the right octane level? 
Listen to your car's engine. If it doesn't knock when you 
use the recommended octane, you're using the right 
grade of gasoline. 



In this case, the best answer would be Choice (B), because pounding a nail in with a hammer 
is generally faster than waiting for glue to dry or screwing a screw in with a screwdriver 
(even in these days of electric screwdrivers). 



WER 




When all else fails, guessing is okay. If you guess, you have a 25 percent chance of guessing 
the right answer. If you leave the answer blank, you have a percent chance. If you're taking 
the computerized version of the ASVAB, you don't have a choice, of course, because you 
must provide an answer before you're presented with the next question. For general guess- 
ing hints, check out Chapter 3. 



Chapter 12 

Mechanical Comprehension 



In This Chapter 

Using the forces of physics 
Figuring out the principles of work 
Manipulating machines to help you work 
Jacking up your test score 



men 




1 

m f your M-16A2 .223 caliber rifle jams on the firing range, knowing how to take it apart and 
«S put it back together will benefit you. Of course, your drill sergeant in basic training will 
be more than happy to teach you this, but how easily you grasp such tasks depends greatly 
on your aptitude for understanding simple mechanical operations. That's the purpose of 
the Mechanical Comprehension (MC) subtest of the ASVAB. 

The questions on this subtest measure your understanding of simple machines and mecha- 
nisms. Many of the questions on this subtest display a diagram, such as a series of gears, 
followed by a question, such as which direction the gears turn or how fast they revolve. 
This subtest is almost all about mechanical physics, so you may want to review some basic 
physics textbooks from your local library. 

Only some military jobs require a good score on this subtest. Turn to Appendix A at the 
back of this book for information about the subtest scores you need to qualify for specific 
military jobs. If you have no interest in taking apart a fighter aircraft or rebuilding a tank, 
you're better off reviewing for the Word Knowledge or Arithmetic Reasoning subtests, 
which make up part of the core exam (the AFQT; see Chapter 1) that you must do well on 
to even qualify for enlistment. 

To ace this subtest, you also have to bone up on your mathematical skills. The Mechanical 
Comprehension subtest often asks you to make calculations based on formulas to explain 
mechanical principles. Don't panic; the formulas are easy to understand, but you do have 
to use math to come up with a final answer. See Chapters 7 and 8 for more information on 
math. In this chapter, you get the mathematical formulas for commonly asked questions on 
the ASVAB, so pay especially close attention to these little beauties. (If the information 
probably isn't on the ASVAB, I don't burden you with it here.) 

The CAT-ASVAB (computerized test) has 16 Mechanical Comprehension questions that 
you're supposed to answer in 20 minutes. For the paper version of the ASVAB, this subtest 
has 25 questions. You have 19 minutes to answer the questions, which is enough time for a 
mechanically oriented individual to tackle this subtest and put a broken clock back together. 
Well, maybe not the whole clock. 



Understanding the Forces of the Universe 

By applying force (a push or pull), you can open the door or close it, speed it up (slam it) or 
slow it down (catch it before it slams), or make it change direction (push it shut when the 
wind blows it open). 



U2 





Part IV: The Whole Ball of Facts: Technical Skills 

* In physics, applying force allows changes in the velocity (the speed and direction) of an 

object. A change in velocity is known as acceleration. Here's the mathematical formula to 
determine force: 

Force = Mass x Acceleration 

Martial artists use this concept all the time. Although a larger fighter may have more size 
(mass), a smaller fighter can usually speed up more quickly (have more acceleration), pos- 
sibly resulting in both fighters' applying the same amount of force. This concept is why 110- 
pound martial artists can break boards and bricks just as well as 200-pound martial artists. 

This section gives you the basics of force that you need to know for the ASVAB. 

He hit me first! The basics of action and reaction 

Sir Isaac Newton sure was one of the sharpest crayons in the box. His third law of motion 
states that for every action (force) in nature, there's an equal and opposite reaction. In 
other words, if object A exerts a force on object B, then object B also exerts an equal and 
opposite force on object A. Notice that the forces are exerted on different objects. 

Take a look at Figure 12-1. As you sit in your chair, your body exerts a downward force on 
the chair, and the chair exerts an upward force on your body. There are two forces result- 
ing from this interaction: a force on the chair and a force on your body. These two forces 
are called action and reaction forces. 



Action 



Figure 12-1: 

An example 

of action 

and reaction 

forces. 




Reaction 



This force can also be used to describe how a motorboat moves through the water. As the 
propellers turn, they push the water behind the boat (action). The water reacts by pushing 
the boat forward (reaction). 



Equilibrium: finding a balance 

Forces are vector quantities. That means that they have both a magnitude (size) and a direc- 
tion associated with them. Forces applied in the same direction as other forces increase the 
total force, and forces that move in opposite directions reduce the total force. In general, an 
object can be acted on by several different forces at any one time. 



Chapter 12: Mechanical Comprehension J A3 



*jABE* 




A very basic concept when dealing with forces is the idea of equilibrium or balance. When 
two or more forces interact so that their combination cancels the other(s) out, a state of 
equilibrium occurs. In this state, the velocity of an object doesn't change. The forces are 
considered to be balanced if the rightward forces are balanced by the leftward forces and 
the upward forces are balanced by the downward forces. 

If an object is at rest and is in a state of equilibrium, then it's at static equilibrium. Static 
means being stationary or at rest. For example, a glass of water sitting on a table is at static 
equilibrium. The table exerts an upward force on the glass to counteract the force of gravity. 



0m* 




Under pressure: Spreading out the force 

Pressure is a measurement of force over an area. Pressure is usually measured in pounds 
per square inch (psi). The formula for deriving pressure is 



Pressure ■ 



Force (in pounds) 
Area ( in inches ) 



*££&> 




If 50 pounds of force is exerted on 10 square inches of surface, the amount of pressure is 
5 pounds per square inch (5 = 50 * 10). 

Consider this: If you're sleeping in bed, the amount of pressure being exerted per square 
inch is much less than when you're standing on your feet. The surface area of the bottoms 
of your feet (supporting all that weight) is much less than the surface area of all your body 
parts that touch the mattress. 

Ever wonder how a person can lie on a bed of nails? The answer involves elementary physics. 
His or her body rests evenly on hundreds of nails; therefore, no individual nail exerts a great 
amount of pressure against the skin. Have you ever seen someone stand on a bed of nails? It's 
unlikely because more pressure is on the feet, and the nails would puncture the feet. 

A barometer is a gauge that measures atmospheric pressure. Normal atmospheric pressure 
is 14.7 psi. A change in air pressure means the weather is about to change. For more infor- 
mation on science and barometric pressure, see Chapter 10. 



Looking at kinds of forces 

Here are some of the forces that act on objects: 

j-" Friction: Resistance to the motion of two objects or surfaces that touch 

v* Gravity: The physical property that draws objects toward the center of the Earth (and 
other objects that have mass) 

p" Magnetism: The property of attracting iron or steel 

j*" Recoil: The property of kicking back when released 

f Static electricity: The production of stationary electrical charges, often the result of 
friction 



In this section, I explain a few of these forces in detail. 



Friction: Resisting the urge to motfe 

When one surface (such as a floor) resists the movement of another surface (the bottom of 
a piano), the result is frictional resistance. (This friction isn't like resisting orders to cut the 



w* 



Part IV: The Whole Ball of Facts: Technical Skills 



grass. That type of resistance may cause friction between you and your dad, but I'm talking 
about a different kind of resistance here.) 

In order to perform work — that is, to get an object to move in the direction you're pushing 
or pulling — sometimes you have to overcome friction by applying more force. For example, 
when you're moving a piano across a smooth, vinyl floor, little friction is produced, so the 
amount of force required to push the piano comes from the piano's weight and the very 
minor friction produced by the smooth floor. But when you're moving a piano across a car- 
peted floor, more friction is produced, so you have to push harder to move the same piano 
the same distance. (See the later section "You Call that Work?!" for more information on 
what's considered work in physics terms.) 

Rolling friction (like the friction that occurs when you roll a wheel along the pavement) is 
always less than sliding friction (which occurs when you shove a piano along the floor). If 
you put wheels on a piano, it's much easier to push! 

You can decrease friction by using a lubricant. Oil, grease, and similar materials reduce fric- 
tion between two surfaces. So theoretically, if you oil the bottom of a piano, it's easier to 
move! Note: Oiling the bottom of your piano isn't recommended (for reasons involving the 
appearance of your floor and piano). 



WER 




Gravity: What goes up must come dovin 

Sir Isaac Newton invented gravity in 1687 when he failed to pay attention while sitting under 
a tree and got bonked on the noggin by an apple. Before that, gravity didn't exist, and every- 
one just floated around. Okay, I'm kidding. Isaac Newton didn't invent gravity. But the 
famous mathematician was the first to study gravity seriously, and he came up with the 
theory (now a scientific law) of how gravity works. 

Newton's law of universal gravitation states that every object in the universe attracts every 
other object in the universe. The Earth produces gravity, and so do the sun, other planets, 
your car, your house, and your body. Even this book you're reading produces gravity. The 
amount (force) of the attraction depends on the following: 

v* Mass: The force of gravity depends on the mass of (amount of matter in) the object. If 
you're sitting in front of your television, you may be surprised to know that the televi- 
sion set is attracting you. However, because the mass of the TV is so small compared 
to the mass of the Earth, you don't notice the physical "pull" toward the television set. 

Note that the force of gravity acting on an object is equal to the weight of the object. Of 
course, other planets have lesser or greater masses than the Earth, so the weight of 
objects on those planets will be different. 

f Distance: Newton's law also says that the greater the distance is between two objects, 
the less the objects will attract each other. In other words, the farther away an object 
is from the Earth (or any large body), the less it will weigh. If you stand at the top of a 
high mountain, you will weigh less than you will at sea level. Don't get too excited 
about this weight-loss technique. Gravitational pull isn't the next big diet craze. The 
difference is incredibly small. Sorry! 

For an object to really lose weight, it must be far away from the Earth (or any other 
large body). When an object is far enough away from these bodies that it experiences 
practically no gravitational pull from them, it is said to experience weightlessness — 
just like the astronauts you see on TV. 

Gravity pulls objects downward toward the center of the Earth, so the old saying "what 
goes up must come down" is appropriate when discussing gravity. If you fire a bullet 
straight up into the air, it will travel (overcoming the force of gravity) until it reaches its 
furthest or highest point, and then it will fall. 



Chapter 12: Mechanical Comprehension J hjy 



Centrifugal force: False gravity 



An object traveling in a circle appears to experience a 
gravitational force. This isn't really gravity, but instead 
it's a concept known as centrifugal force. The amount 
offorce depends onthe mass of the object, the speed of 
rotation, and the distance from the center: 

u* The more massive the object, the greater the force. 

j"* The greater the speed of the object, the greater the 
force. 

C" The greaterthe distance from the center, the greater 
the force. 

The centrifugal force, or effect, on an object is actu- 
ally a fictitious outward force on an object moving along 
a curved path, which can be equal to the centripetal 



force on an object. Centripetal force points toward the 
center of an object's circular path, perpendiculartothe 
direction of motion of an object. 

If you're riding on a merry-go-round onthe playground 
(wee!), you have to exert a constant force to keep from 
flying off. This feeling of being pushed outward isn't due 
to something actually pushing you in that direction but 
to your body's inertia trying to keep you moving in a 
straight line. Because one of Newton's laws states that 
moving objects tend to want to travel in one direction, 
as the merry-go-round turns, your body wants to keep 
traveling in one direction (tangentto the circle, if you like 
math), so you feel you're being pushed outward. 



Applying force to Mo ends: Tension 

Tension force is the force transmitted through a rope, string, or wire when force is applied 
to both ends. The force is the amount of tension directed along the rope, string, or wire 
and pulls equally on the objects at both ends. Tension force is usually measured in either 
pounds-force or newtons (N); 4.45 newtons equal 1 pound-force. See Figure 12-2. 

Elastic recoil: The trampoline of physics 

Liquids and gasses don't have a specific shape, but solid matter does. Solids are perfectly 
happy with the way they look and resist changes in shape. If you exert a force on a solid shape, 
it responds by exerting a force in the opposite direction. This force is called elastic recoil. 

Take a look at Figure 12-3. The cat is standing on a board suspended on two blocks. While 
the board bends, the cat can feel the force of the board trying to regain its original shape. 
If the cat steps off the board, the board will spring back to its normal state. 



Force = 150 pounds-force 








Figure 12-2: 

An example 

of tension 

force. 




Tension = 150 pounds-force 



Weight = 150 pounds 



U6 



Part IV: The Whole Ball of Facts: Technical Skills 



Figure 12-3: 

The concept 

of elastic 

recoil. 




\lou Catt That Work!! 



*JHBE« 




Mechanically speaking, work happens when a force (usually measured in pounds) moving 
over a measurable distance (usually measured in feet) overcomes a resistance. In the 
United States, the unit of measure for work is often called a foot-pound. One foot-pound of 
work occurs when a 1-pound weight is lifted to a height of 1 foot. You can represent this 
concept in equation form: 

Work = Force x Distance 

Work is different from effort; work is the result of effort. You can think of effort as being 
force and of work as being what you produce with that force. 



Overcoming resistance 



The resistance that the work overcomes isn't the same thing as the weight of the object. 
(If you've ever tried to put your freaked-out cat in a cat carrier to go to the vet, you know 
what I mean.) In other words, if you try to move a 1,200-pound piano, you'll probably notice 
a measurable difference between the amount of work it takes to shove it along the floor and 
the amount of work it takes to carry it up the stairs. But don't take my word for it — you can 
demonstrate this concept at home. First, find a 1,200-pound piano and push it across the 
floor. Next, put it on your back and carry it up the stairs. See the difference? (Really, don't 
put the piano on your back. I'm just trying to make a point here.) 

When you move the piano across the floor, you're really working (pushing) against the fric- 
tional resistance (the force that's produced when two surfaces rub together) of the piano 
rather than its full weight. Under these circumstances, the frictional resistance of the piano 
offers less resistance than its full weight. There are times when an object's full weight is less 
than its frictional resistance. Consider trying to push a textbook across a deep-pile carpet. 
Picking the book up and carrying it is easier. (For more about friction, see the earlier sec- 
tion "Friction: Resisting the urge to move.") 



cjABE* 




Gaining porter by Working more quickly 

Power is the rate of work. If Mary Lou is able to lift more 50-pound sacks of potatoes onto the 
truck bed in ten minutes than Joe is, Mary Lou is more powerful than Joe. Mathematically 
speaking, 



Power = 



Work 
Time 



Chapter 12: Mechanical Comprehension I (l7 



cjUBE* 




In this formula, work is usually measured in foot-pounds, time is measured in minutes, and 
power is measured in foot-pounds per minute. However, the unit of measure for power is 
commonly put in terms of horsepower (hp). 

Horsepower is derived from the estimate that an average horse can do 33,000 foot-pounds 
of work in one minute (according to James Watt). Therefore, 1 horsepower = 33,000 foot- 
pounds per minute. One horsepower is also the same as 550 foot-pounds per second. 



Reiyinq on Machines to Help \lou Work 

Ever since Zog crawled out of his cave and invented the wheel to help him carry fur coats to 
his girlfriend, mankind has made use of machines that help him to make work easier. 

In addition to increasing efficiency, machines are also used to help with work that couldn't 
be done otherwise. Think of the mechanisms and machines you use everyday — from the 
simple (like the hinge that allows a door to move easily when you push it open) to the more 
complex (like the hydraulic lift that allows you to lift a car up to check its underside). You 
could move most doors out of the way without hinges, but you couldn't lift a car over your 
head without some help. 



<$m 




Machines give you the ability to magnify and change the direction of forces. When a 
machine multiplies the force you use, it gives you a mechanical advantage. This concept can 
be stated as 



Mechanical Advantage ■■ 



Resistance Output Force 



Effort 



Input Force 



Some simple machines may give you a mechanical advantage of only 1 or 2. This means that 
they enable you to do one or two times the amount of work by expending the same effort. 
But those simple machines are still worth using! Often, even if a machine doesn't multiply 
your effort (or doesn't multiply your effort by much), it can at least spread your effort out 
and make it more effective. 

Machines make work easier by providing some trade-off between the force applied and the 
distance over which it's applied. Keep reading to find out more on some basic types of 
machines. 



Using tetters to qow advantage 

You may not think of the seesaw at the neighborhood park as a machine, but it is. It's a 
lever. Levers are among the simplest machines used to help increase force. 

All levers work by using a fulcrum (point of support) to reduce resistance and multiply the 
effect of effort. Resistance is exerted at one end of the lever (the resistance arm) and effort is 
exerted at the other (the effort arm). The effort arm moves the resistance arm. See Figure 12-4. 



Figure 12-4: 

A simple 
lever. 




U8 



Part IV: The Whole Ball of Facts: Technical Skills 

To determine how much a lever reduces the amount of effort needed to do work, use the 
following formula: 

Length of Effort Arm Resistance Force 




Length of Resistance Arm Effort Force 



*i*!§? 




As you can see, the amount of effort needed to move the lever varies depending on how 
long the effort arm is and how long the resistance arm is. Keep in mind that a short resis- 
tance arm, although easier to move, can't move an object as far through space as a longer 
resistance arm can. 

The mechanical advantage of using a lever can be stated as 

Mechanical Advantage - Effort Arm — 
° Resistance Arm 



If the effort arm is 6 inches and the resistance arm is 3 inches, the mechanical advantage is 2. 
If the effort arm is 6 feet and the resistance arm is 3 feet, the mechanical advantage is still 2. 



tfWEK 




Ramping up the inclined plane 

The inclined plane, also called a ramp, is another very simple machine that makes moving 
an object from one point to another easier. The ramp spreads your work out over a longer 
distance, so less force is needed to do the work. 

For instance, suppose you have to lift a 50-pound barrel to a truck bed that's 3 feet off the 
ground. You would have to use 50 pounds of force for 3 feet to move the barrel. But if you 
put a 6-foot ramp in place and push the barrel up the ramp, you'd only use half as much 
force to get the barrel in the truck (assuming there's no friction) because the mechanical 
advantage of such a ramp is 2. 

The advantage of using a ramp can be expressed as 



Length of Ramp 
Height of Ramp 



Weight of Object Being Moved 
Force Required to Move Object 



&M* 




Wedges are a form of inclined plane and can multiply your effort in much the same way as a 
ramp can. Screws are also inclined planes, only in spiral form. Screw jacks, which you can 
use to lift your house up to build a new foundation, are a combination of a lever and an 
inclined plane. 



Easing your effort: Pulleys and gears 

Pulleys and gears are simple machines that can be used to change the magnitude (size) and 
direction of force. When you ride in an elevator, step onto an escalator, drive your car, or 
wind up your watch, you're using pulleys and gears. 

Block and tackle systems 

When used in a block and tackle arrangement (see Figure 12-5), pulleys make lifting heavy 
objects easier. In block and tackle systems, pulleys can also be used to change the direction 
of your pull. If you tie a 200-pound crate to one end of a rope, run the rope through a pulley, 
and grab the other end of the rope, you can pull down on the rope to lift the crate up. 
Without a pulley, you could pull down on the crate all day, and it wouldn't go up. In this case, 



Chapter 12: Mechanical Comprehension J h Q 



using a simple pulley, the force of your pull must equal the weight of the object being lifted. 
The regular pulley doesn't multiply your force, but it makes the process of lifting easier. 

Using a block and tackle allows you to distribute your force more effectively. Instead of 
hoisting that entire 200-pound crate in one try, you can pull on a rope to lift it a few inches, 
pull on the rope some more to lift it a few more inches, and so on. This makes the work 
easier to perform. 

A block and tackle system can also be used to reduce effort by magnifying force. To help 
understand how this works, look at Figure 12-6: 

v* Example 1 shows a 100-pound box secured to the ceiling by a single line. The weight 
supported by the line is equal to the weight of the box. 

v* In Example 2, the box is secured to the ceiling by using two lines. Each line is support- 
ing one-half the weight of the box. 

v* In Example 3, a single line is threaded through a pulley. Although the line (as a whole) 
is supporting the entire weight of the box, each section of the line is supporting only 
one-half of the box's weight, just as in Example 2. 

v* In Example 4, a man is using this principle to lift the 100-pound box by applying only 
50 pounds of force. In short, this block and tackle system provides the man with a 
mechanical advantage of 2. In receiving a mechanical advantage of force, the man must 
pull the rope farther than if he weren't using a pulley. In this example, the man would 
have to pull 2 feet of rope to raise the box 1 foot. 

Additional pulleys can be added to a block and tackle arrangement to further increase the 
mechanical advantage. Figure 12-7 shows a couple of examples: 

j-" In Example 1, three sections of rope produce a mechanical advantage of 3. Lifting a 
weight with this pulley arrangement requires only K of the effort required to lift the 
weight directly. However, in order to lift the crate 1 foot, you have to pull 3 feet of rope. 

j-" Example 2 illustrates a block and tackle system with six sections of rope. Using this 
arrangement provides you with a mechanical advantage of 6, but you have to pull the 
rope 6 feet for every foot you want to raise the box. 



Figure 12-5: 

A pulley 

used in 

a block 

and tackle 

system. 





Block and tackle 
(pulley) 



EFFORT 



200-pound crate 



150 



Part IV: The Whole Ball of Facts: Technical Skills 



100 lbs. 




Example 1 



Example 2 



Figure 12-6: 

Reducing 

effort by 

using a 

block and 

tackle. 



y~s" 






100 lbs. 



Example 3 



Example 4 



Figure 12-7: 

Two 
examples 
of a block 
and tackle 
arrange- 
ment. 




Example 1 




Example 2 



$*** 




Understanding how gears Work 

Machines often use gears to transmit motion from one place to another. An additional 
advantage of using gears is that they can be used to change direction, increase or decrease 
speed, or increase or decrease force. 

Gears arranged in a series turn in the opposite direction of each other. If you have an even 
number of gears connected in a series, the first and last gear turn in opposite directions. If 
you have an odd number of gears aligned in a series, the first and last gear spin in the same 
direction. Look at Figure 12-8. Gear 1 is rotating counterclockwise, which causes Gear 2 to 
turn clockwise, resulting in Gear 3's spinning counterclockwise, with Gear 4 turning clockwise. 



Chapter 12: Mechanical Comprehension /SI 



Figure 12-8: 

The motion 
of gears 
with an 
even num- 
ber of gears 
aligned in a 
series. 



Ov 




cjftBE/f 




The speed at which a gear rotates (in relation to the driving gear connected to it) depends 
on the number of teeth. In Figure 12-9, Gear 1 has six teeth, and Gear 2 has eight teeth. This 
relation of teeth can be expressed as a ratio of 6:8, which can be further reduced to 3:4. 
That means that Gear 1 has to rotate four times in order for Gear 2 to make three revolu- 
tions. Or expressed another way, for each rotation made by Gear 1, Gear 2 will make three- 
quarters of a revolution. 



6 teeth 



8 teeth 



Figure 12-9: 

The ratio 

of teeth 

between 

two gears 

affects 

rotational 

speed. 




Ratio : 



When gear shafts aren't parallel to one another, bevel gears can be used to connect gears that 
have shafts at different angles. The principles of gear rotation remain the same. Figure 12-10 
shows an example of bevel gears designed to connect shafts having a 90-degree angle to the 
other. 



Figure 12-10: 

Two bevel 

gears meet 

at a right 

angle. 




W? 



152 



Part IV: The Whole Ball of Facts: Technical Skills 



PuNey and belt arrangements 

In addition to magnifying force as part of a block and tackle system, pulleys have another 
use. When connected by a system of belts, pulleys can drive other pulleys. 

Like gears, pulleys are used to transmit motion from one location to another. However, the 
physical properties of pulleys are different from those of gears: 

v* Turning direction: Unless the driving belt is reversed (twisted), pulleys connected in 
series rotate in the same direction. Figure 12-11 illustrates this concept with two sets of 
pulleys. In the first set of pulleys, all the pulleys turn in the same direction (counter- 
clockwise) as the driving pulley. However, in the second set of pulleys, the driving 
pulley and the lower pulley are rotating counterclockwise, but the right-hand pulley is 
rotating in a clockwise direction because the belt is twisted. 



Figure 12-11: 

Pulleys 
rotate in the 
same direc- 
tion unless 
the belt is 
reversed. 



Driving 
Pulley 




Driving 
Pulley 




i*" Speed of rotation: Although the speed of gear rotation is determined by the number of 
teeth, how fast a pulley rotates depends on the diameter of the pulley in relation to the 
diameter of the pulley that's driving it. Have a look at Figure 12-12. Pulley A has a diam- 
eter of 1 inch, Pulley B has a diameter of 2 inches, and Pulley C measures 4 inches in 
diameter. The ratio among the three pulleys is 1:2:4. For every complete revolution 
made by Pulley A, Pulley B makes half of a revolution. Each time Pulley B makes a 
full revolution, Pulley C makes half of a revolution. Thus, for every full revolution of 
Pulley A, Pulley C makes a quarter of a revolution. 



Multiplying yow effort: Wheels and axles 

The wheel-and-axle machine multiplies the effort you use, producing a greater force. When 
you steer a car by using a steering wheel (which is a wheel-and-axle device), a little effort 
exerted on the steering wheel turns the wheels of the car in the direction you desire. Turning 
your car wheels would be a lot more complicated if you didn't have the steering wheel. 

In true wheel-and-axle machines, the wheel and the axle are fixed together and turn at the 
same time. This arrangement multiplies the amount of force you can exert by a consider- 
able amount. 



Chapter 12: Mechanical Comprehension / 3^3 



o 



Figure 12-12: [ 


o S^^*^^ 


Pulley rota- \ 


b y^-^-^^^ 


tion speed 




is based on 




the pulley's 


Pulley A Diameter = 1 inch 


diameter. 


Pulley B Diameter = 2 inches 




Pulley C Diameter = 4 inches 



The relationship between the radius of the wheel and the radius of the area to which force 
is being applied determines the mechanical advantage you receive by using this piece of 
equipment. (Remember, the radius of a circle equals half the diameter; a straight line 
extending from the center of the circle to the edge is the radius of a circle.) A hand drill may 
apply 200 pounds of force for your 10 pounds of effort. (A hand drill uses a gear to convert 
the direction of the force.) See Figure 12-13. 



Figure 12-13: 

A hand drill. 



Force required 

to turn handle 

IE) 



Mechanical advantage of 
wheel and axle = 



Radius of 
wheel's circle 

m 



%w~ 



E 



Width of 
drill tip (W) 




Resistance 
offered by 
material (S) 



Getting a qxip on things With Vises 

Although many mechanisms are designed to transmit motion, some machines have the pur- 
pose of keeping things motionless. Vises are very useful because they can close around 
items and hold them with great force (much greater force than you could do by holding the 
item in your hands). Figure 12-14 shows an illustration of a standard shop vise. 



754 



Part IV: The Whole Ball of Facts: Technical Skills 



Figure 12-14: 

A standard 
shop vise. 




Rotating the handle on the vise causes a screw to turn, which either tightens or loosens the 
vise. A screw is a cylinder wrapped in a continuous spiral. The distance between the ridges 
of the spiral is called the pitch of the thread. The greater the pitch of the thread, the farther 
the jaws of the vice move for each revolution of the handle. However, there's a trade-off. 
Larger pitches require more force to rotate the handle than screws with smaller pitches do. 



Magnifying your force With liquid: Hydraulic jacks 

A hydraulic jack uses a nearly incompressible liquid, such as oil, to exert force in order to 
move an object (see Figure 12-15). As the handle moves, it applies pressure to the oil. 
Because the oil doesn't compress, the oil transmits whatever force is applied to it to the 
work cylinder with no (or little) loss in efficiency. The mechanical advantage is the ratio 
between the diameters of the two cylinders. 

In the figure, the small cylinder has a diameter of 1 inch and the large cylinder has a diame- 
ter of 4 inches. This difference in diameter results in a mechanical advantage of 4. If the 
rocks weigh a total of 100 pounds, only 25 pounds of force has to be applied to the piston in 
the small cylinder in order to lift the load. However, although the force required is reduced 
by a factor of 4, the smaller piston has to move 4 feet for every foot the piston in the larger 
cylinder moves. 



Force 



Figure 12-15: 

A hydraulic 
jack. 



Force 




Chapter 12: Mechanical Comprehension / QQ 




Working \lour Way to a Better Test Score 

When you take the Mechanical Comprehension subtest, you may not know the correct 
answer to a question, or you may not know the mechanical principle involved. You may 
know the mechanical principle but not remember the formula you need to come up with the 
right answer. Never fear — you can still stumble through this test without totally flaming out. 

Using yow observations and common sense 

Questions on this subtest often include illustrations. The ASVAB test-makers expect you to 
look at the illustrated device and guess how it operates. When you run across these types 
of questions, make sure that you understand the illustration. Often, parts of the device are 
labeled. Make certain you read and understand these labels before you try to answer a 
question about the illustration. 

Also, try to use a common-sense approach. You may see the following question: 

Which of the following controls an automatic sump pump? 

(A) mechanical switch 

(B) manual switch 

(C) pneumatic valve 

(D) float 

You may not know the answer to this question, but you can rule out Choice (B), manual 
switch, because the question asks you about an automatic sump pump, and anything 
manual isn't automatic. Eliminating one choice narrows your chances from one in four to 
one in three. Not a bad start, huh? 

A sump pump is used to drain water from an area, and if you know that, you have an even 
better shot at getting this question right. Think about what type of device detects the pres- 
ence of water, and you may guess correctly that Choice (D), float, is the right answer. 

You can answer a lot of the questions correctly if you just think about what you've observed 
in the world around you. Remember, the Mechanical Comprehension subtest also tests your 
knowledge of physical principles of the world around you — questions you may expect to 
find on the General Science subtest. For example, a question may ask something like this: 

If all the following objects are the same temperature, which one will feel coldest on a 
cool day? 

(A) a wooden spoon 

(B) a plastic spoon 

(C) a metal spoon 

(D) a fiberglass spoon 

You don't need to know mechanical or scientific principles to know that a metal spoon will 
feel colder than the other spoons. So it makes sense to select Choice (C) as your answer, 
even if you can't explain the science behind this correct answer. 




156 



Part IV: The Whole Ball of Facts: Technical Skills 



*2£&> 




The nerve endings in your skin detect the difference between your inside body temperature 
and your outside skin temperature. Metal is an excellent conductor of heat, so heat readily 
flows from your hand into the metal. The heat is conducted rapidly away into the bulk of 
the metal, leaving your skin surface relatively cool. That's why metal feels cooler than 
other, less efficient conductors of heat, such as wood, plastic, or fiberglass. 



^PLE 




Using the mathematics of mechanics 

Mechanical principles are based on mathematical principles. Therefore, a screw making 
a complete revolution turns 360 degrees, because a mathematical principle states that 
360 degrees are in a circle, a complete revolution. If you have to know the surface area of a 
floor in order to determine the pounds per square inch that a ton of tile would put on the 
floor, that's a mathematical principle, too (Area = Length x Width). 

Suppose you run across this question: 

A 3-inch-diameter flanged pipe with six holes is being fitted to a base with six holes. What's 
the maximum number of degrees the pipe must be rotated in order to line up the holes? 

(A) 120 degrees 

(B) 180 degrees 

(C) 60 degrees 

(D) 360 degrees 

This isn't really a Mechanical Comprehension question at all — it's a math question. The only 
part that requires mechanical knowledge is knowing that the holes are spaced equally distant 
from one another on a flanged pipe. The answer is 360 degrees * 6 = 60 degrees, Choice (C). 




Guessing With a mechanical mind 

Like most of the other subtests on the ASVAB, you can and should guess on the Mechanical 
Comprehension subtest when you don't know the answer. Check out these tips to help you 
narrow the field: 



v* The amount of force needed to move an object (not including friction resistance) is 
never greater than the weight of the object. Any answer that includes a force that's 
greater than the weight of the object being moved is probably wrong. 

j*" The correct answer is a mechanical answer. For example, if the question asks, "What's 
the purpose of lubricating oil in an engine?" the correct answer won't be "to make the 
parts look shiny." The answer may be "to reduce friction between moving parts." 

u* Any change in a mechanical operation almost always has pluses and minuses associ- 
ated with it. So when a question proposes a change, the correct answer is probably the 
one that specifies the good, the bad, and the ugly. For instance, suppose the question 
says, "Enlarging the wheel on a hand drill will . . . ?" The correct answer is the one that 
says something like "increase the mechanical advantage and decrease the amount of 
effort needed to operate the drill." 



For more general tips on guessing on the ASVAB, flip to Chapter 3. 



Chapter 13 

Electronics Information 



In This Chapter 

Understanding current, voltage, power, and more 
Comprehending electrical flow 
Deciphering circuit diagrams 
Amplifying your test score 



VUBE/? 




■ ^^hen I was around 12 years old, I impressed my parents by taking an old television 
▼ ▼ set apart and putting it back together. I impressed them right up to the point where 
I plugged it in and blew up the garage. But the world of electronics is a bit more complex 
than simply plugging something in and seeing whether it works. I (and the garage) learned 
this lesson the hard way. 

Six years later, when I took the ASVAB, I scored very well on the Electronics Information 
subtest. (Go figure!) This subtest is designed to measure your knowledge of the principles 
of electricity and how these principles are applied in the real world. You may see questions 
about transistors, magnets, engines and motors, and radio and television. (Curiously, there 
are no questions on this subtest concerning the impromptu demolition of garages.) 

You don't have to be an electronics whiz to score well on this subtest. If you're not familiar 
with this information and you want to pursue a military career that requires you to do well on 
this subtest, this chapter is calling your name. You also need to have some familiarity with 
basic mathematical and algebraic principles (see Chapters 7 and 8 for more information). 

Not every military career requires a good score on this subtest. (Turn to Appendix A to find 
out which military jobs require a score on this — and other — subtests.) If the military feels 
that the Electronics Information subtest is important to your desired career, study inten- 
sively for this test. You can even take a course or two at the local community college if you 
don't have a strong enough background in this area. If, however, you don't intend to pursue 
a career that requires a score on this subtest, spend your time studying for other areas of 
the ASVAB. 

You have 9 minutes to answer 20 questions on this subtest on the paper version of the 
ASVAB and 8 minutes to answer 16 questions on the computerized ASVAB. Although 8 or 
9 minutes is sufficient time to answer the questions, it doesn't provide much time for any- 
thing else — if you don't know an answer, guess and go. 



Uncovering the Secrets of Electricity 



One day in 1752, Benjamin Franklin was minding his own business, flying a kite in a storm. 
A key was tied to the kite string and when lightning struck the metal key, Ben was struck by 
the notion that lightning must be electrified air (well, it happened something like that). 



158 



Part IV: The Whole Ball of Facts: Technical Skills 



*J*BEft 




■0*5!? 




Although electricity was just a hobby for Ben Franklin, he made many important contribu- 
tions. As a result of his famous kite flight, he created many of the terms used today when 
folks talk about electricity: battery, conductor, condenser, charge, discharge, uncharged, 
negative, minus, plus, electric shock, and electrician. 

Electricity is a general term for the variety of phenomena resulting from the presence and 
flow of electric current. You can't see electricity running through a wire (but you can cer- 
tainly feel it). You only know electricity is there when you flip on the light switch and the 
light turns on. Even though electricity appears to be pretty mysterious at first glance, scien- 
tists understand a great deal about its properties and how it works. 

Electricity is measured in three different ways: 

j*" Volts: Volts measure the difference of potential between two points. 

f Amperes (amps): Amps measure the number of electrons that move past a specific 
point in 1 second. 

v* Ohms: Ohms measure resistance, including anything that could limit the flow of 
electrons. 

Here are some other electricity terms that are important for you to know for the ASVAB: 

v* Current: Electricity is like water — it flows. Electrical current occurs when electrons 
move from one place to another. The use of conductors, such as copper and water, 
allows the electrons to move freely. Insulators, such as rubber and wood, discourage 
the electric current. 

v* Watt: A watt measures power, the rate at which electrical energy is consumed or trans- 
formed into another type of energy, such as light or heat. 

v* Watt-hour: A watt-hour is the amount of energy used in 1 hour at a rate of 1 watt. Most 
electricity is measured in kilowatt-hours, which is how much energy you'd use if you 
ran a 1,000-watt (1-kilowatt) device for an hour. For example, 10 kilowatt-hours is 
enough energy to run a 10,000-watt speaker system for an hour-long outdoor concert, 
or it could run a 5,000-watt air conditioner for 2 hours or a 1,000-watt waffle iron for 
10 hours. You find watt-hours by multiplying wattage by time (expressed in hours). 

The following sections explain electricity in more detail. 



Measuring Voltage: Do you hai/e the potential} 

A circuit is just the path of an electrical current. A very simple circuit consists of several 
components. For example, it may consist of a battery, one side (terminal) of which is con- 
nected by a conductor (a wire) to an on/off switch, which is connected to a lamp (a light 
bulb) by another wire, which is then connected back to the other side of the battery. As 
long as the switch is off — which means it's set to a position so that there's an open (liter- 
ally an open space) in the circuit — current cannot flow. When you flip the switch, there's a 
short (meaning the open space has been closed), and current can flow from one side of the 
battery, through the closed switch, through the light bulb, and back to the other terminal of 
the battery, all by way of the wires connecting the components. 

Voltage, which is supplied by the battery in this circuit, is the difference of the pressure 
between two points in a circuit. It is sometimes called the voltage drop or difference of poten- 
tial. So, for instance, a 9-volt battery supplies 9 volts of electricity. To see what the voltage 
is anywhere in a circuit, you have to compare the voltage at that point to ground. Ground is 
any part of a circuit (or other object that has electricity running through it) that measures 



Chapter 13: Electronics Information J jQ 



$mn 




volts, such as the case of your radio, the base of a lamp, or the chassis of your car. The 
negative terminal of a 9-volt battery is at ground potential, so the voltage from the negative 
terminal to ground will measure volts. The voltage from the positive terminal to either 
ground or the negative terminal of the battery will measure 9 volts. 

To measure voltage in a circuit, you use a voltmeter or a multimeter, which has several 
meters in one instrument. A voltmeter has two leads. To measure voltage, you place one 
lead somewhere in the circuit and one lead at another location in the circuit. The voltmeter 
tells you what the voltage is between those two points. 

A cell (a storage compartment for electricity in a battery) has a specific voltage. For exam- 
ple, in a particular battery, cells may be 1.5 volts. Therefore, you can figure out the number 
of cells a battery has by dividing the voltage of the battery by 1.5. Pretty handy stuff, huh? 



Examining the current of the electrical riVer 

Electrons are negatively charged, and they attempt to shift from one atom to the next, to the 
next, trying to get to a positive charge, such as the positive side of a battery. They're able to 
shift if the material is a conductor. But if the material is an insulator, the electrons will be 
much, much less able to shift because of the insulating material's molecular structure. 

Electrical current is the flow — or, more precisely, the rate of flow — of electrons in a con- 
ductor. Current flow can be expressed in terms of coulombs (abbreviated C), which mea- 
sure charge. A coulomb is the amount of electricity provided by a current of 1 ampere 
flowing for 1 second. It's called a coulomb because a guy named Charles de Coulomb dis- 
covered it in the late nineteenth century, and the rules say that if you discover something, 
someone will stick your name on it (not sure who made up those rules, though). 

If 1 coulomb (about 6,241,500,000,000,000,000 electrons) flows past a specified point in 
1 second, that's a flow rate of 1 ampere (amp, abbreviated A). An ampere represents the 
strength of a current. For the sake of convenience, electrical currents are measured in 
amps. Typically current is tiny, so small that it's measured in milliamperes; 1 milliampere is 
one-thousandth of an ampere. Current meters, called ammeters, measure the flow of current 
through a circuit. 

The amount of voltage (the difference in potential) and the resistance in a circuit determine 
the number of amperes along a wire — or whatever you're using to conduct the electricity 
from one place to another. More voltage (for instance, a higher-voltage battery) means that 
more amps flow in a wire (or conductor). You can read more about this relationship in the 
next section, which discusses Ohm's law. 



Resistance: Stowing the electrical rii/er 

Current doesn't just flow in any properly working circuit unimpeded. Resistance pops up 
along the way. If the flow of electricity needs to be regulated, resistance is deliberately set 
up in a circuit. If the flow weren't regulated, the motors powering devices like can openers 
and microwave ovens would quickly overheat and melt. (But before that happens, hopefully 
a fuse would blow or a circuit breaker would trip, halting current flow and saving the equip- 
ment.) In a sense, even a wire, such as a filament in a light bulb, is a type of resistance and 
is a way to deliberately create circuit resistance. 



160 



Part IV: The Whole Ball of Facts: Technical Skills 



Adding or removing resistance 

Sometimes a circuit must be opened in order to add or remove resistance. In other words, 
the flow of the electricity must be interrupted in order to physically change the resistance. 
Using a circuit breaker, which is a device that automatically interrupts the electrical current, 
is an example of opening a circuit to control the current. When the circuit breaker trips, the 
electrical device can no longer operate. 

Some devices use a rheostat, which can vary the resistance without opening the circuit — 
the device can continue to work even as the resistance is altered. If an application doesn't 
use all the electricity, the rheostat absorbs it. A dimmer switch on a light is an example of a 
rheostat. You increase the amount of resistance to dim the light and decrease the resis- 
tance to brighten the light. 

Ohm's (avi: Relating resistance to current and Voltage 

The amount of resistance that interferes with the flow is measured in ohms (pronounced just 
like those yoga chants). The symbol for ohm is the Greek letter omega, which looks like an 
upside-down horseshoe: Q. Resistance can be measured by dividing the voltage measured at 
any given point (the voltmeter reading) by the amount of current at the same point in a cir- 
cuit (the ammeter reading). Or you can measure the resistance directly by an ohmmeter. 

If you have a current flowing through a wire, three influences are present: 

v* The amount of voltage, measured in volts 

v* The resistance to the current, measured in ohms 

v* The amount of current, measured in amps 

These three units are always present in a specific relationship to each other. If you know 
the value of any two of the influences, you can find the value of the third. (Yes, this requires 
more math. Sorry.) 

£jftBE# Ohm's law, which was first stated by Georg Simon Ohm, reads, "The current in a circuit 

is directly proportional to the applied voltage and inversely proportional to the circuit 
resistance," but it's actually easier to understand in mathematical terms. When stating the 
relationship mathematically, abbreviations are used, where / is current, E is voltage, and R 
is resistance: 

Current (amperes) = „ — . - .- . A , or / = 4r 
v v J Resistance (ohms) R 

This essentially means that current in a basic circuit is always dependent on the voltage 
and resistance in the circuit. If you use a higher-voltage battery (increase E), the resistance 
doesn't change, but current in the circuit increases. By the same token, if you leave the 
same battery in the circuit but increase the resistance (increase R~), current decreases. 

Here are two other ways to write the same formula, solved for voltage and resistance: 

Iv* Voltage = Current x Resistance, or E = IR 
v* Resistance = p ^-r, or R = 4- 

Ohm's law works exactly the same, no matter which format you use. 




Chapter 13: Electronics Information / 1 



*££&> 




Measuring porter 



Power is measured in watts. One watt is a very small amount of power. It would require 
nearly 750 watts to equal 1 horsepower. One kilowatt represents 1,000 watts. 

A kilowatt-hour (kWh) — the amount of electricity a power plant generates or a customer 
uses — is equal to the energy of 1,000 watts working for one hour. Kilowatt-hours are deter- 
mined by multiplying the number of kilowatts (kW) required by the number of hours of use. 
For example, if you use a 40-watt light bulb 5 hours a day, you've used 200 watt-hours, or 
0.2 kilowatt-hours of electrical energy. 

The term watt was named to honor James Watt, the inventor of the steam engine. 



Getting around to circuits 



Although this section suggests that electricity flows like water, it actually flows more like 
NASCAR. Electricity must be sent along the path of a closed circle (a circuit), just like all 
those NASCAR speedsters roaring around the track. The drivers never actually get any- 
where; they just keep driving in circles. Electrical charges are a lot like that. 

However, electricity does flow like a river in one respect. In general, electricity follows the 
path of least resistance. The conventional way in thinking about the electrical flow of cur- 
rent is based on that the vacancies left by electrical particles "moving" from the positive (+) 
terminal to the negative (-) terminal of a battery. This concept is called conventional cur- 
rent. However, the military teaches current flow based on the flow of the electrons, and elec- 
trons, no matter how you look at it, flow from the negative terminal to the positive terminal 
(see Figure 13-1). 



Figure 13-1: 

A simple Terminal 
electric 
current. 



+ 



If any of the wires leading from one terminal to the other is broken, the circuit is shot — no 
more current. Current can't flow because under most circumstances, the electrons can't 
bridge the open gap in a conductor (the open gap is basically air, and air is an insulator). 

In some cases, current does flow through an insulator — if there's enough difference of 
potential (voltage). When lightning bridges an expanse of air from a cloud to ground (or a 
tree or a golfer), it's because there is a huge amount of voltage, on the order of 100 million 
volts between the source of the lightning and (literally) ground. 

Here's another circuit problem that may come up: A short circuit occurs when any wire acci- 
dentally crosses over another wire, causing the electricity to bypass the rest of the circuit 
and not follow the intended path. 



162 



Part IV: The Whole Ball of Facts: Technical Skills 



Magnetic, electric: No, not your personality 



Certain magnetic effects always accompany an elec- 
tric current, and these effects follow definite laws. In a 
wire, the magnetic lines of force (imaginary lines used 
to explain magnetic effects) are perpendicular to the 
conductor and parallel to each other. 

But when you wrap a wire around a core and pass 
current through it, the wire forms a coil. As the lines of 
force around the core take on a different shape,the field 
around each turn of wire links with the fields from the 
other turns of wire around it. The combined influence 
of all the turns of wire produce a two-pole magnetic 
field, very much like the magnetic field of a simple bar 
magnet — one end of the coil is a north pole; the other 
end is a south pole. 

The strength of the magnetic field depends on several 
factors. Here are the main ones: 

v Number of turns: If you increase the number of 
turns, you increase the field strength. 

J-" Closeness of the turns: The closer the turns, the 
stronger the field. 



j-" Amount of current: If you increase current, you 
increase field strength. 

w Material in the core: Most coils are classified as 
either air or soft iron, based on their cores. Air coils 
are usually wrapped around a piece of cardboard; 
soft-iron coils are wrapped around a piece of iron. 
Soft iron offers a better path for magnetic lines of 
force because its high permeability offers less 
reluctance to magnetic flux, resulting in more lines 
of force. (Think of magnetic flux as a measurement 
of magnetic strength located on a two-dimensional 
surface. A good example of magnetic flux would be 
the magnetic strength of one side of a magnet.) The 
more lines of force, the strongerthe magnetic field. 

Passing a suspended loop of conductive material (wire) 
through a magnetic field creates electromagnetic induc- 
tion, which is the basic principle behind the electric 
generator. When the conductor is standing still, cur- 
rent doesn't flow through it. But when the loop starts to 
rotate clockwise through the lines of force of the mag- 
nets, the lines of force induce free electrons to move 
through the wire. 



Producing electrical effects 



Electric currents can produce different effects. These effects are packaged and sold com- 
mercially. The following is a description of effects produced by current and some of their 
commercial applications: 

v* Chemical effect: Current produces this effect when it passes through a chemical com- 
pound and breaks that compound up. Also called electrolytic decomposition, this phe- 
nomenon is used in electroplating, a process used to cover objects with a very thin 
coating of metal. 

J-" Heat effect: Conducting electricity causes wires to become heated. Heat develops 
because the current must overcome the resistance of the wire. This heat energy can be 
quite obvious or hardly noticeable to touch, depending on the size of the wire and the 
amount of current. 

v* Magnetic effect: When a wire is introduced into a magnetic field, electricity flows 
through the wire and creates a magnetic field that repels a magnet. This effect is used 
to create energy through electromagnetic induction, the basic principle behind the 
electric generator. If the wire is wrapped around an iron core and a current is sent 
through the wire, the iron becomes magnetized. (See the nearby sidebar for more on 
the magnetic effect.) 

j-" Physiological effect: Current produces this effect when it passes through your bicep 
(or any of your muscles for that matter) and causes the muscle to contract. This effect 
is used in medicine. 



Chapter 13: Electronics Information / Oj 



Sitiitchinq Things Up With Alternating 
and Direct Current 

A current doesn't always flow in one direction. A direct current (DC) does — it only and 
always flows in one direction. An alternating current (AC), however, constantly changes 
direction in a regular pattern. Higher voltages are easier to obtain with alternating current, 
and transferring high voltage down a power line is ultimately cheaper than transferring low 
voltage, so most electricity comes in the form of AC. The following sections cover some 
important points about alternating and direct current. 



sfiLSTl^ 




WER 




Figuring out frequency 



The number of times an alternating current changes direction per second is known as its 
frequency. Hertz (Hz) is the unit of measurement for frequency. One hertz (Hz) equals one 
complete cycle per second. In other words, the current makes two complete alternations of 
direction. 

The AC (alternating current, not the air conditioner) in your house probably completes 
60 alternating cycles per second. Therefore, the AC in your house has a frequency of 60 Hz. 
Most electronic devices operate at higher frequencies; therefore, frequencies may be mea- 
sured in kilohertz (kHz, 1,000 hertz), megahertz (MHz, 1 million hertz) or even gigahertz 
(GHz, 1 billion hertz). 

AM radio stations often broadcast in the 530-1,700 kHz range. Television stations may 
broadcast at as low as 7 to as high as 1,002 MHz. Radar operates in the 1-40 GHz range. 

Impedance: Join the resistance! 

Resistance interferes with the flow of current in a circuit. But the flow of current is also 
impeded by two properties of alternating currents: 

v* Capacitive reactance (capacitance): Capacitance is the storage of energy that occurs 
in a nonconductor. This property resists any change in voltage in a circuit. 

u* Inductive reactance (inductance): Inductance is the property that causes an electro- 
motive force (another way of saying voltage) to be induced in a circuit. 

These two types of reactance combine to impede the flow of current. Impedance can be 
expressed as the ratio of electromotive force to the current: 



Impedance = 



Electromotive force 
Current 




Electronic devices often require a specific capacitive or inductive reactance to work. Capacitors 
and inductors are devices used in circuits to provide the type of reactance needed. Capacitors 
are rated in microfarads (uF), and inductors are rated in millihenries (mH). 

You can relate impedance to Ohm's law in reference to AC circuits. Simply substitute resis- 
tance in Ohm's law with impedance and voltage with electromotive force. 



/ ()l) Part IV: The Whole Ball of Facts: Technical Skills 




Sorting out capacitors and inductors 



Capacitors store or hold a charge of electrons. In an AC 
circuit, because AC voltage goes positive and negative 
in each cycle, the capacitor is constantly charging and 
discharging. The rate of the charging and discharging 
acts as opposition to the changing AC voltage — as a 
resistive effect called capacitive reactance. 

Inductors are coils of wire that make use of the proper- 
ties of a magnetic field. The property specifically desired 
is the flow of current through the wire. With full current. 



the magnetic field is at its maximum. However, if you 
take away the current, the field doesn't disappear imme- 
diately. It decays gradually, and the decay continues to 
push electrons in the path they were going. But in an 
AC circuit, the current constantly reverses. The rate of 
changing current flow and the resulting collapse and 
regeneration of the magnetic field in the coil act as 
opposition to changing AC current — a resistive effect 
called inductive reactance. 



Rectifying the situation: Going direct 

Certain electronic circuits are engineered to change alternating current to direct current. 
The process of making the change is called rectification, and the circuits that perform the 
rectification are called rectifiers. 

Rectifiers contain semiconductor diodes, a component made of a material with conductivity 
somewhere between that of a conductor and an insulator. Diodes conduct electricity in only 
one direction. Rectification also often requires inductors and capacitors (see the preceding 
section). 

Rectification helps appliances run at cooler temperatures and allows them to run at vari- 
able speeds. Devices typically need direct current to run properly. The process of rectifica- 
tion changes the incoming AC to DC. 



Turning up the old transistor radio 

A transistor is a semiconductor (an object that conducts electricity poorly at low tempera- 
tures) that controls the flow of electricity in a circuit. It's usually made of germanium or sili- 
con. This electrical device can amplify a signal, which is why it's used in transistor radios. 
Transistors have many properties: 

p" Unlike rectifier diodes (see the preceding section), a transistor doesn't require a 
vacuum to operate. 

f Transistors are small, require little power, and last a long time. 

t* A transistor contains at least three terminals: 

• The emitter is the voltage output. 

• The base acts like a gate, and the voltage at the base controls the flow of current 
through the transistor (and therefore the voltage). 

• The collector is the voltage input. 



Chapter 13: Electronics Information / \)5 



Picture It: becodinq Electrical Circuit Codes 

Electronic circuits can be combined to create complex systems, such as those required to 
operate a stereo system. Block diagrams are used to show the various combined circuits 
that form a complex system. 

Many of the questions on the Electronics Information subtest require you to identify an 
electronic component symbol and know what that component does in an electronic circuit. 
Figure 13-2 shows the most common component symbols. The figure's items are grouped 
based on similarity of functions. For example, cells, batteries, DC power supplies, and AC 
power supplies all have similar functions (they supply power to the circuit). 

So, what do all these electronic doodads do when connected in a circuit? I cover each item 
in the following list: 

v* Wires: Wires are used to pass current from one part of the component to another. 
Wires that are connected to each other are indicated by a dark circle and are called 
joined wires. Sometimes in complex circuit diagrams, it's necessary to draw wires 
crossing even though they aren't connected. In this case, the dark circle is omitted, or 
a hump symbol is drawn to make it clear the wires aren't connected — this is called 
unjoined wires. 

e* Cell: A cell supplies electrical current. Some call this a battery, but technically a battery 
is more than one cell. The large terminal (on the left side of the cell image in Figure 13-2) 
is positive. 

v* Battery: A battery is two or more cells. The large terminal is positive. 

i*" DC power supply: A DC power supply provides direct current. Direct current always 
flows in one direction. 

(-" AC power supply: An AC power supply provides alternating current. Alternating cur- 
rent constantly changes direction at a specific frequency. 

j*" Fuse: A fuse is a safety device that blows (melts) if the current flowing through it 
exceeds a specified value. 

v* Transformer: A transformer consists of two coils of wire linked by an iron core. 
Transformers are used to step up (increase) and step down (decrease) AC voltages. 
No electrical connection exists between the coils. Energy is transferred between the 
coils by the magnetic field in the core. 

i*" Ground: A ground is a connection to the earth. 

v* Transducer: A transducer is a device that converts energy from one form to another. 
Here are various types of transducers: 

• Lighting lamp: Converts electrical energy to light, such as in a light bulb or auto- 
mobile headlight 

• Indicator lamp: Converts electrical energy to light for such uses as a warning 
light on a car's dashboard 

• Motor: Converts electrical energy to kinetic energy (motion) 

• Heater: Converts electrical energy to heat 

• Bells and buzzers: Convert electrical energy to sound 

• Microphone: Converts sound to electrical energy 

• Earphones and speakers: Convert electrical energy to sound 



/ 66 Part IV: The Whole BaM of Facts: Technical Skills 



Wire 



xa 



Joined Wires Unjoined Wires 



Cell 



H ► I 

Battery 



DC Power Supply AC Power Supply 



Fuse 



Transformer 




Inductor 



2-Way Switch 



Ground 



fun- 
Heater 



Push Switch 



Dual On/Off Switch 



Lighting Lamp Indicator Lamp 



Bell 



Push-to-Break 
Switch 



I f ONO 

Relay 



X] X3 id 

Buzzer Microphone Earphone Speaker 

On/Off Switch 

-AWW 1 1— 

Resistor Resistor 



Rheostat 



Potentiometer 



*" 



Figure 13-2: Po | arized Capacitor Variable Capacitor 

Symbols in 

electronic 

circuit 

diagrams. 

^^^^^^^ m Transistor Amplifier 



Preset Variable 
Resistor 



— D+ 



Diode 



V 



Antenna 



Capacitor 



Light-Emitting Diode 
(LED) 



p* Inductor: An inductor is a coil of wire that creates a magnetic field when current passes 
through it. 

p" Switch: Here are several types of switches: 

• Push switch: A push switch allows current to flow only when the button is 
pressed, such as in a doorbell. 

• Push-to-break switch: With this switch, the circuit is normally closed (the device 
is on), and the circuit is open (device is off) only when the button is pressed. 

• On/off switch: An on/off switch allows current to flow only when it's in the closed 
(on) position. 

• Two-way switch: A two-way switch directs the flow of current to one of two 
routes, according to its position. 

• Dual on/off switch: This type is often used to switch main electricity because it 
can isolate both the live and neutral connections. 

• Relay (relay switch): A relay is an electrically operated switch that may operate 
multiple switches at one time. Current flowing through a coil sets up a magnetic 
field, which causes the lever(s) to move, effectively changing the (relay) switch's 
position(s). 



Chapter 13: Electronics Information J 07 



v* Resistor (nonvariable): There are two different versions of the basic resistor symbol. 
Resistors restrict the flow of electric current. Resistors are rated in ohms and have a 
color code on them to indicate their value, tolerance, and sometimes quality. The band 
code is as follows: 

• Black is 

• Brown is 1 

• Red is 2 

• Orange is 3 

• Yellow is 4 

• Green is 5 

• Blue is 6 

• Violet is 7 

• Gray is 8 

• White is 9 

The first and second bands on the resistor are the first two digits in the resistor's 
value. The next band indicates the multiplier (number of zeros after the first two num- 
bers). So if the first band is red, the second is yellow, and the third band is orange, the 
resistor's value is 24,000 ohms. A gold or silver band after the first bands indicates tol- 
erance, and a quality band may follow the tolerance band. 

v* Variable resistor: Variable resistors also restrict the flow of electric current. There are 
several symbols in use in circuit diagrams for standard variable and preset variable 
resistors. Types of variable resistors include the following: 

• Rheostat: A type of variable resistor with two contacts, usually used to control 
current; examples of controlling current would be adjusting lamp brightness or 
adjusting motor speed 

• Potentiometer: A type of variable resistor with three contacts that's used to con- 
trol voltage 

• Preset variable resistor: A device that operates with a small screwdriver or simi- 
lar tool; it's designed to be set when the circuit is made and then left without fur- 
ther adjustment 

j*" Capacitor: Capacitors store electric charge. They're used with resistors in timing cir- 
cuits because it takes time for a capacitor to fill with charge. They're also used in filter 
circuits because capacitors easily pass AC (changing voltage) signals but they block 
DC (constant voltage) signals. Two types of capacitors include the following: 

• Polarized capacitors must be connected the correct way in circuit. 

• Variable capacitors are used most often in radio tuning circuits. 

«*" Diode: Diodes allow electricity to flow in only one direction. The arrow of the circuit 
symbol shows the direction in which the current can flow. Diodes are the electrical 
version of a valve, and early diodes were actually called valves. Light-emitting diodes 
(LEDs) emit light when an electric current passes through them. Specialized diodes, 
called Zener diodes, do allow current in the opposite direction after a threshold is met. 



168 



Part IV: The Whole Ball of Facts: Technical Skills 



v* Transistor: Transistors amplify current. For example, they can be used to amplify the 
small output current from a logic chip so it can operate a lamp, relay, or other high- 
current device. 

i*" Amplifier: An amplifier isn't actually an electronic component but instead is a complex 
circuit. The block diagram symbol shows where an amplifier circuit would be con- 
nected. Amplifier circuits are used to magnify power, current, or voltage. 

v* Antenna: An antenna is a device designed to receive and/or transmit radio signals. 

Circuit diagrams show how electronic components are connected together. These diagrams 
show the connections as clearly as possible with all wires drawn neatly as straight lines. 
The actual layout of the components is usually quite different from the circuit diagram, 
however. Circuit diagrams are useful when testing a circuit and for understanding how it 
works. Figure 13-3 shows a diagram of an adjustable timer circuit. See how many compo- 
nents you can identify. 



Adjustable Timer Circuit 



Figure 13-3: 

An adjust- 
able timer [ 
circuit. 




Eyeinq Some Electronic Information Test Tips 

When it comes to the electronics test, don't feel like you have to know as much as Ben 
Franklin to get a passing score. Just use your common sense. If a question asks, "What's the 
safest way to run an extension cord to a reading light?" the answer "across the middle of 
the floor" is probably going to be wrong. 

You can also figure out quite a few answers if you remember these units of measure: 

f Current: Amperes (or amps) 

v* Voltage: Volts 

i*" Resistance: Ohms 

v* Power: Watts 

i*" Energy: Watt-hours 



Chapter 13: Electronics Information j Q Q 



**!!? 




Memorizing simple principles 



If you commit the following principles to memory, you'll have an easier time succeeding on 
the Electronics Information subtest: 



n" Ohm's law: Current = 



Voltage 



Resistance 

j-" Power (watts) = Voltage (volts) x Current (amperes), or P = EI 

p" Current flows from a negative pole to a positive pole. 

V A closed circuit must exist for electricity to flow. (Think NASCAR.) 

p" Alternating current (AC) changes direction constantly at a constant rate. The number 
of times a current completes two alternations of direction per second is known as its 
frequency; the unit of measurement for frequency is the hertz (Hz). 

p" Electronic devices operate at very high frequencies. 

p" Electronic devices often require a specific capacitive or inductive reactance to work. 
Capacitors and inductors are devices used in circuits to provide the type of reactance 
needed. 

n" Devices that change alternating current to direct current are called rectifiers. 

v A transistor can amplify a signal. 

If you need a good score on this subtest to get your military dream job or you want to 
rebuild that old television set without sacrificing your garage, you may want to check out 
Electronics For Dummies by Gordon McComb and Earl Boysen (Wiley) for additional help. 




Playing the guessing game 



The Electronics Information subtest is the type of test where you either know the answer or 
you don't. But if you don't know the answer, you should still guess. Remember, you don't 
have a lot of time to ponder the answer choices. Guess and move on. To increase your 
chances of guessing correctly, you can often eliminate an incorrect answer. 

Sometimes one answer is obviously wrong, or one answer is more obviously right than 
another. The electronics answer is usually the right answer. Therefore, an answer that has 
to do with how much something costs or how pretty it looks will probably be wrong. 

Not all questions are specifically electronics questions. You may be asked, "A mil measures 
what quantity?" Think about how you've seen that prefix used before, such as in the word 
millimeter. A millimeter, you may remember, is one-thousandth of a meter. So you may be 
safe in assuming that a mil is one-thousandth of an inch. For additional guessing help, flip 
back to Chapter 3. 



170 



Part IV: The Whole Ball of Facts: Technical Skills 



Chapter 14 

Assembling Objects 



In This Chapter 

Checking out the newest ASVAB subtest 
Connecting the dots and putting the pieces together 
Getting your test score into shape 



^M lthough much of the ASVAB measures academic knowledge at the high school level, 
¥ \ Assembling Objects is a subtest that probably doesn't resemble any of your high 
school classes (unless your high school offered a course in Jigsaw Puzzles 101). 

The Assembling Objects subtest is designed to measure your ability to look at pieces of an 
object and determine how those pieces should fit together (technically called visualizing 
spatial relationships'). Spatial skills, which help people figure out maps and interpret techni- 
cal drawings, are important to everyday living as well as for performing well in school and 
on the job. Society today places greater demands on spatial skills, such as interpretation of 
graphs, maps, architectural drawings, and X-rays. 

The Assembling Objects subtest of the CAT-ASVAB consists of 16 graphical problems that 
must be solved in 15 minutes; the paper version of the ASVAB has 25 questions to be solved 
in 16 minutes. That gives you a little less than a minute for each question (not counting any 
time you take out to scratch your head). That's plenty of time to finish if you're good at 
jigsaw puzzles. 

Getting the Picture about Assembling Objects 

The Assembling Objects subtest is relatively new to the ASVAB. It was added when the 
ASVAB was last revised, when the Numerical Operations and Coding Speed subtests were 
deleted. First it was added only to the computerized version of the ASVAB, and then it was 
added to the paper enlistment version about a year later. If you're taking the high school 
version of the ASVAB or the in-service version (Armed Forces Classification Test), you 
won't see this subtest. 

At the time of this writing, only the Navy uses the score from the Assembling Objects sub- 
test for job qualification purposes. Additionally, only a handful of ratings (what the Navy 
calls jobs) require a score in this area. The other branches don't use the results of this sub- 
test at all, but they may in the future. For details about which Navy enlisted jobs require a 
score in this area, see Appendix A. 

The upshot is that unless you're planning to join the Navy, in one of only a handful of Navy 
enlisted jobs, you can safely ignore this entire chapter. Don't say I never gave you a gift. 




172 



Part IV: The Whole Ball of Facts: Technical Skills 



Adopting the AO subtest 



It's interesting that the Navy is the only service to use 
scores from the Assembling Objects subtest, because it 
was an Army study that brought this subtest to life. Way 
back in 1994, the Army concluded a study called Project 
A. (Kind of makes you wonder if the people in charge 
of naming military projects were on vacation that week, 
doesn't it?) Project A was all about trying to improve 
the selection and classification of enlisted personnel. 
The Assembling Objects subtest was a major product 
of this effort. The U.S. Army Research Institute for the 
Behavioral and Social Sciences found Assembling 



Objects questions to be an excellent measure of both 
overall spatial ability and complex problem-solving skills. 

The Army developed two types of Assembling Objects 
questions and tested them under field conditions for a few 
years. A mere ten years later, the Department of Defense 
incorporated the subtests into the ASVAB. (You just got 
to love the speed at which the military makes changes.) 
By that time, the Army had decided that they really didn't 
need to use this new subtest at all. But the Navy said, 
"Hey, that looks pretty cool. .. let's give it a try!" 



Tu/o Types of Questions for the Price of One 

The Assembling Objects subtest has two types of questions, both of which consist of five 
separate drawings. In the first drawing, you see a picture with various disassembled parts, 
followed by four drawings that show the parts assembled or connected. Your task is to 
choose the drawing that shows what the parts may actually look like after they're assem- 
bled or connected properly. 



*3*BE* 




Both types of Assembling Objects problems require you to perform mental rotation — 
a process through which you predict what an array of objects would look like if they were 
rotated or turned by some number of degrees. 



Putting slot A into tab B: Connectors 

The first type of problem presents you with simple geometric figures such as stars, cloud 
shapes, letter shapes, circles, and triangles. In the first drawing, you can see shapes and 
lines labeled with dots and the letters A and B. These letters and dots indicate points of 
attachment. 

The next four drawings show possible solutions of what the shapes would look like if con- 
nected at designated points by the line. The shapes may be reoriented or rotated from what 
you observe in the first drawing. The correct solution shows the line connected correctly to 
reflect the points shown in the first drawing. 

Look at Figure 14-1 and see whether you can solve it. In the first drawing, you see a star and 
a sort of lopsided T. There's a small dot on the short appendage of the T, labeled A, and a 
dot on one of the points of the star, labeled B. 



Figure 14-1: 

Identifying 

points and 

shapes. 



g|T*t tl/ 



Chapter 14: Assembling Objects j 7j 



In Figure 14-1, Choice (A) is the correct solution. Choices (B) and (C) include shapes that 
aren't included in the first drawing, so they're obviously incorrect. Although Choice (D) has 
the correct shapes, they aren't connected at the same points depicted in the first drawing. 

Okay that sounds simple, doesn't it? Don't worry; it gets more complicated (sorry to burst 
your bubble). Figure 14-2 shows the same problem but with a different twist. 



Figure 14-2: 

Rotated 

shapes 

make the 

problem 

harder. 



•T"i 


/ 


k, 


TV 


J 1 



Choice (A) is the correct solution for the problem in Figure 14-2. In this case, the two shapes 
have been repositioned and rotated. 



jttNG/ 




On the flip side: Avoiding mirrors 

Mirroring (or flipping or reflecting) isn't the same as rotation, as Figure 14-3 illustrates. The 
shape in Box B isn't the same as the shape in Box A. It's a mirror image. No matter how you 
rotate the shape in Box A, it will never look like the shape in Box B. Think of it this way — 
although you can turn a jigsaw puzzle piece upside down so the picture side is facing the 
table, it may fit, but that's not the proper method of putting the puzzle together. (It wouldn't 
look very pretty, either). The Assembling Objects subtest is the same way. The possible 
solutions may include shapes that are reflections of a shape shown in the first drawing, but 
they'll never be the correct solution. 



Figure 14-3: 

Figuring out 

mirrored 

shapes 

(AandB) 

and rotated 

shapes. 



T 


T 



T 


>^ 


v^ 


^ 


^ 



**BE» 




Crossing ot/er the right places 

If a shape in the first drawing shows a line that goes through any part of the shape, the cor- 
rect solution must also reflect the same line-shape relationship. Check out Figure 14-4. In 
the first drawing, Point B is in the center of the star. But note the line intersects the star at 
one of its indentations and not one of its points. That means the correct solution shows the 
same intersection. 



Figure 14-4: 

Line-shape 
relation- 
ships. 



•T'i 


> 


•4 


% 


*T 



17 It 



Part IV: The Whole Ball of Facts: Technical Skills 



In this example, Choice (B) is the correct solution. At first glance, Choice (C) looks like it 
could be correct. Can you spot the reason it's not the correct solution? Right! The lopsided 
T shape in the image is a reflection of the shape shown in the first drawing. 

Putting it all together 

You're starting to see the shape of things! (I'm sorry, but these little zingers just keep pop- 
ping out.) Try a couple more, just to get into shape. Look at Figure 14-5. 



Figure 14-5: 

Another 
example of 
spatial rela- 
tionships. 



<2. 

A- 




^ 




In Figure 14-5, did you select Choice (C) as the correct answer? If so, good job! Choice (A) is 
incorrect because the line intersects the triangle at the wrong point and conection point A 
is misplaced. Choice (B) is incorrect because the weird shape is actually a mirror image of 
the shape shown in the first drawing. Choice (D) is incorrect because the points don't corre- 
late to the points depicted in the first drawing. 

Now try Figure 14-6. The first drawing includes a shape that kind of looks like a Y and a 
shape that looks like the letter C. 



Figure 14-6: 

More 

shapes to 

test your 

spatial skills. 



B^ §Ki §hJ l^@ ^ 



The correct answer for the problem shown in Figure 14-6 is Choice (B). Choice (A) is incor- 
rect because the Y shape is a mirror image of the shape shown in the first drawing, and the 
connection points don't correspond to the first drawing's points. Choice (C) is incorrect 
because the Y shape is a mirror image of the shape shown in the first drawing. Choice (D) is 
incorrect because the Y shape is a different shape (the stem is much shorter) than the 
shape shown in the first drawing and because the connection dot on the C shape is in the 
wrong location. 



Solving the jiqsaW puzzle: Shapes 

Many people may find the second type of Assembling Objects problem easier than the con- 
nection problems. This type of problem is very much like a jigsaw puzzle, except it doesn't 
result in a picture of the Statue of Liberty or a map of the United States. Also, there's a heck 
of a lot fewer pieces than that 1,000-piece puzzle your parents kept insisting on buying you. 
The difficulty lies in the fact that you can't use your hands to twist the pieces around on the 
table in order to see how they fit. You have to rotate and move the pieces mentally. 



Chapter 14: Assembling Objects j (5 



In Figure 14-7, the solution is pretty straightforward. 



Figure 14-7: 

A simple 

jigsaw 

example. 







By mentally sliding the shapes in the first drawing together, it's easy to see that they fit 
together to form the picture shown in Choice (A). Look at Figure 14-8. 



Figure 14-8: 












Putting 
the pieces 


A 


A 


A 


A 


A 


together 

with 

rotation. 


/A 




A 


B 


c 


D 



Choice (A) is the correct answer. The figure shown in Choice (A) is the same as the figure 
depicted in Choice (A) of Figure 14-7, except it's been rotated. 

The previous two figures were warm-up exercises — the questions on the ASVAB are harder. 
Check out Figure 14-9 for a better representation of the types of questions on the ASVAB. 



Figure 14-9: 

A harder 

example of 

spatial 

problems. 




Pay attention to the curve of the leaf shape inside the square. It's not bowed out as in 
Choice (B) — the edges have more of a wave shape. Choice (C) has that shape too thin. If 
you selected Choice (D) as the correct solution, give yourself a pat on the back. Examining 
spatial relationships can help locate the correct answer with ease. Try a couple more exam- 
ples to see if you've gotten the hang of it. Check out Figure 14-10. 



Figure 14-10: 

Practice 




















© 


© 





mentally 

rotating and 

relocating 


AA 


6 


& 


AJ 


pieces of 
puzzles. 




A 




B 


C 


D 



176 



Part IV: The Whole Ball of Facts: Technical Skills 



In Figure 14-10, Choice (B) is the correct answer. Mentally rotate and relocate the pieces in 
the first picture until you can see how they fit together to form the shape in Choice (B). In 
the puzzle, three pieces have both a curved edge and a single straight edge. Practice elimi- 
nating choices that lack these characteristics. Choice (A) lacks these shapes, so you can 
discount it right away. Notice the curved pieces are all different sizes. Visualize fitting these 
pieces of the puzzle in your mind and compare the sizes and differences. Now try Figure 14-11. 



Figure 14-11: 

Putting the 
































pieces of 


-A 


1(\ 


























the puzzle 










/ 






^ 






/ 


/ 




together 


l/&n 




J 


v 


with your 
mental spa- 
tial skills. 
































A 






B 






C 








D 





In Figure 14-11, Choice (A) is the correct answer. If you didn't get this one quite right, head 
to Chapter 15 for additional practice and Chapters 16, 18, and 20 for practice examinations. 



Tips far the Assembling Objects Subtest 

In following sections, I offer some tips for improving your score on the Assembling Objects 
subtest. I offer strategies for eliminating wrong answers during the test, and I name some 
ways you can improve your spatial skills in general (which may come in handy the next time 
you have to read a map, too). 




*>*!!? 




Comparing one piece or point at a time 

On the Assembling Objects subtest, you can sometimes improve your odds of getting the 
answer right if you select just one shape from the first drawing and then quickly look at 
each of the choices to see whether that shape is represented there but in a different orien- 
tation. This process can help you quickly eliminate answer choices that are obviously 
wrong. 

On connection-type problems, note the position of the dot on one of the shapes in the first 
drawing and then quickly scan the possible answers, eliminating any choice that depicts the 
dot in a different location or that shows the line passing through the shape at a different 
point than shown in the first drawing. 

Remember to be aware of mirror images — shapes that are reflected (instead of rotated) 
from the image shown in the first drawing. The tricky test-makers often make use of such 
mirror representations to see whether they can trick your eyes. 



Chapter 14: Assembling Objects J 77 




Visualizing success: Practicing 
spatial skills ahead of time 

Researchers at the University of Chicago have determined that your basic foundation for 
spatial skills is established at a very early age, perhaps as young as age 4 or 5. But don't 
worry. That doesn't mean all is lost if your parents never got you that model rocket kit you 
wanted. The same research has concluded that you can still improve spatial skills by engag- 
ing in activities that are spatially orientated. Some of those activities include the following: 

j"* Practicing reading maps: Map reading can help you develop the ability to gauge scales 
of size and direction between related objects (roads, rivers, towns, cities, and so on). 

v^ Putting together jigsaw puzzles: This way is an obvious form of practice for improving 
your spatial perceptions. 

j*"* Playing puzzle games online: Many puzzle games at free online game sites exercise 
the skill of identifying spatial relationships and visual similarities. 

j*"* Playing graphical computer games: Computer games may help you to improve your 
spatial skills. A study conducted in the United Kingdom showed that children who 
played computer games consistently scored higher on spatial aptitude tests than chil- 
dren who didn't play the games. 

c" Sketching: Look at an object or a picture and attempt to sketch it as viewed from a dif- 
ferent view. This exercise can help you to improve your ability to mentally visualize 
angles. 



178 



Part IV: The Whole Ball of Facts: Technical Skills 



Chapter 15 

Facing the Facts: Technical 
Skills Practice Questions 




IN 



In This Chapter 

Taking a stab at General Science questions 
Getting a handle on Auto & Shop Information 
Practicing your Mechanical Comprehension knowledge 
Tuning in to the Electronics Information section of the ASVAB 
Building a better score on an Assembling Objects practice test 



1 

m t's time to see a few examples of what the ASVAB technical skills questions look like. 

«C None of these subtests are used in calculating your AFQT score (the score used to deter- 
mine your general qualification to join the military), but they may be used in computing the 
line score you need to get the military job you want. See Appendix A to determine whether 
you need to do well on any of these subtest areas for the job that you want. 

On the actual paper version of the ASVAB (and on the full-length practice tests in the 
following chapters), you get 25 General Science questions, 25 Auto & Shop questions, 
25 Mechanical Comprehension questions, 20 Electronics Information questions, and 
16 Assembling Objects graphical problems. The CAT-ASVAB gives you 16 General Science 
questions, 16 Electronics Information questions, 11 Auto Information questions, 11 Shop 
Information questions, 16 Mechanical Comprehension Questions, and 16 Assembling 
Objects questions. I don't want you to tire out too quickly, so in this chapter, you get only 
eight sample questions in each area. 



General Science Practice Questions 



General science is a hard topic to study for because the field is so broad. To score well on 
this subtest, you pretty much have to wade through the textbooks and memorize the facts. 
You can also check out Chapter 10 for additional help. See how well you do on the following 
eight practice questions. 

1. If the temperature in Fahrenheit is 212 degrees, the temperature in Celsius is 

(A) degrees. 

(B) 32 degrees. 

(C) 100 degrees. 

(D) 106 degrees. 

Measured in Celsius, the boiling point of water is 100 degrees. If you don't have this 
memorized, you can calculate it. To convert from Fahrenheit to Celsius, use the formula 
C = jy(F- 32). The correct answer is Choice (C). 



/r 



/ 80 Part IV: The Whole BaM of Facts: Technical Skills 



2. A cell nucleus is often referred to as the 

(A) control center. 

(B) cytoskeleton. 

(C) cell membrane. 

(D) chromosome. 

The nucleus contains most of the cell's genetic material and is often referred to as the control 
center of the cell, so the correct answer is (A). 

3. The human circulatory system 

(A) uses air to release energy. 

(B) processes food and eliminates waste. 

(C) moves oxygenated blood throughout the body. 

(D) controls movement of joints. 

The respiratory system uses air to release energy, the digestive system processes food and 
eliminates waste, and the musculoskeletal system controls the movement of joints. The cor- 
rect answer is Choice (C). 

4. Compasses work by 

(A) measuring heat in the air. 

(B) reacting to magnetic fields. 

(C) repulsing wave currents. 

(D) magic. 

A compass is a device that takes advantage of the Earth's magnetic field. Choice (B) is the 
correct answer. 

5. If an atom has one proton and one electron, the atomic number is 

(A) 2 

(B) 10 

(C) 5 

(D) 1 

The atomic number refers to the number of protons an atom has in its nucleus. Choice (D) is 
the correct answer. 

6. The element with the lowest atomic number is 

(A) hydrogen. 

(B) helium. 

(C) lithium. 

(D) uranium. 

Hydrogen has an atomic number of 1. The atomic numbers for the other elements listed are 
2 (helium), 3 (lithium), and 92 (uranium). The correct answer is Choice (A). 



Chapter 15: Facing the Facts: Technical Skills Practice Questions / $ j 



7. Absolute zero is equivalent to 

(A) degrees Kelvin. 

(B) kelvins. 

(C) -273.15 degrees Kelvin. 

(D) -273.15 kelvins. 

Absolute zero is -273.15 degrees Celsius, which is equivalent to kelvins. Temperatures 
stated in the Kelvin scale are measured by using units of kelvins, not degrees. The correct 
answer is Choice (B). 

8. A comet's tail is visible when 

(A) the metal alloys react to the atmospheric change. 

(B) the ice and rock collide. 

(C) the comet is close enough to the sun. 

(D) the comet passes the Kuiper Belt. 

Comets are balls composed mainly of ice and rock. The comet's tail is formed when the ice 
turns into gas from the heat of the sun; therefore, the comet must be closer to the sun for 
the tail to be visible. The correct answer is Choice (C). 

Auto & Shop Information Practice Questions 

If you like to tinker with cars and your idea of a fun weekend is to rebuild the engine, you 
should do well on this subtest without too much additional study. If your idea of fixing your 
car involves calling that guy down the street, a little extra study may be in order. Check out 
Chapter 1 1 for help with this area. 

9. A two-penny nail is 

(A) thicker than a lOd nail. 

(B) shorter than a lOd nail. 

(C) the same thing as a lOd nail. 

(D) harder than a lOd nail. 

Penny, abbreviated d (for the ancient Roman denarius coin), indicates length; a 2d nail is 
shorter than a lOd nail. Choice (B) is the correct answer. 

10. A carburetor has the same function as a/an 

(A) distributor. 

(B) fuel-injection system. 

(C) alternator. 

(D) exhaust system. 

The alternator, exhaust system, and distributor all have very different purposes from the 
carburetor, which combines the fuel and air mixture and sends it to the engine, just as the 
fuel-injection system does. Therefore, Choice (B) is the correct answer. 



/r 



/ 82 Part IV: The Whole BaM of Facts: Technical Skills 



11. An engine's rotational energy is stored by using which mechanical device? 

(A) connecting rod. 

(B) rear axle. 

(C) flywheel. 

(D) cylinder. 

The flywheel accelerates a rotor to a high speed and uses rotational energy to maintain and 
store the energy to keep the engine speed constant as the flywheel and rotor work together. 
The correct answer is Choice (C). As for the other devices, the drive shaft turns the rear 
axle. The cylinder contains the piston that moves the connecting rod that's connected to 
the crankshaft, which turns the flywheel. 

12. A hacksaw is used to cut 

(A) with the grain of wood. 

(B) against the grain of wood. 

(C) round stock. 

(D) metal. 

The hacksaw has a blade specifically designed to cut metal, not wood. Choice (D) is the cor- 
rect answer. 

13. To drive a cold chisel, the best object to use would be 

(A) a frozen hammer. 

(B) a warm sledge. 

(C) a mallet. 

(D) your foot. 

A hammer has a smaller, harder striking surface than a mallet. A mallet won't damage the 
chisel (or the object being chiseled, should the mallet slip off the chisel). A sledge is excep- 
tionally large and heavy and is therefore inappropriate for this use. Temperature of the 
striking object is irrelevant. Choice (C) is the correct answer. 

14. Which of the following is NOT normally part of an automotive tune-up? 

(A) Replace the air filter. 

(B) Replace the spark plugs. 

(C) Replace the CV axles. 

(D) Check the fluids. 

A general automotive tune-up consists of checking/replacing the following: air and fuel filter, 
belts, spark plugs, distributor cap, battery, clutch (if it's manual), engine timing, fluids, igni- 
tion timing, and valves. You can also change the positive crankcase ventilation valve and 
change the points and condenser if you have an older vehicle. Replacing the CV (constant 
velocity) axles is something that is accomplished when they become worn. Choice (C) is the 
correct answer. 



Chapter 15: Facing the Facts: Technical Skills Practice Questions f OD 



15. Antifreeze is used to 

(A) prevent the engine from overheating. 

(B) prevent water in the cooling system from freezing. 

(C) prevent damage to the engine block. 

(D) all of the above 

Antifreeze raises the boiling point of water and lowers the freezing point. This process 
keeps the water in the cooling system from boiling away or freezing. Either condition can 
cause damage to the engine. The correct answer is Choice (D). 

16. The best tool for cutting curves or shapes in wood is a 

(A) ripsaw. 

(B) crosscut saw. 

(C) coping saw. 

(D) pliant saw. 

Coping saws have thin blades with many teeth and are specifically designed to cut curves 
and shapes in wood. The correct answer is Choice (C). 

Mechanical Comprehension Practice Questions 

Mechanical comprehension is all about figuring out how machines and mechanical mechanisms 
operate. A solid background in mechanical physics is a big advantage in scoring well in this 
area. You can flip back to Chapter 12 if you need additional help with mechanical info. Basic 
math skills are also a plus in this area. Test yourself with the next several questions. 

17. The moisture that forms on the inside of a window on a cold day is called 

(A) condensation. 

(B) distillation. 

(C) evaporation. 

(D) tarnation. 

Distillation is the process of extracting or refining a substance using both boiling and con- 
densation. Evaporation is the process of removing moisture from the surface of a liquid — 
the water molecules escape the surface and assume gas form. Tarnation is an interjection 
used to express anger. The correct answer is Choice (A), condensation. 

18. If a 200-pound barrel must be lifted 4 feet to the bed of a box truck, an inclined plane will 
reduce the amount of effort required to move the barrel by half if the inclined plane is 

(A) 2 feet long. 

(B) 6 feet long. 

(C) 8 feet long. 

(D) 9 feet long. 

The formula used for determining how an inclined plane reduces effort is Length of Ramp * 
Height of Ramp = Weight of the Object * Force, or x ± 4 = 200 * 100. The amount of force 
needed to lift the object is equivalent to the object's weight, but the question wants to 
reduce that amount of force to half, so half of the object's weight is 100. Now do the math: 
Xt4x4 = 2x4;x=8. The correct answer is Choice (C). 



/r 



/ SI) Part IV: The Whole BaM of Facts: Technical Skills 



19. Two people are carrying a 100-pound crate on a 2- x 8- x 12-foot board. To distribute the 
load evenly between the two people, the crate should be placed 

(A) 2 feet from the end of the board. 

(B) in the middle of the board. 

(C) 3 feet from the end of the board. 

(D) The load can't be evenly distributed. 

If the weight is placed closer to one person or the other, that person would carry more of 
the load, so the weight should be placed in the middle. Choice (B) is the correct answer. 

20. Wheel A has a diameter of 9 feet. Wheel B has a diameter of 12 feet. If both wheels revolve at 
the same rate, Wheel B will cover a linear distance of 24 feet 

(A) at the same speed as Wheel A. 

(B) more slowly than Wheel A. 

(C) in half the time of Wheel A. 

(D) more quickly than Wheel A. 

Because Wheel A has a smaller circumference, it covers a shorter linear distance than 
Wheel B when turning at the same rate. Thus, Wheel B covers the distance of 24 feet faster 
than Wheel A. Choice (C) is inaccurate because the diameter of Wheel A is not exactly half 
the diameter of Wheel B. If both wheels revolve at the same rate, then Wheel A turns 25% 
more slowly than Wheel B, because wheel A's diameter is three-quarters of wheel B's. 
Choice (D) is the correct answer. 

21. Not including friction, a stationary single pulley gives a mechanical advantage of 

(A) 2. 

(B) 4. 

(C) 3. 

(D) 1. 

A stationary single pulley allows you to change the direction of force but doesn't result in 
an increased mechanical advantage. The correct answer is Choice (D). 

22. Four gears are connected in a series. If Gear #1 is turning clockwise, Gear #4 will turn 

(A) clockwise. 

(B) counterclockwise. 

(C) more quickly than Gear #1. 

(D) more slowly than Gear #1. 

Gears connected in series turn in opposite directions of each other. Gears 1 and 3 rotate 
clockwise, and Gears 2 and 4 rotate counterclockwise. The size of gears is unknown, so 
there isn't enough information to determine whether (C) or (D) is correct. The correct 
answer is Choice (B). 



Chapter 15: Facing the Facts: Technical Skills Practice Questions f OD 



23. The sideways force one feels when a car turns sharply is often called 
A) thrust force. 

(B) angle force. 

(C) centrifugal force. 

(D) positive force. 

Although commonly referred to as centrifugal force, this property isn't actually a force at all 
but is rather a property of inertia, one of Newton's laws of motion. As the car turns, your 
body is trying to continue traveling in a straight line. The correct answer is Choice (C). 

24. When two or more forces act to balance each other out, the condition is called 

(A) equilibrium. 

(B) static recoil. 

(C) gravitational balance. 

(D) concurrent forces. 

When two or more forces interact so that their combination cancels the other(s) out, 
there's a state of equilibrium. In this state, the velocity (speed and direction) of an object 
doesn't change. Choice (A) is the correct answer. 

Electronics Information Practice Questions 

The questions in this section measure your knowledge of basic electronics principles. 
Chapter 13 contains a more in-depth discussion of electronics if you need some help. 
For now, give these questions a try. 

25. What does the abbreviation DC stand for? 

(A) duplicate charge 

(B) direct charge 

(C) direct current 

(D) diode current 

DC stands for direct current. I made up the other choices. The correct answer is Choice (C). 

26. Which of the following is the ohm symbol? 

(A) Z 

(B) A 
(C)O 

(D) n 

Remember, the upside-down horseshoe (the Greek letter omega) is the symbol for ohm, the 
measure of electrical resistance. The correct answer is Choice (D). 



/r 



/ 86 Part IV: The Whole BaM of Facts: Technical Skills 



27. Which of the following has the least resistance? 

(A) iron 

(B) rubber 

(C) silver 

(D) wood 

Silver is the best conductor of electricity of those listed here. Therefore, it offers the least 
resistance to an electric current. The correct answer is Choice (C). 

28. What is the point at which electrical connections (such as two wires) are made? 

(A) terminal 

(B) trigger 

(C) transmitter 

(D) transformer 

A terminal is a device that connects electrical circuits together, a trigger initiates a circuit 
action, a transmitter is a device used to achieve transmission, and a transformer is an induc- 
tor with two or more windings. Windings are magnetic wires that are coated with enamel 
and wrapped around the core of a transformer. The primary winding is driven by transis- 
tors, and the secondary winding is driven by the core's magnetic field, produced by the pri- 
mary winding. Choice (B) is the correct answer. 

29. A device used to amplify a signal is called a 

(A) diode. 

(B) transformer. 

(C) rectifier. 

(D) transistor. 

A diode is a semiconductor that conducts electricity in one direction only; a transformer is a 
device that changes voltage (either "transforming" low voltage to high voltage or high volt- 
age to low voltage); a rectifier is a circuit that changes alternating current to direct current. 
Choice (D) is the correct answer. 

30. The amount of electrical power is measured in units called 

(A) volts. 

(B) amperes. 

(C) watts. 

(D) ohms. 

A watt measures the amount of power, the rate at which energy is produced or used. The 
correct answer is Choice (C). 



Chapter 15: Facing the Facts: Technical Skills Practice Questions I $7 



31. Components designed to store electrical charge are called 

(A) capacitors. 

(B) transformers. 

(C) resistors. 

(D) transistors. 

Capacitors store electric charge. They're used with resistors in timing circuits because it 
takes time for a capacitor to store voltage (to become charged). The correct answer is 
Choice (A). 

32. In an electronic circuit diagram, the symbol used to show wires' connecting is a/an 

(A) X symbol. 

(B) dot. 

(C) dark square. 

(D) T symbol. 

Wires connected to each other are indicated by a darkened circle. The correct answer is 
Choice (B). 



Assembling Objects Practice Questions 

Assembling Objects questions measure your spatial skills. There are two types of questions: 
connecting questions and putting-pieces-together questions. The first type presents you 
with simple geometric figures such as stars, cloud shapes, letter shapes, circles, and trian- 
gles. Your task is to choose the answer that shows the shapes properly connected together 
at the designated points. The second type of question is similar to putting a jigsaw puzzle 
together. Choose the answer that best shows what the shapes in the first drawing would 
look like if assembled together. See Chapter 14 for a complete explanation and illustrated 
examples. 



33. 






^3 



CL^ 



£S 




B 



D 



Note that the bottom figure in the first drawing has a line that intersects the short side of 
the trapezoid shape, so (C) and (D) are wrong. Connection point A is at the tip of the mitten 
shape, so (A) is wrong as well. The correct answer is Choice (B). 



34. 



^ 


R2 


8 


§ 


«J 



B 



D 



Mentally rotate and reposition the shapes in the first drawing until you can see how they fit 
together to form the shape shown in Choice (C) — the correct answer. In first drawing, 
notice that the shape at the upper right resembles a shark fin — it has two sharp points, 
and the third point is curved. Choice (C) is the only image that contains this shape (it's at 
the bottom). 



/r 



/ 88 Part IV: The Whole BaM of Facts: Technical Skills 



35. 



1^ A T 

B 

O ! 


1 


«* 


®Tt 


0— 0- 



36. 



37. 



38. 



39. 



40. 



B 



D 



If you selected Choice (A), you were fooled. The arrow shape shown in Choice (A) is a 
mirror of the shape depicted in the first drawing. The correct answer is Choice (D). 







B 



D 



Mentally rotate and reposition the shapes in the first drawing until you can see how they fit 
together to form the shape shown in Choice (A) — the correct answer. If you had trouble 
with this one, notice that the piece in the center of the upside-down heart should have a 
corner that dips a bit on the left. Choice (C) has the dip in the center, and Choice (D) has it 
on the right, so these answers are wrong. Choice (B) has only three pieces. 








B 



D 



Note that both shapes in the first drawing have lines that intersect the shapes at designated 
points. If you selected Choice (B), your eyes were fooled by mirror images. The correct 
answer is Choice (D). 



B 



D 



Mentally rotate and reposition the shapes in the first drawing until you can see how they fit 
together to form the shape shown in Choice (B) — the correct answer. Here, you can take a 
mental snapshot of the largest shape and look for it in the answers — (B) is the only choice 
that has it. Verify that this is the right answer by recognizing that (B) is also the only 
answer that contains a segment of a circle, at the top. 








B 



D 



Don't be fooled by the mirror shape in Choice (B), because the correct answer is 
Choice (A). 





e 





$ 


$1 



B 



D 



Mentally rotate and reposition the shapes in the first drawing until you can see how they 
fit together to form the shape shown in Choice (C), which is the correct answer. Here, you 
may note that the first drawing contains two shapes that resemble triangles with one side 
curved inward. Choice (C) is the only image that contains those shapes. 



PartV 

Practice ASVAB Exams 



The 5 th Wave 



By Rich Tennant 



<s 



PfcWB*.iN4MT 




IB 



RECRUITER 



"Excuse we. Vm a dog. Can I still take 
tke CAT-ASVA^?'* 



In this part . . . 



m Boing well on the ASVAB requires an effective study 
♦i^plan. You want to concentrate your study time on 
subject areas you may be having problems with. The prac- 
tice examinations in this part are great tools to enhance 
and plan your study program. 

Take the first test in this section to determine your 
strengths and weaknesses. Concentrate most of your 
study efforts on subject areas that are hard for you. When 
you think you've got it down, take the second test to mea- 
sure your improvement. Take the third test right before 
you're ready to take the actual ASVAB to brush up on your 
test-taking skills. 

In this part, you also find a bonus Armed Forces 
Qualification Test (AFQT) practice exam. This practice 
test includes only the four ASVAB subtests that are used 
to make up your AFQT score — the score that determines 
whether you can even join the military branch of your 
choice. 

Not only does taking the sample tests help you understand 
what you need to study, but it also gets you into the test- 
taking mindset. By taking the tests, you get used to the 
format of each subtest. Trust me — these sample tests will 
give you confidence on test day. 



Chapter 16 

Practice Exam 1 




7 his sample test features nine subtests, just like the ASVAB. As you may have guessed, 
the sample tests in this book are paper-based tests. (Yes, I'm a master of the obvious.) 
When you take the actual ASVAB, it may be a paper-based or a computer-based exam. The 
computer version basically has the same subtests as the paper version, but it follows a dif- 
ferent time format and has a different number of questions. 

Another difference with the computer-based test is that you can't skip a question and go 
back to it, and you can't change an answer after you enter it into the computer. Check out 
the computer-based test in greater detail in Chapter 3, and experience computer-based 
practice tests on the CD-ROM. 

To get the most out of this sample test, take it under the same conditions as the real ASVAB: 

v* Allow yourself about 3 hours to take the entire exam, and take the whole thing at one 
time. 

i*" Find a quiet place where you won't be interrupted. 

c" Bring a timer that you can set for various lengths of time, some scratch paper (you get 
two pieces during the exam, but you can get more; just ask for it as needed), and a 
pencil. 

J-" At the start of each subtest, set your timer for the specified period of time. Don't go on 
to the next section until the timer has gone off, and don't go back to a previous section. 
If you finish early, check your work for that section only. 

J-" Use the answer sheet that's provided. 

J-" Don't take a break during any subtest. You can take a short one- or two-minute break 
between subtests if you need it. 

After you complete the entire sample test, check your answers against the answers and 
explanations in Chapter 17. 

Your primary goal with this sample test is to determine your strengths and weaknesses. If 
you miss only one question on the Word Knowledge subtest but you miss 15 on Arithmetic 
Reasoning, you know where to spend your study time. If you're not going to pursue a career 
that requires a score on a particular subtest or the type of knowledge a subtest covers, 
don't worry about your score there. (See Appendix A for information on the subtests that 
various careers require good scores on.) 



192 



Part V: Practice ASVAB Exams 



Answer Sheet for Practice Emm 1 



Subtest 1: General Science 

1®®©® 6®®©® 11®®©® 16®®©® 21®®©® 

2®®©® 7®®©® 12®®©® 17®®©® 22®®©® 

3®®©® 8®®©® 13®®©® 18®®©® 23®®©® 

4®®©® 9®®©® 14®®©® 19®®©® 24®®©® 

5®®©® 10®®©® 15®®©® 20®®©® 25®®©® 

Subtest 2: Arithmetic Reasoning 

1®®©® 7®®©® 13®®©® 19®®©® 25®®©® 

2®®©® 8®®©® 14®®©® 20®®©® 26®®©® 

3®®©® 9®®©® 15®®©® 21®®©® 27®®©® 

4®®©® 10®®©® 16®®©® 22®®©® 28®®©® 

5®®©® 11®®©® 17®®©® 23®®©® 29®®©® 

6®®©® 12®@©@ 18®®©® 24®®©® 30®®©® 



Subtest 3: Word Knowledge 














1 ®@©@ 8 ®@©® 


15 


®@©® 


22 


®@©® 


29 


®@©@ 


2 ®@©® 9 ®®©® 


16 


®@©@ 


23 


®®©@ 


30 


®@©@ 


3®®©® 10®®©® 


17 


®®©® 


24 


®®@® 


31 


®®©® 


4®®©® 11®@©@ 


18 


®@©@ 


25 


®®@® 


32 


®®©@ 


5®®©® 12®@©@ 


19 


®@©@ 


26 


®®©® 


33 


®®©® 


6®®©® 13®@©@ 


20 


®@©® 


27 


®®@® 


34 


®®©® 


7®®©® 14®®©® 


21 


®@©® 


28 


®®©@ 


35 


®®©@ 


Subtest 4: Paragraph Comprehension 














1 ®@©® 4 ®@©® 


7 


®@©® 


10 


®®©® 


13 


®®©® 


2 ®®©® 5 ®®©® 


8 


®®©® 


11 


®®©@ 


14 


®®©® 


3 ®@©® 6 ®@©@ 


9 


®®©@ 


12 


®®©® 


15 


®@©® 


Subtest 5: Mathematics Knowledge 














1 @®©@ 6 ®®©® 


11 


®@©® 


16 


®@©® 


21 


®@©® 


2 ®@©@ 7 ®®©® 


12 


®®©® 


17 


®®©@ 


22 


®®©@ 


3 ®®©® 8 ®®©® 


13 


®®©® 


18 


®®©@ 


23 


®®©® 


4 ®®@® 9 ®@©@ 


14 


®@©@ 


19 


®®@® 


24 


®®©@ 


5®®©® 10®®©® 


15 


®@©® 


20 


®®©® 


25 


®®©® 


Subtest 6: Electronics Information 














1 ®®©® 5 ®@©® 


9 


®@©® 


13 


®®©® 


17 


®®©® 


2 ®@©@ 6 ®®©® 


10 


®®@® 


14 


®®©@ 


18 


®@©@ 


3 ®@©® 7 ®®©® 


11 


®®©@ 


15 


®®©@ 


19 


®®©@ 


4 ®®©® 8 ®@©® 


12 


®@©® 


16 


®®©® 


20 


®®@® 


Subtest 7: Auto & Shop Information 














1 ®®©@ 6 ®@©® 


11 


®@©® 


16 


®®©® 


21 


®@©@ 


2 ®@©@ 7 ®®©® 


12 


®®@® 


17 


®®©@ 


22 


®@©@ 


3 ®@©® 8 ®®©® 


13 


®®©® 


18 


®®©® 


23 


®®©® 


4 ®®©® 9 ®@©@ 


14 


®@©® 


19 


®®©@ 


24 


®®©@ 


5®®©® 10®®©® 


15 


®®©® 


20 


®®@® 


25 


®®©® 



Subtest 8: Mechanical Comprehension 

1®®©® 6®®©® 11®®©® 16®®©® 21®®©® 

2®®©® 7®®©® 12®®©® 17®®©® 22®®©® 

3®®©® 8®®©® 13®®©® 18®®©® 23®®©® 

4®®©® 9®®©® 14®®©® 19®®©® 24®®©® 

5®®©® 10®@©@ 15®®©® 20®®©® 25®®©® 

Subtest 9: Assembling Objects 

1®@©@ 6®®©® 11®®©® 16®®©® 21®@©@ 

2®®©® 7®®©® 12®®©® 17®®©® 22®®©® 

3®®©® 8®®©® 13®®©® 18®®©® 23®®©® 

4®®©® 9®®©® 14®®©® 19®®©® 24®®©® 

5®®©® 10®@©@ 15®®©® 20®®©® 25®®©® 



Chapter 16: Practice Exam 1 f 9j 



Subtest 1: General Science 



Time: 11 minutes for 25 questions 

Directions: This subtest tests your knowledge of general science principles usually covered in high 
school classes. Pick the best answer for each question and then mark the space on your answer 
sheet that corresponds to the letter indicating your choice. 



1. Which planet is named after the Greek god 
who personified the sky? 

(A) Earth 

(B) Mars 

(C) Pluto 

(D) Uranus 

2. An animal that eats only meat is called a(n) 

(A) omnivore. 

(B) herbivore. 

(C) carnivore. 

(D) voracious. 

3. The chemical process in which electrons 
are removed from a molecule is called 

(A) respiration. 

(B) recreation. 

(C) oxidation. 

(D) metabolism. 

4. What is a single unit of quanta called? 

(A) quantum 

(B) quantumonium 

(C) quantus 

(D) quanfactorial 

5. Light waves travel at a rate of about 

(A) 186,000 miles per hour. 

(B) 186,000 miles per minute. 

(C) 18,600 miles per hour. 

(D) 186,000 miles per second. 

6. The largest planet in the solar system is 

(A) Earth. 

(B) Mars. 

(C) Saturn. 

(D) Jupiter. 



7. The intestines are part of the 

(A) circulatory system. 

(B) nervous system. 

(C) respiratory system. 

(D) digestive system. 

8. Joints that hold bones firmly together are 
called 

(A) hinge joints. 

(B) ball and socket joints. 

(C) fixed joints. 

(D) pivot joints. 

9. Of the levels listed, the top or broadest 
level of the classification system for living 
organisms is called the 

(A) class. 

(B) phylum. 

(C) kingdom. 

(D) genus. 

10. Which planet is the brightest object in the 
sky, aside from the sun and moon? 

(A) Saturn 

(B) Pluto 

(C) Venus 

(D) Mercury 

11. The human heart includes 

(A) 2 chambers. 

(B) 3 chambers. 

(C) 4 chambers. 

(D) 5 chambers. 



Go on to next page 



196 



Part V: Practice ASVAB Exams 



12. White blood cells 

(A) produce antibodies. 

(B) fight infections. 

(C) carry oxygen and carbon dioxide. 

(D) both A and B 

13. A measureable amount of protein can be 
found in all of the following foods EXCEPT 

(A) eggs. 

(B) meat. 

(C) peas. 

(D) apples. 

14. What is the most abundant element, by 
mass, in the Earth's crust? 

(A) carbon 

(B) oxygen 

(C) gold 

(D) salt 

15. Osmosis is 

(A) diffusion of water. 

(B) transfer of oxygen. 

(C) low blood sugar. 

(D) protein. 

16. A meter consists of 

(A) 10 centimeters. 

(B) 100 millimeters. 

(C) 100 centimeters. 

(D) 10 millimeters. 

17. One light-year is 

(A) the distance traveled by light in one 
year. 

(B) the brightness of light at 30,000 miles. 

(C) 17 standard Earth years. 

(D) Spock's birthday. 



18. Electrons are particles that are 

(A) positively charged. 

(B) neutral. 

(C) able to move freely. 

(D) negatively charged. 

19. The asteroid belt is located 

(A) around Mercury. 

(B) between Mars and Jupiter. 

(C) inside the orbit of Venus. 

(D) There is no such thing as an asteroid 
belt. 

20. The atomic number of an atom is deter- 
mined by 

(A) the size of its nucleus. 

(B) the number of protons. 

(C) the number of electrons. 

(D) its location in the periodic table. 

21. The "control center" of a cell is called the 

(A) nucleus. 

(B) compound. 

(C) mitochondria. 

(D) atom. 

22. How many planets in the solar system have 
rings? 

(A) one 

(B) two 

(C) three 

(D) four 

23. The temperature at which a substance's 
solid and liquid states exist in equilibrium 
is its 

(A) melting point. 

(B) boiling point. 

(C) anti-freezing point. 

(D) concentration point. 



Go on to next page 



Chapter 16: Practice Exam 1 f Qjy 



24. The atmosphere of Mars is composed 
mostly of 

(A) oxygen. 

(B) carbon dioxide. 

(C) helium. 

(D) Mars has no atmosphere. 



25. Not counting the sun, the closest star to 
the Earth is 

(A) Rigel. 

(B) Proxima Centauri. 

(C) Antares. 

(D) Betelgeuse. 



STOPI 



DO NOTTURNTHE PAGE UNTILTOLDTO DO SO. 
DO NOT RETURN TO A PREVIOUS TEST. 



196 



Part V: Practice ASVAB Exams 



Subtest 2: Arithmetic Reasoning 



Time: 36 minutes for 30 questions 

Directions: This test contains questions about arithmetic. Each question is followed by four possi- 
ble answers. Decide which answer is correct and then mark the space on your answer sheet that 
has the same number and letter as your choice. Use scratch paper for any figuring you want to do. 
A calculator is not allowed. 



4. 



If a car is towed 12 miles to the repair shop 
and the tow charge is $3.50 per mile, how 
much does the tow cost? 

(A) $12.00 

(B) $3.50 

(C) $42.00 

(D) $100.00 

The sum of two numbers is 70. One number 
is 8 more than the other. What's the 
smaller number? 

(A) 31 

(B) 33 

(C) 35 
(D)36 

A sales manager buys antacid in bottles by 
the gross. If he goes through 3 bottles of 
antacid every day, how long will the gross 
last? 

(A) 144 days 

(B) 3 days 

(C) 20 days 

(D) 48 days 

Jenny's test grades are 93, 89, 96, and 98. If 
she wishes to raise her average to 95, what 
does she need to score on her next test? 

(A) 100 

(B) 99 

(C) 97 
(D)95 



5. A waitress earns an average tip of 12% of 
the cost of the food she serves. If she 
serves $375 worth of food in one evening, 
how much money in tips will she earn on 
average? 

(A) $37 

(B) $45 

(C) $42 

(D) $420 

6. How many square feet of carpeting are 
needed to carpet a 12-foot x 12-foot room? 

(A) 24 

(B) 120 
(C)48 
(D) 144 

7. Carpet stain protector costs $0.65 per 
square yard to apply. How much will it cost 
to apply the protector to a 16-foot x 18-foot 
carpet? 

(A) $187.20 

(B) $62.40 

(C) $20.80 

(D) $96.00 

8. A printing plant that produces baseball 
cards has a monthly overhead of $6,000. It 
costs 18 cents to print each card, and the 
cards sell for 30 cents each. How many 
cards must the printing plant sell each 
month in order to make a profit? 

(A) 30,000 

(B) 40,000 

(C) 50,000 

(D) 60,000 



Go on to next page 



Chapter 16: Practice Exam 1 /v7 



9. Joe received an hourly wage of $8.15. His 
boss gave him a 7% raise. How much does 
Joe make per hour now? 

(A) $0.57 

(B) $8.90 

(C) $8.72 

(D) $13.85 

10. Alice leaves her house, driving east at 45 
miles per hour (mph). Thirty minutes later, 
her husband Dave notices she forgot her 
cell phone and sets off after her. How fast 
must Dave travel in order to catch up with 
Alice 3 hours after he leaves? 

(A) 49 mph 

(B) 50.5 mph 

(C) 52.5 mph 

(D) 54 mph 

11. A baker made 20 pies. A Boy Scout troop 
buys one-fourth of his pies, a preschool 
teacher buys one-third of his pies, and a 
caterer buys one-sixth of his pies. How 
many pies does the baker have left? 

( A ) | 
(B)15 
(9 12 
CD) 5 

12. Miriam bought five cases of motor oil on 
sale. A case of motor oil normally costs 
$24.00, but she was able to purchase the oil 
for $22.50 a case. How much money did 
Miriam save on her entire purchase? 

(A) $7.50 

(B) $1.50 

(C) $8.00 

(D) $22.50 

13. A security guard walks the equivalent of six 
city blocks when he makes a circuit around 
the building. If he walks at a pace of eight city 
blocks every 30 minutes, how long will it take 
him to complete a circuit around the building, 
assuming he doesn't run into any thieves? 

(A) 20.00 minutes 

(B) 3.75 minutes 

(C) 22.50 minutes 

(D) 7.5 minutes 



14. The population of Grand Island, Nebraska, 
grew by 600,000 people between 1995 and 
2005, one-fifth more than the town council 
predicted. The town council originally pre- 
dicted the city's population would grow by 

(A) 400,000 

(B) 500,000 

(C) 300,000 

(D) 100,000 

15. Joan is taking an admissions examination. If 
she has to get at least 40 of the 60 questions 
right to pass, what percent of the questions 
does she need to answer correctly? 

(A) 30% 

(B) 40% 

(C) 66^% 

(D) 66|% 

16. A teacher deposits $3,000 in a retirement fund. 
If she doesn't add any more money to the 
fund, which earns an annual interest rate of 
6%, how much money will she have in 1 year? 

(A) $180 

(B) $3,006 

(C) $3,180 

(D) $6,000 

17. The high school track measures one- 
quarter of a mile around. How many laps 
would you have to run in order to run three 
and a half miles? 

(A) 12 

(B)14 

(C)16 

CD) 18 

18. Karl is driving in Austria, where the speed 
limit is posted in kilometers per hour. The 
car's speedometer shows that he's travel- 
ing at a rate of 75 kilometers per hour. Karl 

knows that a kilometer is about -^ of a mile. 

o 

Approximately how many miles per hour is 
Karl traveling? 

(A) 47 

(B) 120 

(C) 50 
(D)53 



Go on to next page 



198 



Part V: Practice ASVAB Exams 



19. A carpenter earns $12.30 an hour for a 
40-hour week. His overtime pay is 1-4 times 
his base pay. If he puts in a 46-hour week, 
how much is his weekly pay? 

(A) $602.70 

(B) $492.00 

(C) $565.80 

(D) $110.70 

20. An office building has 30 employees and 
provides 42 square feet of work space per 
employee. If five more employees are hired, 
how much less work space will each 
employee have? 

(A) 6 square feet 

(B) 7 square feet 

(C) 7.5 square feet 

(D) 36 square feet 

21. Stan bought a monster truck for $2,000 
down and payments of $450 a month for 
five years. What's the total cost of the mon- 
ster truck? 

(A) $4,250 

(B) $29,000 

(C) $27,000 

(D) $34,400 

22. Darla spent $120.37 on groceries in 
January, $108.45 in February, and $114.86 
in March. What was the average monthly 
cost of Darla's groceries? 

(A) $343.68 

(B) $110.45 

(C) $114.86 

(D) $114.56 

23. Keith is driving from Reno to Kansas City to 
meet his girlfriend. The distance between 
the two cities is 1,650 miles. If Keith can 
average 50 miles per hour, how many hours 
will it take him to complete his trip? 

(A) 8 hours 

(B) 30 hours 

(C) 33 hours 

(D) 82 hours 



24. Michael needs 55 gallons of paint to paint 
an apartment building. He would like to 
purchase the paint for the least amount of 
money possible. Which of the following 
should he buy? 

(A) two 25-gallon buckets at $550 each 

(B) eleven 5-gallon buckets at $108 each 

(C) six 10-gallon buckets at $215 each 

(D) fifty-five 1-gallon buckets at $23 each 

25. As a member of FEMA, you're required to 
set up a contingency plan to supply meals 
to residents of a town devastated by a tor- 
nado. A breakfast ration weighs 12 ounces 
and the lunch and dinner rations weigh 

18 ounces each. Assuming a food truck can 
carry 3 tons and that each resident will 
receive 3 meals per day, how many resi- 
dents can you feed from one truck during 
a 10-day period? 

(A) 150 residents 

(B) 200 residents 

(C) 250 residents 

(D) 300 residents 

26. A train headed south for Wichita left the 
station at the same time a train headed 
north for Des Moines left the same station. 
The train headed for Wichita traveled at 

55 miles per hour. The train headed for Des 
Moines traveled at 70 miles per hour. How 
many miles apart are the trains at the end 
of 3 hours? 

(A) 210 miles 

(B) 165 miles 

(C) 125 miles 

(D) 375 miles 

27. A carpenter needs to cut four sections, 
each 3 feet, 8 inches long, from a piece of 
molding. If the board is only sold by the 
foot, what's the shortest length of board 
she can buy? 

(A) 15 feet 

(B) 14 feet 

(C) 16 feet 

(D) 12 feet 



Go on to next page 



Chapter 16: Practice Exam 1 f QQ 



28. Kiya had only one coupon for 10% off one 
frozen turkey breast. The turkey breasts 
cost $8.50 each, and Kiya wanted to buy 
two. How much did she pay? 

(A) $16.15 

(B) $17.00 

(C) $15.30 

(D) $7.65 

29. A recruiter travels 1,100 miles during a 

2 
40-hour workweek. If she spends £ of her 

time traveling, how many hours does she 
spend traveling? 

(A) 22 



30. Your car uses gasoline at the rate of 21 

miles per gallon. If gasoline costs $2.82 per 
gallon and you drive for 7 hours at a speed 
of 48 miles per hour, how much will you 
pay for gasoline for the trip? 

(A) $38.18 

(B) $45.12 

(C) $47.73 

(D) 59.27 



(B)5 



1 



(9 16 

(D)8 



STOPI 



DO NOTTURNTHE PAGE UNTILTOLDTO DO SO. 
DO NOT RETURN TO A PREVIOUS TEST. 



200 



Part V: Practice ASVAB Exams 



Subtest 3: Word Knortiedqe 



Time: 11 minutes for 35 questions 

Directions: This test is about the meanings of words. Each question has a word underlined. You 
may be asked to decide which one of the four words in the choices most nearly means the same 
thing as the underlined word or which one of the four words means the opposite. If the underlined 
word is used in a sentence, decide which of the four choices most nearly means the same thing as 
the underlined word, as used in the context of the sentence. Mark the corresponding space on your 
answer sheet. 



1. Tim promised to meet us at the apex . 
(A) top 

(B) bottom 

(C) canyon 

(D) river 

2. Assimilate most nearly means 

(A) absorb. 

(B) react. 

(C) pretend. 

(D) lie. 

3. Brittle most nearly means 

(A) soft. 

(B) fragile. 

(C) study. 

(D) hard. 

4. Datum most nearly means 

(A) fiscal year date. 

(B) congruence. 

(C) fact. 

(D) positive result. 

5. The exchange student was proficient in 
French, German, and English. 

(A) poor 

(B) knowledgeable 

(C) adept 

(D) exacting 



6. The judge imposed a severe penalty due to 
Tom's actions. 

(A) scheduled 

(B) made an example of 

(C) levied 

(D) questioned 

7. Mary went to the store and bought peanuts 
galore . 

(A) abundant 

(B) salty 

(C) on sale 

(D) roasted 

8. He ran headlong into the fight. 

(A) headfirst 

(B) reluctantly 

(C) happily 

(D) recklessly 

9. Frugal most nearly means 

(A) quiet. 

(B) amazing. 

(C) delayed. 

(D) economical. 

10. The word most opposite in meaning to 
stimulate is 

(A) support. 

(B) arrest. 

(C) travel. 

(D) dislike. 



Go on to next page 



Chapter 16: Practice Exam 1 2 v / 



11. Licit most nearly means 

(A) historical. 

(B) lawful. 

(C) storied. 

(D) willfully. 

12. Vacate most nearly means 

(A) crawl. 

(B) impel. 

(C) exhume. 

(D) leave. 

13. The sergeant gave his reasoned opinion. 

(A) irate 

(B) logical 

(C) impressive 

(D) uninformed 

14. Tacit most nearly means 

(A) loud. 

(B) understood. 

(C) commendable. 

(D) transparent. 

15. The brass was not burnished . 

(A) yellow 

(B) dull 

(C) expensive 

(D) polished 

16. The commodity was sold. 

(A) product 

(B) stock 

(C) idea 

(D) table 

17. Her motives were contrived . 

(A) premeditated 

(B) emotional 

(C) obscure 

(D) amusing 



18. Supplicate most nearly means 

(A) make superior. 

(B) to be unnecessary. 

(C) to beg. 

(D) to be expansive. 

19. The word most opposite in meaning to 
hypocrisy is 

(A) honesty. 

(B) happy. 

(C) angry. 

(D) threatening. 

20. Bob found the peaches to be extremely 
succulent . 

(A) large 

(B) tasteless 

(C) old 

(D) juicy 

21. The Army soldiers were ordered to 
immediate garrison duty. 

(A) field 

(B) combat 

(C) latrine 

(D) fort 

22. Furtherance most nearly means 

(A) advancement. 

(B) finance. 

(C) practicality. 

(D) destruction. 

23. Domicile most nearly means 

(A) office. 

(B) shopping. 

(C) home. 

(D) vacation. 

24. Abrogate most nearly means 

(A) recover. 

(B) aid. 

(C) foreclose. 

(D) abolish. 



Go on to next page 



202 



Part V: Practice ASVAB Exams 



25. Compensation most nearly means 

(A) religion. 

(B) commission. 

(C) boathouse. 

(D) shower. 

26. He gave a brusque account of the events. 

(A) passionate 

(B) lengthy 

(C) uncensored 

(D) concise 

27. The vote resulted in the demise of the 
proposed new law. 

(A) passage 

(B) death 

(C) postponement 

(D) abatement 

28. We commemorated our veterans during 
the ceremony. 

(A) denied 

(B) remembered 
(C) thanked 

(D) took pictures of 

29. Bore most nearly means 

(A) deepen. 

(B) hide. 

(C) burrow. 

(D) jump. 

30. That custom still prevails . 

(A) angers 

(B) persists 

(C) surprises 

(D) excites 



31. Defray most nearly means 

(A) invade. 

(B) obstruct. 

(C) pay. 

(D) reverse. 

32. Chasm most nearly means 

(A) abyss. 

(B) sky. 

(C) mountain. 

(D) valley. 

33. Fundamental most nearly means 

(A) radical. 

(B) religious. 

(C) basic. 

(D) excessive. 

34. Susceptible most nearly means 

(A) travel. 

(B) resistant. 

(C) limited. 

(D) vulnerable 

35. Emblem most nearly means 

(A) symbol. 

(B) picture. 

(C) statue. 

(D) religion. 



STOPI 



DO NOTTURNTHE PAGE UNTILTOLDTO DO SO. 
DO NOT RETURN TO A PREVIOUS TEST. 



Chapter 16: Practice Exam 1 203 



Subtest %: Paragraph Comprehension 



Time: 13 minutes for 15 questions 

Directions: This test contains items that measure your ability to understand what you read. This 
section includes one or more paragraphs of reading material followed by incomplete statements or 
questions. Read the paragraph and select the choice that best completes the statement or answers 
the question. Mark your choice on your answer sheet, using the correct letter with each question 
number. 



An important stage of personal time manage- 
ment is to take control of appointments. 
Determined by external obligation, appointments 
constitute interaction with other people and an 
agreed-on interface between your activities and 
those of others. Start with a simple appointment 
diary. List all appointments, including regular 
and recurring ones. Now, be ruthless and elimi- 
nate the unnecessary. There may be committees 
where you can't productively contribute or 
where a subordinate may be able to participate. 
Eliminate the waste of your time. 

1. Effectively managing your appointments 
allows you to 

(A) spend more time with your 
subordinates. 

(B) delegate responsibility to subordinates. 

(C) make more efficient use of your time. 

(D) attend only the most important 
meetings. 

The U.S. Congress consists of 100 senators 
and 435 representatives. Two senators are 
elected from each state. The number of 
representatives from each state is based on pop- 
ulation, although each state has at least one rep- 
resentative. Senators serve six-year terms, and 
representatives serve two-year terms. 

2. According to this passage, 

(A) there are equal numbers of senators 
and representatives. 

(B) the number of representatives from 
each state is decided by a lottery. 

(C) it's possible for a state to have no 
representatives. 

(D) senators and representatives have 
different term lengths. 



Indo-European languages consist of those 
languages spoken by most of Europe and in 
those parts of the world that Europeans have 
colonized since the 16th century (such as the 
United States). Indo-European languages are also 
spoken in India, Iran, parts of western 
Afghanistan, and in some areas of Asia. 

3. The author of this passage would agree that 

(A) Indo-European languages are spoken in 
areas all over the world. 

(B) Indo-European languages include all 
the languages spoken in the world. 

(C) only Europeans speak Indo-European 
languages. 

(D) Indo-European language speakers can 
easily understand one another. 

In privatization, the government relies on 
the private sector to provide a service. However, 
the government divests itself of the entire process, 
including all assets. With privatized functions, the 
government may specify quality, quantity, and 
timeliness requirements, but it has no control over 
the operations of the activity. Also, the govern- 
ment may not be the only customer. Whoever the 
government chooses to provide the services 
would likely provide the same services to others. 

4. This paragraph best supports the state- 
ment that 

(A) the government must closely supervise 
privatized functions. 

(B) privatized functions consist of a mix- 
ture of government employees, military 
personnel, and private contractors. 

(C) privatized functions are those institu- 
tions that provide services only to a 
government agency. 

(D) privatized functions provide essential 
services to the government. 



Go on to next page 



20U 



Part V: Practice ASVAB Exams 



The success or failure of a conference lies 
largely with its leader. A leader's zest and enthu- 
siasm must be real, apparent, and contagious. 
The leader is responsible for getting the ball roll- 
ing and making the attendees feel as if the meet- 
ing is theirs and its success depends on their 
participation. A good, thorough introduction 
helps establish the right climate. 

5. A good title to this paragraph would be 

(A) "Lead by Example." 

(B) "The Importance of Proper Introductions." 

(C) "Leading a Successful Conference." 

(D) "Conference Participation Basics." 

Cloud seeding is accomplished by dropping 
particles of dry ice (solid carbon dioxide) from a 
plane onto super-cooled clouds. This process 
encourages condensation of water droplets in 
the clouds, which usually, but not always, results 
in rain or snow. 

6. From this passage, it's reasonable to 
assume that 

(A) cloud seeding could be used to end a 
drought. 

(B) cloud seeding is prohibitively expensive. 

(C) cloud seeding is rarely used. 

(D) cloud seeding can be accomplished by 
using regular ice. 

To write or not to write — that is the ques- 
tion. If assigned a writing task, there's no option. 
However, if someone is looking for a specific 
answer, find out if they need a short answer or a 
detailed one. Can the requirement be met with a 
telephone call, e-mail, or short note, or is some- 
thing more necessary? A former CEO of a major 
corporation once commented that he had looked 
at 13,000 pieces of paper in a 5-day period. Think 
how much easier and more economical it would 
be if people would use the telephone, send an 
e-mail, or write a short note. 

7. The main point of this passage is that 

(A) written records are important because 
they provide detailed documentation. 

(B) more businesspeople should invest time 
and energy improving their writing skills. 

(C) writing may not be the best way to 
communicate information. 

(D) it's pointless for businesspeople to 
spend time improving their writing skills. 



The transistor, a small, solid-state device 
that can amplify sound, was invented in 1947. At 
first, it was too expensive and too difficult to 
produce to be used in cheap, mass-market prod- 
ucts. By 1954, though, these cost and production 
problems had been overcome, and the first tran- 
sistor radio was put on the market. 

8. According to this passage, 

(A) there was no market for transistors 
before 1954. 

(B) when transistors could be produced 
cheaply and easily, the transistor radio 
was put on the market. 

(C) transistors were invented in 1947 by 
order of the Department of Defense. 

(D) transistors are still expensive to produce. 

I returned from the City about three o'clock on 
that May afternoon pretty well disgusted with life. I 
had been three months in the Old Country and was 
fed up with it. If people had told me a year ago that 
I would've been feeling like that I should've laughed 
at them; but there was the fact. The weather made 
me liverish, the talk of the ordinary Englishman 
made me sick, I couldn't get enough exercise, and 
the amusements of London seemed as flat as soda 
water that had been standing in the sun. 

9. The author is speaking of his travels in 

(A) Spain. 

(B) Great Britain. 

(C) Germany. 

(D) Scotland. 

Surveys show that the average child under 
the age of 18 watches four hours of television 
per day. Although some of the programming may 
be educational, most isn't. Spending this much 
time watching television interferes with a child's 
ability to pursue other interests, such as reading, 
participating in sports, and playing with friends. 

10. The author of this passage would agree that 

(A) television viewing should be restricted. 

(B) parents who let their children watch 
this much television are neglectful. 

(C) reading, participating in sports, playing 
with friends, and watching television 
should all be given equal time. 

(D) adults over 18 can watch as much tele- 
vision as they want. 



Go on to next page 



Chapter 16: Practice Exam 1 203 



Questions 11 and 12 are based on the following 
passage. 



Questions 13 through 15 are based on the 
following passage. 



High school and college graduates attempt- 
ing to find jobs should participate in mock job 
interviews. These mock interviews help students 
prepare for the types of questions they'll be 
asked, make them more comfortable with 
common interview formats, and help them cri- 
tique their performance before facing a real 
interviewer. Because they're such a valuable aid, 
schools should organize mock job interviews for 
all of their graduating students. 

11. The above passage states that mock job 
interviews 

(A) frighten students. 

(B) should be offered to the best students. 

(C) help prepare students for real job 
interviews. 

(D) should be organized by students. 

12. From the above passage, it is reasonable to 
assume that 

(A) mock interviews can increase a stu- 
dent's confidence when he or she goes 
into a real job interview. 

(B) mock interviews are expensive to 
organize. 

(C) few students are interested in mock 
interviews. 

(D) students don't need job interview 
preparation. 



Due process, the guarantee of fairness in the 
administration of justice, is part of the 5th 
Amendment to the U.S. Constitution. The 14th 
Amendment further requires states to abide by 
due process. After this amendment was enacted, 
the U.S. Supreme Court struck down many state 
laws that infringed on the civil rights guaranteed 
to citizens in the Bill of Rights. 

1 3. According to the above passage, due process 

(A) is an outdated concept. 

(B) guarantees fairness in the justice system. 

(C) never became part of the U.S. 
Constitution. 

(D) is the process by which winning lottery 
tickets are selected. 

14. According to the above passage, it's rea- 
sonable to assume that the 5th Amendment 

(A) is about taxes. 

(B) guarantees due process in all criminal 
and civil cases. 

(C) guarantees due process in federal law. 

(D) should never have become part of the 
Bill of Rights. 

15. The author of the above passage would 
agree that 

(A) without the passage of the 14th 
Amendment, many laws restricting civil 
rights would still exist in various states. 

(B) the Supreme Court overstepped its 
jurisdiction when it struck down laws 
infringing on citizens' civil rights. 

(C) the Supreme Court had every right to 
strike down state laws before the pas- 
sage of the 14th Amendment. 

(D) the 14th Amendment was opposed by 
all states. 



STOPI 



DO NOTTURNTHE PAGE UNTILTOLDTO DO SO. 
DO NOT RETURN TO A PREVIOUS TEST. 



206 



Part V: Practice ASVAB Exams 



Subtest 5: Mathematics Knowledge 



Time: 24 minutes for 25 questions 

Directions: This section tests your ability to solve general mathematical problems. Select the cor- 
rect answer from the choices given, and then mark the corresponding space on your answer sheet. 
Use scratch paper to do any figuring. 



2. 



3. 



4. 



5. 



6. 



If x = 8, what's the value of y in the equation 


7. 


x 3 • x 4 = 


y = (x 2 * 4) - 2? 




(A) x 12 


(A) 14 




(B) 2x 7 


(B)16 




(C) 2x 12 


(0)18 




(D)x 7 


(D)20 








8. 


(x + 4)(x + 2) = 


The cube of 5 is 




(A) x 2 + 6x + 6 


(A) 125 




(B) x 2 + 8x + 8 


(B) 25 




(C) x 2 + 8x + 6 


(C)15 




(D) x 2 + 6x + 8 


(D)50 








9. 


1.5 x 10 3 = 


2.5 x 3 3 = 




(A) 45 


(A) 22.5 




(B) 150 


(B) 75.0 




(C) 1,500 


(C) 67.5 




(D)15 


(D) 675.0 








10. 


Which of the following is a prime number? 


The fourth root of 16 is 




(A) 27 


(A) 4 




(B)ll 


(B)l 




(C)8 


(C)3 




(D)4 


(D)2 








11. 


What's the mode of the following series of 


What's the equation of a line that passes 




numbers? 4 4 8 8 8 10 10 12 12 


through points (0, -1) and (2, 3)? 




(A) 9 


(A)y = 2x-1 




(B)8 


(B) y = 2x + 1 




(9 11 


(C)x = 2y-1 




(D)10 


(D) x = 2y + 1 








12. 


If a = 4, then a 3 * a = 


(12 yards + 14 feet) * 5 = 




(A) 4 


(A) 12 feet 




(B)12 


(B) 5^ feet 




(Q64 


(C) 10 feet 




(D)16 



(D) 2^ yards 



Go on to next page 



Chapter 16: Practice Exam 1 207 



13. 



14. 



15. 



16. 



17. 



18. 



19. 



Solve: 5! 






20. 


If a circle has a radius of 12 feet, what's its 


(A) 25 








circumference most nearly? 


(B) 125 








(A) 24 feet 


(C) 120 








(B) 72 feet 


(D)15 








(C) 75 feet 


(900 x 2) * 6 = 

(A) 30 

(B) 300 






21. 


(D) 36 feet 

An aquarium measures 16 inches long x 
8 inches deep x 18 inches high. What's its 
volume? 


(C) 150 








(A) 2,304 cubic inches 


(D) 3,000 








(B) 128 cubic inches 


If x - 2, then x x (x) = 








(C) 42 cubic inches 


(A) 8 








(D) 288 cubic inches 


(B) 2x x 






22. 


Triangle ABC is a(n) 


(C)4 










(D)6 








A 


If (5 + 1)(6 * 3)(8 - 5) = 


(3 


i- 3)x, then x = 




/3\ 


(A) 12 








/ \ 


(B)3 








/ \ 


(C)4 
CD) 6 








Xl 2 X 


749x^/64 = 








(A) right triangle. 


(A) 56 








(B) obtuse triangle. 


(B)15 








(C) equilateral triangle. 


(C) 42 








(D) isosceles triangle. 


(D) 3,136 






23. 


The sum of the measures of the angles of a 


Which of the following 


fractions is the 




trapezoid is 


largest? 








(A) 360 degrees. 


(A)| 








(B) 540 degrees. 


(B)| 








(C) 180 degrees. 

(D) 720 degrees. 


^>TO 










CD) || 










If 2 + x > 4, then x > 










(A) 6 










(B)2 










(C)4 











(D). 



Go on to next page 



208 Part V: Practice ASVAB Exams 



24. Angles 1 and 2 are 




(A) supplementary. 

(B) complementary. 

(C) both obtuse. 

(D) both right angles. 



25. Convert 24% to a fraction. 
6 



(A) 
(B) 
(C) 
(D) 



25 

J_ 

25 

_6_ 

24 

J_ 
24 



STOPI 



DO NOT TURN THE PAGE UNTILTOLDTO DO SO. 
DO NOT RETURN TO A PREVIOUS TEST. 



Chapter 16: Practice Exam 1 20 z) 



Subtest 6: Electronics Information 



Time 9 minutes for 20 questions 

Directions This test contains questions to challenge your knowledge of electrical, radio, and elec- 
tronics information. Select the correct response from the choices given and then mark the corre- 
sponding space on your answer sheet. 



Ohm's law states 6. 

(A) Voltage = Current x Resistance 

(B) Amperes = Current x Resistance 

(C) Voltage = Resistance * Amperes 

(D) Ohms = Current * Voltage 

A resistor's first three color bands are 

brown, black, and red. What is its value? 7. 

(A) 1,000 ohms 

(B) 500 ohms 

(C) 500 volts 

(D) 50 volts 

In the U.S., all metal equipment, electrical 
or not, connected to a swimming pool 
must be 

Q 

(A) freestanding. 

(B) bonded together. 

(C) certified. 

(D) none of the above 

Voltage can also be expressed as 

(A) watts. 9 - 

(B) amps. 

(C) current. 

(D) electrical potential difference. 

Newer cell phones contain a removable 
memory card, which is often called a 

(A) SIM card. 

(B) DIM chip. 10. 

(C) PIN card. 

(D) PIN chip. 



Made from a variety of materials, such as 
carbon, this inhibits the flow of current. 

(A) resistor 

(B) diode 

(C) transformer 

(D) generator 

This is a type of semiconductor that only 
allows current to flow in one direction. It is 
usually used to rectify AC signals (conver- 
sion to DC). 

(A) capacitor 

(B) inductor 

(C) diode 

(D) transformer 

Radar can operate at frequencies as high as 

(A) 100,000 Hz. 

(B) 100,000 kHz. 

(C) 100,000 MHz. 

(D) 500,000 MHz. 

What do AC and DC stand for in the electri- 
cal field? 

(A) amplified capacity and differential 
capacity 

(B) alternating current and direct current 

(C) accelerated climate and deduced 
climate 

(D) none of the above 

Changing AC to DC is called what? 

(A) capacitance. 

(B) impedance. 

(C) rectification. 

(D) induction. 



Go on to next page 



210 



Part V: Practice ASVAB Exams 



11. A 5,000 BTU air conditioner can efficiently 
cool up to 150 square feet, or a 10-foot x 
15-foot room. What does BTU stand for? 

(A) basic thermal unit 

(B) basic temperature unit 

(C) British thermal unit 

(D) none of the above 

12. Which is the most correct definition of 
current? 

(A) the measure of electrical pressure 

(B) the amount of electricity used in a 
heater 

(C) the electricity used in heating a kilo of 
water 

(D) the presence of electron flow 

13. A device that transforms energy from one 
form to another is called 

(A) a capacitor. 

(B) a transducer. 

(C) a transformer. 

(D) magic. 

14. Which one of the following is an active 
element? 

(A) 15 kQ resistor 

(B) 10 mH inductor 

(C) 25 pF capacitor 

(D) 10 V power supply 

15. A light bulb is 60 watts. Operated at 120 
volts, how much current does it draw? 

(A) 0.5 amperes 

(B) 5.0 amperes 

(C) 50.0 amperes 

(D) 7,200 amperes 

16. A number-12 wire, compared to a number-6 
wire, 

(A) is longer. 

(B) is shorter. 

(C) is smaller in diameter. 

(D) is larger in diameter. 



17. A fuse with a higher-than-required rating 
used in an electrical circuit 

(A) improves safety. 

(B) increases maintenance. 

(C) may not work properly. 

(D) is less expensive. 

18. Neutral wire is always 

(A) whitish or natural. 

(B) black. 

(C) green with stripes 

(D) blue. 

19. To measure electrical power, you would 
use a(n) 

(A) ammeter. 

(B) ohmmeter. 

(C) voltmeter. 

(D) wattmeter. 

20. What will happen if you operate an incan- 
descent light bulb at less than its rated 
voltage? 

(A) The bulb will burn brighter and last 
longer. 

(B) The bulb will burn dimmer and last 
longer. 

(C) The bulb will burn brighter but won't 
last as long. 

(D) The bulb will burn dimmer but won't 
last as long. 



STOPI 



DO NOT TURN THE PAGE UNTIL TOLD TO DO SO. 
DO NOT RETURN TO A PREVIOUS TEST. 



Chapter 16: Practice Exam 1 211 



Subtest 7: Auto & Shop Information 



Time: 1 1 minutes for 25 questions 

Directions: This test is about automobiles, shop practices, and the use of tools. Pick the best 
answer for each question and then mark the corresponding space on your answer sheet. 



1. Overheating the engine can cause all of the 
following problems EXCEPT 

(A) burned engine bearings. 

(B) enlarged pistons. 

(C) melted engine parts. 

(D) improved fuel efficiency. 

2. The device that converts an automobile's 
mechanical energy to electrical energy is 
called the 

(A) converter. 

(B) alternator. 

(C) battery. 

(D) brakes. 

3. A primary advantage of the electronic igni- 
tion system over conventional ignition sys- 
tems is that 

(A) the electronic ignition system is less 
expensive to repair. 

(B) the electronic ignition system requires 
a lower voltage to provide a higher 
voltage for spark 

(C) the electronic ignition system allows 
for use of a lower octane fuel. 

(D) all of the above 

4. The primary purpose of piston rings is to 

(A) seal the combustion chamber and 
allow the pistons to move freely. 

(B) connect the piston to the crankshaft. 

(C) allow fuel to enter the piston cylinder. 

(D) provide lubrication to the piston cylinder. 

5. The crankshaft typically connects to a 

(A) flywheel. 

(B) fuel pump. 

(C) muffler. 

(D) battery. 



6. What component allows the left and right 
wheels to turn at different speeds when 
cornering? 

(A) differential 

(B) camshaft 

(C) valve rotator 

(D) battery 

7. If a car's ignition system, lights, and radio 
don't work, the part that's probably mal- 
functioned is the 

(A) cylinder block. 

(B) water pump. 

(C) carburetor. 

(D) battery. 

8. A good tool to cut intricate shapes in wood 
would be a 

(A) ripsaw. 

(B) hacksaw. 

(C) coping saw. 

(D) pocket knife. 

9. A two-stroke engine will normally be 
found on 

(A) small cars. 

(B) large diesel trucks. 

(C) trucks, vans, and some cars. 

(D) snowmobiles, chainsaws, and some 
motorcycles. 

10. A belt sander would best be used to 

(A) cut wood. 

(B) finish wood. 

(C) shape wood. 

(D) keep your pants up. 



Go on to next page 



212 



Part V: Practice ASVAB Exams 



11. A car equipped with limited-slip differential 

(A) can be readily put into all-wheel (four- 
wheel) drive. 

(B) won't lock up when the brakes are 
applied steadily. 

(C) transfers the most driving force to the 
wheel with the greatest amount of 
traction. 

(D) is rated for off-road driving. 

12. Big block engines generally have 

(A) more than 5.9 L of displacement. 

(B) better gas mileage than small block 
engines. 

(C) less than 6 L of displacement. 

(D) air conditioning. 

13. A good tool for spreading and/or shaping 
mortar would be a 

(A) cement shaper. 

(B) hammer. 

(C) trowel. 

(D) broom. 

14. Plumb-bobs are used to 

(A) clean pipes. 

(B) check vertical reference. 

(C) fix the toilet. 

(D) carve stones. 

15. Rebar is used to 

(A) measure the depth of concrete. 

(B) reinforce concrete. 

(C) stir concrete. 

(D) smooth concrete. 

16. Annular ring, clout, and spring head are 
types of 

(A) hammers. 

(B) saws. 

(C) nails. 

(D) screwdrivers. 



17. A ripsaw cuts 

(A) against the grain of the wood. 

(B) with the grain of the wood. 

(C) most materials, including metal. 

(D) only plastic. 

18. A cam belt is also known as a 

(A) piston. 

(B) timing belt. 

(C) transmission belt. 

(D) lug nut. 

19. To check for horizontal trueness, the best 
tool to use is a 

(A) steel tape rule. 

(B) plumb bob. 

(C) level. 

(D) sliding T-bevel. 

20. A bucking bar is used to 

(A) pull nails. 

(B) pry wood apart. 

(C) form rivet bucktails. 

(D) drive screws. 

21. Washers that have teeth all around the cir- 
cumference to prevent them from slipping 
are called 

(A) shake-proof washers. 

(B) jaw washers. 

(C) flat washers. 

(D) split-lock washers. 



Go on to next page 



Chapter 16: Practice Exam 1 213 



22. The tool below measures 



24. The tool below is used to 




(A) an inside curve. 

(B) an outside curve. 

(C) the depth of a hole. 

(D) the thickness of wire. 

23. The object below is a type of 




(A) finish concrete. 

(B) spread joint compound. 

(C) smooth wallpaper. 

(D) dress wood. 

25. The chisel used to cut metal is 





(D) C ^^<^ 



(A) nut. 

(B) washer. 

(C) screw. 

(D) bolt. 



STOPI 



DO NOTTURNTHE PAGE UNTILTOLDTO DO SO. 
DO NOT RETURN TO A PREVIOUS TEST. 



2U 



Part V: Practice ASVAB Exams 



Subtest 8: Mechanical Comprehension 



Time: 19 minutes for 25 questions 

Directions: This test is about mechanical principles. Many of the questions use drawings to illus- 
trate specific principles. Choose the correct answer and mark the corresponding space on the 
answer sheet. 



1. An induction clutch works by 

(A) magnetism. 

(B) pneumatics. 

(C) hydraulics. 

(D) friction. 

2. If a first-class lever with a resistance arm 
measuring 2 feet and an effort arm measur- 
ing 8 feet are being used, what's the 
mechanical advantage? 

(A) 2 

(B)4 

(C)6 

(D)l 

3. The bottoms of four boxes are shown 
below. The boxes all have the same 
volume. If postal regulations state that the 
sides of a box must meet a minimum 
height, which box is most likely to be too 
short to go through the mail? 



10" 



No. 1 



No. 2 



No. 3 

(A) No. 4 

(B) No. 2 

(C) No. 1 

(D) No. 3 



No. 4 



Looking at the figure below, when Anvil B 
lands on the seesaw, Anvil A will 

Anvil B 




Anvil A 



(A) remain stationary. 

(B) hit the ground hard. 

(C) rise in the air quickly. 

(D) enter the stratosphere. 

Air pressure at sea level is about 15 psi. 
What's the amount of force exerted on the 
top of your head, given a surface area of 
24 square inches? 

(A) 360 pounds 

(B) 625 pounds 

(C) -| pound 

(D) 180 pounds 

The force produced when a boxer's hand 
hits a heavy bag and "bounces" off it is 
called 

(A) response time. 

(B) bounce. 

(C) recoil. 

(D) gravity. 



Go on to next page 



Chapter 16: Practice Exam 1 213 



7. 



10. 



In the figure below, if Gear 1 has 25 teeth 
and Gear 2 has 15 teeth, how many revolu- 
tions does Gear 2 make for every 10 revolu- 
tions Gear 1 makes? 



11. When Cam A completes one revolution, the 
lever will touch the contact point 




(A) about 16| 
(B)12 

(C) about ^ more 

(D) about 20 

A cubic foot of water weighs about 
62.5 pounds. If an aquarium is 18 feet long, 
10 feet deep, and 12 feet wide, what's the 
approximate pressure in pounds per square 
inch (psi) on the bottom of the tank? 

(A) 2 psi 

(B) 4 psi 

(C) 5 psi 

(D) 7 psi 

Springs used in machines are usually 
made of 

(A) plastic. 

(B) bronze. 

(C) nylon fiber. 

(D) steel. 

A clutch is a type of 

(A) universal joint. 

(B) coupling. 

(C) gear differential. 

(D) cam follower. 



Contact point 




Lever arm B 

(A) once. 

(B) never. 

(C) four times. 

(D) twice. 

12. A single block-and-fall is called a 

(A) fixed pulley. 

(B) gun tackle. 

(C) runner. 

(D) sheave. 

13. In the figure below, if the fulcrum support- 
ing the lever is moved closer to the anvil, 
the anvil will be 




fulcrum 

(A) easier to lift and will move higher. 

(B) harder to lift but will move higher. 

(C) easier to lift but will not move as high. 

(D) harder to lift and will not move as high. 



Go on to next page 



216 



Part V: Practice ASVAB Exams 



14. The mechanical advantage of the block- 
and-tackle arrangement shown below is 



15. 



Upper 
Block 



Lower 
Block 



'load 



(A) 2 
(B)4 
(C)6 
(D)l 

In the figure below, if the cogs move up the 
track at the same rate of speed, Cog A will 




(A) reach the top at the same time as 
Cog B. 

(B) reach the top after Cog B. 

(C) reach the top before Cog B. 

(D) have greater difficulty staying on track. 



16. If a house key, a wooden spoon, a plastic 
hanger, and a wool jacket are all the same 
temperature. On a cool day, which one 
feels the coldest? 

(A) key 

(B) spoon 

(C) hanger 

(D) jacket 

17. In the figure below, assume the valves are 
all closed. To fill the tank but prevent it 
from filling entirely, which valves should be 
open? 



Water 
Supply 



Valve 1 



Valve 3 



Valve 2 



Drainage 



Valve 4 



Valve 5 



(A) 1 and 2 only 

(B) 1,2, and 3 only 

(C) 1,2, and 4 only 

(D) 1, 2, 3, and 5 only 

18. If Gear A is turned to the left, 




(A) Gear B turns to the right and Gear C 
turns to the left. 

(B) Gear B turns to the left and Gear C 
turns to the left. 

(C) Gear B turns to the right and Gear C 
turns to the right. 

(D) Gear B turns to the left and Gear C 
turns to the right. 



Go on to next page 



Chapter 16: Practice Exam 1 217 



19. If Gear 1 moves in a clockwise direction, 
which other gears also turn clockwise? 




20. 



(A) 3 and 5 

(B) 3, 4, and 5 
(C) 2 and 5 
(D) 3 and 4 

The pressure gauge in the figure below 
shows a reading of 



21. A way to determine the amount of power 
being used is to 

(A) multiply the amount of work done by 
the time it takes. 

(B) multiply the distance covered by the 
time it takes to move a load. 

(C) divide the amount of work done by 
550 pounds per second. 

(D) divide the amount of work done by the 
amount of time it takes. 

22. A wood tool, a silver tool, and a steel tool 
are placed in boiling water for cleaning. 
Which tool will get the hot the fastest? 

(A) steel 

(B) wood 

(C) silver 

(D) All three are equally hot. 

23. A runner is being used in the figure shown. 
How much effort is the boy who's lifting the 
50-pound anvil using? Disregard friction, 
wind resistance, and the weight of the 
pulley and the rope. 



(A) 15.0 

(B) 19.5 

(C) 21.0 

(D) 23.0 




50 pounds 



(A) 50-pound effort 

(B) 100-pound effort 

(C) 25-pound effort 

(D) 10-pound effort 



Go on to next page 



2 J 8 Part V: Practice ASVAB Exams 



24. In the figure below, at what point was the 
ball traveling most slowly? 



25. In the figure below, which angle is braced 
most solidly? 




B C 




Brace 




Brace 



(A) A 
(B)B 
(C)C 
(D) All are braced equally solidly. 



STOPI 



DO NOTTURNTHE PAGE UNTILTOLDTO DO SO. 
DO NOT RETURN TO A PREVIOUS TEST. 



Chapter 16: Practice Exam 1 £ly 



Subtest 9: Assembling Objects 



Time: 15 minutes for 25 questions 

Directions: The Assembling Objects subtest consists of questions that measure your ability to men- 
tally picture items in two dimensions. Each question is comprised of five separate drawings. The 
problem is presented in the first drawing, and the remaining four drawings are possible solutions. 
Determine which of the choices best solves the problem shown in the first picture and then mark 
the corresponding choice on your answer sheet. 



|Sa a 


\® 


"^ 


^1 


K 



L it! B 


^ 


4> 


ob 


s 





-X 


X 


^ 


^P 



10. 



^ 






ii. rxK: 





4. 




5. 



A 








12. 








ABCD ABCD 

13. 



p. ! 


^7 


^ 


^ 


^ 



6. 



n 



14. 



v ol 


X 


w 


& 





7. e? A s 


A 


B C 


D 


15- ^A i 

T^ B 


ABCD 



16. 




Go on to next page 



220 Part V: Practice ASVAB Exams 



17. 



£3 




22. 



\3 A A 



t^ 



is. n^A * 



F A 



23. 



O^ 




19. 





24 "• i<§u<n. <§u 










20. 



Ai) * 

G 1 


03 


§ 


^O 


*b 



25. 



rJ 







21. 







STOPI 



DO NOT TURN THE PAGE UNTIL TOLD TO DO SO. 
DO NOT RETURN TO A PREVIOUS TEST. 



Chapter 17 

Practice Exam 1: 
Answers and Explanations 



MM/ ith the first practice test out of the way, you're probably anxious to see how well 
▼ ▼ you did. Use the answer keys in this chapter to score yourself on each of the nine 
subtests. Remember, your scores on this practice exam don't equate to scores on the actual 
ASVAB. That's because on the enlistment ASVAB, you get more points for answering harder 
questions correctly than you do for easier questions. The test is scored by comparing your 
raw score to the scores of other people, which produces a scaled score, so missing 20 out 
of 225 questions doesn't mean that your score is 205. The practice exam, however, is a valu- 
able tool for determining which subject areas you need to brush up on. (Turn to Chapter 1 
to find out how the ASVAB is scored.) 



Subtest 1: General Science Answers 




The General Science subtest tests your knowledge of science facts. If you missed a few ques- 
tions, reread the questions and try to figure out where you went wrong. If you missed more 
than a few questions, review Chapter 10. 

General Science is a broad field, but some of the following books may help you: Chemistry 
For Dummies by John T. Moore, Biology For Dummies by Donna Rae Siegfried, Astronomy For 
Dummies, 2nd Edition, by Stephen P. Maran, Weather For Dummies by John D. Cox, and 
Physics I For Dummies by Steven Holzner. You can find additional practice questions in 
Chapter 15. 

l.D 4. A 7. D 10. C 13. D 16. C 19. B 22. D 25. B 

2. C 5. D 8. C 11. C 14. B 17. A 20. B 23. A 

3. C 6. D 9. C 12. D 15. A 18. D 21. A 24. B 



Subtest 2: Arithmetic Reasoning Answers 

Arithmetic Reasoning is one of the four ASVAB subtests that make up your Armed Force 
Qualifying Test (AFQT) score, which determines whether you qualify to join the service 
branch of your choice (check Appendix A to see whether the jobs you're interested in 
require a score in this subtest). If you missed more than five or six questions, dig out 
that old high school math textbook and wrap your brain around some math problems. 
Chapters 7 and 8 may also help you out. 



222 Part V: Practice ASVAB Exams 




Some books that may help you score better on the Arithmetic Reasoning subtest include 
Basic Math & Pre-Algebra For Dummies by Mark Zegarelli, Algebra For Dummies and Algebra II 
For Dummies by Mary Jane Sterling, and Geometry For Dummies by Mark Ryan, as well as the 
related workbooks. You can also check out Mark Zegarelli's SAT Math For Dummies and Scott 
and Lisa Hatch's SAT II Math For Dummies for some math test practice. Also see Chapter 9 for 
additional practice questions. 

1. C. Multiply 12 miles by $3.50 per mile: 12 x $3.50 = $42.00. 

2. A. Let x equal the smaller number and x + 8 equal the larger number. Because the sum of 
the two numbers is 70, you can express this mathematically as x + x + 8 = 70. Now all you 
have to do is solve for x. Combine the like terms: 2x + 8 = 70. Then subtract 8 from both 
sides of the equation: 2x + 8 - 8 = 70 - 8, or 2x = 62. Divide both sides of the equation by 2, 
and you find that x is equal to 31. 

3. D. 144 bottles are in a gross, and 144 + 3 (bottles per day) = 48 days. 

4. B. To determine Jenny's average, add the test scores and divide the sum by the number of 
tests she took. You want to know what she needs on the next test to achieve an average of 
95, so let x equal the unknown score. Set up the equation as (93 + 89 + 96 + 98 + x) + 5 = 95. 
Combine the like terms: (376 + x) ± 5 = 95. Multiplying both sides by 5 results in 376 + x = 475, 
so x = 99. 

Choice (A) is very close to the correct answer, but it isn't the best answer. If Jenny's next 
test score is 100, her average would be raised to 95.2. 

5. B. Multiply the total amount spent on drinks, $375, by 12% (or 0.12) to determine the 
amount of tips: $375 x 0.12 = $45. 

6. D. You determine square footage by multiplying length by width: 12x12= 144. 

7. C. First determine the number of square feet of carpet you want to protect: 16 ft. x 18 ft. = 288 ft. 2 
The carpet stain protector is priced by the square yard, so divide 288 by 9 to convert 
square feet to square yards (because 1 yd. 2 = 3 ft. x 3 ft. = 9 ft. 2 ): 288 ft. 2 * 9 ft. 2 /yd. = 32 yd. 2 

Multiply the number of square yards by the cost of protection per square yard, $0.65, to 
get the correct answer: 32 yd. 2 x $0.65/yd. 2 = $20.80 

8. C. Let x equal the number of cards printed and sold each month. Each card costs $0.18 to 
print and sells for $0.30. Therefore, the cost is equal to 6,000 + 0.18x, and revenue is equal 
to 0.30x. You're looking for the point where revenue is greater than the cost (revenue > 
cost). The inequality is 0.30x > 6,000 + 0.18x. 

Now solve for x. Subtract 0.1 8x from both sides of the inequality and then divide both 
sides by 0.12: 

0.12x> 6,000 

x > 50,000 

The printing plant would have to print and sell at least 50,000 cards per month to make a 
profit. 

9. C. Joe gets a 1% raise. To calculate the new wage, start off by multiplying $8.15 x 0.07 = 
$0.57. Then add that number (the amount of Joe's raise) to his original hourly wage. Joe's 
new hourly wage is $8.15 + $0.57 = $8.72. 

10. C. To find distance, you multiply speed by time. First find how far Alice travels before Dave 
catches up with her. By the time Dave leaves, Alice has already been traveling for half an 

hour. Three hours later, she would've been traveling for 3-^ hours at 45 mph, or 157.5 miles: 

3.5 hr. x 45 mph = 157.5 mi. Dave has three hours to cover this distance. Now find his speed. 
To travel 157.5 miles in 3 hours, Dave would have to travel at 52.5 mph: 157.5 mi. * 3 hr. = 
52.5 mph. 



Chapter 17: Practice Exam 1: Answers and Explanations 223 



11. D. To find the amount of pie purchased (which, by the way, does not give you the final 
answer), you have to add the fractions. But first the fractions need to have a common 
denominator. The denominators (4, 3, and 6), all divide evenly into 12, so use 12 as the 
common denominator. 

To convert the fractions to the least common denominator of 12, do the following: 

Iv3__3_ 
4 3" 12 

I X 4_A 
3 4" 12 

I x 2__6_ 
6 2 "12 

Now you can add the fractions together: 

3.4,2 _ 3 + 4 + 2 _ 9 
12 + 12 + 12~ 12 "12 

Nine-twelfths of 20 pies is the same thing as ^, or 75%, of the 20 pies. That equals 15 pies. 

But that's not what the question asks. One more step: Subtract the pies sold (15) from the 
original 20, leaving 5 pies, which makes Choice (D) correct. 

12. A. Subtract the sale price from the regular price to find how much she saves on each case: 
$24.00 - $22.50 = $1.50. Multiply the answer by the total number of cases to get your final 
answer: $1.50 x 5 = $7.50. 

13. C. Divide 30 by 8 to determine how long the security guard takes to walk one city block: 
30 * 8 = 3.75 minutes. Then multiply 3.75 by 6, the number of blocks it takes to complete 
the circuit. The answer is 22.50 minutes. 

14. B. Let x equal the original number of how much Grand Island would grow. An additional -^ 

6 
would make the population growth ^, or 120%, of x. You can express the equation as 

D 

\.2x = 600,000. To solve for x, divide both sides of the equation by 1.2, which gives you 
x = 500,000. 

15. D. Divide the number of questions she has to get right (40) by the total number of ques- 

9 
tions (60) to reach 66^%. 

16. C. The interest formula says that interest equals principal times rate times time, or / = Prt. 
To determine the amount of interest earned, multiply the principal ($3,000) by the interest 
rate (6%) and the number of years interest accrues (1 year): $3,000 x 0.06 x 1 = $180. Add 
the interest earned to the principal to show how much total money the teacher would 
have: $180 + $3,000 = $3,180. 

17. B. Recognize that if the track is a quarter mile long, then 1 mile equals four laps. Therefore, 
multiply 4 times 3.5 miles; the answer is 14 laps. 

5 5 

18. A. One kilometer is approximately % of 1 mile, so you can multiply 75 x -5-: 75x5 =375, and 

o 

375 * 8 equals about 46.8. Therefore, Karl was traveling at 47 miles per hour. 

19. A. You need to add the carpenter's base pay and overtime pay to find his total pay for the 
week. First find his base pay per week: $12.30/hr. x 40 hr. = $492. Then find his overtime 

rate per hour, which is 1-4 times his base pay: $12.30/hr. x 1.5 = $18.45. Multiply this rate by 

the number of hours of overtime to find his overtime pay: $18.45/hr. x 6 hr. = $110.70. 
Finally, add his base pay and overtime pay to find his total pay for the week: $492.00 + 
$110.70 = $602.70. 

20. A. The office has 1,260 square feet of space (multiply 42 square feet by 30 employees). 
With 35 employees, each employee will have 36 square feet of work space (1,260 * 35), 
which is 6 square feet less than originally. 



221) 



Part V: Practice ASVAB Exams 



21. B. The total cost is the down payment plus 5 years' worth of monthly payments. Five years 
contain 60 months, so multiply $450 (monthly payment) x 60 = $27,000 (total payments). 
Then add $27,000 (total payments) + $2,000 (down payment) = $29,000 (total cost). 

22. D. Add the three monthly amounts to determine the total amount Darla spent on groceries: 
$120.37 + $108.45 + $114.86 = $343.68. Divide the total by 3 to determine the average 
monthly cost: $114.56. 

23. C. Distance equals speed times time, so divide the total distance by Keith's average speed 
to find how long the trip took: 1,650 mi. * 50 mph = 33 hr. 

24. B. Choice (A) doesn't provide enough paint (2 x 25 gal. = 50 gal.), so it's wrong. Now deter- 
mine the cost of each of the other options: 

Choice (B): 11 x $108 = $1,188 

Choice (C): 6 x $215 = $1,290 

Choice (D): 55 x $23 = $1,265 

The lowest price is $1,188, Choice (B). 

25. B. First find how many ounces of rations each truck can hold. One ton is 2,000 pounds, so 
one truck can carry three times that, or 6,000 pounds. There are 16 ounces in a pound, so 
one truck can carry 96,000 ounces: 6,000 lbs. x 16 oz. = 96,000 oz. 

Then figure out how many daily rations are in a truckload. The total daily ration for each 
resident is 12 ounces + 18 ounces + 18 ounces, or 48 ounces. You can express the number 
of daily rations supplied as 96,000 oz. + 48 oz./daily ration = 2,000 daily rations. These 
rations need to last 10 days. Dividing 2,000 by 10 days results in 200 residents who can be 
fed by one truck during this 10-day period. 

26. D. The train headed for Wichita traveled 55 miles/hour x 3 hours =165 total miles. The 
train headed for Des Moines traveled 70 miles/hour x 3 hours = 210 total miles. Adding the 
distances together gives you the number of miles apart the two trains are after three 
hours: 210 + 165 = 375. Another option: You can add the two rates of speed (55 + 70) and 
multiply the sum by 3 hours (125 x 3 hours = 375). 

27. A. Convert the mixed number to inches: 3 feet, 8 inches equals 44 inches (12 in. /ft. x 3 ft. = 
36 in., and 36 in. + 8 in. = 44 in.). Forty-four inches (length each section needs to be) x 4 
(number of sections needed) = 176 inches (total molding needed). To find the amount of 
molding needed in feet, convert 176 inches into feet by dividing 176 inches by 12 inches. 

You get 14-o feet, so the shortest board length necessary is 15 feet. 

28. A. One turkey breast costs $8.50 minus 10% of $8.50 (which is $0.85), or $8.50 - $0.85 = 
$7.65. The other turkey breast is full price, so add the two costs: $7.65 + $8.50 = $16.15. 

29. C. Don't let the number of miles traveled confuse you. You don't use them to solve the 
problem. Finding # of a 40-hour work week is the same thing as multiplying 40 times 2, 
which is 80, and then dividing 80 by 5, which equals 16 hours the recruiter travels weekly. 

30. B. Your first step is to determine the number of miles traveled. Multiply the rate of travel 
by the time: 48 x 7 = 336 mi. The amount of gas used is the total miles driven divided by 
the number of miles per gallon: 336 + 21 = 16 gal. used. At the price of $2.82 per gallon, you 
spent $45.12 for gas: $2.82 x 16 = $45.12. 

Subtest 3: Word Knowledge Answers 

The Word Knowledge subtest is nothing more than a vocabulary test. However, it's very 
important because it's another one of the four subtests used to make up your AFQT score. 
If you find you need to improve your vocabulary, see Chapter 4. 



Chapter 17: Practice Exam 1: Answers and Explanations 22.2 




A couple of other great study references are Vocabulary For Dummies by Laurie E. Rozakis 
(Wiley) and SAT Vocabulary For Dummies by Suzee Vlk (Wiley). Additionally, see Chapter 6 
for more practice questions. 



l.A 


6. C 


11. B 


16. A 


21. D 


26. D 


31. C 


2. A 


7. A 


12. D 


17. A 


22. A 


27. B 


32. A 


3. B 


8. D 


13. B 


18. C 


23. C 


28. B 


33. C 


4. C 


9. D 


14. B 


19. A 


24. D 


29. C 


34. D 


5.C 


10. B 


15. B 


20. D 


25. B 


30. B 


35. A 



Subtest %: Paragraph Comprehension Answers 

Like Word Knowledge, your Paragraph Comprehension score goes toward your AFQT score, 
so pay special attention if you've missed more than a couple of these answers — you need 
some study time (see Chapter 5). Remember that rereading the paragraph several times to 
make sure you have the right answer is perfectly fine. You can find additional practice ques- 
tions in Chapter 6. 

1. C. Effective appointment management eliminates the waste of your time, as the last sen- 
tence of the passage explains. 

2. D. The passage gives the numbers of senators and representatives, so Choice (A) is incor- 
rect. The passage states that each state's population determines the number of represen- 
tatives a state has, so Choice (B) is incorrect. As the passage states, each state has at least 
one representative, so Choice (C) is incorrect. 

3. A. Many languages are excluded from the Indo-European language group, so Choice (B) is 
incorrect. Indians, Iranians, Asians, and Afghans aren't Europeans, so Choice (C) is incor- 
rect. The passage gives no evidence to support Choice (D), which isn't true. 

4. D. Privatized functions operate independently of the government, making Choices (A) and 
(B) incorrect. The passage states that privatized functions may sell goods and services to 
other customers as well as the government, so Choice (C) is also incorrect. Choice (D) is 
the correct answer, because privatized functions do perform essential services to govern- 
ment agencies. 

5. C. Choice (A) — "Lead by Example" — is a good philosophy but isn't pertinent to the main 
point of the passage. Choices (B) and (D) are subpoints, which support the main point of 
the passage, which is how to lead a successful conference (C). 

6. A. You can assume that causing rain or snow would end a drought, Choice (A). Nothing in 
the passage has to do with expense, so Choice (B) is incorrect. The passage says nothing 
about how frequently the process is used, so Choice (C) is incorrect. The passage specifies 
that dry ice (solid carbon dioxide) is used; regular ice (solid water) is a different sub- 
stance, so Choice (D) is wrong. 

7. C. Choices (A) and (B) may be true in certain situations, but they're not the point of this 
particular paragraph. The passage doesn't say anything about working to improve writing 
skills being a waste of time, so Choice (D) is incorrect. The main point of the paragraph is 
that writing may not be the most efficient way of communicating, depending on the situation. 

8. B. Products with transistors weren't widely sold before 1954 because of the expense and 
difficulty of production, not because markets didn't exist, so Choice (A) is incorrect. 
Choice (C) has the right date, but the passage doesn't say who invented the transistor, 



226 



Part V: Practice ASVAB Exams 



so it's wrong as well. Choice (D) is wrong because the passage states that the problem of 
transistors' being expensive to produce was solved by 1954. The last sentence notes that 
the first transistor radio went on the market after cost and production problems were 
overcome, so (B) is the right answer. 

9. B. The words London and Englishman make it clear that the author is speaking of his trav- 
els in England, which is part of Great Britain. 

10. A. The author makes no reference to parents in the passage, so Choice (B) is incorrect. 
The author doesn't imply anything about all these interests requiring equal time, so 
Choice (C) is incorrect. The passage is about children under 18; you can't draw a conclu- 
sion about what the author thinks people over 18 should do, so Choice (D) is incorrect. 

11. C. The passage doesn't say anything about mock job interviews being frightening, so 
Choice (A) is wrong. The passage says that mock job interviews should be available to all 
students, so Choice (B) is wrong. The passage says that schools, not students, should 
organize mock interviews, so Choice (D) is incorrect. 

12. A. Choices (B), (C), and (D) are the opposite of what the paragraph states and implies. 

13. B. Nothing in the paragraph supports Choice (A), which is incorrect. When an amendment 
is passed, it becomes part of the Constitution, so Choice (C) is incorrect. The passage 
doesn't support Choice (D), because the passage doesn't mention anything related to lot- 
tery tickets. The passage defines due process as " the guarantee of fairness in the adminis- 
tration of justice," so (B) is correct. 

14. C. Because the 14th Amendment guarantees due process in states' laws, the 5th Amendment 
must guarantee due process only in federal law, which makes Choice (C) right. Nothing in 
the passage implies that the 5th Amendment is about taxes, so Choice (A) is wrong. Because 
the passage states that the 14th Amendment had to be enacted to require states to abide by 
due process, Choice (B) is incorrect. Choice (D) is neither stated nor implied in the passage. 

15. A. Because the Supreme Court struck down many state laws after the 14th Amendment 
was enacted, it's probably true that these laws would still exist if there'd been no 14th 
Amendment. The passage doesn't support Choices (B), (C), or (D). 

Subtest 5: Mathematics Knortiedqe Answers 

This subtest is also used to calculate your AFQT score, so it's important. If you miss more 
than four or five, consider brushing up on your basic math skills. Chapter 8 can help with this. 

The following books may also be of some help: Algebra For Dummies and Algebra II For 
Dummies by Mary Jane Sterling, Geometry For Dummies and Calculus For Dummies by 
Mark Ryan, and SAT II Math For Dummies by Scott Hatch (all books published by Wiley). 
Chapter 9 also has some additional practice questions. 

1. A. Substitute 8 for x in the equation and then solve fory: 

y = (x 2 + 4) - 2 
= (8 2 * 4) - 2 
= (64 * 4) - 2 
= 16-2 
= 14 

2. A. The cube of 5 is 5 3 , which is 5 x 5 x 5 = 125. 




Chapter 17: Practice Exam 1: Answers and Explanations 22 7 



3. C. Because of the order of operations, you need to find 3 3 first and then multiply by 2.5: 

2.5 x 3 3 

= 2.5(3 x 3 x 3) 
= 2.5x27 
= 67.5 

4. D. Because 2 4 = 16, the fourth root of 16 is 2. 

5. A. To get the equation of the line, you need to know the line's slope and y-intercept. The 
slope of the line is equal to the change in y values divided by the change in x values. The 

change in y values is 3 - (-1) = 4. The change in x values is 2 - = 2. Thus, the slope is % = 2. 
The line passes through the point (0, 1), so to find the intercept, substitute for x and 
-1 for y in the equation y = 2x + b: 

-1 = 2(0) + b 

b = -\ 

Therefore, b = -1, so the full equation is y = 2x- 1. 

6. C. Do what's in parentheses first. You need consistent units of measurement, so convert 
12 yards to feet; then add 14 feet: 

(12 yd. x 3 ft./yd.) + 14 ft. 

= 36 feet + 14 feet 

= 50 feet 

The original problem asks for (12 yards + 14 feet) * 5, so divide by 5 as instructed: 
50 feet -5 = 10 feet. 

7. D. If two powers have the same base, you multiply them by keeping the base the same and 
adding the powers together: x 3 ■ x 4 = x 3 + 4 = x 1 , 

8. D. To find (x + A)(x + 2), you need to multiply every term in the first set of parentheses by 
every term in the second set and then add the results. The acronym FOIL (First, Outside, 
Inside, Last) can help you keep track of which terms you're multiplying: 

v* First: Multiply the first variable in the first set of parentheses by the first variable in the 
second set of parentheses: x(x) = x 2 . 

V Outside: Next, multiply the first variable in the first set of parentheses by the second 
number in the second set of parentheses: x(2) = 2x. So far, the results are x 2 + 2x. 

v* Inside: Now multiply the second number in the first set of parentheses by the first vari- 
able in the second set of parentheses: A(x) = Ax. 

v* Last: Next, multiply the second number in the first set of parentheses by the second 
number in the second set of parentheses: 4(2) = 8. 

The solution is x 2 + 2x + Ax + 8. Combining the like terms results in x 2 + 6x + 8. 

9. C. You need to do powers (exponents) first, so find 10 3 and then multiply by 1.5: 

1.5 xlO 3 

= 1.5x(10xl0x 10) 

= 1.5x1,000 

= 1,500 



228 



Part V: Practice ASVAB Exams 



10. B. A prime number is a number that can be divided evenly by itself or by 1 but not by any 
other number. Choices (A), (C), and (D) can all be divided evenly by other numbers. 

11. B. The mode of a series of numbers is the number that appears in the series the most fre- 
quently. In this case, it's 8. 

12. D. Substitute 4 for all a's in the problem and then solve, doing the powers first: 

43 + 4 

= (4 x 4 x 4) * 4 
= 64*4 
= 16 

13. C. The factorial (!) of a number is the number multiplied by the next-smallest whole 
number, then by the next smallest whole number, and so on down to 1: 

51 = 5x4x3x2x1 = 120 

14. B. Do what's in parentheses first: 

(900 x2)*6 = 1,800*6 = 300 

15. A. Substitute 2 for all x's in the problem and then solve, starting with the powers: 

2 2 (2) = 4(2) = 8 

16. D. The problem asks you to solve (5 + 1)(6 + 3)(8 - 5) = (3 + 3)x for x. Solve the first half of 
the equation, finding the values in parentheses first: 

(6)(2)(3) = 36 

Therefore, the whole equation becomes 36 = (3 + 3)x, which turns into 36 = 6x. Isolate x: 
36 * 6 = 6x * 6 
6 -x 
To check your answer, substitute 6 for x. 

17. A. The square root of 49 is 7; the square root of 64 is 8. And 7 x 8 = 56. 

18. D. Find a common denominator for the fractions. In this case, 80 works for all the fractions. 
Convert all the fractions using the following method: 

2 Y 16_32 
5 16 " 80 

3 V 10_30 
8 10 ~ 80 

X v 8_56 
10 8 " 80 

13 Y 5_65 
16 5 " 80 

Comparing the fractions, you can see that ^jy or li is the largest fraction. 

19. B. Solve as you would solve for any unknown: 

2 +x>4 
2+x-2>4-2 

Therefore, x > 2. To check your answer, substitute 2 for x: 2 + 2 > 4. That's true, so the 
answer is correct. 



Chapter 17: Practice Exam 1: Answers and Explanations ££y 



20. C. Circumference equals n x diameter, and diameter is equal to two times the radius (or 
mathematically, C =nd and d = 2r).For this problem, C = nx24.lt you round n to 3.14, the 
answer is about 75.36, or about 75 feet. 

21. A. Volume equals length x width x height (V - Iwti), so plug in the numbers and solve: 
16x8x18 = 2,304 in. 3 

22. C. In an equilateral triangle, all sides are equal and all angles are equal. 

23. A. All quadrilaterals (four-sided figures) have angles that total 360 degrees. 

24. B. If the sum of two angles equals 90 degrees, they're called complementary angles. 

25. A. 24% = T^Jou further reduce this fraction to -^ by dividing the numerator and 

iuu zo 

denominator by 4. 

Subtest 6: Electronics Information Answers 

The Electronics Information subtest is important only if you plan on a career that requires a 
solid score in this area (check Appendix A to see whether the jobs you're interested in 
require a score in this subtest). Otherwise, spend your time studying for the math- and 
word-related ASVAB subtests. If you do need to score big on this test and you missed more 
than five answers, start brushing up. Start by reviewing the corresponding chapter in this 
book (Chapter 13). 

If you need even more study, check out Electronics For Dummies by Gordon McComb 
(Wiley) or consider enrolling in a quick course at a community college. You can also find 
additional practice questions in Chapter 15. 

1. A. Ohm's law states that Voltage (E) = Current (I) x Resistance (R). All other answer are 
incorrect expressions of this law. 

2. A. You read a resistor's color bands from left to right. The first band denotes the first digit, 
the second band denotes the second digit, and the third band denotes the subsequent 
number of zeros. In this example, brown is one, black is zero, and red means there are two 
additional zeros. 

3. B. Heaters, pumps, stairs, diving boards, railings, and rebar, among other things, must be 
bonded together by a minimum #8 wire for safety purposes. 

4. D. Voltage is commonly used as a short name for electrical potential difference, and it is 
measured in volts. 

5. A. SIM stands for Subscriber Identity Module. The card contains information such as your 
phone number, your billing information, and your address book. The card makes it easier 
to switch from one cell phone to another. 

6. A. A resistor is so named because it resists (or inhibits) the flow of current. 

7. C. A diode has two terminals, the anode and the cathode, which is why it's called a diode. 
It restricts current flow to only one direction. 

8. C. Radar can operate as high as 100,000 MHz (megahertz). 

9. B. Current is the flow of charged particles. The difference between alternating current (AC) 
and direct current (DC) is that the electrons in an AC circuit regularly reverse their direc- 
tion. In a DC circuit, electrons always flow in the same direction. 

10. C. Changing AC to DC is a process called rectification. 

11. C. A British thermal unit (BTU) is a measure of heat energy. 




230 



Part V: Practice ASVAB Exams 



12. D. Current is the presence of electron flow. 

13. B. Transducers, which transform energy, can be switches, strain gauges, temperature sen- 
sors, or inductive switches. 

14. D. Active elements are electronic devices that can create energy (such as voltage supplies 
and current supplies). Passive elements are electronic devices that cannot create energy. 

15. A. Power = Current x Voltage or, written another way, Current = Power * Voltage. Plug in 
the numbers and do the math: 60 watts + 120 volts = 0.5 amperes. 

16. C. The larger the number, the smaller the diameter of the wire. 

17. C. Because fuses are designed to prevent current overload at a specific level, a fuse with a 
high rating may allow a higher current to flow through a circuit not designed to work at 
that higher current, possibly causing damage to the circuit. 

18. A. Neutral wire is always whitish or natural colored. 

19. D. Electrical power is measured in watts, so you use a wattmeter. An ammeter measures 
amps (current). An ohmmeter measures ohms (resistance). A voltmeter measures volts 
(voltage). 

20. B. The bulb will burn dimmer because its full potential isn't used; it'll last longer for the 
same reason. 

Subtest 7: Auto & Shop In formation Answers 

The Auto & Shop Information subtest is fairly straightforward. You either know the informa- 
tion or you don't. Not knowing the info may not matter to you as long as the career you 
want doesn't require a subtest score in this area (check the Appendix to see whether the 
jobs you're interested in require a score in this subtest). But if you do need to do well on 
this subtest and you've missed more than five answers, review the material in Chapter 11. 

Reviewing Auto Repair For Dummies by Deanna Sclar (Wiley) may also help you score better 
on this subtest. Home Improvement All-in-One For Dummies by Roy Barnhart, James Carey, 
Morris Carey, Gene Hamilton, Katie Hamilton, Donald R. Prestly, and Jeff Strong (Wiley) can 
help you get a better handle on basic tools and their uses. You may even want to take a 
class at a nearby community college or at least hang out at the garage and help some 
mechanics for a couple of weeks. See Chapter 15 for some more practice questions. 

l.D 4. A 7. D 10. B 13. C 16. C 19. C 22. B 25. A 

2. B 5. A 8. C 11. C 14. B 17. B 20. C 23. A 

3. B 6. A 9. D 12. A 15. B 18. B 21. A 24. D 



Subtest 8: Mechanical Comprehension Answers 

The Mechanical Comprehension subtest is important only if you want to pursue a military 
career that requires a good score on this subtest (check Appendix A to see whether the 
jobs you're interested in require a score in this subtest). Otherwise, spend your time study- 
ing more important areas of the ASVAB. If you're considering a military job that requires a 
high mechanical aptitude and you missed more than four or five questions on this subtest, 
give Chapter 12 another once over. 




Chapter 17: Practice Exam 1: Answers and Explanations 23 7 



1. A. An induction clutch is a magnetic clutch. When a conductor (wire) is wrapped around a 
core and electricity is passed through the wire, it sets up a magnetic field. The same wire 
also acts as an inductor, which produces inductance, during AC current flow. It's similar to 
resistance in a resistor in that it "resists" current flow, but the value of inductance is based 
on the value of the inductor (written as L) and the frequency of the AC current. Therefore, 
an induction clutch uses magnetism to operate. 

2. B. You can calculate mechanical advantage as Length of Effort Arm + Length of Resistance 
Arm. Simply plug in the numbers: MA = 8-^2 = 4. 

3. D. The box with the largest area on the bottom will have the shortest sides. If Length x 
Width x Height = Volume and all the boxes have equal volume, then the sides must be 
shortest on the box with the largest area on the bottom. Calculate the area of each box 
bottom: 

No. 1 = 20 square inches 

No. 2 = 35 square inches 

No. 3 = 48 square inches 

No. 4 = 27 square inches 

No. 3, which has the largest area, will have the shortest sides. 

4. C. Anvil B's landing on the seesaw will propel Anvil A into the air. 

5. A. Pressure equals force divided by area in square inches (P = F '+ A). You can also state 
this formula as F= A x P. Substitute the known quantities: F = 15 x 24 = 360 pounds. 

6. C. Recoil occurs when an object producing a force is kicked back. 

7. A. To determine the answer, multiply the number of teeth Gear 1 has (D) and the number 
of revolutions it makes (R). Divide that number by the number of teeth Gear 2 has (d) to 
determine the number of revolutions Gear 2 makes (r). Because the gears are proportional, 
this formula shows you the ratio of teeth to revolutions. 

r-DR 

r ~ d 
r _ 25x10 
15 

r _250_50_ 1fi 2 
15 " 3 -1D 3 

8. B. You can determine the pressure of all that water by multiplying the volume of the 
aquarium by the weight of the water. Volume = Iwh. The bottom of the tank is 18 feet long 
by 12 feet wide by 10 feet high for a total volume of 2,160 cubic feet: 18 x 12 x 10= 2,160 ft. 3 
A cubic foot of water weighs approximately 62.5 pounds, so multiply the volume of water 
by 62.5: 2,160x62.5= 135,000. 

That gives an approximate pressure on the bottom of the tank of about 135,000 pounds 
over the entire surface area. The surface area of the bottom of the tank is length x width. 
Convert feet to inches and then find the area: A = (18 ft. x 12 in./ft.) x (12 ft. x 12 in./ft.) = 
216 in. x 144 in. = 31,104 in. 2 . 

Dividing the pressure of 135,000 by the number of square inches of surface area gives an 
approximate psi of 4. 

9. D. Machine springs are usually made of steel, although sometimes they're made of brass or 
other metal alloys. 

10. B. Clutches connect and disconnect parts, so they're a type of coupling. 



232 



Part V: Practice ASVAB Exams 



11. D. When the high point of the cam connects with the lever arm, the lever arm will touch 
the contact point. Two high points on the cam mean the lever arm will touch the contact 
point twice with each revolution of the cam. 

12. C. A single block-and-fall is a way to get mechanical advantage by threading a rope through 
a pulley or stationary point, the load being attached to the end of the rope, and you pulling 
on the other end of the rope, hoisting the load. The device is also called a runner. 

13. C. If the fulcrum is moved closer to the anvil, the length of the effort arm of the lever will 
be increased, making the anvil easier to raise, but the height to which the anvil can be 
raised will be reduced. 

14. A. Because this block-and-tackle arrangement merely changes the direction of the pull, it 
has a mechanical advantage of only 2. 

15. C. The larger cog (Cog A) covers a greater linear distance in a given period of time, so Cog A 
reaches the top first. 

16. A. The key will feel coldest because metal is a better conductor than the other materials. 

17. D. All but Valve 4 should be open. Opening Valves 1 and 2 allows water to enter the tank. 
Opening Valves 3 and 5 prevents water from filling the tank entirely. Opening Valve 4 
allows water to leave the tank. 

18. A. Gears with their teeth together in mesh turn in opposite directions. Gear A turns Gear B 
in the opposite direction (right), and Gear B turns Gear C in the opposite direction (left). 

19. A. Gears with their teeth together in mesh turn in opposite directions. Gear 1 turns clock- 
wise. Gear 2, in mesh with Gear 1, turns counterclockwise. Gear 3, in mesh with Gear 2, 
turns clockwise. Gear 4, in mesh with Gear 3, turns counterclockwise. Gear 5, in mesh with 
Gear 2, turns clockwise. 

20. C. The gauge shows a reading of 21. 

21. D. The formula for determining power is Power = Work * Time. 

22. C. Silver is the best conductor, so it will become hotter faster than the other objects 
because heat transfers faster into materials with greater conductivity than with those with 
lower conductivity. 

23. A. Stationary pulleys give no mechanical advantage, so effort equals the weight of the 
crate, or 50 pounds. 

24. C. At the height of the arc, the ball has no upward momentum, so it goes the slowest at 
that point. 

25. A. The brace on Angle A covers more area of the angle, so it's more solidly braced. 

Subtest 9: Assembling Objects Answers 

If you plan on enlisting in the Navy, check Appendix A to see whether the jobs you're inter- 
ested in require a score in this subtest. For more information about the Assembling Objects 
subtest, see Chapter 14. For additional practice questions, see Chapter 15. 

25. C 



l.C 


4. D 


7. C 


10. B 


13. D 


16. A 


19. C 


22. D 


2. A 


5. A 


8. D 


11. A 


14. C 


17. B 


20. A 


23. B 


3D 


6. A 


9. B 


12. D 


15. B 


18. B 


21. B 


24. C 



Chapter 18 

Practice Exam 2 




i 

■ Ve designed the second practice test so you can see how much you've improved. This 
«S exam is exactly like the first one from Chapter 16, except (of course) the questions are 
different. I hope you used the results from the first practice exam to determine your weak 
areas and spent some time hitting the ol' books and recharging your thinking cap. 

To get the most out of this practice exam, take it like you'd take the real ASVAB under the 
same conditions: 

v* Allow yourself about three hours to take the entire exam, and take the whole thing at 
one time. 

j*" Find a quiet place where you won't be interrupted. 

i*" Bring a timer that you can set for various lengths of time, some scratch paper, and a 
pencil. 

i*" At the start of each subtest, set your timer for the specified period of time. Don't go on 
to the next section until the timer has gone off, and don't go back to a previous section. 
If you finish early, check your work for that section only. 

i*" Use the answer sheet that's provided. 

j*" Don't take a break during any subtest. You can take a short one- or two-minute break 
between subtests if you need it. 

After you complete the entire sample test, check your answers against the answer explana- 
tions and key in Chapter 19. 



23U 



Part V: Practice ASVAB Exams 



Answer Sheet for Practice Emm 2 

Subtest 1: General Science 

1®®©® 6®®©® 11®®©® 16®®©® 21®®©® 

2®®©® 7®®©® 12®®©® 17®®©® 22®®©® 

3®®©® 8®®©® 13®®©® 18®®©® 23®®©® 

4®®©® 9®®©® 14®®©® 19®®©® 24®®©® 

5®®©® 10®®©® 15®®©® 20®®©® 25®®©® 

Subtest 2: Arithmetic Reasoning 

1®®©® 7®®©® 13®®©® 19®®©® 25®®©® 

2®®©® 8®®©® 14®®©® 20®®©® 26®®©® 

3®®©® 9®®©® 15®®©® 21®®©® 27®®©® 

4®®©® 10®®©® 16®®©® 22®®©® 28®®©® 

5®®©® 11®®©® 17®®©® 23®®©® 29®®©® 

6®®©® 12®@©@ 18®®©® 24®®©® 30®®©® 



Subtest 3: Word Knowledge 














1 ®@©@ 8 ®@©® 


15 


®@©® 


22 


®@©® 


29 


®@©@ 


2 ®@©® 9 ®®©® 


16 


®@©@ 


23 


®®©@ 


30 


®@©@ 


3®®©® 10®®©® 


17 


®®©® 


24 


®®@® 


31 


®®©® 


4®®©® 11®@©@ 


18 


®@©@ 


25 


®®@® 


32 


®®©@ 


5®®©® 12®@©@ 


19 


®@©@ 


26 


®®©® 


33 


®®©® 


6®®©® 13®@©@ 


20 


®@©® 


27 


®®@® 


34 


®®©® 


7®®©® 14®®©® 


21 


®@©® 


28 


®®©@ 


35 


®®©@ 


Subtest 4: Paragraph Comprehension 














1 ®@©® 4 ®@©® 


7 


®@©® 


10 


®®©® 


13 


®®©® 


2 ®®©® 5 ®®©® 


8 


®®©® 


11 


®®©@ 


14 


®®©® 


3 ®@©® 6 ®@©@ 


9 


®®©@ 


12 


®®©® 


15 


®@©® 


Subtest 5: Mathematics Knowledge 














1 @®©@ 6 ®®©® 


11 


®@©® 


16 


®@©® 


21 


®@©® 


2 ®@©@ 7 ®®©® 


12 


®®©® 


17 


®®©@ 


22 


®®©@ 


3 ®®©® 8 ®®©® 


13 


®®©® 


18 


®®©@ 


23 


®®©® 


4 ®®@® 9 ®@©@ 


14 


®@©@ 


19 


®®@® 


24 


®®©@ 


5®®©® 10®®©® 


15 


®@©® 


20 


®®©® 


25 


®®©® 


Subtest 6: Electronics Information 














1 ®®©® 5 ®@©® 


9 


®@©® 


13 


®®©® 


17 


®®©® 


2 ®@©@ 6 ®®©® 


10 


®®@® 


14 


®®©@ 


18 


®@©@ 


3 ®@©® 7 ®®©® 


11 


®®©@ 


15 


®®©@ 


19 


®®©@ 


4 ®®©® 8 ®@©® 


12 


®@©® 


16 


®®©® 


20 


®®@® 


Subtest 7: Auto & Shop Information 














1 ®®©@ 6 ®@©® 


11 


®@©® 


16 


®®©® 


21 


®@©@ 


2 ®@©@ 7 ®®©® 


12 


®®@® 


17 


®®©@ 


22 


®@©@ 


3 ®@©® 8 ®®©® 


13 


®®©® 


18 


®®©® 


23 


®®©® 


4 ®®©® 9 ®@©@ 


14 


®@©® 


19 


®®©@ 


24 


®®©@ 


5®®©® 10®®©® 


15 


®®©® 


20 


®®@® 


25 


®®©® 



Subtest 8: Mechanical Comprehension 

1®®©® 6®®©® 11®®©® 16®®©® 21®®©® 

2®®©® 7®®©® 12®®©® 17®®©® 22®®©® 

3®®©® 8®®©® 13®®©® 18®®©® 23®®©® 

4®®©® 9®®©® 14®®©® 19®®©® 24®®©® 

5®®©® 10®@©@ 15®®©® 20®®©® 25®®©® 

Subtest 9: Assembling Objects 

1®@©@ 6®®©® 11®®©® 16®®©® 21®@©@ 

2®®©® 7®®©® 12®®©® 17®®©® 22®®©® 

3®®©® 8®®©® 13®®©® 18®®©® 23®®©® 

4®®©® 9®®©® 14®®©® 19®®©® 24®®©® 

5®®©® 10®@©@ 15®®©® 20®®©® 25®®©® 



Chapter 18: Practice Exam 2 235 



Subtest 1: General Science 



Time: 11 minutes for 25 questions 

Directions: This exam tests your knowledge of general science principles usually covered in high 
school classes. Pick the best answer for each question and then mark the space on your answer 
sheet that corresponds to the question number and the letter indicating your choice. 



1. What is the change in body form that an 
insect undergoes from birth to maturity? 

(A) transformation 

(B) metamorphosis 

(C) trinity 

(D) transmutation 

2. An earthquake that measures 4 on the 
Richter scale would be how many times 
stronger than an earthquake that 
measured 2? 

(A) 2 times stronger 

(B) 4 times stronger 

(C) 10 times stronger 

(D) 100 times stronger 

3. Muscles attach to bone with 

(A) nonconnective tissue. 

(B) ligaments. 

(C) tendons. 

(D) rubber bands. 

4. The male part of a flower is called 

(A) the stamen. 

(B) the pistil. 

(C) the throttle. 

(D) stubborn. 

5. Blood leaving the lungs is 

(A) hydrogenated. 

(B) coagulated. 

(C) watery. 

(D) oxygenated. 



6. Which river is the longest? 

(A) Mississippi 

(B) Nile 

(C) Colorado 

(D) Congo 

7. The branch of science that studies matter 
and energy is called 

(A) chemistry. 

(B) physics. 

(C) oceanography. 

(D) trigonometry. 

8. Which type of cloud's name comes from 
the Latin word meaning "rain"? 

(A) nimbus 

(B) cirrus 

(C) strato 

(D) alto 

9. Deoxyribonucleic acid is better known as 

(A) antacid. 

(B) carbohydrates. 

(C) triglyceride. 

(D) DNA. 

10. The instrument used to measure wind 
speed is 

(A) barometer. 

(B) anemometer. 

(C) altimeter. 

(D) fanometer. 



Go on to next page 



236 



Part V: Practice ASVAB Exams 



11. Electric charges can be 

(A) positive or negative. 

(B) positive or neutral. 

(C) negative or neutral. 

(D) neutral only. 

12. Which planet in the solar system has the 
most moons? 

(A) Neptune 

(B) Saturn 

(C) Jupiter 

(D) Uranus 

13. The law of gravitation was discovered by 

(A) Albert Einstein. 

(B) Isaac Newton. 

(C) Alexander Graham Bell. 

(D) Rod Powers. 

14. Which U.S. space program is responsible 
for putting 12 men on the moon? 

(A) Gemini 

(B) Titan 

(C) Voyager 

(D) Apollo 

15. Animals that eat both plants and animals 
are called 

(A) herbivores. 

(B) carnivores. 

(C) omnivores. 

(D) ambidextrous. 

16. Unlike most other fish, sharks have no 

(A) gills. 

(B) bones. 

(C) liver. 

(D) heart. 



17. What human organ is responsible for 
detoxification of red blood cells? 

(A) liver 

(B) kidneys 

(C) intestines 

(D) stomach 

18. Kinetic energy is the energy that 

(A) is produced by sound waves. 

(B) an object potentially has. 

(C) is possessed by a moving object. 

(D) results from the attraction of two 
magnets. 

19. The terrestrial planets consist of 

(A) Jupiter, Saturn, Uranus, and Neptune. 

(B) Pluto and Neptune. 

(C) Mercury, Venus, Earth, and Mars. 

(D) any planet. 

20. A step-up transformer 

(A) increases the voltage in a power line. 

(B) decreases the voltage in a power line. 

(C) doesn't affect the voltage in a power 
line. 

(D) measures the voltage in a power line. 

21. Which animal has the heaviest brain? 

(A) human 

(B) elephant 

(C) rhinoceros 

(D) sperm whale 

22. The sun is what type of star? 
(A) type 

(B)Gtype 
(C)Ftype 
(D) M type 



Go on to next page 



Chapter 18: Practice Exam 2 2? 7 



23. Molecules are created when 

(A) matter is created. 

(B) matter is destroyed. 

(C) atoms combine together. 

(D) atoms are separated. 

24. An example of an embryonic plant would 
be a 

(A) tree. 

(B) rose. 

(C) seed. 

(D) cabbage. 



25. The vernal equinox is 

(A) the first day of winter. 

(B) near the equator. 

(C) the first day of spring. 

(D) a lunar eclipse. 



STOPI 



DO NOTTURNTHE PAGE UNTILTOLDTO DO SO. 
DO NOT RETURN TO A PREVIOUS TEST. 



238 



Part V: Practice ASVAB Exams 



Subtest 2: Arithmetic Reasoning 



Time: 36 minutes for 30 questions 

Directions: The questions in the arithmetic test are each followed by four possible answers. Decide 
which answer is correct and then mark the space on your answer sheet that has the same number 
and letter as your choice. Use scratch paper for any figuring you need to do. Calculators are not 
allowed. 



3. 



4. 



If you roll two six-sided dice, what's 
the probability of NOT rolling a five on 
either die? 



5. A half-pint of cream is what part of a 
gallon? 



(A) 
(B) 
(C) 
(D) 



j_ 

36 

1 



36 

25 
36 



Jack loaned Bob $1,500 at an annual inter- 
est rate of 1%. After one year, how much 
will Bob owe Jack? 

(A) $105 

(B) $1,500 

(C) $1,605 

(D) $1,507 

A 2-ton truck is taxed at a rate of $0.12 per 
pound. How much is the total tax bill? 

(A) $480 

(B) $240 

(C) $120 

(D) $600 

If ab = 10 and a 2 + b 2 = 30, solve for y in the 
equation y = (a + b) 2 . 

(A) 40 

(B)45 

(C) 50 

(D)55 



(A) 
(B) 
(C) 
(D) 



1 



16 



The cost of a protein bar increased from 
$2.50 to $2.80. The percent increase to the 
$2.80 rate was how much? 

(A) 16% 

(B) 10% 

(C) 15% 

(D) 12% 

An aircraft flies over Boondock Air Force 
Base at 10:20 a.m. At 10:32 a.m., the plane 
passes over Sea Side Naval Air Station, 
120 miles away. How fast is the aircraft 
traveling? 

(A) 400 mph 

(B) 500 mph 

(C) 600 mph 

(D) 700 mph 

Last year, Margot grew 50 bushels of corn 
in her backyard. This year, the yield has 
increased 8%. How many bushels of corn 
did Margot grow this year? 

(A) 56 

(B) 52 

(C) 60 
(D)54 



Go on to next page 



Chapter 18: Practice Exam 2 23^ 



9. Junior has saved money in his piggy bank 
over the winter. He wants to buy a $30 
computer game. If he has 14 one-dollar 
bills, 16 half dollars, 12 quarters, 8 dimes, 
25 nickels, and 10 pennies, how much more 
does he need to borrow from Dad to buy 
the game? 

(A) $27.15 

(B) $2.85 

(C) $2.95 

(D) $1.85 

10. Debbie receives a weekly salary of $80, plus 
a 5% commission on any sales. During the 
week, she has $800 in total sales. What's 
the ratio of her commission to her salary? 

(A) 2:1 

(B) 1:2 

(C) 3:1 

(D) 1:3 

11. How many quart cans can be filled from a 
25-gallon bucket of paint? 

(A) 50 

(B)75 

(C) 100 

(D)80 

12. If a crew of four people can paint the barn 
in three days, how long will it take a crew 
of two people? 

(A) 4 days 

(B) li days 

(C) 8 days 

(D) 6 days 

13. Brian works for five hours and is paid $24. 
Christina works for three hours and is paid 
$10.95. How much more per hour does 
Brian make than Christina? 

(A) $1.15 

(B) $1.25 

(C) $1.35 

(D) $1.37 



14. Margaret is getting married and must be 
ready by 11:15 a.m. If it's now 8:30 a.m., 
how much time does she have to get 
ready? 

(A) 1-^ hours 

(B) 2-i hours 

(C) 2 1 hours 

(D) 2^ hours 

15. An accounting-firm employee is asked to 
shred 900 documents. If he can shred docu- 
ments at a rate of 7 per minute, the number 
of documents remaining after 1-^ hours of 
shredding is 

(A) 630 

(B) 90 

(C) 270 

(D) 810 

16. A home stereo depreciates by 20% each 
year. What's the value of a stereo, pur- 
chased new for $1,200, after two years? 

(A) $768 

(B) $693 

(C) $827 

(D) $654 

17. Janet's old pickup truck can only reach a 
speed of 45 miles per hour. If she drives at 
top speed, how long will it take her to 
reach a city 135 miles away? 

(A) 3 hours 

(B) 2 hours 

(C) 4 hours 

(D) 2^ hours 

18. A blouse normally costs $18.50. How much 
money is saved if the blouse is purchased 
at a 20% discount? 

(A) $1.85 

(B) $14.80 

(C) $4.50 

(D) $3.70 



Go on to next page 



21*0 



Part V: Practice ASVAB Exams 



19. A clerk's weekly salary of $320 is increased 
to $360. The percent increase is 

(A) 10i% 

(B) 11% 

(C) \l\% 

(D) 12% 

20. Two go-carts are racing on a circular track 
with a circumference of 360 feet. Camera 
One is following Go-Cart One, and Camera 
Two is following Go-Cart Two. If the angle 
between the two cameras is 40 degrees, 
how far apart are the two go-carts? 

(A) 30 feet 

(B) 40 feet 

(C) 50 feet 

(D) 60 feet 

21. Dinner at a nice restaurant cost $35.98. If 
Joan gave the cashier $40.00, how much 
change should she get back? 

(A) $5.02 

(B) $4.02 

(C) $3.92 

(D) $1.02 

22. A balloonist circumnavigated the globe in 
13 days, 12 hours, 16 minutes, and 13 sec- 
onds. A plane circumnavigates the globe in 
4 days, 10 hours, 15 minutes, and 7 sec- 
onds. How much longer did it take for the 
balloon to go around the world? 

(A) 12 days, 7 hours, 11 minutes, and 
35 seconds 

(B) 9 days, 2 hours, 1 minute, and 
6 seconds 

(C) 8 days, 14 hours, 16 minutes, and 
6 seconds 

(D) 9 days, 7 hours, 3 minutes, and 
20 seconds 

23. Darlene bought 12 boxes of cookies for 
$48.00. What was the cost of each box of 
cookies? 

(A) $4.00 

(B) $0.48 

(C) $0.40 

(D) $4.80 



24. 



25. 



A tune-up increases a car's fuel efficiency 
by 5%. If a car averaged 20 miles per gallon 
before the tune-up, how many miles per 
gallon will it average after the tune-up? 

(A) 25 

(B) 22 



(C)20 
(D)21 



1 



A lumberjack wishes to drive a spike 
through the center of a tree with a circum- 
ference of 43.96 feet. What's the minimum 
length of the spike needed to go completely 
through the tree, passing through the 
center? 

(A) 14 feet 

(B) 15 feet 

(C) 16 feet 

(D) 17 feet 

26. A bin of hard candy holds 10-4 pounds. How 

3 
many ^-pound boxes of candy can be filled 

from the bin? 

(A) 30 boxes 

(B) 15^- boxes 

(C) 7-| boxes 

(D) 14 boxes 

27. A patio measures 12 feet by 14 feet. How 
many 8-inch-square paving stones are 
needed to pave the patio? 

(A) 21 

(B) 252 

(C) 378 

(D) 168 

28. A computer programmer is making $25,000 
per year, and 28% of her salary is withheld 
for federal and state deductions. How much 
is the computer programmer's net pay? 

(A) $20,000 

(B) $7,000 

(C) $18,750 

(D) $18,000 



Go on to next page 



Chapter 18: Practice Exam 2 %(l / 



29. Pam cuts a pie in half in a straight line. She 
then cuts a line from the center to the edge, 
creating a 55-degree angle. What's the sup- 
plement of that angle? 

(A) 55 degrees 

(B) 125 degrees 

(C) 70 degrees 

(D) 130 degrees 



30. A stack of lumber is 6 feet high. If each 

piece of lumber is 4 inches thick, how many 
pieces of lumber are in the stack? 

(A) 72 

(B)12 

(C)18 

(D)10 



STOPI 



DO NOTTURNTHE PAGE UNTILTOLDTO DO SO. 
DO NOT RETURN TO A PREVIOUS TEST. 



21*2 



Part V: Practice ASVAB Exams 



Subtest 3: Word Knowledge 



Time: 11 minutes for 35 questions 

Directions: This test's questions cover the meanings of words. Each question has an underlined 
word. You may be asked to decide which one of the four words in the choices most nearly means 
the same thing as the underlined word or which one of the four words means the opposite. If the 
underlined word is used in a sentence, decide which of the four choices most nearly means the 
same thing as the underlined word as used in the context of the sentence. Mark the corresponding 
space on your answer sheet. 



1. Abeyance most nearly means 

(A) trustworthiness. 

(B) passion. 

(C) suspension. 

(D) business. 

2. It was a sturdy table. 

(A) well-built 

(B) ugly 

(C) thick 

(D) small 

3. Bullock most nearly means 

(A) ox. 

(B) inattentive. 

(C) lazy. 

(D) panther. 

4. Brevity is the soul of wit. 

(A) beauty 

(B) intelligence 

(C) terseness 

(D) humor 

5. Paradigm most nearly means 

(A) twenty cents. 

(B) model. 

(C) heaven. 

(D) basis. 



6. He facilitated her promotion. 

(A) hindered 

(B) helped 

(C) disliked 

(D) ignored 

7. Quiescence most nearly means 

(A) kill. 

(B) preserve. 

(C) small. 

(D) quiet. 

8. The spectator enjoyed the game. 

(A) competitor 

(B) observer 

(C) referee 

(D) organizer 

9. Joy reclined against the far wall. 

(A) sat 

(B) leaned 

(C) jumped 

(D) paraded 

10. The teacher cited some examples. 

(A) memorized 

(B) finished 

(C) specified 

(D) examined 



Go on to next page 



Chapter 18: Practice Exam 2 2^3 



11. Surround most nearly means 

(A) line. 

(B) benefit. 

(C) encircle. 

(D) speaker. 

12. Illustrious most nearly means 

(A) illustrated. 

(B) famous. 

(C) foolish. 

(D) intelligent. 

13. Habitant most nearly means 

(A) invalid. 

(B) nun. 

(C) seeker. 

(D) dweller. 

14. Tim had a penchant for engaging in 
subterfuge . 

(A) religion 

(B) evasion 

(C) gambling 

(D) danger 

15. Megan found the new shoes to be ghastly . 

(A) hideous 

(B) cute 

(C) large 

(D) comfortable 

16. Rigid most nearly means 

(A) strong. 

(B) weak. 

(C) pliable. 

(D) inflexible. 

17. Billy yearned to join the fraternal 
organization. 

(A) brotherly 

(B) large 

(C) fun 

(D) special 



18. Deplore most nearly means 

(A) accept. 

(B) insult. 

(C) regret. 

(D) salute. 

19. Meager most nearly means 

(A) space. 

(B) sparse. 

(C) brief. 

(D) thirsty. 

20. Weal most nearly means 

(A) happiness. 

(B) blow. 

(C) scream. 

(D) tire. 

21. To be guileless , I think your hair looks ugly. 

(A) helpful 

(B) kind 

(C) frank 

(D) serious 

22. The customs agent confiscated the goods. 

(A) bought 

(B) noticed 

(C) seized 

(D) stole 

23. Dubious most nearly means 

(A) long. 

(B) beautiful. 

(C) articulate. 

(D) doubtful. 

24. Illusion most nearly means 

(A) mirage. 

(B) distant. 

(C) sight. 

(D) perspective. 



Go on to next page 



2M 



Part V: Practice ASVAB Exams 



25. 



26. 



27. 



28. 



29. 



30. 



Beckv developed a sudden craving for ice 
cream. 


31. 


Tom had to provide proof to the judge that 
he was not indigent. 


(A) disgust 




(A) guilty 


(B) passion 




(B) rich 


(C) hatred 




(C) poor 


(D) desire 




(D) ugly 


Enmitv most nearlv means 


32. 


Impertinent most nearlv means 


(A) enemy. 




(A) fun. 


(B) hatred. 




(B) boring. 


(C) anger. 




(C) rude. 


(D) childish. 




(D) impatient. 


Arbor most nearlv means 


33. 


Lustrous most nearlv means 


(A) native. 




(A) expensive. 


(B) tree. 




(B) lazy. 


(C) travel. 




(C) cold. 


(D) delirious. 




(D) bright. 


Thev terminated his contract. 


34. 


Pardon most nearlv means 


(A) bought 




(A) courtesy. 


(B) extended 




(B) excuse. 


(C) sold 




(C) believe. 


(D) ended 




(D) respect. 


Tim always considered Chuck to be a big 
buffoon. 

(A) clown 

(B) help 

(C) liar 

(D) pain 


35. 


Veracious most nearlv means 

(A) fast. 

(B) slow. 

(C) equal. 

(D) truthful. 


Null most nearly means 






(A) zero. 






(B) dull. 






(C) unskilled. 






(D) rapid. 







STOPI 



DO NOTTURNTHE PAGE UNTILTOLDTO DO SO. 
DO NOT RETURN TO A PREVIOUS TEST. 



Chapter 18: Practice Exam 2 2&S 



Subtest %: Paragraph Comprehension 



Time: 13 minutes for 15 questions 

Directions: This test measures your ability to understand what you read. This section includes one 
or more paragraphs of reading material followed by incomplete statements or questions. Read the 
paragraph and select the choice that best completes the statement or answers the question. 



Questions 1 and 2 are based on the following 
passage. 



There is not a single town of any size within 
a distance of forty miles, yet already the rural 
population of this county is quite large. The 
whole country, within a wide circuit north, 
south, east and west, partakes of the same gen- 
eral character; mountain ridges, half tilled, half 
wood, screening cultivated valleys, sprinkled 
with farms and hamlets, among which some 
pretty stream generally winds its way. The 
waters in our immediate neighborhood all flow 
to the southward, though only a few miles to the 
north of our village, the brooks are found run- 
ning in an opposite course, this valley lying just 
within the borders of the dividing ridge. The 
river itself, though farther south it becomes one 
of the great streams of the country, cannot boast 
of much breadth so near its source, and running 
quietly among the meadows, half screened by 
the groves and thickets, scarcely shows in the 
general view. 

1. According to this passage, 

(A) the author lives in a large city. 

(B) the author lives in the country. 

(C) the author lives on the seashore. 

(D) the author lives on Mars. 

2. According to this passage, the brooks are 
running in which direction within the 
author's neighborhood? 

(A) north 

(B) south 

(C) east 

(D) west 



The Panama Canal is a ship canal that cuts 
through the Isthmus of Panama, connecting the 
Atlantic and Pacific oceans. Although several for- 
eign companies tried to build the canal through- 
out the 19th century, none were successful. After 
the U.S. helped Panama revolt against Columbia, 
the U.S. was given rights to the land the canal 
occupied. The U.S. government finished the 
canal in 1914. 

3. According to this passage, 

(A) Panama and Columbia fought a war 
over the Panama Canal. 

(B) the U.S. was given rights to the canal 
land. 

(C) foreign companies built the canal 
before the U.S. stepped in. 

(D) Panama built the canal in 1914. 

Extreme care must be exercised to ensure 
proper handling and cleaning of soiled U.S. flags. 
A torn flag may be professionally mended, but a 
badly torn or tattered flag should be destroyed. 
When the flag is in such a condition that it's no 
longer a fitting emblem for display, destroy it in 
a dignified manner, preferably by burning. 

4. According to this passage, torn flags 
should be 

(A) mended. 

(B) burned. 

(C) destroyed. 

(D) all of the above 



Go on to next page 



246 



Part V: Practice ASVAB Exams 



Medieval guilds were similar to modern-day 
labor unions. These groups of merchants or 
craftspeople set rules regarding economic activ- 
ity in order to protect themselves. Some guilds 
held considerable economic power, but even 
small guilds protected members. Guilds also 
served a social purpose. 

5. According to this passage, guilds 

(A) had only one purpose. 

(B) had little in common with modern 
labor unions. 

(C) exploited workers. 

(D) held considerable economic power. 

After a series of well-publicized failures by 
various inventors, Orville and Wilbur Wright suc- 
ceeded in flying and controlling a heavier-than- 
air craft on December 17, 1903. The War 
Department, stung by its investment in a failed 
effort by Samuel Langley and compounded by 
the Wright's own secretiveness, initially rejected 
the brothers' overtures toward the government 
to buy the aircraft. Prevailing sentiments held 
that the immediate future still belonged to the 
balloon. In August 1908, the two brothers deliv- 
ered the first Army aircraft to the U.S. 
Government. That the U.S. government managed 
to purchase an airplane was a minor miracle. For 
more than four years after the Wright brothers' 
successful flight at Kitty Hawk, North Carolina, 
the government refused to accept the fact that 
man had flown in a heavier-than-air machine. 

6. Which of the following statements is NOT 
supported by the above passage? 

(A) The U.S. Government felt that balloons 
were more practical than airplanes. 

(B) The Wright brothers' own secretive- 
ness contributed to their problems in 
getting the government interested in 
their aircraft. 

(C) The historic flight took place on the 
East Coast. 

(D) It took more than six years for the 
Wright brothers to interest the U.S. 
Government in their airplane. 



If anyone should be inclined to overrate the 
state of our present knowledge of mental life, all 
that would be needed to force him to assume a 
modest attitude would be to remind him of the 
function of memory. No psychologic theory has 
yet been able to account for the connection 
between the fundamental phenomena of remem- 
bering and forgetting; indeed, even the complete 
analysis of that which one can actually observe 
has as yet scarcely been grasped. Today forget- 
ting has perhaps grown more puzzling than 
remembering, especially since we have learned 
from the study of dreams and pathologic states 
that even what for a long time we believed for- 
gotten may suddenly return to consciousness. 

7. The primary subject of this paragraph is 

(A) bowling. 

(B) puzzles. 

(C) memory. 

(D) government service. 

Troy weight is based on a pound of 12 
ounces and an ounce of 480 grains. Common, or 
avoirdupois, weight is based on a pound having 
16 ounces and an ounce having 437.5 grains. A 
common pound has 7,000 grains while a troy 
pound has 5,760. 

8. According to this passage, 

(A) in common weight, an ounce is less 
than 438 grains. 

(B) a troy pound and a common pound are 
the same weight. 

(C) common weight and avoirdupois 
weight are different measures. 

(D) a troy ounce equals 437.5 grains. 



Go on to next page 



Chapter 18: Practice Exam 2 %(l 7 



Good leaders get involved in their subordi- 
nates' careers. People merely obey arbitrary 
commands and orders, but they respond quickly 
and usually give extra effort for leaders who gen- 
uinely care for them. An often neglected leader- 
ship principle in today's environment of 
technology and specialization is knowing the 
workers and showing sincere interest in their 
problems, career development, and welfare. 
Leadership is reflected in the degree of effi- 
ciency, productivity, morale, and motivation 
demonstrated by subordinates. Leadership 
involvement is the key ingredient to maximizing 
worker performance. 

9. A key leadership principle that's often 
ignored is 

(A) leading by example. 

(B) showing sincere interest in the prob- 
lems of the workers. 

(C) ensuring workers have access to the 
most modern technology. 

(D) maximizing worker performance. 

Leukemia is a blood disease in which white 
blood cells in the blood or bone marrow repro- 
duce rapidly, interfering with the body's ability 
to produce red blood cells. Red blood cells are 
needed to perform vital bodily functions. 

10. According to this passage, 

(A) white blood cells perform no vital func- 
tion in the body. 

(B) no treatment for leukemia exists. 

(C) leukemia makes it hard for the body to 
produce red blood cells. 

(D) white blood cells are found only in the 
blood. 



Questions 11 and 12 are based on the following 
passage. 



Any discussion of distinctive military capa- 
bilities would be incomplete without looking at 
their relationship to the Joint Service vision of 
the future. JV 2020 guides all the Services into 
the next century with its vision of future war 
fighting. JV 2020 sets forth four overarching 
operational concepts: dominant maneuver, 
precision engagement, focused logistics, and 
full-dimensional protection. Each of these opera- 
tional concepts reinforces the others. The aggre- 
gate of these four concepts, along with their 
interaction with information superiority and 
innovation, allows joint forces to dominate the 
full range of military operations from humanitar- 
ian assistance through peace operations to the 
highest intensity conflict. 

11. According to the passage above, which 
of the following is NOT an operational 
concept? 

(A) dominant maneuver 

(B) focused logistics 

(C) high intensity conflict 

(D) precision engagement 

12. The document discussed in the above pas- 
sage is primarily about 

(A) military operations of the past. 

(B) present military operations. 

(C) military operations in the future. 

(D) training for future military operations. 



Go on to next page 



248 



Part V: Practice ASVAB Exams 



Questions 13 through 15 are based on the fol- 
lowing passage. 



Genetics is a branch of science dealing with 
heredity. The field is concerned with how genes 
operate and the way genes are transmitted to off- 
spring. Subdivisions in the field include cytoge- 
netics, which is the study of the cellular basis of 
inheritance; microbial genetics, the study of 
inheritance in microbes; molecular genetics, the 
study of the biochemical foundation of inheri- 
tance; and human genetics, the study of how 
people inherit traits that are medically and 
socially important. Genetic counselors are pri- 
marily concerned with human genetics. They 
advise couples and families on the chances of 
their offspring having specific genetic defects. 

13. In the passage above, cytogenetics is 
defined as 

(A) the study of the psychological impact 
of genetics. 

(B) the study of the cellular foundation of 
inheritance. 

(C) the study of molecular genetics. 

(D) the study of human genetics. 



14. According to the passage, genetics 

(A) concerns how genes operate and how 
they're passed along. 

(B) is a field of study populated by quacks, 
fakes, and frauds. 

(C) is a field of study only concerned with 
human genetics. 

(D) is a new field of study. 

15. According to the passage, it's reasonable to 
assume that genetic counseling 

(A) is restricted to the very rich. 

(B) is used to diagnose diseases. 

(C) can be used by parents to learn if their 
offspring are likely to inherit a disease 
one of the parents has. 

(D) can be used by parents to prevent their 
offspring from inheriting a specific 
genetic defect. 



STOP! 



DO NOTTURNTHE PAGE UNTILTOLDTO DO SO. 
DO NOT RETURN TO A PREVIOUS TEST. 



Chapter 18: Practice Exam 2 2 Ix ty 



Subtest 5: Mathematics Knowledge 



Time: 24 minutes for 25 questions 

Directions: This test is a test of your ability to solve general mathematical problems. Select the cor- 
rect answer from the choices given and then mark the corresponding space on your answer sheet. 
Use scratch paper to do any figuring. 



1. 



3. 



4. 



5. 



x^x 4 ) = 
(A)x 6 

(B) x 8 

(C) 2x 6 

(D) 2x 8 

If a rectangle has a perimeter of 36 feet and 
is 4 feet wide, what's its area? 

(A) 56 square feet 

(B) 128 square feet 

(C) 112 square feet 

(D) 16 square feet 

The cube root of 64 is 

(A) 3 

(B)9 

(C)2 

(D)4 

Convert 314,000 to scientific notation. 

(A) 3.14xl0 5 

(B) 3.14xl0- 5 

(C) 314x10 

(D) 31.4x100 

The reciprocal of j, is 
b 

(A)l 
(B)3 
(C)6 



(D) 



1 



6. If 0.05 * x = 1, then* = 

(A) 0.05 

(B) 0.5 

(C) 50.0 

(D) 5.0 

7. Factor x 2 - 6x + 9. 

(A) (x + 6)(x + 6) 

(B) (x-6)(x + 6) 
(C)(x-3) 2 

(D) (x + 3) 2 

8. (3 x 2)(7 - 2)(6 + 2) = (6 x 4)x. What's the 
value of x? 

(A) -5 

(B)5 

(Q10 

(D)l 

9. Solve for x: 2x - 6 = x + 5. 
(A) 3 

0)ii 

(C)7 
(D)5 

10. UI=Prt, and P= $1,000, r= 7%, and t = 1, 
what does /equal? 

(A) $35 

(B) $1,000 

(C) $700 

(D) $70 



Go on to next page 



250 



Part V: Practice ASVAB Exams 



11. 



12. 



13. 



14. 



Solve for x in the equation (x - 7) 2 - 4 = 
(x + l) 2 . 



(A) 2\ 


(B)2| 


(C)4l 


(D)4| 


A circle has a radius of 5 inches. What's its 


approximate area? 


(A) 78.5 inches 


(B) 70.0 inches 


(C) 314.0 inches 


(D) 25.0 inches 



Solve the following inequality: 
|(6x-9) + 4>5x + l. 

(A) x > 6 

(B) x < 6 

(C) x > -3 

(D) x < -3 

A tube has a radius of 3 inches and a height 
of 5 inches. What's its approximate 
volume? 

(A) 34 cubic inches 

(B) 141 cubic inches 

(C) 565 cubic inches 

(D) 45 cubic inches 



15. Triangle ABC (shown below) is a(n) 




(A) right triangle. 

(B) equilateral triangle. 

(C) scalene triangle. 

(D) isosceles triangle. 



16. The following figure what type of 
quadrilateral? 



18. 



19. 



-H- 



,B 



D z 



-H- 



(A) square 

(B) rhombus 

(C) trapezoid 

(D) parallelogram 

17. The angle shown below is a(n) 



(A) complementary angle. 

(B) supplementary angle. 

(C) acute angle. 

(D) obtuse angle. 

Solve for x: -x 2 - x + 30 = 0. 

(A) 4, -8 

(B) -6, 5 

(C) -4, 5 

(D) 6, -3 

A square box has a volume of 64 cubic 
inches. What's the perimeter of one of its 
faces? 

(A) 8 inches 

(B) 16 inches 

(C) 64 inches 

(D) 32 inches 

20. A cube has a volume of 64 cubic inches. 
What's its surface area? 

(A) 16 square inches 

(B) 64 square inches 

(C) 96 square inches 

(D) 32 square inches 



Go on to next page 



Chapter 18: Practice Exam 2 25 1 



21. 


(x 3 )^ 




(A) 3x 3 




(B)x 6 




(Qx 9 




(D) 2x 6 


22. 


4! = 




(A) 16 




(B) 40 




(9 




(D)24 


23. 


If a 3 + b 3 = 




(A) ft 3 - a 3 




(B)x 




(C) a 3 - b 3 




(D)a 



24. What's the sum of the integers from 1 
to 300? 



- x 3 , then b = 



(A) 38,243 


(B) 45,150 


(C) 49,923 


(D) 52,024 


25. (y 2 ) 3 + y 2 = 


(A)y 7 


(B)y 6 


(C)y 8 + y 2 



(D) 3y 2 



STOPI 



DO NOTTURNTHE PAGE UNTILTOLDTO DO SO. 
DO NOT RETURN TO A PREVIOUS TEST. 



252 



Part V: Practice ASVAB Exams 



Subtest 6: Electronics Information 



Time: 9 minutes for 20 questions 

Directions: This part tests your knowledge of electrical, radio, and electronics information. Select 
the correct response from the choices given and then mark the corresponding space on your 
answer sheet. 



4. 



5. 



What is used to measure current that is 
going through a circuit? 

(A) multimeter 

(B) amp gauge 

(C) currentometer 

(D) tri-gauge 

Which of the following isn't a component of 
a DC motor? 

(A) rotor bars 

(B) armature 

(C) field poles 

(D) yoke 

The television broadcast standards in the 
United States is 

(A) NTSC. 

(B) RGB. 

(C) SECAM. 

(D) RTSC. 

In a closed electrical circuit, 

(A) one terminal is always positive, and 
one terminal is always negative. 

(B) both terminals can be positive. 

(C) both terminals can be negative. 

(D) terminals are neither positive nor 
negative. 

Electrical current is counted in what mea- 
surement? 

(A) hertz 

(B) voltage 

(C) amps 

(D) ohms 



6. The following symbol is a/an 



(A) resistor. 

(B) fuse. 

(C) capacitor. 

(D) inductor. 

7. In the United States, what is the specifica- 
tion for an electrical outlet in a bathroom 
near a sink? 

(A) If within 6 feet of a sink, an outlet must 
have a childproof cover. 

(B) If within 2 feet of a sink, an outlet must 
not be GFCI protected. 

(C) If within 6 feet of a sink, an outlet must 
be GFCI protected. 

(D) If within 2 feet of a sink, an outlet must 
also be within reach of the bathtub. 

8. The following symbol is a/an 



rv 




(A) lamp. 

(B) fuse. 

(C) inductor. 

(D) bell. 



Go on to next page 



Chapter 18: Practice Exam 2 253 



9. When a circuit breaker trips, in what posi- 
tion will you find the operating handle? 

(A) on position 

(B) off position 

(C) halfway between on and off 

(D) three-fourths of the way between the 
on position and the off position 

10. Which wire is smallest? 

(A) 00 AWG 

(B) 4 AWG 

(C) 10 AWG 

(D) 12 AWG 

11. Which of the following is the best 
conductor of electricity? 

(A) plastic 

(B) wood 

(C) aluminum 
(D) copper 

12. How many paths of electrical flow can be 
found in a series circuit? 

(A) one 

(B) two 

(C) two or more 

(D) It can't be determined from the 
information given. 

13. A microwave is rated at 1,200 watts. At 
120 volts, how much current does it draw? 

(A) 1 amp 

(B) 10 amps 

(C) 100 amps 

(D) 1,440 amps 

14. Electricians use the term low potential to 
refer to 

(A) electrical circuits with a low potential 
for overload. 

(B) building codes that reduce the risk 
of fire. 

(C) the likelihood of getting a raise this year. 

(D) 600 watts or less. 



15. Which of the following isn't a conductor of 
electricity? 

(A) water 

(B) graphite 

(C) gold 

(D) glass 

16. The ground wire is always 

(A) green. 

(B) black. 

(C) whitish. 

(D) blue. 

17. What does AM mean? 

(A) amp metrics 

(B) alien mothers 

(C) amplitude modulation 

(D) annoid matrix 

18. Silver is a better conductor than copper. 
But copper is more often used because of 

(A) the cost of silver. 

(B) the brittleness of copper. 

(C) the low melting point of silver. 

(D) the tendency of silver to tarnish. 

19. Electronic circuits that produce high 
frequencies are called 

(A) amplifiers. 

(B) regulators. 

(C) transformers. 

(D) oscillators. 

20. If you plug an appliance designed for AC 
into a DC power source, the appliance 

(A) will operate normally. 

(B) will produce excessive heat. 

(C) won't operate. 

(D) will explode into tiny pieces. 



STOPI 



DO NOTTURNTHE PAGE UNTILTOLDTO DO SO. 
DO NOT RETURN TO A PREVIOUS TEST. 



251* 



Part V: Practice ASVAB Exams 



Subtest 7: Auto & Shop In formation 



Time: 11 minutes for 25 questions 

Directions: This test is about automobiles, shop practices, and the use of tools. Pick the best 
answer for each question and then mark the corresponding space on your answer sheet. 



1. If a car uses too much oil, which of the fol- 
lowing parts may be worn? 

(A) camshaft 

(B) connecting rods 

(C) fuel pump 

(D) piston rings 

2. Clean air filters are important because 

(A) dirty filters can cause a decrease in 
fuel mileage. 

(B) they remove pollutants, which can 
decrease engine performance. 

(C) they keep the oil from becoming 
contaminated. 

(D) both A and B 

3. The alternator 

(A) starts the engine. 

(B) supplies power to the battery. 

(C) connects the ignition system to the 
engine. 

(D) can be used as an alternative to 
motor oil. 

4. In which automotive system would you find 
a "wishbone"? 

(A) transmission 

(B) engine 

(C) exhaust 

(D) oil pan 

5. If the electrolyte solution in a battery is too 
low, you should add 

(A) sulfuric acid. 

(B) antifreeze. 

(C) distilled water. 

(D) gasoline. 



6. What area of your car should be 
flushed periodically to maintain optimum 
performance? 

(A) exhaust system 

(B) brake system 

(C) cooling system 

(D) ignition system 

7. The primary purpose of a carburetor is to 

(A) maintain engine timing. 

(B) regulate oil pressure. 

(C) mix fuel and air. 

(D) monitor tire pressure. 

8. Car restorers often seek NOS parts. What 
does NOS stand for? 

(A) Near Original Specifications 

(B) NASCAR Operating Standards 

(C) New Old Stock 

(D) none of the above 

9. To make spark plugs work effectively, the 
coil and breaker 

(A) provide a gap between the electrodes. 

(B) ignite the spark. 

(C) transfer the electricity to the correct 
spark plug. 

(D) create a very high electrical voltage. 

10. Schrader valves can be found in your car's 

(A) tires. 

(B) engine. 

(C) transmission. 

(D) electronic ignition. 



Go on to next page 



Chapter 18: Practice Exam 2 255 



11. A bent frame causes 

(A) improper tracking. 

(B) auto accidents. 

(C) poor visibility 

(D) excessive rust. 

12. In the tire designation 205/55 R 15 92 H, 
what does the "H" signify? 

(A) tread type 

(B) tire height 

(C) maximum sustained speed 

(D) turning radius 

13. When the tightness of screws and/or bolts 
is important, it's best to use 

(A) a screwdriver. 

(B) a torque wrench. 

(C) tin snips. 

(D) a coping saw. 

14. Hammer faces are commonly made of each 
of the following materials EXCEPT 

(A) steel. 

(B) brass. 

(C) plastic. 

(D) lead. 

15. Hammers, mallets, and sledges are all 
striking tools, but mallets and sledges 
don't have 

(A) claws. 

(B) metal parts. 

(C) as much durability. 

(D) heads. 

16. Round objects can be measured most 
exactly using a 

(A) rigid steel rule. 

(B) folding rule. 

(C) set of calipers. 

(D) depth gauge. 



17. The best chisel to use when making a 
circular cut in metal is a 

(A) cold chisel. 

(B) socket chisel. 

(C) butt chisel. 

(D) round chisel. 

18. A pipe wrench is also known as a 

(A) strap wrench. 

(B) hammer. 

(C) plumb-bob. 

(D) Stillson wrench. 

19. Painting on a surface with too much 
moisture 

(A) causes no problems. 

(B) causes bubbling. 

(C) requires an extra coat of paint. 

(D) takes longer. 

20. A tool used to control the location and/or 
motion of another tool is called a 

(A) control tool. 

(B) jig. 

(C) nail. 

(D) static rectifier. 

21. An 8-point saw 

(A) has 7 teeth per inch. 

(B) weighs 8 ounces. 

(C) can saw 8 kinds of material. 

(D) is 8 inches long. 

22. Concrete is made by mixing 

(A) cement and sand. 

(B) cement, sand, and water. 

(C) cement and water. 

(D) cement, sand, gravel, and water. 



Go on to next page 



250 Part V: Practice ASVAB Exams 



23. Which of the following tools isn't used to 
cut metal? 



(A) 



25. The following tool is a(n) 



(B) 





24. The following tool is used to 



(A) Phillips screwdriver. 

(B) Allen wrench. 

(C) socket wrench. 

(D) offset screwdriver. 




(A) cut tile. 

(B) cut wire. 

(C) turn screws. 

(D) cut bolts. 



STOPI 



DO NOTTURNTHE PAGE UNTILTOLDTO DO SO. 
DO NOT RETURN TO A PREVIOUS TEST. 



Chapter 18: Practice Exam 2 25 7 



Subtest 8: Mechanical Comprehension 



Time: 19 minutes for 25 questions 

Directions: This test is about mechanical principles. Many of the questions use drawings to illus- 
trate specific principles. Choose the correct answer and mark the corresponding space on the 
answer sheet. 



A simple pulley gives a mechanical advan- 
tage of 

(A) 2 

(B)3 

(C)l 

(D) unknown 

The baskets are balanced on the arm in the 
figure below. If cherries are removed from 
Basket B, then to rebalance the arm, 



Basket A 



Basket B 



fulcrum 

(A) the fulcrum will have to be moved to 
the right. 

(B) Basket B will have to be moved to 
the right. 

(C) Basket A will have to be moved to 
the left. 

(D) Basket A will have to be moved to 
the right. 



3. If both Wheel A and Wheel B revolve at the 
same rate in the figure below, Wheel A will 
cover a linear distance of 12 feet 




(A) faster than Wheel B. 

(B) slower than Wheel B. 

(C) in about the same time as Wheel B. 

(D) half as quickly as Wheel B. 

If a force of 200 pounds is exerted over an 
area of 10 square inches, what's the psi? 

(A) 10 

(B)15 

(C) 20 

(D)200 



Go on to next page 



258 



Part V: Practice ASVAB Exams 



5. In the following figure, if you move Anvil A 
toward the middle of the seesaw, Anvil B 
will 




V 





Anvil A / \ Anvil B 



(A) remain stationary. 

(B) move toward the ground. 

(C) rise in the air. 

(D) lose weight. 

6. If a ramp measures 6 feet in length and 
3 feet in height, an object weighing 
200 pounds requires how much effort 
to move using the ramp? 

(A) 200 pounds 

(B) 100 pounds 

(C) 50 pounds 

(D) 300 pounds 

7. A micrometer is used to measure 

(A) small changes in temperature. 

(B) changes in psi. 

(C) thicknesses to a few thousandths of 
an inch. 

(D) objects invisible to the unaided eye. 

8. If the weight is removed from Side B of the 
seesaw, what happens to the weight on 
Side A? 



S 



^g^^r ^^^ 



Side A 



Side B 



(A) The weight will never move from Side B. 

(B) The weight on Side A will move up in 
the air. 

(C) The weight on Side A will move toward 
the ground. 

(D) Nothing will happen. 



9. The force produced when two objects rub 
against each other is called 

(A) gravity. 

(B) recoil. 

(C) magnetism. 

(D) friction. 

10. Normally, atmospheric pressure is 
approximately 

(A) 14.7 psi 

(B) 23.2 psi 

(C) 7.0 psi 

(D) 10.1 psi 

11. For Gear A and Gear B to mesh properly in 
the following figure, 



12. 




(A) they must be the same size. 

(B) they must turn at different rates. 

(C) they must both turn in the same 
direction. 

(D) their teeth must be of equal size. 

Torsion springs 

(A) produce a direct pull. 

(B) exert no pull. 

(C) produce a twisting action. 

(D) coil but do not uncoil. 



Go on to next page 



Chapter 18: Practice Exam 2 250 



13. To move a 400-pound crate from the floor 
of a warehouse to the bed of a truck 4-feet 
off the ground, the most efficient device to 
use is a 

(A) lever. 

(B) inclined plane. 

(C) fixed pulley. 

(D) jackscrew. 

14. Water in an engine can cause damage in 
winter weather because 

(A) it can vaporize. 

(B) water expands when it freezes. 

(C) ice is heavier than water. 

(D) cold water creates more steam than 
warm water. 

15. The weight of the load is being carried on 
the backs of the two anvils shown in the 
figure. Which anvil is carrying the most 
weight? 



D_ 



16. 




Anvil A 




Anvil B 



(A) Anvil A 

(B) Anvil B 

(C) Both are carrying an equal amount of 
weight. 

(D) It can't be determined without more 
information. 



When the block-and-tackle arrangement 
shown in the figure is used to lift a load, 
all the following parts remain stationary 
EXCEPT 



Upper 
Block 



Lower 
Block 



LOAD 



(A) the upper hook. 

(B) the upper block. 

(C) the lower block. 

(D) all the parts move. 

17. In the following figure, what effort (E) must 
be applied to lift the anvil? 




(A) 7.0 pounds 

(B) 9.0 pounds 

(C) 21.0 pounds 

(D) 10.5 pounds 



Go on to next page 



260 



Part V: Practice ASVAB Exams 



18. In the figure below, for each complete revo- 
lution the cam makes, how many times will 
the valve open? 

cylinder 



19. 



-valve 



XL 



cam - 




r/o 



(A)l 
(B)6 
(9 3 
(D)2 

In the following figure, assume the valves 
are all open. Which valves need to be 
closed for the tank to fill up completely? 



Water 
^Supply/ 



Valve 1 



l Valve 3 



Valve 2 



-Drainage 



Valve 4 



Valve 5 



(A) 3 and 4 only 

(B) 3, 4, and 5 

(C) 2, 3, and 4 

(D) 4 only 



20. If Gear A turns left in the figure below, 
Gear B 



21. 




(A) won't turn. 

(B) turns left. 

(C) turns right. 

(D) It can't be determined. 

If Gear 1 makes 10 complete clockwise 
revolutions per minute in the figure below, 
then 




(A) Gear 2 makes 10 complete clockwise 
revolutions per minute. 

(B) Gear 2 makes 20 complete counter- 
clockwise revolutions per minute. 

(C) Gear 2 makes 5 complete counter- 
clockwise revolutions per minute. 

(D) Gear 3 keeps Gear 2 from making any 
revolutions. 



Go on to next page 



Chapter 18: Practice Exam 2 26 / 



22. For the fuel to travel from Reservoir A to 
Reservoir B, passing through Filters C and 
D on the way, which valves must be open? 



Filter C 

I. 



Reservoir A 



w 



JM 



"e: 



7ZL 



*M 



:s_ 




24. If a water tank on a toilet keeps overflow- 
ing, the problem is probably a 

(A) defective float. 

(B) clogged pipe. 

(C) crimped chain. 

(D) improper seal. 

25. In the figure below, the board holds the 
anvil. The board is placed on two identical 
scales. Each scale reads 



(A) 1, 2, 4, and 8 

(B) 1, 2, and 3 

(C) 6, 7, and 8 

(D) 4, 6, and 7 

23. A yellow flame on a gas furnace indi- 
cates that 

(A) everything is fine. 

(B) the fuel-air mixture is too rich. 

(C) the fuel-air mixture is too lean. 

(D) the gas pressure is too low. 




scales are identica 



(A) 24 
(B)10 
(C)12 
(D)40 



STOPI 



DO NOTTURNTHE PAGE UNTILTOLDTO DO SO. 
DO NOT RETURN TO A PREVIOUS TEST. 



262 



Part V: Practice ASVAB Exams 



Subtest 9: Assembling Objects 



Time: 15 minutes for 25 questions 

Directions: The Assembling Objects subtest consists of questions that measure your ability to men- 
tally picture items in two dimensions. Each question is comprised of five separate drawings. The 
problem is presented in the first drawing and the remaining four drawings are possible solutions. 
Determine which of the choices best solves the problem shown in the first picture, then mark the 
corresponding choice on your answer sheet. 



4- f 

A 1 

B B 


S 


t-*- 


% 


r 





\ 


<^> 


^ 


i 



^ 


© 





o 


© 



A B 



10. 






3. 






11. 







4. 



AD 

GO 


<$> 


V 


SP 


<p 



5. 



t) i 



.r"? 




-2-^t 



12. 



13. 





A B 



A B 





ll'W 



6. 



< ^ 



14. 



M2 


€^ 


ffo 


€^ 


-f 



|A A 

& 1 


J 


^ 


^ 


^ 



15. 



s- ! 


\rf 


£ 


sH? 


^ 



A B 




16. 



/TV Ka KJ K) Ra 



Go on to netf jm#e 



Chapter 18: Practice Exam 2 263 



17. 







18. 



~A,a* 

+ " B 


\ 


iH- 


j/r 


V 



19. 
















A 


B 


C 


D 


20. 


J>A ' 


5 


^ 


J 


<n 


^ 



21. 



> 
























/\ 




^^x ^ 


/ 




> 


/( 




> 


A 




> 




/( 






c 


\> 


/- 


± — ^ 




^- 


i^s^ 




/- 


>_^- 




/- 


>^^ 





22. 



0» b 


^ 


4--0 


^3 


o-^ 



23. 



« o 


OS) 


G© 


CED 


OS) 



Go on to netf page 



261} Part V: Practice ASVAB Exams 



24. 



Ma * 


®y\ 


® 


As 


^ 




STOPI 



DO NOTTURNTHE PAGE UNTILTOLDTO DO SO. 
DO NOT RETURN TO A PREVIOUS TEST. 




Chapter 19 

Practice Exam 2: 
Answers and Explanations 



Were are the answers and explanations for the practice exam in Chapter 18. Read over 
each question from Chapter 18 as you check your answers. Doing so reminds you 
what the question is about and serves as a helpful review. If you look at each question and 
the possible answers, you can also identify some of the traps that you may run across on 
theASVAB. 

You don't have to be an algebra ace to determine whether you're making progress through 
your review efforts. Simply compare the number of wrong answers you got on Practice 
Exam 1 (Chapter 16) against the number of wrong answers you got on this test. If you put 
the work in, you'll probably find that you made fewer errors on Practice Exam 2. 

By the time you've scored Practice Exam 2, you should have a good idea of your strengths 
and weaknesses. If some subjects or subtests still give you problems, keep studying — just 
follow the cross-references for some tips and additional practice questions. If you find you 
need in-depth study, check out Chapter 17, where I name some books on various subjects 
covered in the subtests. 

The ASVAB is technically scored by comparing your raw score to the scores of other people, 
which produces a scaled score. Turn to Chapter 2 to find out how the ASVAB is scored. 



Subtest 1: General Science Answers 



The answers to the questions on the General Science subtest are fairly straightforward — 
you either know the answer or you don't. This can be a hard subject to study for because 
General Science includes the entire scope of scientific disciplines. The good news is you 
may not even have to score well on this subtest — it depends on the job you're interested 
in. See Appendix A for military jobs that require a good General Science score. You can find 
additional science practice questions in Chapter 15. 

l.B 6. B 11. A 16. B 21. D 



2D 


7. B 


12. C 


17. A 


22. B 


3. C 


8. A 


13. B 


18. C 


23. C 


4. A 


9. D 


14. D 


19. C 


24. C 


5. D 


10. B 


15. C 


20. A 


25. C 



266 



Part V: Practice ASVAB Exams 



Subtest 2: Arithmetic Reasoning Answers 

This subtest is one of the most important because it makes up a portion of your AFQT 
score, the score that determines your overall mental qualifications to enlist in the military. 

If you think you need more in-depth study, review Chapters 7 and 8 or see whether you can 
find some high school-level math textbooks at your local library. Chapter 9 has some addi- 
tional practice questions. 

1. D. For each die, the probability of rolling a 5 is 1 out of 6 that is,^ , so the probability of 
not rolling a 5 is 1 - -~, or -~. With two dice, the probability of not rolling a 5 is -~ x ^, or -^ . 

2. C. Multiply $1,500 by 7%, or 0.07, and get $105. Then add $105 to $1,500 to find the answer, 
$1,605. 

Sometimes you can actually save time by not working the problem. In this problem, simply 
recognizing that the answer has to be more than $1,500 makes it obvious that Choices (A) 
and (B) are wrong. It also should be obvious that 1% of $1,500 has to be more than $7, so 
Choice (D) is also wrong. That only leaves the correct answer, Choice (C). 

3. A. Two tons = 4,000 pounds; 4,000 x $0.12 = $480. 

4. C. y = (a + ft) 2 . Expanding the equation results in y = a 2 + b 2 + lab. You know that a 2 + b 2 = 30 
and ab = 10. When you substitute these known values into the equation, you get y = 30 + 2(10). 
Solving for y results in y = 50. 

5. C. There are 2 pints in a quart, and 4 quarts make up a gallon; therefore, a gallon contains 
2x4 = 8 pints, or 16 half-pints. One half-pint equals tf of a gallon. 

6. D. First subtract the old cost from the new cost: $2.80 - $2.50 = $0.30. Then divide the dif- 
ference by the old cost to find the percent difference: $0.30 + $2.50 = 0.12 = 12%. 

1 5 

7. C. The aircraft travels 120 miles in 12 minutes, which is 4 of an hour. Therefore, in # (or 

1 hour), it would travel 5 x 120, or 600 miles. The aircraft is traveling 600 miles per hour. 

8. D. Multiply 50 bushels by 8% to find the yield increase in bushels: 50 x 0.08 = 4. Add 4 bush- 
els (the amount of the increase) to 50 bushels (the original yield) to determine that an 8% 
increase equals 54 bushels. 

9. B. Convert the change to dollars or fractions of dollars and add: 

14 dollars = $14.00 

16 half dollars =$8.00 

12 quarters = $3.00 

8 dimes = $0.80 

25 nickels =$1.25 

10 pennies =$0.10 
= $27.15 

Subtract the total from $30.00 to determine how much money Junior has to borrow: 

$30.00 -$27.15 = $2.85. 

10. B. Her commission for the week was $40 (because 0.05 x 800 = 40). The ratio of her com- 
mission to her salary is 40:80, which can be reduced to 1:2. 

11. C. A gallon consists of 4 quarts, and 4x 25 = 100. 



Chapter 19: Practice Exam 2: Answers and Explanations 2 O 7 



12. D. Four members is twice as many as two members. Multiply the number of days it would 
take four people to paint by 2 (that is, 3 x 2 = 6) to determine how long it would take two 
people to do the same task. 

13. A. Brian's hourly wage is 24 * 5 = $4.80. Christina's hourly wage is 10.95 * 3 = $3.65. 
$4.80 -$3.65 = $1.15. 

14. C. The amount of time from 8:30 a.m. to 11:15 a.m. is 2 hours, 45 minutes. From 8:30 a.m. 

o 

until 10:30 a.m. is 2 hours. From 10:30 until 11:15 is 45 minutes, or 4 of an hour, for a total of 
o 4 

2 j hours. 

15. C. At a rate of 7 documents per minute, the employee can shred 630 documents in 90 min- 
utes. How do you come up with that number? Multiply 7 by 90 (the number of minutes in 
li hours). Subtract 630 from 900 total documents to determine that after li hours of 
shredding, 270 documents remain. 

16. A. If the stereo depreciates 20%, the value of the stereo then becomes 80% of its original 
value. After depreciation, the value of the stereo the first year is $960 (0.8 x 1,200). The 
value of the stereo after the second year is $768 (0.8 x 960). $768 is Choice (A) 

17. A. Divide the distance (135 miles) by the speed (45 miles per hour) to determine that Janet 
will take 3 hours to reach the city. 

18. D. Multiply the price of the blouse by the amount of the discount: $18.50 x 0.20 = $3.70. 

19. C. Subtract the original salary from the new salary to get the difference in salary: $360 - 
$320 = $40. Then divide the difference in salary ($40) by the original salary ($320) to deter- 
mine the percent increase: 40 + 320 = 0.125 = 12.5%. 

20. B. A circle is 360 degrees, so 40 degrees is i of a circle (360° ± 40° = 9). To get the answer, 

multiply the circumference of the track by -^ : 360 ft. x ^ = 40 ft. . 

21. B. Subtract $35.98 from $40.00 to get $4.02. 

22. B. Subtract the time of the plane from the time of the balloon to determine how much 
longer it took the balloonist: 

13 days 12 hours 16 minutes 13 seconds 

- 4 days 10 hours 15 minutes 7 seconds 

9 days 2 hours 1 minute 6 seconds 

23. A. Divide the total cost by the number of boxes purchased to determine the cost per box: 

$48 ^ 12 = $4. 

24. D. Multiply 20 x 0.05 to determine how many more miles per gallon the car will get. The 
answer is 1. Then add the number of additional miles per gallon the car will get to the orig- 
inal number of miles per gallon the car gets to reach the new average: 1 + 20 = 21. 

25. A. The minimum length of spike is equal to the diameter of the tree. To find the diameter 
of the tree, use the formula, C = nd, where C = 43.96 and n = 3.14: 43.96 = 3.14 xd;d = 43.96 + 
3.14; d= 14. 

1 3 1 

26. D. Divide 104 by 4. You can perform this operation by multiplying 104 by the reciprocal 

of -|: 10^ x 4 = ^ x 4 = "^- Divide 84 by 6 - and the answer is 14. 

27. C. First figure out how many stones will be needed along the 12-foot side of the patio and 
then how many stones will be needed along the 14-foot side of the patio. Then multiply 
those two numbers together to get the total number of stones required. Here's the math: 
Convert 12 feet to inches: 12 ft. x 12 in./ft. = 144 inches. The paving stones are 8 inches 
square, so divide 144 inches by 8 inches (144 + 8), which gives you 18 stones. 



268 



Part V: Practice ASVAB Exams 



Do the same math for the 14-foot length: 14 ft. x 12 in./ft. = 168 in., and 168 * 8 = 21. 
Therefore, 21 stones are needed on the 14-foot side. 

Now multiply the stones: 18 stones x 21 stones = 378 stones, which is Choice (C). 

28. D. Calculate the amount of the deduction by multiplying her salary by the percent deducted: 
$25,000 x 28% = $25,000 x 0.28 = $7,000. Subtract that product from the salary to determine 
the net pay: $25,000 - $7,000 = $18,000. 

29. B. When the sum of two angles is 180 degrees, the angles are said to be supplementary to 
each other. To find the supplement, subtract 55 from 180: 180 - 55 = 125. 

30. C. Multiply the height of the stack in feet by 12 to determine the height of the stack in 
inches: 6 x 12 = 72 inches. Divide that number by 4 inches, the thickness of each board, 
to determine the number of pieces of lumber in the stack: 72 * 4 = 18. 

Subtest 3: Word Knowledge Answers 

Your score on the Word Knowledge subtest is important — it counts toward your AFQT 
score (see Chapter 1). 

If your score on the Word Knowledge subtest has improved since you took the first test, 
congratulations! If not, don't be too surprised. Improving your score on this subtest in a 
short period of time is difficult, but it can be done. Review the information from Chapter 4 
and set aside time each day (maybe several times a day, depending on how soon you plan 
on taking the ASVAB) to memorize words, roots, prefixes, and suffixes. Chapter 6 has some 
additional practice questions for you to sink your dictionaries into. 

l.C 8. B 15. A 22. C 29. A 



2. A 


9.B 


16. D 


23. D 


30. A 


3. A 


10. C 


17. A 


24. A 


31. C 


4. C 


11. C 


18. C 


25. D 


32. C 


5.B 


12. B 


19. B 


26. B 


33. D 


6. B 


13. D 


20. A 


27. B 


34. B 


7. D 


14. B 


21. C 


28. D 


35. D 



Subtest 4: Paragraph Comprehension Answers 

Doing well on the Paragraph Comprehension subtest is really important if you want to enlist 
in the military — this score counts toward your AFQT score. If you're missing more answers 
than you should, review the info in Chapter 5 and concentrate on improving your analytical 
reading skills. For example, when you're reading the evening newspaper, ask yourself what 
the main point of an article is. Or when you finish a news story, set the paper down and try 
to remember what the President said about the budget deficit. Think of this technique as a 
workout for your mind. You can find more practice questions in Chapter 6. 



Chapter 19: Practice Exam 2: Answers and Explanations £Oy 



1. B. The author is describing a quaint country setting. 

2. B. The passage states that the brooks in the village run south, so the answer is Choice (B). 
A few miles north, the brooks run in an opposite direction (north). 

3. B. The passage states that Panama revolted against Columbia, not that they fought over the 
canal, so Choice (A) is incorrect. The passage states that the foreign companies were unsuc- 
cessful in building the canal, so Choice (C) is incorrect. The United States, not Panama, built 
the canal, so Choice (D) is wrong. In the next to last sentence, the passage states that the 
U.S. was given rights to the land the canal occupied, making Choice (B) the correct answer. 

4. D. According to the passage, a torn U.S. flag can be professionally mended, but a severely 
torn flag should be destroyed. The preferred method of destruction is by burning. 

5. D. The passage states that guilds had economic and social purposes, so Choice (A) is 
incorrect. The passage states that guilds were similar to labor unions, so Choice (B) is 
incorrect. The passage states that guilds protected merchants and craftspeople; it says 
nothing about exploiting workers, so Choice (C) is incorrect. The third sentence states 
that some guilds held considerable economic power, but even small guilds protected mem- 
bers, making Choice (D) the correct answer. 

6. D. According to the passage, it took more than four years for the government to believe 
that anyone had flown a heavier-than-air craft. The historic flight was in December 1903, 
and the Wright brothers delivered the first aircraft to the government in August 1908, 
4.5 years later. The passage supports all the other statements. 

7. C. Freud comments on the characteristics of memory throughout the entire passage. 

8. A. The passage describes how troy and common weights are different, so Choice (B) is incor- 
rect. Common and avoirdupois are the same system, so Choice (C) is incorrect. A troy ounce 
is 480 grains, so Choice (D) is incorrect. Choice (A) is the correct answer because the second 
sentence states that a common ounce is 437.5 grains, which is just shy of than 438 grains. 

9. B. The passage doesn't address leading by example or use of technology by workers, so 
Choices (A) and (C) are incorrect. Maximizing worker performance is a result of leadership 
involvement, not a principle of leadership, making Choice (D) incorrect. The correct 
answer, showing interest in workers' problems, is in the third sentence of the passage. 

10. C. The passage doesn't support Choices (A) or (B). The passage states that white blood 
cells are found in blood and bone marrow, so Choice (D) is wrong. The correct answer, 
Choice (C), can be found in the first sentence. The passage states that leukemia interferes 
with "the body's ability to produce red blood cells." 

11. C. High intensity conflict is listed as a type of military operation (in the last sentence), not 
one of the four operational concepts. 

12. C. The JV 2020 guides all the military services with its vision of future war fighting. 
Although Choice (D) is close, the passage doesn't specifically reference military training. 

13. B. Cytogenetics is the study of the cellular basis of inheritance; the text doesn't support 
Choices (A), (C), or (D). 

14. A. Nothing in the passage supports Choices (B) or (D). Although human genetics is an 
important subfield of genetics, nothing in the passage suggests that it's the only concern of 
geneticists. Microbial genetics, as the passage mentions, is a subfield in genetics that has 
nothing to do with humans, so Choice (C) is incorrect. Choice (A) is the correct answer — 
the second sentence mentions genes and their transmission to offspring. 

15. C. Nothing in the passage supports Choices (A), (B), or (D). Choice (C) is the correct 
answer because the last sentence in the passage states, "[Genetic counselors] advise cou- 
ples and families on the chances of their offspring having specific genetic defects." Note it 
does not state that genetic counselors use genetics to prevent offspring from inheriting 
defects, which is what Choice (D) states, making Choice (D) an incorrect answer. 



270 



Part V: Practice ASVAB Exams 



Subtest 5: Mathematics Knowledge Answers 

Although the military doesn't expect you to be the next Einstein, a solid grasp of mathemat- 
ics is important because math skills make up half of your AFQT score. If you're still strug- 
gling on this subtest, it's time to hit the books. (Actually, as much as you may feel like it, 
I don't recommend that you actually hit the books — just study them.) See Chapter 9 for 
some more fun practice questions. 

1. A. If two exponents have the same base, you can multiply them by keeping the base and 
adding the exponents together: x^x 4 ) = x 2+4 = x 6 , 

2. A. To find area, multiply length times width (A = Iw). You have the width, so you need to 
find the length. For a rectangle, P = 2/ + 2w, so plug in the values you know and solve for /. 
To determine the length, subtract two times the width from the perimeter: 36 - 2(4) = 36 - 
8 = 28. Divide the answer by 2 to determine the length of one side: 28 * 2 = 14. Then multi- 
ply length times width to determine the area: A - 14 x 4 = 56. 

3. D. The cube of 4 is 4 x 4 x 4 = 64, so 4 is the cube root of 64. 

4. A. To convert this number to scientific notation, move the decimal point to the left until 
it's to the immediate right of the first number, while counting the number of moves. In this 
case, you move it five places. The result is then multiplied by 10 raised to the power of the 
number of places the decimal point was moved. The exponent is positive here because the 
original number, 314,000, is larger than 3.14. Choice (B), 3.14 x 10" 5 , is equal to 0.0000314, 
so it's incorrect. 

5. C. A reciprocal is the number by which a number can be multiplied to produce 1. The 
reciprocal -~ is 6, because ^ x 6 = 1. 

6. A. You start with 0.05 * x - 1. Multiply both sides of the equation by x: 0.05 = korx= 0.05. 
Check by substituting 0.05 for x in the original equation. 

7. C. x 2 - 6x + 9 = (x - 3)(x -3) = (x - 3) 2 

8. C. You start with (3 x 2)(7 - 2)(6 + 2) = (6 x 4)x Solve the left side of the equation first. 
(6)(5)(8) = (30)(8) = 240. Therefore, 240 = (6 x 4)x, which equals 240 = 24x. Now isolate x by 
dividing both sides of the equation by 24: 240 + 24 = 24x * 24, or 10 = x Check your answer 
by substituting 10 for x in the original equation. 

9. B. Isolate x on one side of the equation. Subtract x from both sides of the equation and 
then add 6 to both sides: 

2x - 6 = x + 5 

x-6 = 5 

x= 11 

Check by substituting 11 for x in the original equation. 

10. D. Solve for/. /= (1,000)(7%)(1), or/= (1,000)(0.07)(1) = 70. 

11. B. Multiply out what's in parentheses. Then simplify and solve for x: 

(x-7) 2 -4 = (x + l) 2 

(x-7)(x-7)-4 = (x + l)(x + l) 

x 2 -7x-7x + 49-4 = x 2 +x + x + l 

x 2 -14x + 45 = x 2 +2x + l 

-14x + 45 = 2x + l 

-16x = -44 

x 4 z 4 



Chapter 19: Practice Exam 2: Answers and Explanations 271 



12. A. The area of a circle is A = nr 2 . A = 7t5 2 . The number n is approximately 3.14, so 3.14 x 25 
means A is approximately 78.5 square inches. 

o 

13. D. Distribute the %, simplify, and solve for x. Note that when you divide by a negative 

number, you have to switch the direction of the inequality sign. 
|(6x-9) + 4>5x + l 

Ax - 6 + 4 > 5x + 1 
Ax - 2 > 5x + 1 
Ax > 5x + 3 
-x> 3 
x<-3 

14. B. For cylinders, Volume = nr^h. In this problem, V - 7i(3 2 )(5). Assume n is approximately 
3.14. Vis approximately equal to (3.14)(9)(5), or 141 cubic inches. 

15. A. A right triangle has one right angle (one 90° angle). 

16. D. Parallelograms have opposite sides of equal length. 

17. D. Angles measuring more than 90 degrees are obtuse angles. 

18. B. This is a quadratic equation, which you solve by factoring. First factor out the -1; then 
factor the quadratic expression and solve for x: 

-x 2 -x + 30 = 



l(x 2 +x-30) = 




x 2 + x - 30 = 




(x+6)(x-5) = 




x+6 = 


x-5 = 


x = -6 


x = 5 



19. B. Volume equals length times width times height (V - Iwti). In this case, V - 64, so one 
edge of the box is 4 inches long (because 4 is the cube root of 64: 64 = 4 x 4 x 4). Find the 
perimeter by adding the four sides together: 4 + 4 + 4 + 4= 16. 

20. C. You calculate volume by multiplying length times width times height (V - Iwti). Because 
the edges are equal on a cube, each edge is 4 inches (because 4x4x4 = 64). The area of 
one face of the cube is 4 x 4 = 16 square inches, and because a cube has 6 sides, you multi- 
ply 16 x 6 to find the surface area of the cube, 96 square inches. 

21. C. (x 3 ) 3 is the same as 0c 3 )(x J )(Jc 3 ). Multiply exponents with the same base by keeping the 
base and adding the exponents: (* 3 )0c 3 )(jc s ) = x 3+3+3 = x 9 . 

22. D. 4! (4 factorial) = 4x3x2x1 = 24. 

23. B. You start with a 3 + b 3 = a 3 + x 3 . To solve, subtract a 3 from both sides of the equation and 
then take the cube root: 

a 3 + b 3 - a 3 

6 3 =x 3 

b - x 



272 



Part V: Practice ASVAB Exams 



24. B. The formula to find the sum of a finite arithmetic sequence is 5 = y(a + b), where n is the 

number of terms, a is the first term in the sequence, and b is the last term in the sequence. 
In this case there are 300 terms (n), and the first term is 1 and the final term is 300. 

5 = | (a + b) 
5=300(1 + 300) 

5 = 150(301) 
5 = 45,150 

25. B. (y 2 ) 3 is the same as (y 2 )(y 2 )(y 2 ). Multiply exponents with the same base by keeping the 
base and adding the exponents: (y 2 ~)(y 2 ~)(y 2 ~) = y 2 * 2 * 2 = y 6 . The second y 2 in the equation 
cannot be added into the first term because now they are not like terms, so the answer is 
Choice (B), y 6 + y 2 . Tricky, eh? 

Subtest 6: Electronics Information Answers 

If you're having difficulty defining the difference between AC and DC, you may want to 
spend some additional time studying basic electronic information. Reviewing Chapter 13 
can help. You can also wrap your wires around the practice questions in Chapter 15. 

On the other hand, you may not be interested in a military job that requires a decent score 
on this subtest (see Appendix A), in which case, don't bother. 

1. A. A multimeter includes several pieces of test equipment, including an ammeter, which 
measures inline current. 

2. A. Rotor bars are only on AC induction motors, not DC motors. 

3. A. NTSC stands for National Television Analog Committee and, although gradually being 
replaced by ATSC (Advanced Television Systems Committee), NTSC is currently the broad- 
cast standard in the U.S. Choice (B) is incorrect because RGB stands for red, green, and 
blue — the colors of light used to create an image. Although most televisions use this stan- 
dard, it is not a broadcast standard. Choice (C) is incorrect because SECAM (Sequentiel 
couleur avec memoire, or sequential color with memory) is a standard used in other coun- 
tries. Choice (D) is RTSC, which stands for Raytheon Technical Services Company and is 
obviously not the correct answer. 

4. A. In a closed circuit, one terminal is always positive, and the other is always negative. 

5. C. Amperes (or amps) are the unit of measure of electric current. Hertz is the unit of mea- 
surement of frequency, not current. Current equals voltage divided by resistance. 
Resistance is measured in ohms. Therefore, neither voltage nor ohms can be the unit of 
measure for current. 

6. B. The symbol is a fuse. Fuses are designed to blow (melt) if the current flowing through it 
exceeds a specified value. 

7. C. This is code prescribed by the NEC (National Electric Code). Outlets within 6 feet of a 
sink need to be GFCI protected for safety reasons. 

8. A. The symbol is a lamp. A lamp is a transducer that converts electrical energy to light. 

9. C. Conventional circuit breaker handles have four positions: on, off, trip, and reset. When 
tripped, the handle moves to the middle position. 

10. D. The smaller the wire, the larger the number. 



Chapter 19: Practice Exam 2: Answers and Explanations 2 /3 



11. D. Plastic does not conduct, and wood is a poor conductor. Aluminum is a good conductor 
but not better than copper. 

12. A. A series circuit has only one path, so if you break the circuit's path at any point, elec- 
tricity stops flowing. An example of a series circuit is a string of Christmas lights that no 
longer works if a single bulb burns out. 

13. B. /(current) = Power (watts) + Effort (volts). In this case, / = 1,200 + 120 = 10 amperes. 

14. D. Potential equals voltage; low potential is anything less than 600 watts. 

15. D. Glass is an insulator. Other insulators include plastics, paper, and rubber. 

16. A. Ground wires are always green. 

17. C. Amplitude modulation (AM) was the first type of audio modulation to be used in radio. 
It works well with high frequency (HF) and Morse code. 

18. A. Silver is a better conductor, but it's more brittle than copper and more expensive. 

19. D. Oscillators produce high frequencies. An amplifier changes the amplitude of a signal. 
A regulator is a circuit that maintains a constant voltage. A transformer is a device that 
changes (transforms) the voltage at its input side to a different voltage on its output side. 

20. B. When DC is applied to an AC appliance, the amount of resistance is less, so more cur- 
rent flows through the wire and heat builds up. 

Subtest 7: Auto & Shop In formation Answers 

You need to do well on this subtest to qualify for certain military jobs (see Appendix A). If 
you care about those jobs and you're missing more than a few questions on this subtest, it's 
time for more extreme measures — like taking your mother's car apart and putting it back 
together (or going back over Chapter 11). 

Drive back to Chapter 15 for more practice questions. 

l.D 6.C 11. A 

2. D 7. C 12. C 

3. B 8. C 13. B 

4. A 9. D 14. C 

5. C 10. A 15. A 



Subtest 8: Mechanical Comprehension Answers 

If you need to do well on the Mechanical Comprehension subtest (as in you're hoping for a 
military career that requires a score for this subtest) but you're still missing more answers 
than you should be, ask yourself whether your math skills need work. Go back to Chapters 7 
and 8 if they do. Many of the formulas you need to know for this subtest require an under- 
standing of arithmetic and basic algebra. 



16. C 


21. A 


17. D 


22. D 


18. D 


23. B 


19. B 


24. D 


20. B 


25. B 



27b 



Part V: Practice ASVAB Exams 




Usually, improving your arithmetic and basic-algebra skills will improve your score on the 
Mechanical Comprehension subtest. Improving your knowledge of physics is also beneficial. 
Take a gander at Chapter 12 and the practice questions in Chapter 15. 

1. C. A simple pulley gives no mechanical advantage, although it does make work easier by 
spreading out the work needed over several tries. The mechanical advantage is 1. 

2. D. Moving Basket A to the right counterbalances the loss of cherries from Basket B. 

3. A. Wheel B has to make more revolutions to cover the same ground as Wheel A, so it 
covers the distance more slowly. 

4. C. You can calculate psi as Pressure = Force * Area. So in this problem, P = 200 * 10 = 20. 

5. B. If you move Anvil A toward the center, Anvil B will move toward the ground. 

6. B. The formula to determine mechanical advantage of an inclined plane is Length of Ramp * 
Height of Ramp = Weight of Object * Effort. Plugging in the numbers gives you 

6_200 
3" E 

6£ = 600 

£ = 100 

7. C. Micrometers measure very small but not microscopic objects. 

8. C. Reducing the weight on Side B will cause Side A to move toward the ground. 

9. D. Objects rubbing together produce friction. 

10. A. Normal atmospheric pressure (the average atmospheric pressure at sea level) is 14.7 psi. 

11. D. Gears of unequal size can mesh properly as long as their teeth are of equal size. 

12. C. Torsion springs coil or uncoil and produce a twisting action, not a direct pull; in other 
words, torsion springs apply torque. 

13. B. To move a heavy object a few feet in height, the inclined plane is the most efficient 
device (of those listed) to use. Note: The mechanical advantage of an inclined plane is 
equal to the slope of the plane divided by the height. The longer the slope is (compared to 
the height), the greater the mechanical advantage will be. 

14. B. Water expands when it freezes, possibly damaging engine components. 

15. A. The load is closer to Anvil A, so it's carrying the greater portion of the weight. 

16. C. All the listed parts remain stationary except the lower block. 

17. A. Apply the leverage formula: Length of Effort Arm * Length of Resistance Arm = 

Resistance Force * Effort Force: 

9 _ 21 
3" £ 

o_21 

a ~ E 

3£ = 21 

£ = 7 



Chapter 19: Practice Exam 2: Answers and Explanations 2 75 



18. C. The valve will open each time a high point of the cam hits it. The cam has three high 
points, so the valve will open three times per revolution. 

19. A. Closing only Valves 3 and 4 keeps the water from leaving the tank. 

20. C. Gears in mesh always turn in opposite directions. 

21. B. If Gear 1 turns at 10 rpm, then Gear 2, which is half the size, turns twice as fast, at a rate 
of 20 rpm. 

22. A. Opening Valves 1, 2, 4, and 8 allows the fuel to travel through the filters. Opening Valves 
1, 2, and 3 doesn't allow the fuel to travel through Filter D. Opening Valves 6, 7, and 8 
doesn't allow the fuel to travel through the filters. Opening Valves 4, 6, and 7 doesn't allow 
fuel to travel to Reservoir B. 

23. B. A yellow flame indicates too much fuel or not enough air. More air should be allowed to 
enter and mix with the gas. Thus, the fuel-air mixture is too rich. 

24. A. The float measures the water level in the tank. If the tank overflows, the float is proba- 
bly defective. 

25. C. The 20-pound anvil and the 4-pound board weigh 24 pounds total or, divided by 2, 
12 pounds per scale. 

Subtest 9: Assembling Objects Answers 

So far, only the Navy has elected to use scores from the Assembling Objects subtest and 
only for a few jobs. If you're planning on joining the Navy and you're interested in a Navy 
career that requires a score on this subtest (see Appendix A), review Chapter 14 for help on 
improving your score. For additional practice questions, see Chapter 15. 

l.B 6.A 11. C 16.D 21. A 



2. A 


7. B 


12. B 


17. C 


22. D 


3. C 


8. C 


13. A 


18. B 


23. C 


4.D 


9. D 


14. B 


19. C 


24. B 


5. C 


10. D 


15. D 


20. C 


25. D 



276 



Part V: Practice ASVAB Exams 



Chapter 20 

Practice Exam 3 



i 

■ suggest you take the third practice exam a week or so before you're scheduled to take 
«S the real ASVAB. Use it to refresh your memory of the material or to cram for any of the 
subtests that you have to do better on than you've been doing. 



$JABE* 




Don't forget to use the test-taking strategies and the guessing tips in each of the subtest 
chapters earlier in this book. Chapter 3 provides additional information on how to improve 
your score just by using smart test-taking strategies. 

This sample test features nine subtests and follows the same format as the actual ASVAB. 
To get the most out of this sample test, take it under the same conditions as the real ASVAB: 

v* Allow yourself about three hours to take the entire exam, and take the whole thing at 
one time. 

i*" Find a quiet place where you won't be interrupted. 

i*" Bring a timer that you can set for various lengths of time, some scratch paper, and a 
pencil. 

i*" At the start of each subtest, set your timer for the specified period of time. Don't go on 
to the next section until the timer has gone off, and don't go back to a previous section. 
If you finish early, check your work for that section only. 

J-" Use the answer sheet that's provided. 

j-" Don't take a break during any subtest. You can take a short one- or two-minute break 
between subtests if you need it. 

After you complete the entire test, check your answers against the answer keys and expla- 
nations in Chapter 21. Then compare the results to your results on Practice Exams 1 and 2. 
You should see some improvement. 



21 S 



Part V: Practice ASVAB Exams 



Answer Sheet for Practice Emm 3 

Subtest 1: General Science 

1®®©® 6®®©® 11®®©® 16®®©® 21®®©® 

2®®©® 7®®©® 12®®©® 17®®©® 22®®©® 

3®®©® 8®®©® 13®®©® 18®®©® 23®®©® 

4®®©® 9®®©® 14®®©® 19®®©® 24®®©® 

5®®©® 10®®©® 15®®©® 20®®©® 25®®©® 

Subtest 2: Arithmetic Reasoning 

1®®©® 7®®©® 13®®©® 19®®©® 25®®©® 

2®®©® 8®®©® 14®®©® 20®®©® 26®®©® 

3®®©® 9®®©® 15®®©® 21®®©® 27®®©® 

4®®©® 10®®©® 16®®©® 22®®©® 28®®©® 

5®®©® 11®®©® 17®®©® 23®®©® 29®®©® 

6®®©® 12®@©@ 18®®©® 24®®©® 30®®©® 



Subtest 3: Word Knowledge 














1 ®@©@ 8 ®@©® 


15 


®@©® 


22 


®@©® 


29 


®@©@ 


2 ®@©® 9 ®®©® 


16 


®@©@ 


23 


®®©@ 


30 


®@©@ 


3®®©® 10®®©® 


17 


®®©® 


24 


®®@® 


31 


®®©® 


4®®©® 11®@©@ 


18 


®@©@ 


25 


®®@® 


32 


®®©@ 


5®®©® 12®@©@ 


19 


®@©@ 


26 


®®©® 


33 


®®©® 


6®®©® 13®@©@ 


20 


®@©® 


27 


®®@® 


34 


®®©® 


7®®©® 14®®©® 


21 


®@©® 


28 


®®©@ 


35 


®®©@ 


Subtest 4: Paragraph Comprehension 














1 ®@©® 4 ®@©® 


7 


®@©® 


10 


®®©® 


13 


®®©® 


2 ®®©® 5 ®®©® 


8 


®®©® 


11 


®®©@ 


14 


®®©® 


3 ®@©® 6 ®@©@ 


9 


®®©@ 


12 


®®©® 


15 


®@©® 


Subtest 5: Mathematics Knowledge 














1 @®©@ 6 ®®©® 


11 


®@©® 


16 


®@©® 


21 


®@©® 


2 ®@©@ 7 ®®©® 


12 


®®©® 


17 


®®©@ 


22 


®®©@ 


3 ®®©® 8 ®®©® 


13 


®®©® 


18 


®®©@ 


23 


®®©® 


4 ®®@® 9 ®@©@ 


14 


®@©@ 


19 


®®@® 


24 


®®©@ 


5®®©® 10®®©® 


15 


®@©® 


20 


®®©® 


25 


®®©® 


Subtest 6: Electronics Information 














1 ®®©® 5 ®@©® 


9 


®@©® 


13 


®®©® 


17 


®®©® 


2 ®@©@ 6 ®®©® 


10 


®®@® 


14 


®®©@ 


18 


®@©@ 


3 ®@©® 7 ®®©® 


11 


®®©@ 


15 


®®©@ 


19 


®®©@ 


4 ®®©® 8 ®@©® 


12 


®@©® 


16 


®®©® 


20 


®®@® 


Subtest 7: Auto & Shop Information 














1 ®®©@ 6 ®@©® 


11 


®@©® 


16 


®®©® 


21 


®@©@ 


2 ®@©@ 7 ®®©® 


12 


®®@® 


17 


®®©@ 


22 


®@©@ 


3 ®@©® 8 ®®©® 


13 


®®©® 


18 


®®©® 


23 


®®©® 


4 ®®©® 9 ®@©@ 


14 


®@©® 


19 


®®©@ 


24 


®®©@ 


5®®©® 10®®©® 


15 


®®©® 


20 


®®@® 


25 


®®©® 



Subtest 8: Mechanical Comprehension 

1®®©® 6®®©® 11®®©® 16®®©® 21®®©® 

2®®©® 7®®©® 12®®©® 17®®©® 22®®©® 

3®®©® 8®®©® 13®®©® 18®®©® 23®®©® 

4®®©® 9®®©® 14®®©® 19®®©® 24®®©® 

5®®©® 10®@©@ 15®®©® 20®®©® 25®®©® 

Subtest 9: Assembling Objects 

1®@©@ 6®®©® 11®®©® 16®®©® 21®@©@ 

2®®©® 7®®©® 12®®©® 17®®©® 22®®©® 

3®®©® 8®®©® 13®®©® 18®®©® 23®®©® 

4®®©® 9®®©® 14®®©® 19®®©® 24®®©® 

5®®©® 10®@©@ 15®®©® 20®®©® 25®®©® 



Chapter 20: Practice Exam 3 2 70 



Subtest 1: General Science 



Time: 11 minutes for 25 questions 

Directions: This test challenges your knowledge of general science principles usually covered in 
high school classes. Pick the best answer for each question and then mark the space on your 
answer sheet that corresponds to the question number and the letter indicating your choice. 



The moon completes a revolution around 
the Earth approximately every 

(A) 28 days. 

(B) 365 days. 

(C) 24 hours. 

(D) 7 days. 

Carcinogens are chemicals that cause 

(A) high blood pressure. 

(B) genome mutations. 

(C) blood clots. 

(D) diabetes. 

A Paramecium is 

(A) a one-celled organism. 

(B) algae. 

(C) bacteria. 

(D) a many-celled organism. 

What substance is essential for the func- 
tion of the thyroid gland? 

(A) potassium chloride (salt) 

(B) hemoglobin 

(C) calcium 

(D) iodine 

The brainstem controls 

(A) vision. 

(B) voluntary muscle movements. 

(C) your sense of balance. 

(D) some involuntary activities. 



10. 



Which element is the most abundant one in 
the atmosphere? 

(A) oxygen 

(B) nitrogen 

(C) helium 

(D) hydrogen 

Minerals are necessary for 

(A) respiration. 

(B) eliminating waste. 

(C) preventing night blindness. 

(D) metabolic function. 

What's the only metallic element found as a 
liquid at room temperature? 

(A) bromine 

(B) tellurium 

(C) mercury 

(D) silver 

Which of the following isn't a type of 
telescope? 

(A) reflecting 

(B) convexing 

(C) refracting 

(D) catadioptric 

A dekagram 

(A) is larger than a kilogram. 

(B) is smaller than a kilogram. 

(C) is the same as a kilogram. 

(D) doesn't exist. 



Go on to next page 



280 



Part V: Practice ASVAB Exams 



11. The aurora borealis can be seen only in the 

(A) winter. 

(B) summer. 

(C) Southern Hemisphere. 

(D) Northern Hemisphere. 

12. The three important properties of sound 
waves are 

(A) wavelength, speed, and crest. 

(B) speed, frequency, and reflection. 

(C) wavelength, frequency, and vibration. 

(D) wavelength, frequency, and speed. 

13. Between which two planets can most of the 
asteroids in the solar system be found? 

(A) Mars and Jupiter 

(B) Saturn and Jupiter 

(C) Earth and Mars 

(D) Mercury and Venus 

14. At room temperature, an element is a 

(A) gas. 

(B) liquid or gas. 

(C) gas or solid. 

(D) liquid, gas, or solid. 

15. The elements hydrogen and helium com- 
prise what percentage of almost all matter 
in the universe? 

(A) 75% 

(B) 82% 

(C) 90% 

(D) 98% 

16. Compounds are created when 

(A) atoms of two or more like elements are 
combined. 

(B) atoms of two or more different ele- 
ments are combined. 

(C) two or more molecules are combined. 

(D) a molecule decomposes. 



17. What theory suggests the universe will 
come to an end when its ever-increasing 
rate of expansion causes all matter to fly 
apart? 

(A) The Big Rip 

(B) The Big Bang 

(C) The Big Crunch 

(D) The Big Easy 

18. A watt-hour measures 

(A) how much electricity is converted. 

(B) the number of electrons moving past a 
specific point. 

(C) resistance. 

(D) voltage. 

19. Which of the following planets, known as 
gas giants, have no rings? 

(A) Neptune 

(B) Jupiter 

(C) Uranus 

(D) They all have rings. 

20. Gas particles move 

(A) more slowly than liquid particles. 

(B) more slowly than solid particles. 

(C) more quickly than liquid particles. 

(D) at the same rate as all other particles. 

21. Absolute zero is 

(A) degrees Fahrenheit. 

(B) degrees Celsius. 

(C) -273 degrees Celsius. 

(D) -32 degrees Fahrenheit. 

22. Radiology is employed when doing which 
of the following? 

(A) using a Magnetic Resonance Imaging 
machine 

(B) using a blood pressure cuff 

(C) blood typing 

(D) breathing 



Go on to next page 



Chapter 20: Practice Exam 3 28 1 



23. Which of the following statements is NOT 
true? 

(A) The human female chin is usually more 
rounded or pointed than the human 
male chin. 

(B) The human female pelvis is usually nar- 
rower than the human male pelvis. 

(C) The human male skull is usually larger 
than the human female skull. 

(D) The human male skull has a larger 
brow ridge than the human female 
skull. 

24. A lunar eclipse occurs when 

(A) the Earth moves into the moon's 
shadow. 

(B) the sun blocks the moon from view. 

(C) the Earth moves into the sun's shadow. 

(D) the moon moves into the Earth's 
shadow. 



25. What chemical can be used to detect 
blood, even if it's been wiped from a 
surface? 

(A) luminol 

(B) cyanide 

(C) ninhydrin 

(D) alcohol 



STOPI 



DO NOTTURNTHE PAGE UNTILTOLDTO DO SO. 
DO NOT RETURN TO A PREVIOUS TEST. 



282 



Part V: Practice ASVAB Exams 



Subtest 2: Arithmetic Reasoning 



Time: 36 minutes for 30 questions 

Directions: This test is about arithmetic. Each question is followed by four possible answers. 
Decide which answer is correct and then mark the space on your answer sheet that has the same 
number and letter as your choice. Calculators are not permitted. Use scratch paper for any figuring 
you need to do. 



4. 



A baker sells a dozen donuts for $3.99. The 
cost to make three donuts is $0.45. How 
much is the total profit on 5 dozen donuts? 

(A) $17.70 

(B) $13.20 

(C) $2.19 

(D) $10.95 

Your piggy bank contains $19.75 in dimes 
and quarters. There are 100 coins in all. 
How many dimes are there? 

(A) 25 

(B) 30 

(C) 35 
(D)40 

A bricklayer charges $8 per square foot to 
lay a patio. How much would it cost for the 
bricklayer to lay a 12-foot by 16-foot patio? 

(A) $960 

(B) $192 

(C) $224 

(D) $1,536 

Terry earns three times more per hour 
than Tim. Tim earns $2 more per hour than 
Angie. As a group, they earn $43 per hour. 
What's Angie's hourly wage? 

(A) $7.00 

(B) $8.00 

(C) $9.00 

(D) $10.00 



7. 



If four people can run eight machines, how 
many machines can two people run? 

(A) 2 

(B)4 

(C)l 

(D)3 

The price of daily admission at an amuse- 
ment park is $36. The park sells an unlim- 
ited season pass for $240. How many trips 
would you need to make with the season 
pass in order for it to cost less than paying 
the daily admission rate? 

(A) 6 

(B)7 

(C)8 

(D)9 

A plumber needs four lengths of pipe, each 
3 feet, 6 inches long. Pipes are sold by the 
foot. How many feet does he need to buy? 

(A) 15 

(B)16 

(C)14 

(D)12 

The product of two consecutive odd num- 
bers is 399. What are the numbers? 

(A) 17 and 19 

(B) 19 and 21 

(C) 21 and 23 

(D) 25 and 27 



Go on to next page 



Chapter 20: Practice Exam 3 283 



9. A personal trainer earns a 65% commission 
on her training sales. If she sells $530 worth 
of training, how much commission does 
she make? 

(A) $874.50 

(B) $34.45 

(C) $344.50 

(D) $185.50 

10. A rectangle is 1 inch longer than it is wide. 
Its diagonal is 5 inches. What's the width of 
the rectangle? 

(A) 2 inches 

(B) 3 inches 

(C) 4 inches 

(D) 5 inches 

11. A treasure map is drawn to a scale of 

2 inches equals 3 miles. On the map, the 
distance between Point A and X-marks-the- 
spot is 9-k inches. How many actual miles 
does this represent? 

(A) 20 i miles 

(B) 14-| miles 

(C) 6^ miles 

(D) 19 miles 

12. A painter has painted a picture on a piece 
of canvas that measures 10 by 14 inches. 
To accommodate a frame, he has left an 
unpainted margin of 1 inch all the way 
around. What part of the canvas has been 
painted? 

(A) 96% 

(B) 91% 

(C) 65% 

(D) 69% 

13. A dog trainer is building a rectangular dog 
run that measures 9 by 16 feet. If she wants 
to fence the perimeter of the run, how 
many feet of chain link fence will she need? 

(A) 144 feet 

(B) 25 feet 

(C) 32 feet 

(D) 50 feet 



14. 



15. 



16. 



17. 



A rectangle is li times as long as it is wide. 

The perimeter of the rectangle is 100 inches. 
What's the length of the rectangle? 

(A) 20 inches 

(B) 30 inches 

(C) 40 inches 

(D) 45 inches 

Miguel passed seven of his history quizzes 
and failed three. The fraction of quizzes he 
passed is correctly expressed as 

(A) J 

0)4 



(Q 

(D) 



10 



A 3-yard-long ribbon was used to trim four 
dresses. Each dress used the same amount 
of ribbon. How much ribbon was used for 
each dress? 

(A) 1 yard 

(B) | yard 

(C)iyard 
(D) | yard 

Kelly bought a painting at an antiques sale 
for $500 and the following day she was able 
to sell it for an additional $30. What per- 
centage of the sale price was her profit? 

(A) 5% 

(B) 6% 

(C) 7% 
(D)4% 



18. A bin of bolts at the hardware store con- 
tains 7 dozen bolts when full. The stock 
clerk is supposed to reorder bolts when 

the bin is 4 full. How many bolts are in the 

bin when it's time to reorder? 

(A) 14 bolts 

(B) 1 bolt 

(C) 84 bolts 

(D) 12 bolts 



Go on to next page 



28b 



Part V: Practice ASVAB Exams 



19. Two bicyclists head toward each other 
from the opposite ends of Main Street, 
which is 6 miles long. The first biker 
started at 2:05 going 12 mph. The second 
biker began peddling 4 minutes later at a 
rate of 14 mph. What time will they meet? 

(A) 2:13 

(B) 2:24 

(C) 2:21 

(D) 2:34 

20. A recipe calls for 8 ounces of black beans 
or red beans. The cheapest option to buy 
and use would be 

(A) two 4-ounce cans of black beans at 
$0.79 each. 

(B) one 8-ounce can of red beans at $1.49. 

(C) two 3-ounce cans of black beans at 
$0.59 each. 

(D) three 3-ounce cans of red beans at 
$0.65 each. 

21. A street vendor sells $25.70 worth of pret- 
zels on Friday, $32.30 on Saturday, and 
$31.80 on Sunday. He spends a fourth of the 
money over the weekend. How much 
money does he have left? 

(A) $89.80 

(B) $22.45 

(C) $44.90 

(D) $67.35 

22. A recruit has $30.00. He saw some camou- 
flage socks for $3.95 a pair. How many pairs 
of socks can he buy? 

(A) 9 

(B)7 

(C)6 

(D)4 

23. A crate containing a puppy weighs 

60 pounds, 5 ounces. The puppy weighs 
43 pounds, 7 ounces. How much does 
the crate alone weigh? 

(A) 16 pounds, 8 ounces 

(B) 16 pounds, 2 ounces 

(C) 17 pounds 

(D) 16 pounds, 14 ounces 



24. 



25. 



26. 



27. 



28. 



In a manufacturing plant that produces new 
computers, a 0.15 probability exists that a 
computer will be defective. If five comput- 
ers are manufactured, what's the probabil- 
ity that all of them will be defective? 

(A) 7.6 

(B) 0.60 

(C) 0.00042 

(D) 0.000076 

A house contains one 12-foot x 14-foot bed- 
room, one 12-foot x 10-foot bedroom, and 
one 8-foot x 12-foot bedroom. What's the 
total amount of carpeting needed to carpet 
all three bedrooms? 



(A) 383 square yards 

(B) 128 square yards 

(C) 88 square yards 

(D) 43 square yards 



Rafael can type 9 pages an hour. How long 
will it take him to type 126 pages? 

(A) 14 hours 

(B) 9 hours 

(C) 7 hours 

(D) 16 hours 

In a 60-minute gym class, 48 girls want to 
play volleyball, but only 12 can play at a 
time. For each player to get the same 
amount of playing time, how many minutes 
should each person play? 

(A) 1^ minutes 

(B) 6 minutes 

(C) 30 minutes 

(D) 15 minutes 

The movie-rental store charges $2.00 for 
the first day a rented DVD is overdue and 
$1.25 for each day after that. If a person 
paid $8.25 in late fees, how many days was 
the DVD overdue? 

(A) 7 days 

(B) 6 days 

(C) 4 days 

(D) 5 days 



Go on to next page 



Chapter 20: Practice Exam 3 285 



29. Janet is trying to watch her weight. A half- 
cup of pudding has 150 calories. The same 
amount of broccoli has 60 calories. How 
much broccoli can Janet eat to equal the 

same number of calories in the i cup of 
pudding? 

(A) 2 cups 

(B) 2^ cups 

(Ql^cups 
(0)1-1 cups 



30. The neighbor's dog barks at a raccoon 
every 15 minutes at night. If he first barks 
at 10 p.m., when you're trying to fall asleep, 
how many times will he have barked by 
2 a.m., when you give up trying to sleep 
and decide to read a book instead? 

(A) 16 times 

(B) 132 times 

(C) 17 times 

(D) 15 times 



STOPI 



DO NOT TURN THE PAGE UNTILTOLDTO DO SO. 
DO NOT RETURN TO A PREVIOUS TEST. 



286 



Part V: Practice ASVAB Exams 



Subtest 3: Word Knortiedqe 



Time: 11 minutes for 35 questions 

Directions: This test is about the meanings of words. Each question has an underlined word. You 
may be asked to decide which one of the four words in the choices most nearly means the same 
thing as the underlined word or which one of the four words means the opposite. If the underlined 
word is used in a sentence, decide which of the four choices most nearly means the same thing as 
the underlined word as used in the context of the sentence. Mark the corresponding space on your 
answer sheet. 



1. Lackadaisical most nearly means 6. 

(A) flowerless. 

(B) listless. 

(C) promiscuous. 

(D) suitable. 

2. The fruit was edible . 7. 

(A) waxy 

(B) expensive 

(C) foreign 

(D) digestible 

3. Universities and colleges should be 

designed to cater to the philomaths . 8. 

(A) athletes 

(B) scholars 

(C) teachers 

(D) faculty 

4. Pretense most nearly means 9. 

(A) politeness. 

(B) dishonesty. 

(C) stress. 

(D) appearance. 

5. At an early age Jane showed a proclivity for 10. 
music and dancing. 

(A) predisposition 

(B) interest 

(C) dislike 

(D) fever 



Her conversation was incoherent . 

(A) eloquent 

(B) succinct 

(C) unintelligible 

(D) amusing 

The week following Joe DiMaggio's death 
was filled with often mawkish eulogies. 

(A) long 

(B) sentimental 

(C) boring 

(D) detailed 

She established proof. 

(A) offered 

(B) invented 

(C) demanded 

(D) demonstrated 

Ephemeral most nearly means 

(A) short-lived. 

(B) mythical. 

(C) dead. 

(D) exceptional. 

Avocation most nearly means 

(A) hobby. 

(B) occupation. 

(C) vacation. 

(D) education. 



Go on to next page 



Chapter 20: Practice Exam 3 28 7 



11. Kvetch most nearly means 

(A) assert. 

(B) yell. 

(C) complain. 

(D) argue. 

12. Her eyesight was acute . 

(A) sharp 

(B) poor 

(C) unusual 

(D) tested 

13. Inamorata most nearly means 

(A) boyfriend. 

(B) mistress. 

(C) best friend. 

(D) acquaintance. 

14. Her thoughts on the matter were 
inconsequential . 

(A) profound 

(B) disturbing 

(C) irrelevant 

(D) confused 

15. Debouch most nearly means 

(A) emerge. 

(B) fight. 

(C) relax. 

(D) capture. 

16. He was an amateur astronomer. 

(A) veteran 

(B) novice 

(C) interested 

(D) pleased 

17. She had no idea how to react to her ludic 
boyfriend. 

(A) playful 

(B) cheating 

(C) crazy 

(D) lazy 



18. The rose was crimson . 

(A) blooming 

(B) colorful 

(C) fragrant 

(D) red 

19. The word most opposite in meaning to 
benison is 

(A) theft. 

(B) replaceable. 

(C) curse. 

(D) heavy. 

20. She was exempt from gym class. 

(A) banned 

(B) excused 

(C) tired 

(D) refreshed 

21. The eldritch light of the desert can play 
tricks on your eyes. 

(A) bright 

(B) wavering 

(C) strange 

(D) yellow 

22. Defective most nearly means 

(A) flawed. 

(B) noticeable. 

(C) rare. 

(D) durable. 

23. Allot most nearly means 

(A) plow. 

(B) assign. 

(C) property. 

(D) test. 

24. The doctor gave the patient a cursory 
examination. 

(A) in-depth 

(B) painful 

(C) unnecessary 

(D) superficial 



Go on to next page 



288 



Part V: Practice ASVAB Exams 



25. Arcanum most nearly means 

(A) rare. 

(B) secret. 

(C) tangible. 

(D) false. 

26. Her answer was terse . 

(A) defensive 

(B) angry 

(C) lengthy 

(D) brief 

27. The dulcet songs of the band got the atten- 
tion of the audience. 

(A) harmonious 

(B) love 

(C) jazzy 
(D) loud 

28. He was arrested on a misdemeanor charge. 

(A) theft 

(B) serious 

(C) petty crime 

(D) bogus 

29. Embonpoint most nearly means 

(A) plumpness. 

(B) height. 

(C) quickness. 

(D) cold. 

30. He concocted a story about me. 

(A) told 

(B) rehearsed 

(C) invented 

(D) remembered 



31. He spent his days searching fruitlessly for 
that chimera , his true self. 

(A) personality 

(B) enigma 

(C) talent 

(D) monster 

32. Her former home was in Colorado. 

(A) previous 

(B) current 
(C) second 
(D) abandoned 

33. Mulct most nearly means 

(A) complain. 

(B) play. 

(C) work. 

(D) defraud. 

34. My voice is strident . 

(A) soft 

(B) melodious 

(C) harsh 

(D) baritone 

35. Raffish most nearly means 

(A) clean. 

(B) serene. 

(C) tawdry. 

(D) expensive. 



STOPI 



DO NOTTURNTHE PAGE UNTILTOLDTO DO SO. 
DO NOT RETURN TO A PREVIOUS TEST. 



Chapter 20: Practice Exam 3 280 



Subtest %: Paragraph Comprehension 



Time: 13 minutes for 15 questions 

Directions: This test measures your ability to understand what you read. This section includes 
paragraphs of reading material followed by incomplete statements or questions. Read each para- 
graph and select the choice that best completes the statement or answers the question. Mark your 
choice on your answer sheet by using the correct letter with each question number. 



Because leadership is charged with bringing 
new ideas, methods, or solutions into use, inno- 
vation is inextricably connected with the pro- 
cess of being an effective leader. Innovation 
means change, and change requires leadership. 
Leaders must be the chief transformation offi- 
cers in their organizations and learn everything 
there is to know about the change before it even 
takes place. Furthermore, they must learn how 
to deal with the emotions that result from the 
chaos and fear associated with change. 

1. According to the passage, 

(A) leaders should resist making changes 
that subordinates are likely to resist. 

(B) innovation and change are distinctly 
different processes. 

(C) it's not necessary for the leader to 
know everything about a change before 
it's implemented. 

(D) change is often associated with panic 
and disorder. 

Cougars are the most wide-ranging big cats 
in North America, inhabiting a wide variety of 
environments. A cougar, also called a puma or a 
mountain lion, lives about 18 years in the wild, 
can jump 20 feet (in distance) at a time, and can 
range 50 miles when on the prowl for food. 

2. According to this passage, 

(A) a cougar isn't the same thing as a 
mountain lion. 

(B) cougars are an endangered species. 

(C) cougars live in many areas of North 
America. 

(D) cougars live only a few years in the 
wild. 



A helping relationship refers to interactions 
in which the counselor makes a determined effort 
to contribute in a positive way to the counselee's 
improvement. In counseling, the counselor estab- 
lishes a helping relationship by drawing on 
practices that help the counselee live more in 
harmony with himself or herself and others and 
with a greater self-understanding. The relation- 
ship develops because the counselee needs assis- 
tance, instruction, or understanding. 

3. Which of the following statements is NOT 
supported by the passage? 

(A) Successful counseling requires devel- 
oping a relationship. 

(B) Most counselees initially reject advice 
given by the counselor. 

(C) Counseling helps a counselee develop a 
greater understanding of him/herself. 

(D) Counseling relationships are developed 
by relying on helpful practices. 

Many small cities and towns rely on volunteer 
fire departments to put out fires. A professional 
fire department, however, has more training, 
more expertise, and more experience in fighting 
fires and investigating their causes. In many 
cases, it's worthwhile for even very small towns 
to hire professional firefighters. 

4. According to this passage, it's reasonable 
to assume that 

(A) volunteer firefighters have less train- 
ing, expertise, and experience than 
professional firefighters. 

(B) volunteer firefighters have the skills 
and resources to investigate the causes 
of fires. 

(C) professional firefighters don't know 
what causes fires. 

(D) a professional fire department is cost- 
prohibitive for small towns. 



Go on to next page 



290 



Part V: Practice ASVAB Exams 



The idea being an alarming one, he scrambled 
out of bed and groped his way to the window. He 
was obliged to rub the frost off with the sleeve of 
his dressing-gown before he could see anything 
and could see very little then. All he could make 
out was that it was still very foggy and extremely 
cold and that there was no noise of people run- 
ning to and fro and making a great stir, as there 
unquestionably would've been if night had beaten 
off bright day, and taken possession of the world. 

5. This story takes place 

(A) in Ireland. 

(B) on a calm summer evening. 

(C) on a winter night. 

(D) both A and C. 

Epidemiology is the study of what causes 
diseases, injuries, and other physiological 
damage to humans and why such problems 
occur. Epidemiologists examine where and when 
disease outbreaks occur. By using statistics and 
other scientific methods, epidemiologists deter- 
mine what factors affect the frequency and 
severity of disease patterns. The primary goal of 
epidemiology is to control or prevent outbreaks 
of disease — other goals are subordinate. 

6. What would be the best title for this passage? 

(A) "Epidemiology: The Study of Disease 
Patterns" 

(B) "Goals for the Future of Epidemiology" 

(C) "Using Statistical Methods in 
Epidemiology" 

(D) "Employment Outlook for 
Epidemiologists" 

Buddhism is a religion that must be viewed 
from many angles. Its original form, as preached 
by Gautama in India and developed in the early 
years succeeding and as embodied in the sacred 
literature of early Buddhism, isn't representative 
of the actual Buddhism of any land today. 

7. According to this passage, 

(A) most Buddhists live in India. 

(B) Buddhist teachings have changed over 
the years. 

(C) Buddhism draws its teachings from 
early Christianity. 

(D) Buddhist temples can be found in any 
land of the world. 



Questions 8 and 9 are based on the following 
passage. 



Many criminal-law statutes permit more 
severe punishment of a person convicted of a 
crime if he or she intended to harm another 
person. For example, voluntary manslaughter 
carries a heavier penalty than involuntary man- 
slaughter in most states. Planned crimes are also 
punished more severely than spur-of-the- 
moment crimes. 

The problem is that juries find it difficult to 
know what the intent of a person was at the time 
he or she committed a crime. Many defendants 
will deny that they intended to harm the other 
person and claim that any harm that occurred 
was "accidental." The law asks too much of 
juries when it expects them to determine what a 
person was thinking. Juries should only be asked 
to weigh objective evidence. 

8. The author of this passage would agree that 

(A) laws should not punish people based 
on intention. 

(B) juries aren't intelligent enough to weigh 
evidence. 

(C) more laws should distinguish between 
crimes committed with intent and crimes 
committed on the spur of the moment. 

(D) lawyers will lie about anything. 

9. According to this passage, 

(A) most states don't distinguish between 
voluntary and involuntary manslaughter. 

(B) punishing people more severely for vol- 
untary manslaughter is unconstitutional. 

(C) it's difficult for juries to determine a 
defendant's intentions at the time a 
crime was committed. 

(D) prosecutors can, through careful ques- 
tioning, show a defendant's intention at 
the time a crime was committed. 



Questions 10 through 12 are based on the fol- 
lowing passage. 



Ergonomics is the science of designing and 
arranging workspaces so that people and objects 
interact efficiently and safely. Lack of attention 
to ergonomics causes thousands of workers to 



Go on to next page 



Chapter 20: Practice Exam 3 2 ^ / 



suffer repetitive stress injury, eye fatigue, muscle 
soreness, and many other medical problems 
each year. 

Adequate lighting, well-designed chairs, and 
clutter-free work areas contribute to effective 
ergonomic design. The opportunity to take short 
breaks every hour or two, especially for desk- 
bound workers, is also helpful. It's also impor- 
tant for workers to avoid performing the same 
movements over and over for hours at a time. 
Variety in the type of work being done can 
decrease the chance of injury. 

10. According to this passage, 

(A) ergonomics can cause injuries. 

(B) ergonomics is about designing and 
arranging workspaces efficiently and 
safely. 

(C) ergonomics is expensive and time- 
consuming. 

(D) few people experience problems due to 
poor ergonomics. 

11. According to this passage, 

(A) adequate lighting and well-designed 
chairs, although important, have noth- 
ing to do with ergonomics. 

(B) repetition in the type of work people 
do helps them accomplish their tasks 
safely and efficiently. 

(C) short breaks aren't important for desk- 
bound employees because they do 
little heavy labor. 

(D) ergonomic design also includes keeping 
work areas well-lit and clutter-free. 

12. According to this passage, it's reasonable 
to assume that 

(A) employers should invest in ergonomic 
design to protect workers. 

(B) lack of ergonomic design isn't 

dangerous. 

(C) labor unions have opposed ergonomic 
design. 

(D) poor design is responsible for most 
employee accidents. 



Questions 13 through 15 are based on the fol- 
lowing passage. 



Electricity is the most inefficient and costly 
way to heat a home. One kilowatt-hour of elec- 
tricity creates about 3,400 British thermal units 
(BTUs). (BTUs are a standard heat measure- 
ment.) The price of electricity per kilowatt-hour 
is between $0.10 and $0.25 or between $29.35 
and $73.13 per million BTUs. 

In contrast, fuel oil, which produces 140,000 
BTUs per gallon, costs about $8.33 to $13.89 per 
million BTUs. Natural gas, which produces 
100,000 BTUs per therm, can be purchased for 
$5.00 to $22.50 per million BTUs. Oak firewood, 
which produces 26,000,000 BTUs per cord, costs 
$5.77 to $13.46 per million BTUs. 

Choosing the right heating method for your 
home, based on the cost of fuel, may be more 
expensive at installation but will be cheaper in 
the long run. 

13. According to the passage, a BTU 

(A) is an unusual method of measuring 
heat. 

(B) stands for "British thermal unit." 

(C) is the abbreviation for a "big thermal 
unit." 

(D) can heat a 9 x 12 room. 

14. According to the passage, 

(A) heating with fuel oil is always cheaper 
than other methods. 

(B) oak firewood produces fewer BTUs per 
dollar than the other types of fuel. 

(C) natural gas costs more than all other 
fuels except oak firewood. 

(D) electricity is always the most expensive 
way to heat a house. 

15. The title of this passage should be 

(A) "Choosing the Right Heating Method" 

(B) "Heating Methods for Houses" 

(C) "Know Your BTUs" 

(D) "Price List for Fuel" 



STOPI 



DO NOT TURN THE PAGE UNTIL TOLD TO DO SO. 
DO NOT RETURN TO A PREVIOUS TEST. 



292 



Part V: Practice ASVAB Exams 



Subtest 5: Mathematics Knowledge 



Time: 24 minutes for 25 questions 

Directions: This section tests your ability to solve general mathematical problems. Select the cor- 
rect answer from the choices given and then mark the corresponding space on your answer sheet. 
Use scratch paper to do any figuring you want. 



5. 



If y = 6, then 2y x y 




6. 


x(x 2 ) = 


(A) 12 






(A)x 2 


(B) 72 






(B) 2x 


(C)18 






(C) 2x 2 


(D) 242 


squals 


7. 


(D)x 3 


If 0.05x = 1, then .re 


J(5 + x) 2 = 


< A >A 






(A) 5-x 


(B) 20 
(Q10 
(D)5 






(B) 5 + x 
(C)V5-^x" 






(D)^ + >/x 


^25x 2 = 




8. 


(3 x 3)(5 - 3)(6 + 2) = x 2 . What's the value 
of X? 


(A)x 






(A) 6 


(B)x 2 






(B)12 


(C) 5x 






(C) 144 


(D) -5x 2 






(D)64 


Factor: 9X 3 + 18x 2 - 


x-2 


9. 


If -5x = 25, x equals 


(A) (9x 2 - l)(x + 2) 






(A) -5 


(B) (9x 2 + l)(x-2) 






(B)5 


(C) (9x 2 + 2)(x-l) 






(C)10 


(D) (9x 2 - 2)(x + 1) 






(D)0 


Solve for x: 5x + 7 = 


6(x-2)-4(2x-3) 


10. 


A circle measures 12 feet in diameter. 


(A)l 






What's its area to the nearest foot? 


(B)-l 






(A) 452 


(C)2 






(B) 24 


(D)-2 






(C) 113 
(D)48 



Go on to next page 



Chapter 20: Practice Exam 3 2 ^? 



n. 



12. 



13. 



14. 



15. 



A square box has 6-inch sides. What's its 
volume? 

(A) 18 cubic inches 

(B) 216 cubic inches 

(C) 12 cubic inches 



16. In the following figure, the sum of Angles 1 
and 2 equals 




(D) 36 cubic inches 












A circle has a diameter of 10 inches 


What's 




(A) 180 degrees 

(B) 90 degrees 

(C) 45 degrees 

(D) 360 degrees 


its approximate area? 

(A) 7t(10 2 ) 

(B) tc(25) 








(C) 7i(5) 






17. 


Solve for x: 3(2x - 5) - 2(4* + 1) = -5(x + 3) - 


(D) ti(10 2 )(10) 








(A)0 


A cylinder has a diameter of 12 inches and 




0)1 


a height of 10 inches. What's its approxi- 




(C)2 


mate volume? 








(D)3 


(A) 4,521 cubic inches 

(B) 120 cubic inches 






18. 


A cube has a volume of 64 cubic inches. 
What's the length of one side of the cube? 


(C) 1,130 cubic inches 








(A) 4 inches. 


(D) 1,440 cubic inches 








(B) 16 inches. 


Triangle ABC is a(n) 








(C) 8 inches. 

(D) 32 inches. 


B 






19. 


(x 3 )^ 


c / \ a 








(A)* 5 
(B)x 6 


A 1 2 ) 








(C)x 9 


A b 


^C 






(D) 2x 3 


(A) equilateral triangle. 

(B) right triangle. 

(C) scalene triangle. 

(D) isosceles triangle. 






20. 


If i inches of rain fall in one minute, how 
many inches fall in h hours? 

(A) ih * 60 

(B) 60/ - h 

(C) ih 


The angles of the following quadrilateral 




(D) 60/ft 


A 






B 




21. 


If x = y, then 6 + 4(x - y) = 




J 


L 




(A) 6xy + 4 














(B) 6 + 4xy 




~l 


r 








(C) lOx-lOy 
(D)6 


D 






c 



(A) are all right angles. 

(B) each equal 45 degrees. 

(C) are all unequal. 

(D) total 180 degrees. 



Go on to next page 



2% 



Part V: Practice ASVAB Exams 



22. ^820 is a number between 

(A) 20 and 30. 

(B) 10 and 20. 

(C) 80 and 90. 

(D) 40 and 50. 

23. (x + 2)(x + 2) = 

(A) x 2 + 2x + 4 

(B) x 2 + 4x + 4 

(C) x 2 + 4x + 2 

(D) x 2 + 2x + 



24. 



25. 



Evaluate the expression 6a - 3x - 2y if 

a = -3, x = -7, and y = 4. 



(A) -5 






(B) -40 






(CD 31 






(D)40 






(x + 4)(3. 


r + 5) 


= 


(A) 3x 2 + 


9x + 


20 


(B) 3x 2 + 


17x- 


H 15 


(C) 3x 2 + 


17x- 


f20 


(D)3x2 + 


9x + 


20 



STOP! 



DO NOTTURNTHE PAGE UNTILTOLDTO DO SO. 
DO NOT RETURN TO A PREVIOUS TEST. 



Chapter 20: Practice Exam 3 2 ty5 



Subtest 6: Electronics Information 



Time: 9 minutes for 20 questions 

Directions: This section tests your knowledge of electrical, radio, and electronics information. 
Select the correct response from the choices given and then mark the corresponding space on your 
answer sheet. 



1. What effect does a speaker wire's gauge 
have on speaker sound quality? 

(A) higher gauge wires are thicker with 
better sound quality 

(B) lower gauge wires are thicker with 
better sound quality 

(C) lower gauge wires are thicker with 
lesser sound quality 

(D) higher gauge wires are thicker with 
lesser sound quality 

2. What's the primary advantage of a quad- 
band cell phone over a dual-band cell 
phone? 

(A) transmission strength 

(B) coverage area 

(C) reception strength 

(D) smaller phone size 

3. When working with electricity, you should 
assume that all electrical equipment is 
alive unless you know for certain other- 
wise. This prevents 

(A) damage to circuits. 

(B) personal injury. 

(C) unnecessary labor. 

(D) overheating the equipment. 

4. The heat effect of current occurs 

(A) when the pressure of the current in the 
wire breaks up impurities in the wire, 
creating heat. 

(B) when the current in the wire decays 
electrons, causing them to move more 
quickly, creating heat. 

(C) when the current overcomes resistance 
in the wire, creating heat. 

(D) The heat effect of current is only theo- 
retical; it has never been proven 

to exist. 



5. What special type of diode is commonly 
used to regulate voltage? 

(A) capacitor 

(B) transistor 

(C) Zener 

(D) LED 

6. This symbol means 



(A) ohm. 

(B) ampere. 

(C) high voltage. 

(D) wattage. 

7. Electromotive force is another way of 
saying 

(A) frequency. 

(B) watts. 

(C) cycles per second. 

(D) voltage. 

8. A primary advantage of using a Li-Ion bat- 
tery instead of a NiMH battery in your cell 
phone is 

(A) Li-Ion batteries are lighter. 

(B) Li-Ion batteries last longer. 

(C) Li-Ion batteries don't interfere with 
signal quality. 

(D) none of the above. 



Go on to next page 



296 



Part V: Practice ASVAB Exams 



9. Transistors contain at least three terminals 
called the 

(A) base, emitter, and collector. 

(B) base, positive terminal, and negative 
terminal. 

(C) emitter, amplifier, and collector. 

(D) base and two gates. 

10. To control a light fixture from two different 
wall switches, you should use 

(A) a single-pole switch and a four-way 
switch. 

(B) two three-way switches. 

(C) two four-way switches. 

(D) two single-pole switches. 

11. A transistor is also called a(n) 

(A) rectifier. 

(B) cathode. 

(C) amplifier. 

(D) semiconductor. 

12. This symbol means 



(A) ground. 

(B) resistor. 

(C) diode. 

(D) battery. 

13. To decrease capacitance, capacitors 

(A) should have less voltage applied to them. 

(B) should be connected in parallel. 

(C) should be connected in series. 

(D) should be eliminated. 

14. A resistor marked 2.5K ohms has the 
value of 

(A) 2.5 ohms. 

(B) 250 watts. 

(C) 2,500 ohms. 

(D) 25,000 ohms. 



15. A 9-volt transistor contains 

(A) 1 cell. 

(B) 6 cells. 

(C) 9 cells. 

(D) 3 cells. 

16. The hot wire is always 

(A) purple. 

(B) green. 

(C) whitish. 

(D) black. 

17. How wide is the full AT motherboard? 

(A) 11 inches 

(B) 11.5 inches 

(C) 12 inches 

(D) 12.5 inches 

18. The following symbol represents a(n) 



(A) relay. 

(B) on-off switch. 

(C) push switch. 

(D) connected wire. 

19. If a 120-volt current is protected by a 25-amp 
circuit breaker, what's the largest number of 
watts an appliance can safely use? 

(A) 1,200 watts 

(B) 1,800 watts 

(C) 3,000 watts 

(D) 3,600 watts 

20. The following symbol represents a 



A 



(A) rheostat. 

(B) capacitor. 

(C) relay. 

(D) potentiometer. 



STOPI 



DO NOTTURNTHE PAGE UNTILTOLDTO DO SO. 
DO NOT RETURN TO A PREVIOUS TEST. 



Chapter 20: Practice Exam 3 2 i) 7 



Subtest 7: Auto & Shop In formation 



Time: 11 minutes for 25 questions 

Directions: This test contains questions about automobiles, shop practices, and the use of tools. Pick 
the best answer for each question and then mark the corresponding space on your answer sheet. 



1. A symptom of worn piston rings is 

(A) a knocking and pinging sound when 
driving. 

(B) soft and spongy acceleration. 

(C) the smell of exhaust in the car. 

(D) an engine using excessive amounts 
of oil. 

2. What term refers to the rebuilding of an 
engine to precise factory specifications? 

(A) blueprinting 

(B) speccing 

(C) gold rebuild 

(D) silver rebuild 

3. The number of cranks a crankshaft has on 
a V-8 engine is 

(A) 6. 

(B)4. 

(0)3. 

(D)8. 

4. When an engine runs on after the ignition 
key is turned off, it's called 

(A) dieseling. 

(B) sputtering. 

(C) ignition recharge. 

(D) ignition malfunction. 

5. If a radiator fails, the engine 

(A) will idle roughly. 

(B) may burn fuel less efficiently. 

(C) works hard to maintain speed. 

(D) can quickly overheat. 



6. On modern automobile engines, what's the 
purpose of the intake manifold? 

(A) It regulates airflow to the cooling 
system. 

(B) It provide airflow to the air-conditioner 
and heater. 

(C) It connects the air/fuel management 
device to the head. 

(D) It regulates fuel pump pressure. 

7. Brake systems work by 

(A) applying friction to the wheels to stop 
their rotation. 

(B) reversing power to the wheels. 

(C) applying pressure to the axle. 

(D) interrupting power to the transmission. 

8. Which of the following isn't a component of 
the cooling system? 

(A) heater core 

(B) radiator 

(C) thermostat 

(D) hydrator 

9. A catalytic converter 

(A) combines the fuel-air mixture. 

(B) reduces dangerous exhaust emissions. 

(C) converts the up-and-down motion of 
the pistons to rotary motion. 

(D) charges the battery when the engine is 
in operation. 

10. If the steering wheel vibrates at high 
speeds, the most likely problem is 

(A) front end alignment. 

(B) front tire balance. 

(C) cracked steering column. 

(D) overinflated tires. 



Go on to next page 



298 



Part V: Practice ASVAB Exams 



11. During the compression stroke on a four- 
cycle engine, 

(A) the intake valve opens to fill the cylin- 
der with fuel. 

(B) the burning fuel mixture forces the 
piston to the bottom of the cylinder. 

(C) the intake valve closes, and the piston 
moves to the top of the cylinder. 

(D) the exhaust valve releases the 
burned gas. 

12. On older cars, the air filter can be found 

(A) on top of the engine. 

(B) under the engine. 

(C) behind the engine. 

(D) on the left or right side of the engine. 

13. Glazing is the process of 

(A) cutting glass to size. 

(B) using putty to hold glass to a window 
frame. 

(C) polishing glass before using. 

(D) removing glass from a window. 

14. A wrench with fixed, open jaws is called a(n) 

(A) adjustable wrench. 

(B) Allen wrench. 

(C) socket wrench. 

(D) open-end wrench. 

15. All hammers have a 

(A) head, face, and handle. 

(B) head, toe, and handle. 

(C) head and foot. 

(D) head and claw. 

16. To determine the number of threads per 
inch on a fastener, use a 

(A) depth gauge. 

(B) thread gauge. 

(C) thickness gauge. 

(D) wire gauge. 



17. To chip or cut wood in close, the best tool 
is a 

(A) screwdriver. 

(B) butt chisel. 

(C) framing chisel. 

(D) mortising chisel. 

18. Machine screws 

(A) are made by machines. 

(B) can be used interchangeably with wood 
screws. 

(C) fasten metal parts. 

(D) are machined to fine tolerances. 

19. Double-headed nails are used 

(A) to reinforce a joint. 

(B) on temporary construction. 

(C) to make frames for furniture. 

(D) when a larger striking surface is 
needed. 

20. To thin oil-based paint, use 

(A) turpentine. 

(B) baby oil. 

(C) benzene. 

(D) varnish. 

21. When finishing a piece of wood, it's best to 
sand 

(A) diagonal to the grain. 

(B) against the grain. 

(C) with the grain. 

(D) in small circles. 

22. To transfer an angle, the best tool to use is a 

(A) square. 

(B) caliper. 

(C) level. 

(D) sliding T-bevel. 



Go on to next page 



Chapter 20: Practice Exam 3 2>^ 



23. The following tool is a(n) 



25. The following tool is used to 




(A) pipe wrench. 

(B) socket wrench. 

(C) adjustable crescent wrench. 

(D) box-end wrench. 

24. Which of the following screw heads 
requires a Phillips screwdriver? 



(A) punch holes. 

(B) drive nails. 

(C) measure thickness. 

(D) set nails. 





(A) 



(C) 





(B) 



(D) 



STOPI 



DO NOTTURNTHE PAGE UNTILTOLDTO DO SO. 
DO NOT RETURN TO A PREVIOUS TEST. 



300 



Part V: Practice ASVAB Exams 



Subtest 8: Mechanical Comprehension 



Time: 19 minutes for 25 questions 

Directions: This test is about mechanical principles. Many of the questions use drawings to illus- 
trate specific principles. Choose the correct answer and mark the corresponding space on the 
answer sheet. 



Helical gears have 

(A) straight teeth. 

(B) slanted teeth. 

(C) teeth of unequal size. 

(D) no advantage over spur gears. 

In the following figure, which pillar sup- 
ports the greater load of the anvil? 





1» 




\ i \ 


1 -^ 1 


A 




B 



(A) Pillar A 

(B) Pillar B 

(C) Both pillars support the anvil equally. 

(D) It's impossible to determine from the 
information given. 



4. 



Wheel A has a diameter of 10 feet. Wheel B 
has a diameter of 8 feet. If both wheels 
revolve at the same rate, Wheel B will 
cover a linear distance of 16 feet 

(A) at the same time as Wheel A. 

(B) more slowly than Wheel A. 

(C) in twice the time as Wheel A. 

(D) faster than Wheel A. 

What effort must be used to lift a 30-pound 
anvil (see the following figure) using a first- 
class lever? (Don't include the weight of 
the lever in your calculations.) 




30 pound 
anvil 

(A) 10 pounds 

(B) 15 pounds 

(C) 50 pounds 

(D) 5 pounds 



Go on to next paqe 



Chapter 20: Practice Exam 3 *%01 



5. What mechanical advantage does the block- 
and-tackle arrangement in the following 
figure give? 




(A)l 
(B)3 
(C)2 
(D)4 

6. If a ramp is 8 feet long and 4 feet high, how 
much effort is required to move a 
400-pound object up the ramp? 

(A) 35 pounds 

(B) 150 pounds 

(C) 800 pounds 

(D) 200 pounds 

7. 33,000 foot-pounds of work done in one 
minute is called 

(A) a job for an enlisted soldier. 

(B) 1 horsepower. 

(C) 330 psi. 

(D) meaningful force. 

8. A 130-pound woman is wearing shoes with 
high heels that measure 1-inch square. If 
the woman is standing on one heel, what 
psi does the heel exert as it rests on the 
ground? (Disregard atmospheric pressure 
from your calculations.) 

(A) 130 

(B) 65 

(C) 260 
(D)ll 



9. Clothes from the dryer stick together 
because of 

(A) gravity. 

(B) magnetism. 

(C) friction. 

(D) static electricity. 

10. An aneroid barometer measures 

(A) atmospheric pressure. 

(B) water pressure. 

(C) hydraulic-fluid pressure. 

(D) the ambient temperature. 

11. If Gear A is revolving in a clockwise 
manner, as in the following figure, Gear B 



12. 




(A) remains stationary. 

(B) revolves in a clockwise manner. 

(C) revolves in a counterclockwise manner. 

(D) turns more slowly than Gear A. 

Springs are used for the following purposes 
EXCEPT 

(A) to store energy for part of a mechanical 
cycle. 

(B) to force a mechanical component 
to maintain contact with another 
component. 

(C) to reduce shock or impact. 

(D) to increase the weight of a mechanism. 



Go on to next page 



302 



Part V: Practice ASVAB Exams 



13. The floats in Tubes A and B measure spe- 
cific gravity. Which tube contains the liquid 
with the higher specific gravity? 



14. 



15. 



A B 

(A) Tube A 

(B) Tube B 

(C) It can't be determined. 

(D) Both Tube A and Tube B have the same 
specific gravity. 

Universal joints are used to 

(A) connect ball bearings. 

(B) fix two shafts so they don't pivot or 
rotate. 

(C) connect shafts in a U-shape. 

(D) couple two shafts set at different 
angles. 

The try-cock in the following schematic 
measures 



16. The steel plate below is held in place by 
different machine screws, each indicated 
by different symbols. How many different 
types of machine screws have been used? 





©@(D 



@0@@«o 



□ 








(A) 6 
(B)15 
(C)5 
(D)9 

17. The amount of force (F) needed to balance 
the lever in the following figure is most 
nearly 



10 pound 5 pound 
anvil anvil 

(A) 15 pounds. 

(B) 13 pounds. 

(C) 7.5 pounds. 

(D) 20 pounds. 




Water line 



try-cock 



(A) temperature of water. 

(B) pressure of water. 

(C) pressure of steam buildup. 

(D) level of water. 



Chapter 20: Practice Exam 3 3 Q 2 



18. With one complete revolution of the cable 
winch shown below, the load will move 

Drum circumference 24 inches 



20. 





(A) 12 inches. 

(B) 6 inches. 

(C) 24 inches. 

(D) 36 inches. 

19. In the following figure, assume the valves 
are all closed. Which valves need to be 
open to fill the tank entirely? 



Water 
Supply 



Valve 1 



Valve 3 



Valve 2 



Drainage 



Valve 4 



Valve 5 



(A) 1 and 2 only 

(B) 1 only 

(C) 1, 2, and 3 

(D) 2 only 



21. 



22. 



23. 



If Gear 1 in the following figure makes 
10 complete clockwise revolutions per 
minute, then 




(A) Gear 2 makes 2 clockwise revolutions 
per minute. 

(B) Gear 3 makes 8 clockwise revolutions 
per minute. 

(C) Gear 3 makes 30 clockwise revolutions 
per minute. 

(D) Gear 3 makes 9 counterclockwise revo- 
lutions per minute. 

A gear and pinion have a ratio of 1 to 4. If 
the gear makes 200 revolutions per minute, 
the speed of the pinion is 

(A) 50 rpm. 

(B) 800 rpm. 

(C) 400 rpm. 

(D) 200 rpm. 

The gas gauge in an automobile relies on 
what mechanical device to measure the 
amount of gas in the tank? 

(A) ball and cock 

(B) automatic valve 

(C) float 

(D) mechanical switch 

Using a runner gives you a mechanical 
advantage of 

(A) 4. 

(B)2. 

(C)3. 

(D)l. 



Go on to next page 



30U 



Part V: Practice ASVAB Exams 



24. For the valve shown in the figure below to 
open once each second, the cam must 
revolve at a rate of 



25. The following figure represents a water 
tank. Which of the following statements is 
NOT true? 



valve 



Water 
Supply 



XL 



carn- 



al 




Valve 1 



(A) 6 rpm. 

(B) 10 rpm. 

(C) 15 rpm. 

(D) 3 rpm. 



Valve 3 



Water level 



Drainage 



Valve 2 



Valve 4 Valve 5 



(A) If Valves 1 and 2 are open and Valves 3, 
4, and 5 are closed, the tank will even- 
tually overflow. 

(B) If all valves are open, the water will 
remain at a constant level as long as 
the rate of intake is equal to the rate of 
discharge. 

(C) Water in the tank will rise if Valves 1 
and 2 are open and Valves 3 and 4 are 
closed. 

(D) The tank will empty entirely if Valves 1 
and 2 are closed and Valves 4 and 5 are 
open. 



STOPI 



DO NOTTURNTHE PAGE UNTILTOLDTO DO SO. 
DO NOT RETURN TO A PREVIOUS TEST. 



Chapter 20: Practice Exam 3 3 Q£y 



Subtest 9: Assembling Objects 



Time: 15 minutes for 25 questions 

Directions: The Assembling Objects subtest consists of questions that measure your ability to men- 
tally picture items in two dimensions. Each question is comprised of five separate drawings. The 
problem is presented in the first drawing and the remaining four drawings are possible solutions. 
Determine which of the choices best solves the problem shown in the first picture, and then mark 
the corresponding choice on your answer sheet. 



0* 


X 


<r 


% 


% 



2. 



3. 



^h 



>=^| 









ft 



r^> 



p^ o o e> 



10. 



*A A 


% 


grQ 


X 


<^J 



11. 



m 


A 


A 


4> 


X 



12. 



A B C D 



/ B B 


X 


&-^ 


X 


(X 



13. 



A B C D 




14. 



A B C D 



> 






A B C D 



X 

aA 


V 


V 


V 


A 



A B C D 



a. 

Ob 




f 


i" 


\ 


H 



A B C D 



AH 


X 


it 


/ 


V 



A B C D 



■ — | A A 

r b b 


<^^t? 


% 


I 


1 



A B C D 



















A 


B 


C 


D 


V. 




A 


B 


% 

C 


D 


15. 


Q* j 

d- 1 


□^ 


X 


^> 


% 



A B C D 



o 

X 



3> 



16. 



Q?s x (ID £> 



A B C D 



Go on to next page 



3 06 Part V: Practice ASVAB Exams 



17. 








18. CV A A 



f> B 




19. 






20. 





^ 


N& 


4^ 


^7 



21. 









23. 



P/ 







Go on to netf jm#e 



Chapter 20: Practice Exam 3 $07 



24. 



^ 1 

0= B 


^ 


^ 


r° 


*\ 



25. 







STOPI 



DO NOTTURNTHE PAGE UNTILTOLDTO DO SO. 
DO NOT RETURN TO A PREVIOUS TEST. 



308 



Part V: Practice ASVAB Exams 



Chapter 21 

Practice Exam 3: 
Answers and Explanations 



M^ ead over each question from Chapter 20 as you check the answer key. I hope you did 
w ▼well on this practice exam. If you find you need to study more for any subtest, follow 
the cross-references. (And for in-depth study, check out some of the book recommenda- 
tions in Chapter 17.) 



Subtest 1: General Science Answers 

If you're still having problems figuring out the difference between an isotope and an ion, 
remember you may not have to do well on this subtest. It depends on the military career 
you're interested in. (See Appendix A for a list of military jobs that require a competent 
General Science score.) If this subtest is important to your military career aspirations, con- 
sider putting in some extra study. You can find additional information in Chapter 10, and 
Chapter 15 has a few more practice questions. 

l.A 6. B 11. D 16. B 21. C 



2. B 


7. D 


12. D 


17. A 


22. A 


3. A 


8. C 


13. A 


18. A 


23. B 


4.D 


9. B 


14. D 


19. D 


24. D 


5. D 


10. B 


15. D 


20. C 


25. A 



Subtest 2: Arithmetic Reasoning Answers 

You have to do well on this subtest to qualify for military enlistment — your score from the 
Arithmetic Reasoning subtest counts toward your AFQT score. If you're still doing poorly on 
this test, you may want to postpone taking the ASVAB until you have more study time under 
your belt (and perhaps take a math class or two). You may also want to review Chapters 7 
and 8 and the practice questions in Chapter 9. 

1. D. Multiply $0.45 (the cost of making three donuts) by 4 to find the cost of making a dozen 
donuts: $0.45 x 4 = $1.80. Then subtract the cost of making one dozen donuts from the sell- 
ing price of one dozen donuts to get the profit on one dozen donuts: $3.99 - $1.80 = $2.19. 

Because the baker sold five dozen donuts, multiply the profit on one dozen donuts times 5 
to determine the profit on five dozen donuts: $2.19 x 5 = $10.95. 



310 



Part V: Practice ASVAB Exams 



2. C. Let x equal the number of dimes. Then 100 - x represents the number of quarters. You 
have $0.10x in dimes and $0.25(100 - x~) in quarters, so set up your equation and solve for x: 

0.10x + 0.25(100 -x) = 19.75 

0.10x+25-0.25x = 19.75 

-0.15* = -5.25 

x = 35 

3. D. First determine the square footage of the patio: 12 feet x 16 feet = 192 square feet. Then 
multiply this number by the cost per square foot to determine what the bricklayer 
charges: 192 x $8 = $1,536. 

4. A. Let x equal Angie's hourly wage; x + 2 would then represent Tim's hourly wage, and 
3(x + 2) would represent Terry's hourly wage. Set up your equation and solve for x: 



x + (x + 


2) + 


3(x +2) = 


= 43 


x + x + 2 


+ 3x 


+ 6 = 


= 43 






5x 


+ 8 = 
5x - 

x -- 


= 43 
= 35 

= 7 



5. B. Two people is half as many as four people. Multiply the number of machines four people 
can run by i to determine how many machines two people can run: 8x^ = 4. 

6. B. Let x equal the number of daily tickets you would purchase; 36x equals the daily ticket 
cost: 

240 < 36x 
~36~ <x 

o 
You would need to use the ticket more than 6-=j times (or 7 times) for it to be cheaper to 

use the season ticket. 

7. C. You can convert the lengths of the pipes from feet and inches to inches and then divide 
the total inches needed by 12 to get the total number of feet of pipe needed. However, the 
easiest and fastest way to do this problem is to realize that 3 feet, 6 inches is 3.5 feet. 
Multiply the number of pipes needed by 3.5 feet to get the number of feet of pipe needed. 

4x3.5= 14 

8. B. The fastest way to solve this is to simply multiply the possible choices together (19x21 = 
399). However, you can also solve this with algebra. Let x equal the first number and x + 2 
equal the second number: 

x(x + 2) = 399 

x 2 + 2x = 399 

This is a quadratic equation that you can solve by setting it equal to zero and factoring. 

x 2 + 2x - 399 = 



(x-19)(x + 21) = 




x-19 = or 


x+21 =0 


x= 19 


x = -21 


x + 2 = 21 


x+2 = -19 



Two solutions are possible: 19 and 21, and -21 and -19. Because the latter pair isn't one of 
the answer choices, the first pair is the correct answer. 



Chapter 21: Practice Exam 3: Answers and Explanations 37 7 



9. C. Multiply her total sales by her percent commission to find her commission: $530 x 0.65 = 
$344.50. 

10. B. The formula for the length of the diagonal of a rectangle is d 2 = I 2 + w 2 (this is the 
Pythagorean theorem, where / and w are the sides of a right triangle and d is the hypote- 
nuse). In this case, d - 5 and / = w + 1. Substituting the known values into the formula 
results in 5 2 = (w + l) 2 + w 2 . 

5 2 = (w + Y)(w + 1) + w 2 

25 = w 2 + 2w + 1 + w 2 

25 = 2u; 2 + 2w + 1 

This equation is a quadratic equation, which you can solve by setting it equal to zero and 
factoring. 

= 2u; 2 + 2w - 24 

±(0)=±(2w 2 + 2w-24~) 

= w 2 + w-l2 

= (w- 3)(u; + 4) 

(£>-3 = or u; + 4 = 

w = 3 w = -4 (not a possible solution) 

11. B. If 2 inches = 3 miles, then 1 inch equals 1.5 miles: 3 + 2 = 1.5. Multiply 1.5 miles x 9.5 inches 
to determine the actual distance: 1.5 x 9.5 = 14.25, or 144- miles. 

12. D. The area of the entire piece of canvas = 10 inches x 14 inches = 140 square inches. The 
portion painted on equals 8 inches x 12 inches = 96 square inches. (This is determined by 
subtracting 2 inches — 1 inch on each side — from the length of each side to account for 
the margins.) Divide 96 by 140 to determine that about 68.596 of the canvas is covered with 
paint. You can round up to 6996. 

13. D. Calculate perimeter by adding the lengths of all four sides of a quadrilateral: 9 + 9 + 16 + 
16 = 50 feet. 

14. B. The formula for the perimeter of a rectangle is P = 2/ + 2w. In this case, P = 100 and 
/ = \.5w. Set up your equation and solve for w: 

100 = 2(1.5«;) + 2w 

100 = 3w + 2w 

100 = 5w 

w = 20 

The width of the rectangle is 20 inches. Because the length is l-k times the width, 1.5 x 20 = 30. 

15. C. The total number of quizzes is 10. If he passed seven of them, the fraction would be 

7 

expressed as y^r. 

16. D. Divide the amount of ribbon (in yards) used by the number of dresses to determine how 
much ribbon (in yards) was used in each dress: 3 + 4 = 4. Three-quarters of a yard of 
ribbon was used to make each dress. 

17. B. Divide $30 by $530 ($500 + $30) to determine the percentage of the sale price that the 
profit comprised: 30 * 530 = 0.056 = 696. 

18. A. First find how many bolts a full bin contains: 7 x 12 = 84 bolts. Then multiply the total 

number of bolts in a full bin by 4 to find how many bolts are in the bin when it's -4 full: 

i 84 6 6 

84x4- = -^-= 14 bolts. 
6 6 



312 



Part V: Practice ASVAB Exams 



Tip: A shortcut is to find -~ of a dozen first and then multiply by 7, the number of dozens. That 
is, 12 x ^ = 2 bolts, and 2 x 7 = 14 bolts. You can this because multiplication is commutative — 
you can multiply the numbers in any order. 

19. C. The first bike got a =-mile head start (12 mph x ^yhr. = ^ = = mi.). Therefore, by the 
time the second bike leaves, there are 5-=^ miles between them (6--^). Their combined rate 
of travel is 12 + 14 = 26 mph. Let t = the number of hours the second bike travels. 



26f = 


•4 




26f = 


26 

' 5 




t = 


26 . 
" 5 ' 


26 
1 


t = 


. 26 x 
" 5 X 


1 
26 


t = 


1 
5 




One-fifth of an hour 


meet at 2:21. 





60 4- 5 =12 minutes. The second bike left at 2:09, so both bikes will 

20. B. Choice (B) is the cheapest option that gives you enough beans. Calculate each answer 
option and compare: 

V Choice (A): 2 x $0.79 = $1.58 

^ Choice (B): $1.49 

j-" Choice (C): Two 3-ounce cans give you less than 8 ounces, so this answer can't be 
correct. 

V Choice (D): 3 x $0.65 = $1.95 

21. D. Add the sales amounts together: $25.70 + $32.30 + $31.80 = $89.80. Then multiply the 
total sales by -j to determine how much money he has left: $89.80 x 0.75 = $67.35. 

22. B. Divide $30.00 by $3.95. The whole number is the number of pairs of socks the recruit 
could buy: $30.00 * $3.95 = 7.59, or 7 pairs of socks. 

23. D. Subtract 43 pounds, 7 ounces (the weight of the puppy) from 60 pounds, 5 ounces (the 
weight of the crate). Converting an additional pound of the crate to ounces makes the sub- 
traction possible. Sixteen ounces make a pound, so 60 pounds, 5 ounces is the same as 

59 pounds, 21 ounces. Do the subtraction: 

59 pounds 21 ounces 
- 43 pounds 7 ounces 



16 pounds 14 ounces 



24. D. The probability that all five computers will be defective is 0. 15x0. 15x0. 15x0. 15x0. 15 = 
0.0000759 (round up to 0.000076). 

25. D. Find the area of each bedroom and add them together: 12x14= 168; 12x10 = 120; 8x12 = 
96; and 168 + 120 + 96 = 384 square feet. Then, because 9 square feet make up a square yard, 
divide the total area in square feet by 9 to determine the number of square yards needed: 
384 + 9 = 42.6 square yards. You can round up to 43 square yards. 

26. A. Divide the total number of pages to be typed by the number of pages Rafael can type 
per hour to find the number of hours it will take him to type the pages: 126 pages * 9 pages 
per hour = 14 hours. 



Chapter 21 : Practice Exam 3: Answers and Explanations A J A 



27. D. Divide the group of 48 girls by the number of girls who can play at the same time: 

48 + 12 = 4. This means four groups of girls have to share the 60 minutes: 60 minutes + 4 = 
15 minutes. Thus, each girl plays for 15 minutes. 

28. B. Subtract the first day's late charge from the total: $8.25 - $2.00 = $6.25. Then divide that 
amount by $1.25 to determine the number of additional days the movie was overdue: $6.25 + 
$1.25 = 5. Add those 5 days to the first day the movie was late to find that the movie was 

6 days overdue. 

29. D. Divide the number of calories in the pudding by the number of calories in the broccoli: 
150 + 60 = 2.5. Janet can eat 2.5 times the amount of broccoli as she can eat pudding for 
the same number of calories. Multiply 2.5 by 0.5 cup (the amount of pudding that con- 
tains 150 calories) to find how many cups of broccoli she can eat for 150 calories: 

2.5 x 0.5 = 1.25 = licups. 

30. C. The dog is barking every 15 minutes, or 4 times per hour. The time between 10 p.m. and 
2 a.m. is 4 hours. Multiply the total number of hours in the time period by 4 barks per 
hour. Then add 1 because the dog barked at the beginning of the period also: (4 x 4) + 1 = 
16+1 = 17. 



Subtest 3: Word Knowledge Answers 



The Word Knowledge subtest is another one of the "big four" that counts toward your AFQT 
score. If you're not seeing the improvement in your scores that you need to see, work with a 
partner who can quiz you on vocabulary. Review your vocabulary words intensely, even 
several times a day, to ensure your success on this subtest. See Chapter 4 for more help on 
improving your word knowledge. Additional practice questions are available in Chapter 6, 
and you also have a chance to practice this subtest in the practice AFQT in Chapter 22. 

l.B 8. D 15. A 22. A 29. A 



2D 


9. A 


16. B 


23. B 


30. C 


3.B 


10. A 


17. A 


24. D 


31. D 


4. B 


11. C 


18. D 


25. B 


32. A 


5. A 


12. A 


19. C 


26. D 


33. D 


6. C 


13. B 


20. B 


27. A 


34. C 


7. B 


14. C 


21. C 


28. C 


35. C 



Subtest %: Paragraph Comprehension Answers 

Because the military bigwigs use the Paragraph Comprehension subtest to determine 
whether you even qualify for enlistment (it counts toward your AFQT score), you need to 
do well here. If you're still struggling, remember to take your time when you read the pas- 
sages. And after you read each question, you can quickly reread the passage just to make 
sure you're on the money. The information is in the paragraph; you just have to concentrate 
to pull it out. Turn to Chapter 5 and the practice questions in Chapter 6 if you still need 
additional help to pull off a good score on this subtest. An additional opportunity to prac- 
tice taking this subtest is in the next chapter. 



3U 



Part V: Practice ASVAB Exams 



1. D. The last sentence in the passage states that chaos and fear are associated with change, 
making Choice (D) the correct choice. The passage states that leaders must learn to deal 
with negative emotions connected with change, making Choice (A) incorrect. The second 
sentence makes it clear that innovation means change, so Choice (B) is incorrect. The 
third sentence clearly states that leaders must learn everything there is to know about the 
change, making Choice (C) the wrong choice. 

2. C. The first sentence says cougars are wide-ranging big cats in North America, making 
Choice (C) correct. The passage states that pumas, mountain lions, and cougars are the 
same thing, so Choice (A) is incorrect. Nothing in the passage supports Choice (B), which 
says cougars are endangered. The passage states that cougars live about 18 years in the 
wild, so Choice (D) is incorrect. 

3. B. The counseling process works because the counselee feels the need for assistance, 
instruction, or understanding. Therefore, Choice (B) — counselees initially reject the 
advice of their counselors — isn't supported by the passage. The other three choices are 
all supported by the content of the paragraph. 

4. A. The second sentence says that professional fire departments have more training, exper- 
tise, and experience. Therefore, the reader can infer that volunteer departments have less 
training, expertise, and experience than professionals. The passage says that profession- 
als, not volunteers, have the skills needed to investigate fires, so Choice (B) is incorrect. 
The passage states that professional firefighters have more experience investigating the 
causes of fires, so Choice (C) is incorrect. The passage states that hiring professional fire- 
fighters is worthwhile, so Choice (D) is incorrect. 

5. C. The passage doesn't state the locale of the story, so Choices (A) and (D) are incorrect. 
The references to extreme cold and lack of light makes Choice (B) an incorrect answer. In 
sentence three, the author says it was still very foggy and extremely cold, so Choice (C) is 
the answer. 

6. A. The main point of the passage is to define epidemiology, as evidenced by the opening 
sentence. Epidemiology is the study of what causes diseases. Choices (B), (C), and (D) 
aren't the main points of the passage. 

7. B. The only statement that's supported by the passage is Choice (B), which says Buddhist 
teachings have changed. In fact, this sentence is the primary theme of the passage. The 
other choices aren't supported by information contained in the paragraph. 

8. A. Choice (B), which questions jurors' intelligence, isn't supported by the passage. 
Choice (C), which asks for more laws that take intent into consideration, is the opposite 
of what the author argues. The text doesn't support Choice (D), which calls lawyers liars. 
The first line of the passage states that statutes permit more severe punishment of a 
person convicted of a crime if he or she intended to harm another person. The last sen- 
tence says juries should only be asked to weigh objective evidence, so the author would 
no doubt agree that laws should not punish people based on intention, Choice (A). 

9. C. The passage says that most states punish voluntary manslaughter more severely than 
involuntary manslaughter, so Choice (A) is incorrect. The argument that punishing people 
more severely for voluntary manslaughter is unconstitutional isn't made in the passage, so 
Choice (B) is incorrect. The passage doesn't support Choice (D), which says prosecutors 
can establish intent. The first sentence of the second paragraph states that juries find it 
difficult to know what the intent of a person was at the time he or she committed a crime, 
so Choice (C) is correct. 

10. B. Lack of attention to ergonomics, not ergonomics itself, can cause injury, so Choice (A) 
is incorrect. The passage doesn't support Choice (C), which discusses cost and time. The 
passage states that many people suffer injuries when sufficient attention isn't paid to ergo- 
nomics, so Choice (D) is incorrect. The first says, "Ergonomics is the science of designing 
and arranging workspaces so that people and objects interact efficiently and safely," so 
Choice (B) is correct. 



Chapter 21 : Practice Exam 3: Answers and Explanations j 1 5 



11. D. The passage states that adequate lighting and well-designed chairs are part of ergo- 
nomic design, so Choice (A) is incorrect. The passage states that repetitious work can 
cause injury, so Choice (B) is incorrect. The passage states that desk-bound workers 
should take breaks, so Choice (C) is incorrect. The first sentence in the second paragraph 
states that adequate lighting, well-designed chairs, and clutter-free work areas contribute 
to effective ergonomic design, so Choice (D) is correct. 

12. A. The passage makes it clear that lack of ergonomic design is dangerous, so Choice (B) 
is incorrect. Nothing in the passage supports Choice (C), which brings up labor unions. 
Although the passage claims that lack of ergonomic design causes injury, nothing in the pas- 
sage supports Choice (D). The passage's second sentence says that lack of attention to ergo- 
nomics causes thousands of workers to suffer repetitive stress injury, eye fatigue, muscle 
soreness, and many other medical problems, so it's reasonable to assume that employers 
should invest in ergonomic design to protect workers, which makes Choice (A) correct. 

13. B. The passage says that BTUs are the standard measure of heat, so Choice (A) is incor- 
rect. BTU stands for British thermal unit, so Choice (C) is incorrect. Nothing in the passage 
supports Choice (D), which names room dimensions. 

14. D. The passage shows that fuel oil can be more expensive than other heating methods, 
so Choice (A) is incorrect. Oak firewood is sometimes less expensive than other types of 
fuel, so Choice (B) is incorrect. Natural gas can sometimes cost less than firewood, so 
Choice (C) is incorrect. The first sentence states that electricity is always the most ineffi- 
cient and costly way to heat a home, which makes Choice (D) correct. 

15. A. The main point of this passage deals with choosing the right fuel based on price; only 
Choice (A) summarizes this point. Choices (B), (C), and (D) are less important points. 
Because after discussing costs by BTU of the various fuels, the final sentence makes 
Choice (A) correct. 

Subtest 5: Mathematics Knotfiedqe Answers 

The Mathematics Knowledge subtest is used to determine whether you qualify for enlist- 
ment, so you need to do well. If you're still missing too many questions, you may need to 
take more drastic measures, such as enrolling in a basic-algebra class at a local community 
college. If your scores are improving, keep hitting the books and testing yourself up until the 
day of the ASVAB. Turn to Chapter 7 and the practice questions in Chapter 9 for more infor- 
mation. The practice AFQT in Chapter 22 also gives you a chance to gauge your progress. 

1. B. Substitute 6 for y in the equation: 2(6) x6=12x6 = 72. 

2. B. Divide both sides of the equation by 0.05 to isolate x: 0.05* 4- 0.05 = 1 * 0.05, or x = 20. To 
check your answer, substitute 20 for x in the original equation. 

3. C.V25x 2 =V( 5x ) 2 = 5x 

4. A. 9X 3 + 18x 2 - x - 2 = 9x 2 (x + 2) - l(x + 2) = (9x 2 - l)(x + 2). 

The fully factored answer is (3x - l)(3x + l)(x + 2), but that's not what the question asks. 
Make sure you look for the answer to the question. 

5. B. Solve for x: 

5x + 7 = 6(x - 2) - 4(2* - 3) 
5x+ 7 = 6x-12-8x+ 12 
5x + 7 = -2x 
Ix + 7 = 
7x = -7 
x = -\ 



316 



Part V: Practice ASVAB Exams 



6. D. If two powers have the same base, they can be multiplied by keeping the base and 
adding the exponents together. In this case, x is the same as x 1 : x + x 2 = x 1+2 - x 3 , 

7. B. This is so easy that it may tempt you to think that the correct answer is too obvious. 
The square root of (5 + x) 2 is simply 5 + x. 

8. B. You start with (3 x 3)(5 - 3)(6 + 2) = x 2 . First solve the left side of the equation: (9)(2)(8) = 
144. So x 2 = 144. Find the square root of each side: x = 12. 

9. A. You start with -5x = 25. Isolate x by dividing each side of the equation by -5: x = -5. 

10. C. The area of a circle equals n times the radius squared. The radius is half the diameter, 
which is 12 in this problem. Plug in the known values: A = jxr 2 ; A = jt6 2 = 36ti. If n is approxi- 
mately 3.14, the area of the circle is approximately 3.14 x 36, or 113 square feet. 

11. B. Volume equals length x width x height (V '= Iwli): 6x6x6 = 36 x6 = 216 cubic inches. 

12. B. The area of a circle is A = nr 2 . Radius is half the diameter, so the radius is 5. In this 
problem, A = ji5 2 = 25jt. 

13. C. For cylinders, Volume = ni^Qti). Because the radius is half the diameter, you can calculate 
the problem this way: V= 7t(6 2 )10 = 7t(36)10. If % is approximately 3.14, then 3.14 x 36 x 10 = 
1,130 cubic inches. 

14. D. In an isosceles triangle, sides a and c are equal, and angles 1 and 2 are equal. 

15. A. Rectangles have four equal angles, and all angles are right angles. 

16. A. Supplementary angles always equal 180 degrees. 

17. A. Solve for x: 

3(2x - 5) - 2(4x + 1) = -5(x + 3) - 2 
6x-15-8x-2 = -5x-15-2 
-2x-17 = -5x-17 
3x-17 = -17 
3x=0 
x=0 

18. A. Volume equals length x width x height (V = Iwli). Finding the cube root of 64 shows that 
each edge measures 4 inches (4x4x4 = 64). 

19. B. (x 3 ) 2 is the same as (x 3 )(x 3 ). To multiply exponents with the same base, keep the base 
and add the exponents: (x 3 )(x 3 ) = x 3 * 3 = x 6 . 

20. D. To find out how much rain falls in an hour, multiply the amount that falls in one minute 
by 60, because 60 minutes make up an hour. In h hours, the amount of rain is 60//?. 

21. D. Because x = y, you can plug inx for eachy in the problem. Therefore, 6 + 4(x-x) = 
6 + 4(0) = 6 + = 6. 

22. A. The problem asks for the square root of 810. You know that 20 2 = 400 and 30 2 = 900, so 
the range of 20 to 30 is correct. 

23. B. The problem asks you to find (x + 2)(x + 2). Using the FOIL method (First, Outside, Inside, 
Last), systematically multiply each term in the first set of parentheses by each term in the 
second set of parentheses: 

Iv* Multiply the First terms: x(x) = x 2 . 
j-" Multiply the Outside terms: x(2) = 2x. 



Chapter 21: Practice Exam 3: Answers and Explanations 377 



Iv* Multiply the Inside terms: 2(x) = 2x. 
u* Multiply the Last terms: 2x2 = 4. 

Now add all the products together, and you get x 2 + 2x + 2x + 4. Add like terms to get the 
final answer of x 2 + Ax + 4. 

24. A. Replace the unknowns with the numbers given: 6(-3) - 3(-7) - 2(4) = -18 + 21 - 8 = -5. 

25. C. The problem asks you to find (x + 4)(3x + 5). Use the FOIL method: 
v* Multiply the First terms: x(3x) = Zx 2 . 
v* Multiply the Outside terms: x(5) = 5x. 
v* Multiply the Inside terms: 4(3x) = 12x. 
W Multiply the Last terms: 4(5) = 20. 

Now add all the products together, and you get 3x 2 + 5x + 12x + 20. Add like terms to get 
the final answer, 3x 2 + 17x + 20. 

Subtest 6: Electronics In formation Answers 

If you need to do well on the Electronics Information subtest to qualify for a certain military 
career (see Appendix A) and you're still missing questions, review Chapter 13 and spend 
some time memorizing key electronics concepts, including the mathematical formulas (like 
Power = Voltage x Current) that help you solve all kinds of electronics problems. 

You can find additional practice questions can be found in Chapter 15. 

1. B. Unless a specific gauge is specified by the speaker manufacturer, you should always 
choose lower gauges for better sound quality. 

2. B. There are four frequency bands used throughout the world. A quad-band cell phone 
would be able to access any of these frequency bands. 

3. B. The greatest concern when dealing with electricity is personal injury. 

4. C. Heat effect occurs when electrical current must overcome the resistance of the wire. 
Heat effect can be quite obvious or very subtle. 

5. C. Like other diodes, Zener diodes allow current in only direction, except if the voltage 
across it is greater than a threshold voltage (called breakdown voltage). Current also flows 
through the Zener diode in the opposite direction. This action allows the Zener diode to 
regulate voltage. 

6. A. The symbol stands for ohm. 

7. D. Electromotive force is the difference of potential, so the term is another way of saying 
voltage. 

8. A. Lithium-ion (Li-Ion) batteries are much lighter than nickel metal hydride (NiMH) batteries. 

9. A. The three terminals a transistor must have are the base, emitter, and collector. 

10. B. To control a light fixture from two different positions, use two three-way switches. 

1 1. C. Amplifier is another name for transistor. 



318 



Part V: Practice ASVAB Exams 



12. A. The symbol means ground. 

13. C. Capacitance is the ability to hold an electrical charge. Connecting capacitors in series 
reduces the capacitance. 

14. C. 2.5K ohms is 2,500 ohms. K = 1 kilo, or 1,000. 

15. B. A cell is equal to about 1.5 volts, so 9 * 1.5 = 6. 

16. D. Primary live wires are black. 

17. C. A motherboard is the physical arrangement in a computer that contains the computer's 
basic circuitry and components. Motherboards come in various sizes and component 
arrangements referred to as form factors to fit standard case sizes, components required, 
and so on. AT is a standard form factor that has a width of 12 inches and a length of 

13.8 inches. 

18. B. An on-off switch allows current to flow only when it's in the closed (on) position. 

19. C. Determine the wattage that could cause the circuit breaker to trip with this formula: 
Watts = Volts x Amps, or 120 x 25 = 3,000 watts. 

20. A. A rheostat is a type of variable resister that's usually used to control voltage. 

Subtest 7: Auto & Shop Information Answers 

If you have your heart set on fixing Jeeps and tanks or doing other related military jobs (see 
Appendix A) and you're still struggling on this test, review Chapter 11 and do the practice 
questions in Chapter 15. 



ID 


6. C 


11. C 


16. B 


21. C 


2. A 


7. A 


12. A 


17. B 


22. D 


3D 


8. D 


13. B 


18. C 


23. C 


4. A 


9. B 


14. D 


19. B 


24. B 


5. D 


10. B 


15. A 


20. A 


25. A 



Subtest 8: Mechanical Comprehension Answers 

If you need to do well on the Mechanical Comprehension subtest, don't forget to apply your 
math skills to the concepts. (A little extra physics study wouldn't hurt, either.) But simply 
using your common sense can help you quite a bit, too. For example, you may not know 
exactly why a metal spoon feels colder than a wooden spoon when they're at the same tem- 
perature, but at least you know that it feels colder. And knowing that may help you answer 
a question correctly. See Chapter 12 and the additional practice questions in Chapter 15 for 
more information. 

1. B. The teeth of helical gears are slanted. 

2. B. The anvil is closer to Pillar B, so Pillar B bears more weight. 

3. B. Wheel B is smaller. It has to make more revolutions than Wheel A to cover the same 
amount of distance, so it will take longer. 



Chapter 21: Practice Exam 3: Answers and Explanations A f Q 



4. A. E stands for effort needed. Here's how to set up the equation: 30 (weight of the anvil) x 2 
(length of resistance arm) = x(6) (length of effort arm). Do a little multiplication, and you get 
60 = 6x. To isolate x, divide each side by 6: 10 = x. 

5. A. A fixed, simple pulley gives no mechanical advantage, so its mechanical advantage 
number is 1. 

6. D. The formula to determine the mechanical advantage of an inclined plane is Length of 
Ramp * Height of Ramp = Weight of Object * Effort: 

8 400 

4 E 
8E = 400(4) 
8£ = 1,600 

£• = 200 

7. B. One horsepower equals 33,000 foot-pounds per minute. 

8. A. Pressure = Force * Area. Psi stands for pounds per square inch, so you don't have to 
change the units of measurement. Just plug in your numbers: P = 130 lbs. * 1 in. 2 = 130 psi. 

9. D. Static electricity is the buildup of electrical charge on surfaces, which causes materials 
to "stick" together this way. 

10. A. An aneroid barometer measures atmospheric pressure. 

11. C. Meshed gears always turn in opposite directions. 

12. D. Springs are used for all the listed purposes except to add weight. 

13. B. Specific gravity is a comparison between the density of a liquid and the density of water. 
The liquid with the higher specific gravity will have a float that rises higher. 

14. D. Universal joints are used to connect shafts that aren't in the same plane. 

15. D. Try-cocks are valves that measure water level. Water seeks a level throughout a system, 
so in the schematic, the try-cock correctly indicates the water level. 

16. A. There are six different symbols, so six different types of machine screws were used. 

17. B. To determine the amount of force the anvils exert, first multiply the length of the resis- 
tance arm (as it applies to the anvil) by the weight of each anvil and add the products 
together. The 10-pound anvil is supported by the entire weight of the resistance arm, so 5 x 
10 = 50. The 5-pound anvil is being supported by 3 feet of the resistance arm, so 3 x 5 = 15. 
Add 'em up: 50 + 15 = 65. This number is equal to the length of the resistance arm times 
effort (force), or 65 = 5F. To isolate F, divide both sides by 5: 13 = F. 

18. C. One revolution of the winch will move the weight 24 inches, the circumference of the 
winch drum. 

19. A. Valves 1 and 2 need to be open to fill the tank. 

20. B. The number of revolutions a gear makes is inversely proportional to its difference in 
size from the gear that's turning it. Gear 1 makes 10 clockwise revolutions per minute. 
Gear 2 is half the size of Gear 1, so to determine the number of revolutions it makes, multiply 
the number of revolutions Gear 1 makes by the inverse (reciprocal) of -k, which is y (or 
just 2): 10x2 = 20. Therefore, Gear 2 makes 20 counterclockwise revolutions per minute. 

c 

Gear 3 is 2.5 times (or ^ times) the size of Gear 2. To determine the number of revolutions 

5 
Gear 3 makes, multiply the inverse of 4f by the number of revolutions Gear 2 makes: 

2 40 

■w x 20 = -¥- = 8 revolutions per minute. 

21. B. The pinion turns four times as often as the gear: 4 x 200 = 800 rpm. 



320 



Part V: Practice ASVAB Exams 



22. C. A float indicates the level of liquid in a container. 

23. B. Using a runner (a single, moveable pulley) gives a mechanical advantage of 2. 

24. A. Because 60 seconds comprise a minute, the valve must open 60 times per minute. The 
cam will open the valve 10 times per revolution, and 60 + 10 = 6. The cam must make 6 rev- 
olutions per minute to raise the valve 60 times per minute. 

25. D. Because Valve 4 is above the bottom of the tank, some water will remain in the tank 
below the level of the valve, so the tank will never be completely empty. 

Subtest 9: Assembling Objects Answers 

At present, only the Navy uses the scores from this subtest. If you plan to sail the Seven 
Seas and you want one of the few Navy jobs that requires you to put parts A and B together, 
you may want to go over the practice subtests again. For additional practice questions, see 
Chapter 15. 



l.C 


6. D 


11. D 


16. C 


21. B 


2. A 


7. C 


12. A 


17. A 


22. D 


3. A 


8. C 


13. C 


18. B 


23. D 


4. B 


9. B 


14. B 


19. A 


24. A 


5.C 


10. A 


15. D 


20. C 


25. C 



Chapter 22 

Practice AFQT Exam 



i 

■ f you're wondering what in the world this exam is doing in a book on the ASVAB, 
«S don't be confused. The Armed Forces Qualification Test, or AFQT, is part of the ASVAB — 
in a way, it's a test within a test. Your scaled AFQT score is derived from four subtests of 
the ASVAB, and it determines your overall mental qualification to join the service branch of 
your choice. Each of the five branches of military service has set its own minimum AFQT 
score in order to qualify for enlistment. The four subtests that can make or break your 
chances of joining the military are Arithmetic Reasoning (AR), Word Knowledge (WK), 
Paragraph Comprehension (PC), and Mathematics Knowledge (MK). 

Because I like you (and because you were kind enough to buy this book), I've included an 
extra chance for you to evaluate your communication and math skills before you head over 
to the MEPS (military entrance processing station) or your school or the local National 
Guard Armory for the real deal. 

After you complete the entire practice test, check your answers in Chapter 23. 



.VABE/f 




Your goal here is to determine where you may still need to spend some more study time. If 
you miss only one question on the Word Knowledge subtest but you miss 15 on Arithmetic 
Reasoning, you may want to dedicate some extra study time to further develop your math 
skills before you take the actual ASVAB. 



322 



Part V: Practice ASVAB Exams 



Answer Sheet for Practice AFQT Exam 



Subtest 1: Arithmetic Reasoning 



1 ®@©® 


7 ®®©® 


13 ®®©® 


19 ®®©® 


25 ®@©@ 


2 ®(D©® 


8 ®®©® 


14 ®®©® 


20 ®®©@ 


26 ®@©@ 


3 ®®©® 


9 ®®©® 


15 ®®©® 


21 ®®©@ 


27 ®@©® 


4 ®®©® 


10 ®@©@ 


16 ®®©® 


22 ® ® © ® 


28 ® ® © ® 


5 ®@©® 


11 ®@©@ 


17 ®@©® 


23 ® ® © ® 


29 ® ® © ® 


6 ®®©® 


12 ®@©@ 


18 ®®©@ 


24 ® ® © ® 


30 ® ® © ® 


Subtest 2: Word Knowledge 








1 ®®©@ 


8 ®®©® 


15 ®®©® 


22 ® ® © ® 


29 ® ® © ® 


2 ®®©® 


9 ®®©® 


16 ®®©® 


23 ®®©® 


30 ®®©@ 


3 ®®©@ 


10 ®®@® 


17 ®®©® 


24 ®®©® 


31 ®@©® 


4 ®®©® 


11 ®@©® 


18 ®®©® 


25 ®®©® 


32 ® ® © ® 


5 ®®©@ 


12 ®@©@ 


19 ®@©® 


26 ® ® © ® 


33 ® ® © ® 


6 ®®@® 


13 ®@©@ 


20 ®@©® 


27 ®®@® 


34 ® ® © ® 


7 ®@©® 


14 ®®©@ 


21 ®@©@ 


28 ®®©® 


35 ® ® © ® 


Subtest 3: Paragraph Comprehension 








1 ®®©@ 


4 ®®©® 


7 ®@©® 


10 ®®©® 


13 ®®©® 


2 ®®©® 


5 ®®©@ 


8 ®®@® 


11 ®®©@ 


14 ®@©@ 


3 ®®©® 


6 ®®©@ 


9 ®®©® 


12 ®@©@ 


15 ®@©@ 


Subtest 4: Mathematics Knowledge 








1 ®®©@ 


6 ®®©® 


11 ®®©® 


16 ®®©® 


21 ®®©@ 


2 ®®©® 


7 ®®©® 


12 ®®©® 


17 ®®©® 


22 ®@©@ 


3 ®®©® 


8 ®@©@ 


13 ®®©@ 


18 ®®©@ 


23 ®@©@ 


4 ®®©® 


9 ®@©@ 


14 ®®©® 


19 ®®©® 


24 ® ® © ® 


5 ®@©® 


10 ®@©@ 


15 ®®©@ 


20 ®®©® 


25 ® ® © ® 



Chapter 22: Practice AFQT Exam 2 2? 



Subtest 1: Arithmetic Reasoning 



Time: 36 minutes for 30 questions 

Directions: This test contains questions about arithmetic. Each question is followed by four possi- 
ble answers. Decide which answer is correct and then mark the space on your answer sheet that 
has the same number and letter as your choice. Use scratch paper for any figuring you need to do. 
Calculators are not allowed. 



1. If a barber is capable of cutting the hair of 
35 people per day and he works 7 days per 
week, how many haircuts could he give 
during the months of April, May, and June? 

(A) 3,185 

(B) 3,150 

(C) 2,545 

(D) 2,555 

2. If you type 45 words per minute, how many 
words can you type in 12 minutes? 

(A) 490 

(B) 540 

(C) 605 

(D) 615 

3. Tom is flying a kite at the end of a 500-foot 
string. His friend Kathy is standing directly 
under the kite 300 feet away from Tom. 
How high is the kite flying? 

(A) 300 feet 

(B) 350 feet 

(C) 400 feet 

(D) 450 feet 

4. Amy wants to fence in a yard using 400 feet 
of fencing. If she wants the yard to be 

30 feet wide, how long will it be? 

(A) 170 feet 

(B) 175 feet 

(C) 180 feet 

(D) 185 feet 



5. A three-digit code must be used to access a 
computer file. The first digit must be an A 
or a B. The second digit must be a number 
between and 9. The final digit is a single 
letter from the alphabet from A to Z. How 
many possible access codes can there be? 

(A) 38 

(B) 468 

(C) 520 
(D)640 

6. The sun is 93 million miles from Earth, and 
light travels at a rate of 186,000 miles per 
second. How long does it take for light from 
the sun to reach the Earth? 

(A) 5 minutes 

(B) 6-4 minutes 

(C) 7 minutes 

(D) 8-^ minutes 

7. A tanning-bed pass for unlimited tanning 
costs $53 per month this year, but it was 
only $50 per month last year. What was the 
percentage of increase? 

(A) 5% 

(B) 5.5% 

(C) 6% 

(D) 6.5% 

8. Eleven plus forty-one is divided by a number. 
If the result is thirteen, what's the number? 

(A) 2 

0)4 

(C)6 

(D)8 



Go on to next page 



321, 



Part V: Practice ASVAB Exams 



9. Mark received an hourly wage of $9.25. His 
boss gave him a 496 raise. How much does 
Mark make per hour now? 

(A) $9.29 

(B) $9.62 

(C) $9.89 

(D) $9.99 

10. How many pounds of nails costing $7 per 
pound must be mixed with 6 pounds of 
nails costing $3 per pound to yield a mix- 
ture costing $4 per pound? 

(A) 2 pounds 

(B) 2.5 pounds 

(C) 3 pounds 

(D) 3.5 pounds 

11. Theodore has 24 baseball cards. He sells 4- 
of his cards to Tom and ^ of his cards to 
Larry, and his Mom accidently throws away 
^ of his cards. How many baseball cards 
does Theodore have left? 

(A) 2 
(B)18 

(9 12 

(D)6 

12. Theresa bought five karaoke CDs on sale. 
A karaoke CD normally costs $24, but she 
was able to purchase the CDs for $22.50 
each. How much money did Theresa save 
on her entire purchase? 

(A) $7.50 

(B) $1.50 

(C) $8.00 

(D) $22.50 

13. On a trip to the beach, you travel 200 miles 
in 300 minutes. How fast did you travel? 

(A) 30 mph 

(B) 40 mph 

(C) 50 mph 

(D) 60 mph 



14. Twenty-one students, or 6096 of the class, 
passed the final exam. How many students 
are in the class? 

(A) 45 

(B)40 

(C) 35 

(D)30 

15. Joan invests $4,000 in an account that 
earns 3/6 simple interest. How much will 
Joan have in the account in 10 years? 

(A) $4,500 

(B) $4,800 

(C) $5,200 

(D) $5,400 

16. A rectangle has a perimeter of 36 inches. 
Its length is 3 inches greater than twice the 
width. What's the rectangle's length? 

(A) 5 inches 

(B) 13 inches 

(C) 18 inches 

(D) 20 inches 

17. A backyard is 50 feet by 100 feet. What's its 
area? 

(A) 150 square feet 

(B) 500 square feet 

(C) 2,500 square feet 

(D) 5,000 square feet 

18. Eric is driving a car in which the speedome- 
ter is calibrated in kilometers per hour (kph). 
He notes that his car is traveling at a rate of 
75 kph, when he passes a speed limit sign 
stating the limit is 40 miles per hour (mph). 

He knows that a kilometer is about -^ of a 

mile. If a police officer stops him at this point, 
how many miles per hour over the limit will 
the ticket read? 

(A) 5 

(B)7 

(C)9 

(D)ll 



Go on to next page 



Chapter 22: Practice AFQT Exam 2 25 



19. Three apples and twice as many pears add 
up to one-half the number of grapes in a 
fruit basket. How many grapes are in the 
basket? 

(A) 8 

(B)18 

(C) 28 

(D)38 

20. Apples are on sale for "Buy 2 pounds get 

1 free." How many pounds must Janet pur- 
chase to get 2 pounds free? 

(A) 2 pounds 

(B) 4 pounds 

(C) 6 apples 

(D) 3 pounds 

21. If four pipes of equal length measure 44 feet 
when they're connected together, how long 
is each pipe? 

(A) 11 feet 

(B) 4 feet 

(C) 22 feet 

(D) 9 feet 

22. A German Shepherd and an Alaskan 
Malamute are both headed toward the 
same fire hydrant. The German Shepherd 
is 120 feet away from the hydrant, and the 
Alaskan Malamute is 75 feet away from the 
hydrant. How much closer to the hydrant 
is the Alaskan Malamute? 

(A) 45 feet 

(B) 25 feet 

(C) 75 feet 

(D) 195 feet 

23. A recruit reporting to boot camp took a bus 
from her home to the military processing 
center in another city. The trip took 

14 hours. If she left at 6 a.m., what time 
did she arrive at the processing center? 

(A) 7 p.m. 

(B) 12 a.m. 

(C) 8 p.m. 

(D) 9 p.m. 



24. A farmer sold 3 pints of strawberries for 
$1.98 each, 5 pints of raspberries for $2.49 
each, and a bushel of peaches for $5.50 at 
his roadside stand. How much money did 
the farmer make? 

(A) $9.97 

(B) $23.89 

(C) $18.39 

(D) $18.91 

25. A librarian wants to shelve 532 books. If 
four books fit on a 1-foot length of shelving, 
how many feet of shelving does she need to 
shelve all the books? 

(A) 13 

(B)45 

(C) 33 

(D) 133 

26. A student buys a science textbook for 
$18.00, a math textbook for $14.50, and a 
dictionary for $9.95. What's the total cost 
of the books? 

(A) $27.95 

(B) $42.45 

(C) $41.95 

(D) $38.50 

27. Debra works an 8-hour shift on Friday. How 
many minutes does she work on Friday? 

(A) 480 minutes 

(B) 800 minutes 

(C) 240 minutes 

(D) 400 minutes 

28. Six people can run three machines in the 
factory. How many machines can 18 people 
run? 

(A) 7 

(B)9 

(C)6 

(D)8 



Go on to next page 



326 



Part V: Practice ASVAB Exams 



29. 



On a map drawn to scale, i inch equals 
1 mile. What length on the map equals 
5 miles? 

(A) 2.5 inches 

(B) 5.0 inches 

(C) 10.0 inches 

(D) 1.5 inches 



30. A man bought a pair of jeans for $23.00, a 
shirt for $14.95, and two ties for $7.98 each. 
What was the total cost of his clothing? 

(A) $53.91 

(B) $45.93 

(C) $51.99 

(D) $54.50 



STOPI 



DO NOTTURNTHE PAGE UNTILTOLDTO DO SO. 
DO NOT RETURN TO A PREVIOUS TEST. 



Chapter 22: Practice AFQT Exam $27 



Subtest 2: Word Knortiedqe 



Time: 11 minutes for 35 questions 

Directions: This test has questions about the meanings of words. Each question has an underlined 
word. You need to decide which one of the four words in the choices most nearly means the same 
thing as the underlined word and then mark the corresponding space on your answer sheet. 



1. The abhorrent smell from the lake over- 
powered the picnickers gathered on the 
shore. 

(A) strong 

(B) pleasant 

(C) offensive 

(D) tantalizing 

2. Belie most nearly means 

(A) pleasure. 

(B) rule. 

(C) pretend. 

(D) misrepresent. 

3. The water was calm that day with detritus 
slowly moving in the small eddies. 

(A) fish 

(B) lily pads 

(C) plants 

(D) debris 

4. The prime minister was always cautious 
about leaving his redoubt in Belgrade. 

(A) city 

(B) stronghold 

(C) house 

(D) country 

5. Mike was afraid he might be ostracized for 
stepping out of line. 

(A) banished 

(B) scolded 

(C) assaulted 

(D) arrested 



9. 



10. 



The hotel was specifically designed for the 
wayworn traveler. 

(A) lost 

(B) weary 

(C) demanding 

(D) happy 

The park has no showers and no potable 
water. 

(A) usable 

(B) clear 

(C) drinkable 

(D) tasty 

Decamp most nearly means 

(A) to backpack. 

(B) to leave. 

(C) to doubt. 

(D) to act with abandon. 

Glorious most nearly means 

(A) splendid. 

(B) particular. 

(C) delayed. 

(D) contentious. 

Hypocrisy most nearly means 

(A) hyperactivity. 

(B) god-like. 

(C) insincerity. 

(D) criticalness. 



Go on to next page 



328 



Part V: Practice ASVAB Exams 



11. Mallet most nearly means 

(A) sermon. 

(B) participate. 

(C) hammer. 

(D) fish. 

12. Hosiery most nearly means 

(A) dangerous. 

(B) illegal. 

(C) stocking. 

(D) automatic. 

13. Hale most nearly means 

(A) old. 

(B) healthy. 

(C) customary. 

(D) uninformed. 

14. Magnitude most nearly means 

(A) importance. 

(B) peculiar. 

(C) alone. 

(D) tantamount. 

15. His vapid presentation earned him a C in 
the class. 

(A) mediocre 

(B) plagiarized 

(C) dull 

(D) polished 

16. Percival was unpopular at the meeting 
because he imparted so much extraneous 
data. 

(A) extensive 

(B) unwelcome 

(C) superfluous 

(D) radical 

17. She was often solicitous of her father's 
feelings. 

(A) careful 

(B) ignorant 

(C) forgetful 

(D) abusive 



18. I could never get over her liquid blue, 
limpid eyes. 

(A) bright 

(B) clear 

(C) attentive 

(D) dull 

19. The goal of the treaty is to develop interna- 
tional amity and reciprocal trade. 

(A) agreement 

(B) friendship 

(C) standards 

(D) understanding 

20. He often bragged about the bravery of his 
favorite cohort . 

(A) person 

(B) teacher 

(C) companion 

(D) employee 

21. Speechlessness most nearly means 

(A) well spoken. 

(B) silence. 

(C) restlessness. 

(D) talkative. 

22. Indigenous most nearly means 

(A) poor. 

(B) rich. 

(C) immigrant. 

(D) native. 

23. Illusive most nearly means 

(A) insignificant. 

(B) deceptive. 

(C) useful. 

(D) hidden. 

24. Hesitate most nearly means 

(A) slam. 

(B) slow to act. 

(C) foreclose. 

(D) end. 



Go on to next page 



Chapter 22: Practice AFQT Exam ?2^ 



25. Gravity most nearly means 

(A) planet. 

(B) relationship. 

(C) earn. 

(D) seriousness. 

26. Fondle most nearly means 

(A) stir. 

(B) handle. 

(C) ogle. 

(D) radiate. 

27. Fete most nearly means 

(A) festival. 

(B) criticize. 

(C) approve. 

(D) eat. 

28. Encore most nearly means 

(A) play. 

(B) applause. 

(C) repetition. 

(D) excite. 

29. Diverse most nearly means 

(A) various. 

(B) hidden. 

(C) nestled. 

(D) pastime. 

30. Detest most nearly means 

(A) anger. 

(B) hate. 

(C) surprise. 

(D) excite. 



31. Tim was known as a smart aleck, able to 
deliver acerbic one-liners with no effort. 

(A) funny 

(B) cheap 

(C) sharp 

(D) poetic 

32. It took a great degree of inexorable force to 
break into the cavern. 

(A) strong 

(B) unyielding 

(C) acute 

(D) powerful 

33. Attendents were stationed at intervals, 
with the obvious intent to hector those 
who moved too slowly. 

(A) hurry 

(B) harass 

(C) encourage 

(D) note 

34. Reggy was as gauche in this group of polite 
company as he always had been. 

(A) funny 

(B) entertaining 

(C) tactless 

(D) embarrassed 

35. Confident most nearly means 

(A) assured. 

(B) positive. 

(C) intelligent. 

(D) educated. 



STOPI 



DO NOTTURNTHE PAGE UNTILTOLDTO DO SO. 
DO NOT RETURN TO A PREVIOUS TEST. 



330 



Part V: Practice ASVAB Exams 



Subtest 3: Paragraph Comprehension 



Time: 13 minutes for 15 questions 

Directions: This test contains items that measure your ability to understand what you read. This 
section includes one or more paragraphs of reading material followed by incomplete statements or 
questions. Read the paragraph and select the choice that best completes the statement or answers 
the question. Mark your choice on your answer sheet by using the correct letter with each question 
number. 



On June 22, 1944, President Franklin Delano 
Roosevelt signed into law one of the most signifi- 
cant pieces of legislation ever produced by the 
United States government: The Servicemembers' 
Readjustment Act of 1944, commonly known as 
the GI Bill of Rights. By the time the original GI 
Bill ended in July 1956, 7.8 million World War II 
veterans had participated in an education or 
training program, and 2.4 million veterans had 
home loans backed by the Veterans 
Administration (VA). 

1. The G.I. Bill provided 

(A) free housing, training, and education. 

(B) medical coverage, education, and assis- 
tance to veterans. 

(C) home loan guarantees, training, and 
education for many former military 
members. 

(D) a means to exempt veterans from 
Social Security taxes. 

You can put up to $3,000 a year into an 
individual retirement account (IRA) on a tax- 
deductible basis if your spouse isn't covered 
by a retirement plan at work or as long as your 
combined income isn't too high. You also can put 
the same amount tax-deferred into an IRA for a 
nonworking spouse if you file your income tax 
return jointly. 

2. The maximum amount that a married 
couple could possibly save in a tax-deferred 
IRA during a year is 

(A) $3,000. 

(B) $6,000. 

(C) $9,000. 

(D) The question can't be answered based 
on the information contained in the 
passage. 



Presidential appointments are an ongoing 
effort. Some of a president's appointments 
require Senate confirmation. These appoint- 
ments are for positions throughout the federal 
government, for the Cabinet and subcabinet, for 
members of regulatory commissions, for ambas- 
sadorships, for judgeships, and for members of 
numerous advisory boards. 

3. Which of the following statements is NOT 
true? 

(A) All presidential appointments require 
Senate confirmation. 

(B) A position on a regulatory commission 
is an example of a presidential 
appointment. 

(C) Presidential appointments happen 
throughout the president's term in office. 

(D) All of the above statements are true. 



Go on to next page 



Chapter 22: Practice AFQT Exam 22 f 



A link between advertising and alcohol 6. 

consumption is intuitively compelling but hasn't 
been consistently supported by research. Because 
alcohol advertising is pervasive, econometric 
studies may not be sensitive to change or assess 
in a range where change actually makes a differ- 
ence. In dealing with advertising, partial bans 
aren't likely to be effective, and total bans aren't 
practical. Advertising bans in one medium also are 
weakened by substitution of increased advertising 
in alternative media and/or other promotions. 



4. The author of this passage believes that 

(A) advertisement of alcoholic beverages 
should be illegal. 

(B) partial bans on alcohol advertising 
could be effective in some cases. 

(C) bans on alcohol advertising aren't 
likely to work. 

(D) clear links have been established 
between alcohol consumption and 
advertising. 



Which of the following statements is NOT 
supported by the passage? 

(A) Bankruptcy must be filed in a federal 
court. 

(B) Bankruptcy is designed to help individ- 
uals and protect businesses. 

(C) Businesses can be reorganized or 
liquidated through bankruptcy. 

(D) Bankruptcy must be filed in the 
bankruptcy court. 



Questions 7 and 8 are based on the following 
passage. 



The etymology of the word or name Alabama 
has evoked much discussion among philological 
researchers. It was the name of a noted southern 
Indian tribe whose habitat when first known to 
Europeans was in what is now central Alabama. 
One of the major waterways in the state was 
named for this group and from this river, in turn, 7. 

the name of the state was derived. According to 
some investigations, the tribal name Alabama 
must be sought in the Choctaw tongue, because 
it isn't uncommon for tribes to accept a name 
given them by a neighboring tribe. 

5. The state of Alabama was named after 

(A) a Choctaw Indian tribe. 8. 

(B) European settlers. 

(C) a river. 

(D) an Indian chief. 

Each of the 94 federal judicial districts 
handles bankruptcy matters, and in almost all 
districts, bankruptcy cases are filed in the bank- 
ruptcy court. Bankruptcy cases can't be filed in 
state court. Bankruptcy laws help people who 
can no longer pay their creditors get a fresh start 
by liquidating their assets to pay their debts or 
by creating a repayment plan. Bankruptcy laws 
also protect troubled businesses and provide for 
orderly distributions to business creditors 
through reorganization or liquidation. 



The U.S. Department of Justice has prepared 
a report about hate crimes in the United States 
between 1997 and 1999. In 6096 of hate crime inci- 
dents, the most serious offense was a violent 
crime, most commonly intimidation or simple 
assault. The majority of incidents motivated by 
race, ethnicity, sexual orientation, or disability 
involved a violent offense, while two-thirds of 
incidents motivated by religion involved a prop- 
erty offense, most commonly vandalism. Younger 
offenders were responsible for most hate crimes. 
Thirty-one percent of violent offenders and 4696 
of property offenders were under age 18. 



Most property offense hate crimes were 
motivated by 

(A) religion. 

(B) race. 

(C) sexual orientation. 

(D) abortion. 

The majority of hate crimes during this 
period can be classified as 

(A) property offenses. 

(B) violent crimes. 

(C) assault. 

(D) intimidation. 



Go on to next page 



332 



Part V: Practice ASVAB Exams 



Linewatch operations are conducted near 
international boundaries and coastlines in areas 
of Border Patrol jurisdiction to prevent the ille- 
gal entry and smuggling of aliens into the United 
States and to intercept those who do enter ille- 
gally before they can escape from border areas. 
Signcutting is the detection and the interpreta- 
tion of any disturbances in natural terrain condi- 
tions that indicate the presence or passage of 
people, animals, or vehicles. 

9. The activity that's designed to detect 
changes in the natural environment, which 
may indicate passage of illegal aliens, is 
called 

(A) linewatching. 

(B) signcutting. 

(C) Border Patrol Operations. 

(D) Terrain Observation. 

Wales was in ancient times divided into three 
parts nearly equal, consideration having been 
paid, in this division, more to the value than to 
the just quantity or proportion of territory. 
They were Venedotia, now called North Wales; 
Demetia, or South Wales, which in British is 
called Deheubarth, that is, the southern part; and 
Powys, the middle or eastern district. Roderic 
the Great, or Rhodri Mawr, who was king over all 
Wales, was the cause of this division. He had 
three sons, Mervin, Anarawt, and Cadell, amongst 
whom he partitioned the whole principality. 

10. Wales was divided into divisions because 

(A) natural boundaries such as rivers and 
mountains made the division necessary. 

(B) Wales was too large for the King to 
oversee personally. 

(C) the King of Wales wanted his sons 
to rule. 

(D) all of the above 



Questions 11 and 12 are based on the following 
passage. 



The fierce and warlike tribe, called the Huns, 
who'd driven the Goths to seek new homes, 
came from Asia into Southeastern Europe and 
took possession of a large territory lying north of 
the River Danube. During the first half of the fifth 
century, the Huns had a famous king named 
Attila. He was only 21 years old when he became 
their king. But although he was young, he was 
very brave and ambitious, and he wanted to be a 
great and powerful king. As soon as his army was 
ready, he marched with it into countries, which 
belonged to Rome. He defeated the Romans in 
several great battles and captured many of their 
cities. The Roman Emperor Theodosius had to 
ask for terms of peace. Attila agreed that there 
should be peace, but soon afterwards he found 
out that Theodosius had formed a plot to 
murder him. He was so enraged at this that he 
again began war. He plundered and burned cities 
wherever he went, and at last the emperor had 
to give him a large sum of money and a portion 
of the country south of the Danube. 

11. A good title for the above paragraph 
would be 

(A) "The Burning of Rome" 

(B) "Emperor Theodosius" 

(C) "Attila the Hun" 

(D) "Rome for Dummies" 

12. After terms of peace were offered, Attila 
resumed the war against Rome because 

(A) he discovered the emperor wanted to 
assassinate him. 

(B) he wanted to further expand his 
kingdom. 

(C) the emperor of Rome offered too little 
money in the peace terms. 

(D) Danube, his second-in-charge, advised 
him not to accept the peace terms. 



Go on to next page 



Chapter 22: Practice AFQT Exam 33% 



Questions 13 through 15 are based on the fol- 
lowing passage. 



In the military, as in all professions, the issue 
of competence is directly relevant to profes- 
sional integrity. Because human life, national 
security, and expenditures from the national 
treasury are so frequently at issue when the mili- 
tary acts, the obligation to be competent isn't 
merely prudential. That obligation is a moral 
one, and culpable incompetence here is clearly a 
violation of professional integrity. Part of the 
social aspect of professional integrity involves 
the joint responsibility for conduct and compe- 
tence shared by all members of the profession. 
Only fellow professionals are capable of evaluat- 
ing competence in some instances; hence, fellow 
professionals must accept the responsibility of 
upholding the standards of the profession. 
Fellow military members can spot derelictions of 
duty, failures of leadership, failures of compe- 
tence, and the venalities of conduct that inter- 
fere with the goals of the military mission. Often, 
the obligations of professional integrity may be 
pitted against personal loyalties or friendships; 
and, where the stakes for society are so high, 
professional integrity should win out. 



14. Professional competence is 

(A) a moral obligation. 

(B) directly relevant to professional 
integrity. 

(C) essential because military operations 
impact human life, national security, 
and use of taxpayer funds. 

(D) all of the above 

15. The author of the passage would agree that 

(A) friendship must often take a back seat 
to professional integrity. 

(B) only fellow professionals should evalu- 
ate competence. 

(C) professional competence is a direct 
result of effective training programs. 

(D) all of the above 



13. One word that best describes the primary 
theme of this passage would be 

(A) proficiency. 

(B) equality. 

(C) evaluations. 

(D) relationships. 



STOPI 



DO NOT TURN THE PAGE UNTILTOLDTO DO SO. 
DO NOT RETURN TO A PREVIOUS TEST. 



33d 



Part V: Practice ASVAB Exams 



Subtest ft: Mathematics Knowledge 



Time: 24 minutes for 25 questions 

Directions: This section is a test of your ability to solve general mathematical problems. Select the 
correct answer from the choices given and then mark the corresponding space on your answer 
sheet. Use scratch paper to do any figuring. Calculators are not allowed. 



2. 



3. 



4. 



5. 



Solve for x: 5x - 2x = Ix + 2x - 24 


6. 


(12 yards + 14 feet) * 2 = 


(A) 2 




(A) 25 feet 


(B)-2 




(B) 12 feet 


(C)4 




(C) 32 feet 


(D)-4 




(D) 8 feet 


The cube of 6 is 


7. 


x 3 (x 3 ) = 


(A) 125 




(A)* 9 


(B) 225 




(B) 2x 9 


(C) 216 




(C) 2x 6 


(D) 238 




(D)x 6 


In the equation 3x + ly = 21, at what point is 
the x-axis intersected? 


8. 


4 +1 f + 4= 


(A) (7, 0) 




(A)6l 


(B) (0, 7) 

(C) (0, 4) 




©4 


(D) (4, 0) 




(C)5± 


x + y = 6 and x - y = 4. Solve for x. 




(D)7l 

1.5 x 10 2 = 

(A) 45 

(B) 150 

(C) 1,500 

(D) 15 


(A) 3 
(B)5 
(C)7 
(D)8 

Solve for y: 4(y + 3) + 7 = 3 


9. 


(A) 2 
(B)-2 
(C)4 
(D)-4 


10. 


The average of 54, 61, 70, and 75 is 

(A) 50 

(B) 52 

(C) 55 
(D)65 



Go on to next page 



Chapter 22: Practice AFQT Exam ?35 



11. 



12. 



13. 



14. 



2 feet, 4 inches + 4 feet, 8 inches 

(A) 6 feet, 8 inches 

(B) 7 feet 

(C) 7 feet, 2 inches 

(D) 8 feet 

If x - 4, then x 4 * x = 

(A) 12 

(B) 36 
(C)64 
(D) 72 

3x> 14 + 6x 



Solve for x: 5 ■ 

(A) x > -1 

(B) x < -1 

(C) x > -1 

(D) x < -1 

(900 x 3) * 6 = 

(A) 45 

(B) 450 

(C) 55 

(D) 550 

15. If x = 2, then x^x*) = 

(A) 16 

(B) 2x* 
(C)8 
(D)24 

16. Solve for x.x 2 - 2x- 15 = 

(A) 4, -2 

(B) 3, -3 

(C) 5, -3 
(D)-l,l 

17. 



49 W64: 

1 



(A) 
(B) 
(C) 
(D) 



18. If5y 2 = 80, then y is 

(A) a positive number. 

(B) a negative number. 

(C) either a positive or negative number. 

(D) an imaginary number. 

19. If 2 + x > 15, what's the value of x? 

(A) x < 13 

(B) x > 13 
(C)x>13 
(D)x<13 

20. If a circle has a radius of 15 feet, what is its 
circumference most nearly? 

(A) 24 feet 

(B) 72 feet 

(C) 94 feet 

(D) 36 feet 

21. What's the volume of a box measuring 
12 inches long by 8 inches deep by 

10 inches high? 

(A) 960 cubic inches 

(B) 128 cubic inches 

(C) 42 cubic inches 

(D) 288 cubic inches 

22. The following figure is a(n) 



_EL 



(A) parallelogram. 

(B) obtuse triangle. 

(C) trapezoid. 

(D) rectangle. 



Go on to next page 



336 



Part V: Practice ASVAB Exams 



23. The sum of the measures of the angles of a 
parallelogram is 

(A) 360 degrees. 

(B) 540 degrees. 

(C) 180 degrees. 

(D) 720 degrees. 

24. What is the prime factorization of 100? 

(A) 2 x 50 

(B) 2 2 x 5 2 

(C) 4 x 25 

(D) 25 2 



25. -jm is an example of a(n) 

(A) real number. 

(B) imaginary number. 

(C) irrational number. 

(D) sloping number. 



STOPI 



DO NOTTURNTHE PAGE UNTILTOLDTO DO SO. 
DO NOT RETURN TO A PREVIOUS TEST. 



Chapter 23 

Practice AFQT Exam: 
Answers and Explanations 



M M se this answer key to score the Practice AFQT Exam in Chapter 22. Note that the 
%rw actual AFQT is scored by comparing your raw score to the scores of other people, 
which produces a scaled score. Turn to Chapter 1 to find out how the AFQT score is 
derived from the Arithmetic Reasoning, Word Knowledge, Paragraph Comprehension, and 
Mathematics Knowledge subtests. Keep in mind that these four subtests determine whether 
you can even get into the military. If you find you're still struggling in any of these subtest 
areas, you may want to concentrate some additional study effort before knocking on the 
recruiter's door to say, "I'm ready!" 

Subtest 1: Arithmetic Reasoning Answers 

Mathematical word problems can be tough for some people. You have to develop a skill for 
determining which factors are relevant to the problem and then be able to convert those 
factors into a mathematical formula to arrive at a correct solution. Yikes! No wonder there 
are so many math books on the market! If you still need work on this subtest, I recommend 
a few good math books in Chapter 17. Reviewing Chapters 7 and 8 and the additional prac- 
tice questions in Chapter 9 may also help. 

1. A. There are 30 days in April, 31 days in May, and 30 days in June for a total of 91 days. 
Multiply the number of days by the number of haircuts per day: 91 x 35 = 3,185. 

2. B. Multiply the number of words you can type per minute (45) by the number of minutes 
you'll be typing (12): 45 x 12 = 540. 

3. C. Visualize a right triangle, where the kite string represents the hypotenuse and the line 
between Tom and Kathy represents one of the legs. The Pythagorean theorem states that if 
you know the length of two sides of a right triangle, you can determine the length of the 
third side using the formula a 2 + b 2 = c 2 . In this case, 300 2 + b 2 = 500 2 . Solve for b: 

90, 000 + b 2 =250,000 

b 2 =250,000-90,000 
b 2 = 160,000 

6 = ^160,000 

b = 400 

4. A. The formula used to determine the perimeter of a rectangle is P = 2(1 + w). The width is 30, 
and the perimeter is 400. Plug in the numbers and solve for I: 

400 = 2(1 + 30) 

400 = 2/ + 60 

340 = 2/ 

/= 170 



338 



Part V: Practice ASVAB Exams 



5. C. There are two possibilities for the first digit (A or B), 10 possibilities for the second digit 
(0 to 9), and 26 possibilities for the third digit. Using the multiplication principle, 2 x 10 x 
26 = 520 possible access codes. 

6. D. The distance formula is distance equals rate times time, or d - rt. Substitute the known 
values: 93,000,000 = 186,000?. Therefore, t = 500 seconds. Divide 500 by 60 to convert to 
minutes: 500 seconds * 60 seconds/minute = 8=, minutes. 

7. C. The difference in the price is $3, so divide the difference by the original price: $3 + $50 = 
0.06, or 6%. 

8. B. Let x - the unknown number. Set up the formula as = 13. Then solve for x: 

11 + 41 _ io 



X 




52- 

X 


= 13 


52 = 


= 13* 


X- 


= 4 



9. B. Mark received a 4% raise, so to calculate the new wage, start off by taking $9.25 x 0.04 = 
$0.37. Then add that number (the amount of Mark's raise) to his original hourly wage. 
Mark's new hourly wage is $9.25 + $0.37 = $9.62. 

10. A. Let x = the number of nails costing $7 per pound. The total cost of the mixture (M) 
equals the sum of the cost for each type of nail, or M = A + B, where A = Ix, B - 3(6), and 
M = 4(6 + x). Substitute the known values into the equation and solve for x: 

4(6 + x) = Ix + 18 

24 + Ax = Ix + 18 

24-18 = Ix-Ax 

6 = 3x 

x = 2 

11. D. You need to add the fractions, so convert the different denominators to a common 
denominator — 4, 3, and 6 all divide evenly into 12, so use 12 as the common denominator. 

To convert 4-, divide 12 (the new common denominator) by 4 (the original common 

13 3 

denominator) to get 3. Then multiply 4- by # (another way of saying 1). The product is j^ 

13 4o 1412 

(that is, -r = yt). Do the same calculation for the other fractions: ^t = to and -~ = ===y 

To add the fractions, first add the new numerators together: 3 + 4 + 2 = 9. Place the added 

Q 

numerator over the new denominator, 12, and you can see that ^ of the cards have been 
sold or lost. You can reduce ^ to |, which equals 75%. Now find 75% of 24: 24 x 0.75 = 18. 

Eighteen of 24 cards have been sold or lost, so subtract to find the remaining cards: 24 - 18 = 
6 cards remaining. 

12. A. Subtract the sale price from the regular price: $24.00 - $22.50 = $1.50. Multiply the differ- 
ence by the number of CDs to find out how much Theresa saved altogether: $1.50 x 5 = $7.50. 

13. B. First convert the 300 minutes to hours by dividing by 60 (300 minutes + 60 minutes/ 
hour = 5 hours). Use the distance formula (d = rf) and substitute the known values: 200 = 5r; 
r = 40 mph. 

14. C. Let x = the number of people in the class; 60% of x = 21, so 0.60x = 21, andx = 35. 

15. C. Use the interest formula (I - Prt) to determine the amount of interest earned, where the 
principal (P) is 4,000, the rate (r) is 0.03 (3%), and the time (t) is 10. / = 4,000(0.03)(10), or 
/= $1,200. Add the interest earned to the original amount invested: $4,000 + $1,200 = $5,200. 



Chapter 23: Practice AFQT Exam: Answers and Explanations $3 9 



16. B. A rectangle's perimeter is determined by the formula P =2(1 + w). The length of this rect- 
angle is 3 + 2w. Substituting the known values into the formula results in 

36 = 2(w + 3 + 2w) 

36 = 2(3w + 3) 

18 = 3w + 3 

I5 = 3w 

w = 5 

The length is 3 + 2w, so / = 3 + 2(5) = 13. 

17. D. The area of a rectangle is the length x the width of the rectangle, or A = Iw: 50 x 100 = 
5,000 square feet. 

5 5 375 

18. B. A kilometer is % of a mile, so multiply 75 x % = ^-^-. Divide 375 by 8 to determine that 

Eric was traveling at about 47 miles per hour, 7 mph over the 40 mph posted limit. 

19. B. Let x = the number of grapes; 3 apples and 6 pears (twice the number of apples) equals 
i of x, or 

3 + 6 = -ix 

9=2* 
2(9) = x 
x = 18 

20. B. If Janet must purchase 2 pounds of apples to get 1 free pound, then to get 2 free pounds, 
she would need to purchase twice as many apples, or 4 pounds of apples. 

21. A. All pipes are equal in length, so divide the total length, 44 feet, by the total number of 
pipes, 4. The answer, 11, is the length of each individual pipe. You can check this answer 
by multiplying: 4 x 11 = 44. 

22. A. Subtract the Malamute's distance from the German Shepherd's distance (120 - 75 = 45) 
to determine how much closer the Malamute is to the hydrant. 

23. C. Simply add 14 hours to 6 a.m. to reach 8 p.m. Twelve hours from 6 a.m. is 6 p.m., and 
two hours after that brings you to 8 p.m. 

24. B. Multiply 3 pints of strawberries at $1.98 (3 x $1.98 = $5.94), 5 pints of raspberries at 
$2.49 (5 x $2.49 = $12.45), and 1 bushel of peaches at $5.50 (1 x $5.50 = $5.50). Add the 
products together to determine the amount of cash the farmer earned: $5.94 + $12.45 + 
$5.50 = $23.89. 

25. D. Divide 532 by 4 to determine how many feet of shelving will be needed: 532 books * 4 
books/foot =133 feet. 

26. B. Simply add the cost of all the books: $18.00 + $14.50 + $9.95 = $42.45. 

27. A. 8 hours x 60 minutes/hour = 480 minutes. 

28. B. Eighteen people can run three times the number of machines six people can run, because 
18 = 3x6 (divide 18 by 6). Six people can run three machines, so multiply 3x3 machines = 

9 machines. Therefore, 18 people can run nine machines. 

1 115 5 

29. A. Multiply the scale measurement for 1 mile Qk inch per mile) by 5 miles: ix5 = yXy = ^. 

Divide 5 by 2, and you get 2.5 inches. 

30. A. Simply add the cost of all the items: $23.00 + $14.95 + $7.98 + $7.98 = $53.91. 



360 Part V: Practice ASVAB Exams 



Subtest 2: Word Knowledge Answers 



I hope you did well on this subtest. (I was crossing my fingers the whole time!) If not, you 
may want to take another gander at Chapter 4 and the practice questions in Chapter 6. 



l.C 


8. B 


15. C 


22. D 


29. A 


2D 


9. A 


16. C 


23. B 


30. B 


3D 


10. C 


17. A 


24. B 


31. C 


4. B 


11. C 


18. B 


25. D 


32. B 


5. A 


12. C 


19. B 


26. B 


33. B 


6. B 


13. B 


20. C 


27. A 


34. C 


7.C 


14. A 


21. B 


28. C 


35. A 



Subtest 3: Paragraph Comprehension Answers 

If you still need to boost your score, engage in some more reading practice. Improving your 
vocabulary can also help improve your reading comprehension skills. See Chapter 5 for 
some tips. You may also want to try a few of the practice questions in Chapter 6. 

1. C. According to the passage, millions of veterans received home loan guarantees, education, 
and training, making Choice (C) the correct answer. Be careful here, because Choice (A) is 
tempting, but nothing in the passage indicates that the housing, education, and training were 
totally free. 

2. B. The paragraph states that the maximum amount one can place into a tax-deferred IRA is 
$3,000, plus an additional $3,000 if the spouse isn't employed. The question asks about a 
couple, so add $3,000 + $3,000 to get $6,000. 

3. A. Although many presidential appointments require Senate confirmation, not all do. The 
passage mentions only some appointments, so Choice (A) is an incorrect statement. 

4. C. The author specifically states that partial bans on alcohol advertising isn't likely to be 
effective and that total bans wouldn't be practical. 

5. C. According to the passage, a river was named after the Alabama Indian Tribe, and the 
state derived its name from this river. 

6. D. The first sentence states that bankruptcy is usually (not always) filed in bankruptcy 
court, making Choice (D) an incorrect statement. 

7. A. The second sentence states that most violent crimes were motivated by such factors as 
race and sexual orientation, and most property crimes were motivated by religion. The 
question refers to property crimes, so Choice (A) is the right answer. 

8. B. Sixty percent of all hate crimes during the period were violent crimes. Assault and intim- 
idation are examples of violent crimes. 

9. B. The last sentence in the passage describes the signcutting operation. 

10. C. The rationale for the division is explained in the final sentence, which mentions the 
king's sons. The passage makes no reference to the size of Wales or the natural boundaries. 

11. C. The primary subject of this paragraph is Attila, who was king of the Huns. 



Chapter 23: Practice AFQT Exam: Answers and Explanations ju, J 



12. A. Attila agreed to peace but soon after discovered that the Roman emperor had launched 
a plot to kill him. 

13. A. The primary theme of the passage is stated in the first sentence. Proficiency is closest in 
meaning to the word competence, which is the primary theme of the passage. 

14. D. The passage directly supports all the statements. 

15. A. The author specifically states that when pitted against friendship, professional integrity 
should win out. The author explains that only fellow professionals can evaluate other pro- 
fessionals in some (not all) cases, making Choice (B) incorrect. Choice (C) isn't supported 
by information in the passage. 

Subtest ft: Mathematics Knotfiedqe Answers 

It's too bad the ASVAB folks don't allow calculators! That would make this subtest a breeze. 
Don't be discouraged. The problems are purposely designed so that they can be solved using 
only a scratch paper, the ol' No. 2 pencil, and a little brain sweat. If you're still having diffi- 
culty, give Chapter 7 another gander. There are additional practice questions in Chapter 9. 

1. C. 5x - 2x = Ix + 2x - 24; 3x = 9x - 24; -6x = -24; 6x = 24; x = 4 

2. C. The cube of 6 is 6 3 = 6 x 6 x 6 = 216. 

3. A. The equation 3x + ly = 21 is the equation for a line, and a line intersects the x-axis at the 
point where the y-coordinate is (y = 0). Substitute for the y variable in the equation and 
solve for x: 3x + ly = 21; 3x + 7(0) = 21;3x=21;x=7. The point's coordinates are (7, 0). 

4. B. This problem gives you is a system of equations — two equations with two variables. 
You can find x by solving one of the equations for y and plugging that value into the other 
equation. First, x + y = 6, so y = 6 - x. Substitute this known value for y in the second equa- 
tion and solve for x: 

x-y = 4 

x-(6-x) = 4 

x- 6 + x = 4 

2x-6 = 4 

2x= 10 

x = 5 

5. D. 

4(y + 3) + 7 = 3 
4y + 12 + 7 = 3 
4y+ 19 = 3 
4y = -16 
v = -4 

6. A. Convert 12 yards, 14 feet to feet: (12 yards x 3 feet/yard) + 14 feet = 36 feet + 14 feet = 
50 feet. Divide by 2 as instructed: 50 feet * 2 = 25 feet. 

7. D. If two powers have the same base, the numbers can be multiplied by keeping the base 
the same and adding the powers (exponents) together: x 3 (x 3 ) = x 3 * 3 = x 6 . 

8. B. Convert to the lowest common denominator (which is 10), and then add: 

aJ_j_ i 2. i q_3_ _ a _2__i_i _4_i "i_3_ _ q 9 
4 5 + 1 5 +-5 10" 4 10 +i 10 +,3 10" 5 10- 



342 



Part V: Practice ASVAB Exams 



9. B. Following the order of operations, you have to find the power (10 2 ) before multiplying: 

1.5 x 10 2 = 1.5 x (10 x 10) = 1.5 x 100 = 150. 

10. D. Add the numbers and then divide by the number of terms to find the mean. The sum is 
54 + 61 + 70 + 75 = 260. You have four numbers, and 260 * 4 = 65. 

11. B. 2 feet + 4 feet = 6 feet, and 4 inches plus 8 inches = 12 inches (the equivalent to 1 foot). 
Therefore, you have a total of 7 feet. 

12. C. Your first reaction may be to substitute 4 for each x and then do the math. But you can 
save yourself some work (and time) by first dividing x 4 by x, which is x 3 . Then substitute 
4 for each x, and you find that the equation is now 4x4x4. Then multiply: 4 x 4 is 16, and 
16 x 4 is 64, which is your final answer. 

13. B. As you solve this inequality, remember that when you multiply or divide an inequality 
by a negative number, you need to reverse the direction of the inequality sign: 

5-3x> 14 + 6x 

5-3x-6x>14 

-9x>14-5 

-9x>9 

x<-l 

14. B. (900 x 3) * 6 = 2,700 * 6 = 450. 

15. A. Substitute 2 for all the x's: x x (x x ) = 2 2 x 2 2 = 4 x 4 = 16. 

16. C. This is a quadratic equation that you can solve by factoring and setting each factor 
equal to zero: 

x 2 -2x-15 = 

(x-5)(x + 3) = 

x-5 = or x+3 = 

x = 5 x = -3 

17. D. Find the square roots before dividing. The square root of 49 is 7, and the square root of 
64 is 8. Now divide: 7 ■*■ 8 = X. 

18. C. The square root of a positive number can be either positive or negative. For instance, 
the square root of 16 is ±4, because 4 2 and (-4) 2 both give you the positive number 16. 

19. C. Solving this equation doesn't require multiplying or dividing by a negative number, so 
the inequality sign remains the same: 2 + x > 15; x > 13. 

20. C. Circumference equals n times diameter, and diameter is equal to two times the radius. 
In other words, C = nd, and d = 2r. Thus, C = 7i(2)(15) = 7i30. If you round n to 3.14, the 
answer is 94.2, or about 94 feet. 

21. A. Volume equals length times width times height (V= Iwh). Plug in the numbers and solve: 
V= 12 x 8 x 10 = 960 cubic inches. 

22. C. In a trapezoid, two of four sides are parallel to each other. 

23. A. All quadrilaterals have angles that total 360 degrees. 

24. B. Here's how you may have found the prime factors of 100: 100 = 4 x 25 = 2x2x5x5 = 
2 2 x 5 2 . 

25. B. The square root of a negative number doesn't exist as far as real numbers are concerned. 
In mathematics, this is called an imaginary number, and it's represented by the letter ;'. 



Part VI 

The Part of Tens 



The 5 th Wave 



By Rich Tennant 




" Okay— novr tha-t the paramedic is here viith 

the defibrillator and swelling salts, prepare 

■bo open your test booklets...* 



In this part . . . 



■# ou can put your pencil down now — no more quizzes 
^r- or sample tests like the ones in Part V. This part is 
the Part of Tens, which features my personal ASVAB top- 
ten lists. I couldn't write a For Dummies book and not 
include this part. 

I give you important information for doing well on the 
ASVAB, offer some pointers for memorizing crucial con- 
cepts, and point you in the right direction for finding 
more information if you need it. This part presents mate- 
rial in quick tidbits so you can get into the chapters and 
then get out. 

Good luck taking your ASVAB. I hope you get stationed in 
Hawaii! 



Chapter 24 

Ten Surefire Ways to Fail the ASVAB 



In This Chapter 

Avoiding common ASVAB-preparation mistakes 
Steering clear of other people's mistakes 



Technically, you can't fail the ASVAB — it's not a pass/fail test but instead a tool the mili- 
tary uses to measure your potential for learning military duties and military occupa- 
tions. But realistically, each of the branches has established minimum AFQT scores to 
qualify for enlistment and minimum line scores to qualify for certain military jobs. If you 
don't qualify to join the service branch of your choice or don't qualify for the job you want, 
you haven't technically failed, but you may have to take the test again (after some study 
sessions) to get into the branch of service your heart desires. 

But if you avoid the mistakes outlined in this chapter, you can improve your chances of 
qualifying for enlistment and getting the military job of your dreams. 

Choosing Not to Study at All 

Many people think that they don't need to study for the ASVAB. They assume that because 
they studied many of the subjects in high school, they'll do fine even if they just wing it. 

This train of thought isn't true (and it's kind of crazy). Why wouldn't you study? At the very 
least, brushing up on vocabulary and math concepts definitely helps you score higher on 
the ASVAB. Using a calculator is a no-no on the ASVAB, so you may want to revisit some 
math tricks for doing calculations by hand. Auto, Shop, and Mechanical Comprehension 
aren't required high school courses, so these subjects may be completely new to you and 
require additional attention. 

Using study guides like this one not only gives you an idea of what to expect but also allows 
you to sharpen some skills that may have gotten a little dull. 

Failing to Realize Hotf Scores Are Used 

The military powers that be use the nine subtests on the ASVAB to determine which mili- 
tary jobs you qualify for. If you don't know how the scores are used, you can't decide which 
parts of the exam are most important for you to study. 

Check out Chapters 1 and 2 for an explanation of how the military uses ASVAB subtest 
scores to determine your qualifications. Also head to Appendix A to see the scores that you 
need to get into the various branches of and careers in the military. 



366 



Part VI: The Part of Tens 



Studying (or Unnecessary Subtests 




If you don't want to be a mechanic in the military, what are you doing studying for the Auto 
& Shop Information subtest? You should be spending your time on the math and vocabulary 
review, because the math and vocabulary subtests of the ASVAB are used to compute the 
all-important AFQT score, which determines whether you can join the military branch of 
your choice. 

It's easier to study subject areas that you find easy or have an interest in, but if you're already 
an electronics whiz, don't waste your time studying a subject area that you're already going to 
ace. Spend your time studying subject areas that you aren't quite so confident of. 



Losing Focus 





I'm not going to sugarcoat this: The ASVAB is tiring. You have to take nine subtests that 
cover some really diverse subjects. You have about three hours to complete the actual test, 
so if you lose focus while you're taking the test, time has a tendency of slipping away, and 
you may not get to all the questions. It's hard, but keep your mind focused on the task at 
hand throughout the whole test. It'll be over soon. 

Here are some tips that can help you maintain focus: 

v* Arrive at the test location with time to spare. This gives you a few minutes to sit and 
relax before you have to dive into the test questions. 

J*" Leave your baggage at the door. Don't worry about whether you'll get the military job 
you want or whether you'll pass the physical the next morning. You'll have plenty of 
time to worry about that after you've finished the test. 

v* Concentrate on one subtest at a time. Don't waste time rehashing the questions on the 
previous subtest or trying to anticipate the questions on the next subtest. Focus on the 
subtest you're taking at the time. 

v* Take a few moments to relax and refocus between subtests. If you finish a subtest 
with time to spare, close your eyes for a bit and take some deep breaths before you 
begin the next subtest. 

When you answer the final question on one subtest on the CAT-ASVAB, you move immedi- 
ately to the next subtest. If the timer on the computer screen says you have a few minutes 
of time left on the subtest, use that time to relax and refocus before submitting that final 
answer. 



Panicking 0</er Time 




Yes, you have only a limited time to do the test, but don't worry about it. The more you panic, 
the more likely you are to make mistakes. Just work at a steady pace, and you'll do fine. 

Don't spend too much time on any single question. If you're drawing a blank, make a guess 
and move on. (See "Making Wild Guesses or Not Guessing at All," later in this chapter.) 



Chapter 24: Ten Surefire Ways to Fail the ASVAB 2 A "J 



If you're taking the CAT-ASVAB, there will be a timer on the computer screen, counting 
down the number of minutes you have to finish that subtest. If you're taking the pencil and 
paper version of the test, check the clock on the wall. The proctor will generally write the 
start and finish time for the current subtest on a chalkboard. Keep your eye on the time 
remaining, but don't panic over it. 

Deciding Not to Check the Answers 

You should always double-check your answers before you commit to them — you don't 
want to be tripped up by silly mistakes. Don't mark your answer and then check your work. 
Check your work first. 

Do not second-guess yourself (see "Changing Answers," later in this chapter). Just check for 
accuracy (like in mathematical equations). Be sure to mark your answer sheet correctly, 
too, verifying that the number of the question matches the number on your answer sheet 
(you don't have to worry about this with the computerized version of the ASVAB). Getting 
just one question off can mess up the rest of the answer sheet. 

Making Wild Guesses or Not Guessing at Alt 

Take the time to eliminate answers you know are incorrect before choosing among the 
remaining answer options. And here's the number one rule: Don't leave any blank spaces. In 
most cases, guessing if you have to is the way to go — at least you have a higher chance at 
getting the right answer, as opposed to a percent chance if you leave the answer blank. If 
you can eliminate answers you know are wrong before guessing, you increase your chances 
of answering correctly even more. For tips on smart guessing, see Chapter 3. 



Changing Answers 



After you double-check your math, decide that Choice (C) is correct, and mark it on the 
answer sheet, don't change your answer on the paper version of the ASVAB! You're almost 
certain to change a right answer to a wrong one when you play that game. Plus, you can 
drive yourself crazy by second-guessing (and third- and fourth-guessing) your decision. 
Mark the answer and move on. 



Memorizing the Practice Test Questions 

Don't waste your time trying to memorize the practice questions in this book. I can almost 
guarantee you won't see any of the practice questions in this book (or any other study guide) 
on the actual ASVAB. Military test materials are highly controlled items, and no author of an 
ASVAB preparation book has access to them. In fact, military members and military civilian 
employees who disclose actual ASVAB test questions or answers can go to jail — and I'm not 
planning on going to the big house any time soon! 

Just use the practice questions in this book as a measurement tool of which subject areas 
you should spend your time concentrating on. 



3liS 



Part VI: The Part of Tens 



Misunderstanding the Problem 



Make sure that you know what the question wants from you and then give the question 
what it wants. If the problem asks for the sum of two numbers, don't multiply the numbers. 
Don't mistake a division sign for an addition sign. By familiarizing yourself with the types of 
questions on the ASVAB, you'll be able to zero in on what you're supposed to do a lot more 
quickly than those poor folks who didn't have the brilliant idea to buy this book. 



Chapter 25 

Ten Tips for Doing Well on the AFQT 



In This Chapter 

Using your scratch paper wisely 
Reading carefully 
Performing math operations 
Making sure your answers make sense 



■ ommonly referred to as the ASVAB score, the Armed Forces Qualification Test (AFQT) 
^^ score is actually computed only from the reading and math skills subtests of the 
ASVAB. The AFQT score determines whether you're even qualified to enlist in the service of 
your choice. (For the full scoop, see Chapter 1.) The ten concepts presented in this chapter 
help you score better on the four subtests of the ASVAB that are used to calculate your 
AFQT score. 



As Soon as the Test Starts, Write Dou/n 
What \lou're Likely to Forget 

You can't bring your own scratch paper to the test, but the test proctors will give you as 
many sheets of scratch paper as you want. Not only is your scratch paper useful to take the 
place of the calculator you're not allowed to use, but you can also use it to write notes at 
the very beginning of the test, things that you're worried you may forget. For example, if 
you're worried that you'll forget the math order of operations, write it down first thing. 

Read All the Answer Choices before bedding 

I think that those people who write the ASVAB questions must go through a special course 
about being tricky. Many of the answer choices given on the ASVAB are "close but no cigar." 
In other words, these tricksters often try to pull a fast one over on you with an incorrect 
answer that's almost correct but not quite. 

The best defense against this type of trickery is to read each answer choice completely, 
even if you think the first or second choice looks plausible. You're looking for the answer 
that's most correct. 




350 



Part VI: The Part of Tens 




Don't Expect Perfect Word Matches 

jjjftBEfl The Word Knowledge subtest of the ASVAB contains questions that ask you to find the 

word that is closest in meaning to a given word. Don't get confused and think that you have 
to find the word that means exactly the same thing as the given word. Just follow the direc- 
tions. Because some of the answer options may have similar meanings, you need to choose 
the answer that's closest in meaning to the given word — the answer that's most right. 

Read Passages before the Questions 

Some of the Paragraph Comprehension questions can be tricky. The question asks you to 
come to a conclusion based on the information presented in the paragraph. The only way 
to do this effectively is to understand the entire paragraph and what the author wants to 
convey. If you instead read the question first, you may find yourself wasting time by looking 
for information that isn't directly stated. 

Reread to Find Specific Information 

The Paragraph Comprehension subtest often asks you to find specific information in a 
passage. Go back and reread the paragraph. You shouldn't have to guess what this informa- 
tion is — it's in the passage, or you can easily deduce it from the passage. For instance, if a 
paragraph includes the sentence, "Six out of ten smokers will contract some form of 
cancer," and a question asks, "How many smokers won't contract some form of cancer?" 
you can easily deduce that four is the correct number. 

Base Conclusions Only on What \lou Read 

You may have to draw inferences or conclusions from what you've read. You must use only 
the information presented in the paragraph to reach this conclusion instead of relying on 
your own ideas and opinions. In other words, ask yourself, "Would the author agree with 
this statement, based on what he or she has written in this paragraph?" Apply this test to 
each answer option to choose the best answer. 

Change Percents to Decimals 

To perform math operations, you often have to change a percent to a fraction or a decimal. 
To change a percent to a fraction, multiply the percent by j^ and drop the percent sign: 

KO/ _ c _1 5_ 

0/0-0 100" 100 

To change a percent to a decimal, move the decimal point over two places to the left and 
drop the percent sign: 

5% = 0.05 



Chapter 25: Ten Tips for Doing Well on the AFQT 2^ / 



Understand Inverses 



Inverse operations are opposite operations. The opposite of addition is subtraction, and vice 
versa. And the opposite of multiplication is division, and vice versa. 

But when it comes to numbers, the term inverse is not the same as opposite. The opposite of 
5 is -5, but the inverse of 5 is -w. When you deal with numbers, think of writing the inverse of 

5 1 

a number as standing the number on its head: The inverse (reciprocal) of 5, or y, is -?. 



Remember Hou) Ratios, Rates, 
and Scales Compare 

You need to understand the differences among ratios, rates, and scales: 

v* Ratio: A ratio represents any relationship between two objects. If Luis invests $10 in 
Lotto tickets and Joe invests $20 in Lotto tickets, then for every dollar Luis invests, 
Joe invests two. That's a ratio of 1:2. 

i*" Rate: A rate is an expression of the relationship between two unlike elements. For exam- 
ple, if Anna's car can travel a distance of 450 miles per tank of gas and her gas tank 
holds 15 gallons, then her car consumes gas at a rate of 30 miles per gallon, or 30 mpg 
(miles and gallons being unlike elements). Mathematically, 450 (miles) * 15 (gallons in 
the tank) = 30 miles to the gallon. 

j*" Scale: A scale expresses a relationship between two like elements, although the 
units of measure may differ. A map drawn to scale may use 1 inch to represent 1 mile. 
Although an inch and a mile aren't the same unit of measure, they measure the same 
thing (distance). 



Make Sure \lour Answers Are Reasonable 

On the Arithmetic Reasoning and Mathematics Knowledge subtests, you have a very short 
amount of time to answer each question, approximately a minute. But if you use your time 
wisely, you should have plenty of time to double-check and make sure you've chosen the 
correct answer. 



0t&£H 




Your answers should make sense, or they're probably wrong. For example, if you're required 
to compute the average speed that a car maintains during a 2,000 mile trip and your answer 
is 2,000 mph, your answer is probably not correct. If a question includes a formula (such as 
2 + x - 10), plug in your answer for the variable and see whether both sides are equal. So if 
you answered 8, plug in 8 for x and find that 10 = 10. That means your answer checks out. 

Those crazy test-writers who designed the test questions often include wrong answers that 
you may chose if you make a common mistake when solving the problem. Double-checking 
your answers allows you to catch your errors. 



352 



Part VI: The Part of Tens 



Chapter 26 

Ten Ways to Boost Your 
Math and English Skills 



In This Chapter 

Getting used to doing math correctly by hand 
Knowing key rules and formulas 
Expanding your vocabulary 
Becoming a better reader 



T 

m he ASVAB includes two math and two English subtests: Mathematics Knowledge, 
m Arithmetic Reasoning, Word Knowledge, and Paragraph Comprehension. These four 
subtests are probably the most important subtests of the ASVAB, because they comprise 
your AFQT score, which is the score that determines whether or not you qualify to join the 
branch of your choice. (Check out Chapter 1 for more info on the AFQT and Chapter 22 for 
a sample AFQT. You can also peruse my book AFQT For Dummies [published by Wiley].) 

Because these four subtests are so important, this chapter includes ten surefire ways to 
build your skills in these critical areas before you take the test. 



Practice Doing Math Problems 



The best way to get a firm grasp of certain types of math is by doing math problems and not 
simply reading them. Take advantage of the practice math questions in this book, and visit 
the public library to see what kind of high school math textbooks it has to lend. The more 
you do math, the better you'll get at it. 



Put Au/ay \lour Calculator 



You're not allowed to use a calculator when you take the ASVAB, so the time to get used to 
solving basic math problems without one is now, not during the test. You may have been 
taught to rely on a calculator for high school math, but you have to leave your calculator at 
home. Practice working out problems by hand, and make sure you know your multiplication 
tables and other basic calculations. The ASVAB math questions are written by people who 
know how to compute 2 + 2 in their heads. 



351* 



Part VI: The Part of Tens 




As you solve math problems by hand, you can get a feel for what works and what doesn't. 
For instance, some questions ask you to find the square root of a number, which you may 
find hard without a calculator. But a little logic can help. If you know the square root of 9 is 
3 (3 x 3 = 9) and you know the square root of 16 is 4 (4 x 4 = 16), then you can conclude that 
the square root of 12 falls between 3 and 4. 



Memorize the Order of Operations 



0t&£H 




Mathematical equations with multiple steps must be solved in a specific order. Otherwise, 
you won't get the correct answer. Memorize the order in which you do certain calculations 
when you're solving equations, and practice applying these rules well before test day. 

When solving an equation involves multiple steps, the correct order of operations is 

1. Whatever's within parentheses (and other grouping symbols) 

If you have multiple parentheses nested inside each other, do the innermost set first. 
On the ASVAB, the other grouping symbols you run across are the fraction bar and the 
square root sign. Do what's beneath the square root bar before taking the root. Do 
any operation above the fraction bar and any operation below the fraction bar before 
dividing. 

2. Exponents 

3. Multiplication and division 

Operate from left to right. 

4. Addition and subtraction 

Again, work from left to right. 

For example, 3 + 2x3 isn't equal to 5 x 3, or 15. The correct answer is 9. You first do the 
multiplication and then the addition. You can remember order of operations as "Please 
Excuse My Dear Aunt Sally," or PEMDAS. Check out Chapter 7 for more explanation. 



Knou) l/our Geometry Formulas 



You encounter some math questions that require you to calculate area, perimeter, and 
volume on the ASVAB. Memorize the following area formulas: 

u* Area of a rectangle: For any rectangle, Area = Length x Width: A - Iw. 

v* Area of a triangle: For triangles, Area = Base x Height (or altitude) divided by 2: 
A = ^bh. 

e* Area of a circle: For circles, area is n (approximately 3.14) times the radius squared: Anr 2 . 

Know these perimeter and circumference formulas: 

v* Perimeter of polygon (a shape with straight sides): Calculate the perimeter of any 
quadrilateral (four-sided figure) or triangle by adding the lengths of all the sides 
together. 

v* Circumference of a circle: Find the circumference of a circle by multiplying n times the 
diameter: C = nd. Note: Diameter equals 2 times the radius. 



Chapter 26: Ten Ways to Boost Your Math and English Skills $55 



And know these formulas for the volume of 3-D solids: 



j*" Volume of a box: Find the volume of a rectangular solid by multiplying Length x Width x 
Height: V=lwh. 

U* Volume of a cylinder: Find the volume of a cylinder by multiplying the area of the 
circular base (n times the base's radius squared) by the cylinder's height: V= tuP-h. 



Keep a Word List 




The English language has well over 200,000 words in it, so nobody expects you to know 
them all. However, the ASVAB writers do expect you to have a good grasp of many vocabu- 
lary words. One way to improve your vocabulary is to keep a word list. 

How does a word list work? As you read, write down the words that you don't know. Quickly 
look them up in the dictionary. You can then apply your word list in your day-to-day life. Of 
course, you can't remember every single word, but you can focus on mastering one new 
word every day and using it in conversation. Check out Chapter 4 for more tips on building 
your vocabulary. 

Don't waste your time and choose little-known words, such as absquatulate (which means to 
leave hurriedly or secretly). You're unlikely to see obscure words on the ASVAB, but you 
should make a practice of learning the meaning of as many common English words as possible. 



Study Latin and Greek 



You can skip the grammar and pronunciation, but you should get to know some of the 
roots, prefixes, and suffixes that English has borrowed from Latin and Greek. These word 
parts are the building blocks of much of the English language, and they can give you clues 
about what words mean. 

If you see an unfamiliar word on the Word Knowledge section, try to figure out its root. 
For example, if you know the meaning of mercy, you can figure out the meaning of merciful. 
Remember that prefixes and suffixes can be added onto a root to change the word's mean- 
ing or function. Here are some examples: 

v* Changing meaning: The prefix a- usually means opposite, so the word atypical means 
the opposite of typical, not a typical thing. 

v* Changing parts of speech: Establish is a verb meaning to make stable or to prove, 

whereas establishment (with a suffix) is a noun meaning a thing that has been established. 

For a list of common word parts you should know, check out Chapter 4. 



Use Flashcards 



Flashcards help you remember important facts through the process of spaced repetition. 
Learning psychologists agree that this is one of the most effective methods of memorizing 
new information. Plus, it's cheap — all you need is a set of blank index cards and a pen to 
create your very own studying machine. 



356 



Part VI: The Part of Tens 



You can use flashcards to improve both your mental math and vocabulary — write down 
vocab words, roots, prefixes, and suffixes, practice matching square roots and square num- 
bers, or just make sure you know your math formulas. 



Read More, Watch TV Less 



The best way to improve your reading comprehension is simple: Read more. If you spend 
four hours a day watching TV or surfing the Web, you can instead use those four hours to 
read a novel or the newspaper or a book about car repair — whatever interests you the 
most. You'll be surprised at how fast your reading speed and comprehension improve with 
just a little daily practice. 

Practice Finding Main and Supporting Points 

All writing should have a point. The main point is the thing that the writer wants you to 
take away from his or her words. Some passages include more than one point. Usually, such 
passages have one main point and one or more subpoints that support the main idea. As 
you're reading passages on the ASVAB (and in real life), you want to be able to easily iden- 
tify the main point. You should practice identifying the points during your own reading 
sessions. Read each paragraph and then ask yourself what information the author is trying 
to convey to you. 



Use a Study Guide 



In my humble opinion, there is no better commercial ASVAB study guide available than this 
one. Read the chapters carefully and then utilize the practice tests to see where you need 
more study. 

ajtfNG/ Use the practice questions only to test your own knowledge of the subject. Don't expect to 

*/Ji^ see the same questions on the actual ASVAB. Those test-makin' hooligans who write the 
actual ASVAB tests keep a close eye on commercial study guides, like this one, and try to 
avoid having the same questions. 




*>£!? 




Appendix 

Matching ASVAB Scores 
to Military Jobs 



T 

m he military has hundreds of enlisted job opportunities, ranging from washing and 
m sewing clothing items to translating foreign languages. Each of the military services has 
established its own individual line score requirements (a combination of various ASVAB 
subtest scores) for specific enlisted jobs. The tables in this appendix show the minimum 
line scores that the services have established for entry-level enlisted jobs. 



Just because you achieve the minimum ASVAB line score for the job of your choice doesn't 
mean you'll absolutely get that job. Other factors are considered, including the current 
needs of the service, security clearance qualification, and medical exam results. 

The charts in this appendix are as accurate as they can be at press time. However, military 
jobs and qualification standards are subject to change with little or no notice. For the most 
up-to-date information and for complete job descriptions and qualification factors, see your 
local military recruiter or visit the military enlisted-job pages on the About.com U.S. Military 
Information site (usmilitary. about . com). 



Army Enlisted Jobs 



The Army calls its enlisted jobs Military Occupation Specialties (MOS), and more than 150 
such specialties exist for entry-level recruits. Table A-l shows entry-level Army MOSs and 
the ASVAB line scores required to qualify for the jobs. Scan the table and see whether you 
find a job that interests you. 

Line scores are abbreviated as follows: Clerical (CL), Combat (CO), Electronics (EL), Field 
Artillery (FA), General Maintenance (GM), General Technical (GT), Mechanical Maintenance 
(MM), Operators and Food (OF), Surveillance and Communications (SC), and Skilled 
Technical (ST) scores. See Chapter 2 for an explanation of which ASVAB subtest scores are 
used to calculate each of the line scores. 



358 ASVAB For Dummies, 3rd Edition 





















Table A-1 


Army Enlisted Jobs and Requi 


red ASVAB Scores 




MOS 


Title 


Score 


MOS 


Title 


Score 


MOS 


Title 


Score 


11B 


Infantryman 


CO-90 


11C 


Indirect Fire 
Infantryman 


CO-90 


13B 


Cannon 
Crewmember 


FA-95 


13C 


Tactical 
Automated 
Fire Control 
Systems 
Specialist 


FA-95 


13D 


Field Artillery 
Automated 
Tactical Data 
Systems 
Specialist 


FA- 100 


13E 


Cannon Fire 

Direction 

Specialist 


FA-95 


13F 


Fire Support 
Specialist 


FA- 100 


13M 


Multiple 

Launch 

Rocket 

System 

Crewmember 


OF-105 


13P 


Multiple 

Launch 

Rocket 

System 

Operations/ 

Fire Direction 

Specialist 


FA- 100 


13R 


Field Artillery 
Firefinder 
Radar 
Operator 


EL-100 

and 
SC-100 


13W 


Field Artillery 
Meteor- 
ological 
Crewmember 


EL-95 


14E 


PATRIOT 

Missile 

System 

Enhanced 

Operator/ 

Maintainer 


MM-105 


14J 


Early 
Warning 
System 
Operator 


MM-100 

and 

GT-100 


14M 


Man 
Portable 
Air Defense 
System 
Crewmember 


OF-90 


14R 


Bradley 

Linebacker 

Crewmember 


OF-100 


14S 


AVENGER 
Crewmember 


OF-90 


14T 


PATRIOT 

Launching 

Station 

Enhanced 

Operator/ 

Maintainer 


OF-100 


15B 


Aircraft 
Power Plant 
Repairer 


MM-105 


15D 


Aircraft 

Powertrain 

Repairer 


MM-105 


15F 


Aircraft 
Electrician 


MM-105 


15G 


Aircraft 

Structural 

Repairer 


MM-105 


15H 


Aircraft 

Pneudraulics 

Repairer 


MM-105 


15J 


Aircraft 

Armament/ 

Electrical/ 

Avionics 

Systems 

Repairer 


EL-100 

and 

MM-105 


15M 


Utility 

Helicopter 

Repairer 


MM-105 


15N 


Avionic 
Mechanic 


EL-95 


15P 


Aviation 

Operations 

Specialist 


ST-95 


15Q 


Air Traffic 

Control 

Operator 


ST-100 


15R 


AH-64 Attack 

Helicopter 

Repairer 


MM-100 


15S 


0H-58D 

Helicopter 

Repairer 


MM-100 


15T 


UH-60/ 

Helicopter/ 

Utility 

Airplane 

Repairer 


MM-105 


15U 


Medium 

Helicopter 

Repairer 


MM-105 


15V 


Scout/ 

Observation/ 

Attack 

Helicopter 

Repairer 


MM-105 


15X 


AH-64 

Armament/ 

Electrical 

Systems 

Repairer 


EL-100 



Appendix: Matching ASVAB Scores to Military Jobs jjy 



MOS 


Title 


Score 


MOS 


Title 


Score 


MOS 


Title 


Score 


15Y 


AH-64D 


EL-100 


18B 


Special 


GT-110 


18C 


Special 


GT-110 




Armament/ 


and 




Forces 


and 




Forces 


and 




Electrical/ 


MM-105 




(Weapons) 


CO-100 




(Engineer) 


CO-100 




Avionics 


















Systems 


















Repairer 
















18D 


Special 


GT-100 


18E 


Special 


GT-110 


19D 


Cavalry 


CO-90 




Forces 


and 




Forces 


and 




Scout 






(Medical) 


CO-100 




(Communi- 
cations) 


SC-100 








19K 


Ml Armor 
Crewman 


CO-90 


21 B 


Combat 
Engineer 


CO-90 


21 C 


Bridge Crew- 
member 


CO-90 


21 D 


Diver 


GM-100 
and 
ST- or 
GT-110 


21 E 


Heavy 

Construction 
Equipment 
Operator 


GM-90 


21 F 


Crane 
Operator 


GM-90 


21 G 


Quarrying 
Specialist 


GM-95 


21 J 


General 
Construction 
Equipment 
Operator 


GM-90 


21 K 


Plumber 


GM-90 


21 L 


Lithographer 


ST-85 


21 M 


Firefighter 


GM-90 


21 P 


Prime Power 

Production 

Specialist 


ST-110 


21 Q 


Transmis- 
sion and 
Distribution 
Specialist 


EL-95 


21 R 


Interior 
Electrician 


EL-95 


21T 


Technical 

Engineering 

Specialist 


ST-95 


21 U 


Topographic 
Analyst 


ST-100 


21V 


Concrete 
and Asphalt 
Equipment 
Operator 


GM-90 


21 W 


Carpentry 
and Masonry 
Specialist 


GM-90 


25B 


Information 


ST-100 


25C 


Radio 


SC-100 


25D 


Telecom- 


SC-90 




Systems 






Operator/ 


and 




munications 


and 




Operator/ 






Maintainer 


EL-100 




Operator/ 


EL-90 




Analyst 












Maintainer 




25F 


Network 


SC-105 


25L 


Cable 


SC-90 


25M 


Multimedia 


SC-95 




Switching 


and 




Systems 


and 




Illustrator 


and 




Systems 


EL-105 




Installer/ 


EL-90 






EL-95 




Operator/ 






Maintainer 












Maintainer 
















25P 


Microwave 
Systems 
Operator/ 
Maintainer 


EL-110 


25Q 


Multichannel 
Transmis- 
sion Systems 
Operator/ 
Maintainer 


SC-100 

and 
EI-100 


25R 


Visual 

Information 

Equipment 

Operator/ 

Maintainer 


EL-110 


25S 


Satellite 
Communi- 
cation 
Systems 
Operator/ 
Maintainer 


EL- 120 


25U 


Signal 
Support 
Systems 
Specialist 


ST-95 

and 
EL-95 


33W 


Military 

Intelligence 

Systems 

Maintainer/ 

Integrator 


ST-115 



(continued) 



360 



ASVAB For Dummies, 3rd Edition 



Table A-1 (continued) 


MOS Title 


Score 


MOS 


Title 


Score 


MOS 


Title 


Score 


37F Psychological 
Operations 
Specialist 


ST- 105 


38A 


Civil Affairs 
Specialist 


ST- 100 


42A 


Human 

Resources 

Specialist 


CL-95 


42F Information 
Systems 
Technician 


CL-105 


42L 


Adminis- 
trative 
Specialist 


CL-95 


42R 


Band 
Member 


N/A 



44C Finance 
Specialist 



CL-105 45B Small Arms/ 
Artillery 
Repairer 



GM-90 



45D 



Self- 
propelled 
Field Artillery 
Turret 
Mechanic 



52D Power- 
Generation 
Equipment 
Repairer 



GM-100 52F 



Turbine 
Engine Drive/ 
Generator 
Repairer 



GM-100 



45G 


Fire Control 

System 

Repairer 


EL-95 


45K 


Armament 
Repairer 


GM-100 


45N 


M60A1/A3 
Tank Turret 
Mechanic 


MM-100 


46Q 


Journalist 


GT-110 


46R 


Broadcast 
Journalist 


GT-110 


52C 


Utilities 

Equipment 

Repairer 


GM-100 



GM-100 56M Chaplain CL-95 

Assistant 



63A 



M1 ABRAMS 

Systems 

Maintainer 



MOO 63B 



Light-Wheel 

Vehicle 

Mechanic 



MM -90 63D 



Artillery 
Repairer 



MM-105 



63G 


Fuel and 


MM-105 


63H 


Track 


MM-90 


63J 


Quarter- 


MM-90 




Electrical 






Vehicle 






master and 






Systems 






Repairer 






Chemical 






Repairer 












Equipment 
Repairer 




63M 


M2/3 Bradley 

Fighting 

Vehicle 

System 

Maintainer 


MM-105 


63N 


M60A1/A3 
Tank System 
Mechanic 


MM-100 


63S 


Heavy- 
Wheel 
Vehicle 
Mechanic 


MM-105 


63W 


Wheel 

Vehicle 

Repairer 


MM-90 


63Y 


Track 

Vehicle 

Mechanic 


MM-105 


63S 


Heavy- 
Wheel 
Vehicle 
Mechanic 


MM-105 


68D 


Operating 

Room 

Specialist 


ST-95 


68E 


Dental 
Specialist 


ST-95 


68G 


Patient 
Admini- 
stration 
Specialist 


CL-95 



68H 



Optical 

Laboratory 

Specialist 



GM-100 68J 



Medical 

Supply 

Specialist 



CL-95 



68K 



Medical 

Laboratory 

Specialist 



ST-110 



68M Hospital 

Food Service 
Specialist 



OF-100 



68P 



Radiology 
Specialist 



ST-110 



68Q 



Pharmacy 
Specialist 



ST-95 



68R 



Veterinary 
Food 

Inspection 
Specialist 



ST-100 



68S 



Preventive 

Medicine 

Specialist 



ST-105 



68T 



Animal Care 
Specialist 



ST-105 



Appendix: Matching ASVAB Scores to Military Jobs 3 v / 



MOS 


Title 


Score 


MOS 


Title 


Score 


MOS 


Title 


Score 


68V 


Respiratory 
Specialist 


ST- 105 


68W 


Healthcare 
Specialist 


ST-95 


68X 


Mental 
Health 
Specialist 


ST- 105 


74D 


Chemical 

Operations 

Specialist 


ST-95 


88H 


Cargo 
Specialist 


GM-90 


88K 


Watercraft 
Operator 


MM-100 



88L 



Watercraft 
Engineer 



1M-105 88M 



Motor 

Transport 

Operator 



OF-90 



88N 



Transpor- 
tation 

Management 
Coordinator 



CL-100 



88P 



Railway 

Equipment 

Repairer 



MM-100 



88T Railway 
Section 
Repairer 



1M-90 88U 



Railway 

Operations 

Crewmember 



MM-95 



89B 



94D 



Ammunition 
Specialist 



ST-100 



89D 



Air Traffic 
Control 
Equipment 
Repairer 



Explosive 


GM-105 


92A 


Automated 


Ordnance 






Logistical 


Disposal 






Specialist 


(EOD) 








Specialist 









EL-105 



94E 



Radio and 
Communica- 
tions 
Security 
Repairer 



EL-110 



94F 



Special 
Electronic 
Devices 
Repairer 



CL-95 



92F 


Petroleum 

Supply 

Specialist 


CL-90 

and 

OF-90 


92G 


Food Service 
Specialist 


OF-90 


92L 


Petroleum 
Laboratory 
Specialist 


ST-105 


92M 


Mortuary 

Affairs 

Specialist 


GM-90 


92R 


Parachute 
Rigger 


GM-90 

and 

CO-90 


92S 


Fabric Repair 
Specialist 


GM-85 


92W 


Water 

Treatment 

Specialist 


GM-90 


92Y 


Unit Supply 
Specialist 


CL-95 


94A 


Land Combat 
Electrician 


EL-105 



EL-105 



94H 



Test 

Measure- 
ment and 
Diagnostic 
Equipment 
Support 
Specialist 



EL-110 



94K 



Automatic 
Test 

Equipment 
Operator and 
Maintainer 



EL-110 



94L 



Avionic 
Communi- 
cations 
Equipment 
Repairer 



EL-100 



94M Radar 
Repairer 



EL-110 



94P 



Multiple 
Launch 
Rocket 
Repairer 



EL-100 



94R 



Avionic 

Radar 

Repairer 



EL-100 



94S Patriot 
System 
Repairer 



EL-100 



94T 



Avenger 

System 

Repairer 



EL-100 



94Y 



Integrated 
Family 
of Test 
Equipment 
Operator and 
Maintainer 



EL-110 



(continued) 



362 



ASVAB For Dummies, 3rd Edition 





















Table A-1 (continued) 


MOS 


Title 


Score 


MOS 


Title 


Score 


MOS 


Title 


Score 


96B 


Intelligence 
Analyst 


ST- 105 


96D 


Imagery 
Analyst 


ST-95 


96H 


Common 
Ground and 
Station (CGS) 
Operator 


SC-95 

and 

ST-105 


96R 


Ground 
Surveillance 
Systems 
Operator 


EL-85 

and 

SC-95 


96U 


Tactical 

Unmanned 

Aerial 

Vehicle 

Operator 


SC-105 


97E 


Human 

Intelligence 

Collector 


ST-95 


97L 


Translator/ 
Interpreter 


ST-95 


98C 


Signals 

Intelligence 

Analyst 


ST-105 


98G 


Cryptologic 
Linguist 


ST-95 


98H 


Communi- 
cations 
Locator/ 
Interceptor 


ST-95 


98J 


Electronic 
Intelligence 
Interceptor/ 
Analyst 


ST-95 


98K 


Signals 
Collection/ 
Identification 
Analyst 


ST-105 



Air Force Enlisted Jobs 



The United States Air Force has about 120 entry-level enlisted jobs for new recruits. The Air 
Force refers to enlisted jobs as Air Force Specialty Codes (AFSC). Table A-2 shows the Air 
Force entry-level AFSCs and the line scores required to qualify for each job. The table is 
organized by AFSC number, so browse the table and see which AFSCs pique your interest. 

Line scores are abbreviated as follows: General (G), Electronic (E), Mechanical (M), and 
Administrative (A). See Chapter 2 for information on which ASVAB subtest scores are used 
by the Air Force to calculate the various line scores. 



Table A-2 


Air Force Enlisted Jobs and Requi 


red ASVAB Scores 




AFSC 


Title 


Score 


AFSC 


Title 


Score 


AFSC 


Title 


Score 


1A0X1 


In-Flight 
Refueling 


G-55 


1A1X1 


Flight 
Engineer 


M-47 

or 

E-38 


1A2X1 


Aircraft 
Loadmaster 


G-55 


1A3X1 


Airborne 
Communi- 
cations and 
Electronic 
Systems 


E-70 


1A4X1 


Airborne 
Battle 

Management 
Systems 


G-55 


1A5X1 


Airborne 
Missions 
Systems 


E-70 


1A7X1 


Aerial Gunner 


M-60 

or 

E-45 


1A8X1 


Airborne 

Cryptologic 

Linguist 


G-72 


1C0X1 


Airfield 
Management 


A-41 


1C1X1 


Air Traffic 
Control 


G-55 

and 

M-55 


1C2X1 


Combat 
Control 


G-44 


1C3X1 


Command 
Post 


G-49 


1C4X1 


Tactical Air 
Command and 
Control 


G-49 


1C5X1 


Aerospace 
Control and 
Warning 
Systems 


G-55 


1C6X1 


Space 

Systems 

Operations 


E-60 



Appendix: Matching ASVAB Scores to Military Jobs j03 



AFSC Title 



Score AFSC 



Title 



Score AFSC 



Title 



Score 



1N0X1 Operations 
Intelligence 



G-57 



1N1X1 



Imagery 
Analysis 



G-66 



1N2X1 



Communica- 
tions Signals 
Intelligence 
Production 



G-53 



1N3XX Cryptologic 
Linguist 



G-72 



1N4X1 



Network 

Intelligence 

Analysis 



G-62 



1N5X1 



Electronic 
Signal 
Intelligence 
Exploitation 



G-72 



1N6X1 Electronic 
Systems 
Security 
Assessment 



G-62 



1T0X1 



Survival, 
Evasion, 
Resistance, 
and Escape 
Operations 



G-55 



1T1X1 



Aircrew Life 
Support 



G-34 



1T2X1 Pararescue 



G-44 



1W0X1 Weather 



G-66 

and 

E-50 



2A0X1 



Avionics Test 
Stations and 
Components 



E-70 



2A3X1 



A-10, F-15, 
and U-2 
Avionics 
Systems 



E-70 



2A3X2 



F-16, F-117, 

RQ-1,and 

CV-22 

Avionics 

Systems 



E-70 



2A3X1 



Tactical 
Aircraft 
Maintenance 



2E0X1 Ground Radar 
Systems 



E-70 



2E1X1 



Satellite, 
Wideband, 
and Telemetry 
Systems 



E-70 



2E1X2 



Meteoro- 
logical and 
Navigations 
Systems 



-47 



2A5X1 


Aerospace 
Maintenance 


M-47 


2A5X2 


Helicopter 
Maintenance 


M-56 


2A5X3 


Integrated 

Avionics 

Systems 


E-70 


2A6X1 


Aerospace 
Propulsion 


M-56 


2A6X2 


Aerospace 

Ground 

Equipment 


M-47 

and 

E-28 


2A6X3 


Aircrew 

Egress 

Systems 


M-56 


2A6X4 


Aircraft Fuel 
Systems 


M-47 


2A6X5 


Aircraft 

Hydraulic 

Systems 


M-56 


2A7X1 


Aircraft 

Metals 

Technology 


M-47 


2A7X2 


Nondestruc- 
tive 
Inspection 


M-42 


2A7X3 


Aircraft 

Structural 

Maintenance 


M-47 


2A7X4 


Survival 
Equipment 


M-40 



E-70 



2E1X3 Ground Radio 
Communi- 
cations 



E-70 



2E1X4 



Visual 

Imagery and 
Intrusion 
Detection 
Systems 



E-70 



2E2X1 



Computer, 
Network, 
Switching, 
and Crypto- 
graphic 
Systems 



E-70 



2E6X2 Communi- 
cations Cable 
and Antenna 
Systems 



M-47 2E6X3 



Telephone 
Systems 



E-45 



2F0X1 Fuels 



M-47 

and 

G-38 



2G0X1 



Logistics 
Plans 



A-56 2M0X1 



Missile 
and Space 
Systems 
Electrical 
Maintenance 



E-70 



2M0X2 



Missile 
and Space 
Systems 
Maintenance 



-47 



(continued) 



361, 



ASVAB For Dummies, 3rd Edition 



Table A-2 (continued) 



AFSC Title 



Score AFSC 



Title 



Score AFSC 



Title 



Score 



2M0X3 



2T3X1 



3C0X2 



Missile 
and Space 
Facilities 



E-50 



2P0X1 



Precision 
Measurement 
Equipment 
Laboratory 



E-70 2R0X1 



Maintenance 
Data Systems 
Analysis 



Special 
Purpose 
Vehicle and 
Equipment 
Maintenance 



M-47 2T3X2 



Special 
Vehicle 
Maintenance 



M-40 2T3X5 



Vehicle Body 
Maintenance 



Computer 

Systems 

Programming 



G-64 3C1X1 



Radio Com- 
munication 
Systems 



A-41 3C1X2 



Electro- 
magnetic 
Spectrum 
Management 



G-55 



2R1X1 


Maintenance 
Scheduling 


G-44 


2S0X1 


Material 
Management 


A-41 

or 

G-44 


2S0X2 


Supply 

Systems 

Analysis 


A-47 


2T0X1 


Traffic 
Management 


A-35 


2T1X1 


Vehicle 
Operations 


M-40 


2T2X1 


Air Trans- 
portation 


M-47 

and 

A-28 



M-56 



2T3X7 


Vehicle 


A-41 


2W0X1 


Munitions 


M-55 


2W1X1 


Aircraft 


M-60 




Management 






Systems 


or 




Armament 


or 




and Analysis 








G-55 




Systems 


E-45 


2W2X1 


Nuclear 
Weapons 


M-60 


3A0X1 


Information 
Management 


A-28 


3C0X1 


Computer 

Systems 

Operations 


G-64 



G-44 



3C2X1 



3E1X1 



3V0X1 



Computer 

Systems 

Control 



E-70 



3C3X1 



Computer 
Systems 
Planning and 
Implemen- 
tation 



G-62 3M0X1 Services 



Heating, 


M-47 


3E2X1 


Pavement and 


Ventilation, 


or 




Construction 


Air Condi- 


E-28 




Equipment 


tioning, and 








Refrigeration 









M-40 3E3X1 Structural 



Visual 
Information 



G-44 



3V0X2 



Still 
Photograph 



G-44 



3V0X2 



Visual 
Information 
Production- 
Documen- 
tation 



G-24 



3N0X1 


Public Affairs 


G-72 


3N0X2 


Radio and TV 
Broadcasting 


G-72 


3N1X1 


Regional 
Band 


A-21 

or 

G-24 


3P0X1 


Security 
Forces 


G-33 


3E0X1 


Electrical 
Systems 


E-28 


3E0X2 


Electric 

Power 

Production 


M-56 

and 

E-40 



-47 



3E4X1 


Utilities 
Systems 


M-47 


3E4X2 


Liquid Fuel 

Systems 

Maintenance 


M-47 


3E4X3 


Pest 
Management 


G-38 


3E5X1 


Engineering 


G-49 


3E6X1 


Operations 
Management 


G-44 


3E7X1 


Fire 
Protection 


G-38 


3E8X1 


Explosive 
Ordnance 
Disposal 


G-64 

and 

M-60 


3E9X1 


Readiness 


G-62 


3S0X1 


Personnel 


A-41 



G-62 



Appendix: Matching ASVAB Scores to Military Jobs jOS 



AFSC 


Title 


Score 


AFSC 


Title 


Score 


AFSC 


Title 


Score 


4A0X1 


Health 

Services 

Management 


G-44 


4A1X1 


Medical 
Materiel 


G-44 


4A2X1 


Biomedical 
Equipment 


E-70 
and 
M-60 


4B0X1 


Bio- 
environmental 
Engineering 


G-49 


4C0X1 


Mental Health 
Services 


G-55 


4D0X1 


Diet Therapy 


G-44 


4E0X1 


Public Health 


G-44 


4H0X1 


Cardio- 
pulmonary 
Lab 


G-44 


4J0X2 


Physical 
Medicine 


G-49 


4M0X1 


Aerospace 
Physiology 


G-44 


4N0X1 


Aerospace 

Medical 

Service 


G-44 


4N1X1 


Surgical 
Services 


G-44 


4P0X1 


Pharmacy 


G-44 


4R0X1 


Diagnostic 
Imaging 


G-44 


4T0X1 


Medical 
Laboratory 


G-62 


4T0X2 


Histo- 
pathology 


G-44 


4T0X3 


Cytotech- 
nology 


G-44 


4V0X1 


Optometry 


G-55 


4Y0X1 


Dental 
Assistant 


G-44 


4Y0X2 


Dental Lab 


G-66 


5R0X1 


Chaplain 
Assistant 


G-44 

or 

A-35 



6C0X1 Contracting 



G-72 



6F0X1 



Financial 
Management 
and 
Comptroller 



G-57 



9S100 



Technical 

Applications 

Specialist 



and 
E-85 



NaVy Enlisted Jobs 



The Navy calls its enlisted jobs ratings and has about 75 types of jobs available for entry- 
level recruits. This branch doesn't use line scores for job-qualification purposes. Instead, 
the Navy combines scores from the various ASVAB subtests for each of its enlisted ratings. 

Table A-3 (in ratings order) shows combinations of ASVAB subtest scores that are required 
to qualify for Navy enlisted jobs. Peruse the list and see which jobs may best suit you. The 
ASVAB subtests are abbreviated as follows: General Science (GS), Arithmetic Reasoning (AR), 
Word Knowledge (WK), Paragraph Comprehension (PC), Auto & Shop Information (AS), 
Mathematics Knowledge (MK), Mechanical Comprehension (MC), Electronics Information 
(EI), Assembling Objects (AO), and Verbal Expression (VE) scores. 



Table A-3 


Navy Enlisted Jobs and Required ASVAB Scores 




Rating 


Title 


Score 


Rating 


Title 


Score 


Rating 


Title 


Score 


ABE 


Aviation 


AR + 


ABF 


Aviation 


AR + 


ABH 


Aviation 


MC + 




Boatswain's 


MC + 




Boatswain's 


MC + 




Boatswain's 


AR + 




Mate — 


AS = 




Mate — Fuels 


AS = 




Mate — 


MC + 




Equipment 


130 






130 




Handling 


AS = 
130 


AC 


Air Traffic 


AR + 


AD 


Aviation 


AR + 


AE 


Aviation 


AR + 




Controller 


2MK 




Machinist's 


MK + 




Electrician's 


MK + 






+ GS 




Mate 


EI + 




Mate 


EI + 






= 210 






GS = 
190 






GS = 
218 



(continued) 



366 



ASVAB For Dummies, 3rd Edition 



Table A-3 (continued) 



Rating Title 



Score Rating Title 



Score Rating Title 



Score 



AECF 


Advanced 


AR + 


AG 


(Aviation) 


VE + 


AIR- 


Aircrew 


VE + 




Electronics 


MK + 




Aerographer's 


MK + 


CREW 


Program 


AR + 




Computer 


EI + 




Mate 


GS = 






MK + 




Field 


GS = 
222 






165 






MC = 
210 or 
VE + 
AR + 
MK + 
AS = 
210 


AM 


Aviation 


VE + 


AME 


Aviation 


AR + 


AO 


Aviation 


AR + 




Structural 


ARE + 




Structural 


MC + 




Ordnanceman 


MK + 




Mechanic 


MK + 
AS = 
210 or 
VE + 
AR + 
MK + 
MC = 
210 




Mechanic — 
Equipment 


AS = 

164 






EI + 
GS = 
190 


AS 


Aviation 


AR + 


AT 


Aviation 


AR + 


AW 


Aviation 


AR + 




Support 


2MK 




Electronics 


MK + 




Warfare 


2MK 




Equipment 


+ GS 




Technician 


EI + 




Systems 


+ GS 




Technician 


= 200 






GS = 
218 or 
MK + 
EI + 
GS = 
156 




Operator 


= 196 


AZ 


Aviation 


VE + 


BM 


Boatswain's 


VE + 


BU 


Builder 


AR + 




Maintenance 


AR = 




Mate 


AR + 






MC + 




Administra- 


103 






MK + 






AS = 




tionman 








AS = 
175 or 
MK + 
AS + 
A0 = 
135 






140 


CE 


Construction 


AR + 


CM 


Construction 


AR + 


CS 


Culinary 


VE + 




Electrician 


MK + 
EI + 
GS = 
200 




Mechanic 


MC + 
AS = 
158 




Specialist 


AR = 
88 


CS(SS) 


Culinary 


AR + 


CTA 


Cryptologic 


VE + 


CTI 


Cryptologic 


VE + 




Specialist 


MK + 




Technician — 


MK + 




Technician — 


MK + 




(Submarine) 


EI + 
GS = 
200 or 
VE + 
AR + 
MK + 
MC = 
200 




Administra- 
tion 


105 




Interpretive 


GS = 
150 



Appendix: Matching ASVAB Scores to Military Jobs 3 v 7 



Rating Title 



Score Rating Title 



Score Rating Title 



Score 



CTM 



Cryptologic 
Technician — 
Maintenance 



MK + 
EI + 
GS + 
AR = 
218, 
MK + 
EI + 
GS = 
156, 
MK 
= 57 
and 
AR = 
57 



CTN 



Cryptologic 


AR + 


Technician — 


2MK 


Networks 


+ GS 




= 222 



CTR 



Cryptologic 


VE + 


Technician — 


AR = 


Collection 


110 



CTT 


Cryptologic 


CTT 


DC 


Damage 


VE + 


EA 


Engineering 


AR + 




Technician — 


VE + 




Controlman 


MC + 




Aide 


2MK 




Technical 


MK + 
GS = 
165 






AS = 

158 






+ GS 
= 210 


EM 


Electrician's 


AR + 


EN 


Engineman 


MK + 


EO 


Equipment 


AR + 




Mate 


2MK 
+ GS 
= 196 






AS = 

96 




Operator 


MC + 
EI + 
GS = 
204 


EOD 


Explosive 


AR + 


ET 


Electronics 


AR + 


ET(SS) 


Electronics 


AR + 




Ordnance 


VE = 




Technician 


MK 




Technician 


MK + 




Disposal 


109 
and 
MC = 
51 






+ EI 
+ GS 
= 218 
and 
MK 
+ EI 
+ GS 
= 156 
and 
MK 
= 57 
and 
AR = 
57 




(Submarine) 


EI + 
GS = 
222 or 
VE + 
AR + 
MK + 
MV = 
222 


FC 


Fire 


AR + 


FT(SS) 


Fire Control 


AR + 


GM 


Gunner's Mate 


AR + 




Controlman 


MK 
+ EI 
+ GS 
= 218 
and 
MK 
+ EI 
+ GS 
and 
MK 
= 57 
and 
AR = 
57 




Technician 
(Submarine) 


MK + 
EI + 
GS = 
222 or 
VE + 
AR + 
MK + 
MV = 
222 






MK + 
EI + 
GS = 
204 



(continued) 



368 



ASVAB For Dummies, 3rd Edition 



Table A-3 (continued) 



Rating Title 



Score Rating Title 



Score Rating Title 



Score 



GSE 


Gas Turbine 


VE + 


GSM 


Gas Turbine 


AR + 


HM 


Hospital 


VE + 




Systems 


AR + 




Systems 


MK + 




Corpsman 


MK + 




Technician — 


MK + 




Technician — 


EI + 






GS = 




Electrical 


MC = 
204 or 
AR + 
MK + 
EI + 
GS = 
204 




Mechanical 


GS = 
204 






149 


HT 


Mull 


VE + 


IC 


Interior 


GS + 


IS 


Intelligence 


VE + 




Technician 


MC + 
AS = 

158 




Communi- 
cations 
Electrician 


AR + 
MK + 
El = 
212 




Specialist 


AR = 

103 


IT 


Information 


VE + 


MA 


Master at 


AR + 


MC 


Mass 


VE + 




System 


MK + 




Arms 


WK 




Communi- 


AR + 




Technician 


CS = 
163 or 
VE + 
MK = 
110 






= 100 

and 
WK = 
45 




cations 
Specialist 


109 


MM 


Machinist's 


VE + 


MM(SS) 


Machinist's 


VE + 


MN 


Mineman 


VE + 




Mate 


AR + 
MK + 
AS = 
200 or 
VE + 
AR + 
MK + 
A0 = 
205 




Mate 
(Submarine) 


AR + 
MK + 
MC = 
210 






MC + 
AS = 

158 


MR 


Machinery 


AR + 


MT 


Missile 


AR + 


ND 


Navy Diver 


AR + 




Repairman 


MC + 
AS = 

158 




Technician 


MK + 
EI + 
GS = 
222 or 
VE + 
AR + 
MK + 
MC = 
222 






VE = 

103 
and 
MC = 
51 


NUC 


Nuclear 


AR + 


OS 


Operations 


VE + 


PC 


Postal Clerk 


VE + 




Program 


MK + 
EI + 
GS = 
252 or 
VE + 
AR + 
MK + 
MC = 
252 




Specialist 


MK + 
CS = 
157 or 
AR + 
2MK 
+ GS 
= 210 






AR = 

108 


PR 


Aircrew 


VE + 


PS 


Personnel 


VE + 


QM 


Quartermaster 


VE + 




Survival 


MC + 




Specialist 


MK = 






AR = 




Equipment- 


AS = 






105 or 






97 




man 


158 






VE + 
MK + 
CS = 
157 









Appendix: Matching ASVAB Scores to Military Jobs < Q Q 



Rating Title 



Score Rating Title 



Score Rating Title 



Score 



SH 



Ship's 
Serviceman 



VE + 
AR = 
96 



SK 



Storekeeper 



VE+ SK(SS) 

AR = 

103 



Storekeeper 
(Submarines) 



Marine Carps Enlisted Jabs 



AR + 
MK + 
EI + 
GS = 
200 or 
VE + 
AR + 
MK + 
MC = 
200 or 
VE + 
AR + 
MK + 
MC = 
200 



SN(SS) 


Seaman 


AR + 


STG 


Sonar 


AR + 


ST(SS) 


Sonar 


AR + 




(Submarine) 


MK + 




Technician 


MK 




Technician 


MK + 






EI + 




(Surface) 


+ EI 




(Submarine) 


EI + 






GS = 






+ GS 






GS = 






200 or 






= 218 






222 or 






VE + 






and 






VE + 






AR + 












AR + 






MK + 












MK + 






MC = 












MC = 






200 












200 


SO 


Special 


GS + 


SW 


Steelworker 


VE + 


TM 


Torpedoman's 


AR + 




Warfare 


MC + 






MC + 




Mate 


2MK 




Operator 


El = 






AS = 






+ GS 




(SEAL) 


165 or 
VE + 
MK + 
MC + 

cs = 

220 






140 






= 194 


UT 


Utilitiesman 


AR + 
MK + 
EI + 
GS = 
200 


YN 


Yeoman 


VE + 
MK = 
105 or 
VE + 
MK + 
CS = 
157 


YN(SS) 


Yeoman 
(Submarine) 


AR + 
MK + 
EI + 
GS = 
200 or 
VE + 
AR + 
MK + 
MC = 
200 



The United States Marine Corps needs a few good men (and women) to fill about 120 enlisted 
entry-level job specialties. Like the Army, the Marine Corps calls its enlisted jobs Military 
Occupation Specialties (MOS). The Marine Corps has only three line scores, and they're 
abbreviated in Table A-4 as follows: Mechanical Maintenance (MM), Electronics (EL), and 
General Technical (GT). 



See Chapter 2 for information regarding which subtest scores of the ASVAB are used to 
comprise these line scores. 



^ JQ ASVAB For Dummies, 3rd Edition 



Table A-4 


Marine 


Corps Enlisted Jobs and Reqi 


uired ASVAB Scores 


MOS 


Title 


Score 


MOS 


Title 


Score 


MOS 


Title 


Score 


0121 


Personnel 
Clerk 


CL-100 


0151 


Administrative 
Clerk 


CL-100 


0161 


Postal Clerk 


CL-90 


0231 


Intelligence 
Specialist 


GT-100 


0241 


Imagery 
Analysis 
Specialist 


GT-100 


0251 


Interrogator/ 
Debriefer 


GT-100 


0261 


Geographic 
Intelligence 
Specialist 


EL-100 


0311 


Rifleman 


GT-80 


0313 


LAV Crewman 


GT-90 


0321 


Reconnais- 
sance Man 


GT-105 


0341 


Mortarman 


GT-80 


0351 


Assaultman 


GT-80 



0352 Antitank GT-90 0411 Maintenance GT-100 0431 Logistics/ 

Assault Management Embarkation 

Guided Specialist and Combat 

Missileman Service 

Support (CSS) 
Specialist 



0451 Air Delivery 
Specialist 



GT-100 0481 



Landing 
Support 
Specialist 



GT-95 

and 

MM-100 



0511 MAGTF 
Planning 
Specialist 



GT-100 



GT-110 



0612 Field Wireman EL-90 



0613 



Construction 
Wireman 



EL-90 



0614 



Unit Level 
Circuit Switch 
(ULCS) 
Operator/ 
Maintainer 



EL-100 



0621 Field Radio EL-90 0622 Mobile 

Operator Multichannel 

Equipment 
Operator 



EL-100 



0624 



High 

Frequency 
Communica- 
tion Central 
Operator 



EL-100 



0626 Fleet SATC0M EL-100 0627 Ground 

Terminal Mobile Forces 

Operator SATC0M 

Operator 



EL-100 0811 Field Artillery GT-90 

Cannoneer 



0842 



Field Artillery 

Radar 

Operator 



GT-105 



0844 



Field Artillery 
Fire Control 



GT-105 



0847 



Artillery 
Meteoro- 
logical Man 



GT-105 



1141 



Electrician 



EL-90 



1142 



Electrical 
Equipment 
Repair 
Specialist 



EL-100 



1161 



Refrigeration 
Mechanic 



1833 Assault 

Amphibious 
Vehicle (AAV) 
Crewman 



GT-90 



2111 



Small Arms 

Repairer/ 

Technician 



MM-95 



2131 



Towed 
Artillery 
Systems 
Technician 



MM-105 



1171 


Hygiene 

Equipment 

Operator 


MM-85 


1181 


Fabric Repair 
Specialist 


MM-85 


1316 


Metalworker 


MM-95 


1341 


Engineer 

Equipment 

Mechanic 


MM-95 


1345 


Engineer 

Equipment 

Operator 


MM-95 


1361 


Engineer 
Assistant 


GT-100 


1371 


Combat 
Engineer 


MM-105 


1391 


Bulk Fuel 
Specialist 


MM-85 


1812 


MIAITank 
Crewman 


GT-90 



MM-95 



Appendix: Matching ASVAB Scores to Military Jobs j7 1 



MOS Title 



Score 



MOS Title 



Score 



MOS Title 



Score 



2141 



2887 



Assault 
Amphibious 
Vehicle (AAV) 
Repairer/ 
Technician 



MM-105 2146 



Main Battle 
Tank(MBT) 
Repairer/ 
Technician 



MM-105 



2147 



Light Armored 
Vehicle (LAV) 
Repairer/ 
Technician 



Counter 
Mortar Radar 
Repairer 



EL-115 



3043 



Supply 

Administration 

and 

Operations 

Clerk 



MM-105 



2161 


Machinist 


MM-105 


2171 


Electro- 
Optical 
Ordnance 
Repairer 


MM-105 

and 

EL-105 


2311 


Ammunition 
Technician 


GT-100 


2336 


Explosive 
Ordnance 
Disposal 
Technician 


GT-110 


2621 


Communica- 
tions Signal 
Collection/ 
Manual Morse 
Operator/ 
Analyst 


GT-100 


2631 


Electronic 

Intelligence 

(ELINT) 

Intercept 

Operator/ 

Analyst 


GT-100 


2651 


Special 

Intelligence 

System 

Administrator/ 

Communicator 


GT-100 


267X 


Cryptologic 
Linguist 


GT-105 


2811 


Telephone 
Technician 


EL-115 


2818 


Personal 

Computer 

(PC)/Tactical 

Office 

Machine 

Repairer 


EL-115 


2821 


Computer 
Technician 


EL-115 


2822 


Electronic 
Switching 
Equipment 
Technician 


EL-115 


2831 


Multichannel 

Equipment 

Repairer 


EL-115 


2832 


Multichannel 

Equipment 

Technician 


EL-115 


2834 


Satellite 
Communi- 
cations 
(SATCOM) 
Technician 


EL-115 


2841 


Ground Radio 
Repairer 


EL-115 


2844 


Ground Com- 
munications 
Organizational 
Repairer 


EL-115 


2846 


Ground Radio 

Intermediate 

Repairer 


EL-115 


2871 


Test 

Measurement 
and Diagnostic 
Equipment 
Technician 


EL-115 


2881 


Communica- 
tion Security 
Equipment 
Technician 


EL-115 


2886 


Artillery 
Electronic 
System 
Repairer 


EL-115 



GT-110 3051 Warehouse GT-90 

Clerk 



3052 


Packaging 
Specialist 


GT-80 


3112 


Traffic 

Management 

Specialist 


GT-90 


3361 


Subsistence 
Supply Clerk 


GT-90 


3381 


Food Service 
Specialist 


GT-90 


3432 


Finance 
Technician 


GT-110 


3441 


NAF Audit 
Technician 


GT-110 


3451 


Fiscal/Budget 
Technician 


GT-110 


3521 


Organizational 

Automotive 

Mechanic 


MM -95 


3531 


Motor Vehicle 
Operator 


MM -85 



(continued) 



3 72 ASVAB For Dummies, 3rd Edition 





















Tabli 


s A-4 (continued) 














MOS 


Title 


Score 


MOS 


Title 


Score 


MOS 


Title 


Score 


4066 


Small 
Computer 
Systems 
Specialist 


GT-110 


4067 


Programmer 


GT-110 


4113 


Morale, 

Welfare, 

Recreation 

(MWR) 

Specialist 


GT-110 


4341 


Combat Cor- 
respondent 


GT-105 

and 
VE-40 


4421 


Legal Services 
Specialist 


GT-100 


46XX 


Visual 
Information 


GT-100 


55XX 


Band 


GT-50 


5711 


Nuclear 

Biological 

and Chemical 

(NBC) 

Defense 

Specialist 


GT-110 


5811 


Military 
Police 


GT-100 


5821 


Criminal 
Investigator 


GT-110 


5831 


Correctional 
Specialist 


GT-100 


5937 


Aviation 

Radio 

Repairer 


EL-105 


5942 


Aviation Radar 
Technician 


EL-105 


5952 


Air Traffic 

Control 

Navigational 

Aids 

Technician 


EL-105 


5953 


Air Traffic 
Control Radar 
Technician 


EL-105 


5954 


Air Traffic 
Control Com- 
munications 
Technician 


EL-105 


5962 


Tactical Data 

Systems 

Equipment 

(TDSE) 

Repairer 


EL-105 


5963 


Tactical Air 
Operations 
Module 
Repairer 


EL-105 


6042 


Individual 
Material 
Readiness List 
(IMRL) Asset 
Manager 


GT-100 


6046 


Aircraft 
Maintenance 
Administration 
Specialist 


GT-100 


6048 


Flight 

Equipment 

Technician 


MM-105 


6061 


Aircraft 

Intermediate 

Level 

Hydraulic/ 

Pneumatic 

Mechanic 


MM-105 


6071 


Aircraft 
Maintenance 
Support 
Equipment 
(SE) Mechanic 


MM-105 


6091 


Aircraft 

Intermediate 

Level 

Structures 

Mechanic 


MM-105 


611X 


Helicopter 
Mechanic 


MM-105 


61 2X 


Helicopter 
Power Plants 
Mechanic 


MM-105 


61 5X 


Helicopter/ 
Tiltrotor 
Airframe 
Mechanic 


MM-105 


61 7X 


Helicopter 
Crew Chief 


MM-105 


621 X 


Fixed-Wing 

Aircraft 

Mechanic 


MM-105 


622X 


Fixed-Wing 
Aircraft 
Power Plants 
Mechanic 


MM-105 


6232 


Fixed-Wing 
Aircraft Flight 
Mechanic 


MM-105 


625X 


Fixed-Wing 
Aircraft 
Airframe 
Mechanic 


MM-105 


628X 


Fixed-Wing 

Aircraft 

Safety 

Equipment 

Mechanic 


MM-105 



Appendix: Matching ASVAB Scores to Military Jobs A 73 



MOS Title 



Score 



MOS Title 



Score 



MOS Title 



Score 



63XX 



Aircraft Com- 
munications/ 
Navigation/ 
Electrical/ 
Weapon 
Systems 
Technician 



EL-105 



64XX 



Aircraft Com- 


EL-105 


6511 


Aircraft 


munications/ 






Ordnance 


Navigation 






Technician 


Systems 








Technician 








Automated 


GT-100 


6821 


Weather 


Information 






Observer 


Systems (AIS) 








Computer 








Operator 









GT-105 



6672 Aviation 

Supply Clerk 



7251 



GT-100 



6673 



Air Traffic 
Controller 



GT-105 



7314 



Unmanned 
Aerial Vehicle 
(UAV) Air 
Vehicle 
Operator 



GT-105 



7371 



Aerial 
Navigator 



GT-105 



7011 


Expeditionary 
Airfield 
Systems 
Technician 


MM-105 


7041 


Aviation 

Operations 

Specialist 


GT-100 


7051 


Aircraft 
Firefighting 
and Rescue 
Specialist 


MM-95 


7212 


Low Altitude 
Air Defense 
(LAAD) 
Gunner 


GT-90 


7234 


Air Control 
Electronics 
Operator 


GT-105 


7242 


Air Support 
Operations 
Operator 


GT-100 



GT-110 



7381 



Airborne 

Radio 

Operator/ 

In-flight 

Refueling 

Observer/ 

Loadmaster 



GT-110 



Coast Guard Enlisted Jabs 



The smallest U.S. Military service, the Coast Guard, has only 19 types of entry-level jobs for 
enlisted members. Like the Navy, the Coast Guard calls its enlisted jobs ratings. Also like the 
Navy, the Coast Guard doesn't use line scores for job qualification purposes. Instead, it uses 
the sums of various ASVAB subtest scores. 



Table A-5 shows combinations of ASVAB subtest scores that are required to qualify for Coast 
Guard enlisted jobs. The ASVAB subtests are abbreviated as follows: General Science (GS), 
Arithmetic Reasoning (AR), Word Knowledge (WK), Paragraph Comprehension (PC), Auto & 
Shop Information (AS), Mathematics Knowledge (MK), Mechanical Comprehension (MC), 
Electronics Information (EI), Assembling Objects (AO), and Verbal Expression (VE) scores. 



37b 



ASVAB For Dummies, 3rd Edition 



Table A-5 



Coast Guard Enlisted Jobs and Required ASVAB Scores 



Rating Title 



Score Rating Title 



Score Rating Title 



Score 



AMT 


Aviation 


AR + 


AST 


Aviation 


VE + 


AV 


Avionics 


MK 




Mainten- 


MC 




Survival 


MC 




Technician 


+ EI 




ance 


+ AS 




Technic- 


+ AS 






+ GS 




Technician 


+ EI 
= 213 
(mini- 
mum 
AR = 
52) 




ian 


= 159 
(mini- 
mum 
AR = 
52) 






= 171 

(mini- 
mum 
AR = 
52) 


BM 


Boatswain's 


VE + 


DC 


Damage 


VE + 


EM 


Electrician's 


MK 




Mate 


AR = 

101 




Control- 
man 


MC + 
AS = 
152 




Mate 


+ EI 
+ GS 
= 152 
(mini- 
mum 
AR = 
52) 


ET 


Electronics 


MK 


FS 


Food 


VE + 


GM 


Gunner's 


AR + 




Technician 


+ EI 
+ GS 
= 171 
(mini- 
mum 
AR = 
52) or 
AFQT 
= 66 




Service 
Specialist 


AR = 
106 




Mate 


MK + 
EI + 
GS = 
208 


HS 


Health 


VE + 


IT 


Informa- 


MK 


MK 


Machinery 


AR + 




Services 


MK + 




tion 


+ EI 




Technician 


MC + 




Technician 


GS = 

154 




Systems 
Technic- 
ian 


+ GS 
= 171 
(Mini- 
mum 
AR = 
52) 






AS = 
150 or 
VE + 
AR = 

106 


MST 


Marine 

Science 

Technician 


VE + 
AR = 
115 
(mini- 
mum 
MK = 
58) 


MU 


Musician 


N/A 


OS 


Operations 
Specialist 


VE + 
AR = 

106 


PA 


Public 


VE + 


PS 


Port 


VE + 


SK 


Storekeeper 


VE + 




Affairs 


AR = 




Security 


AR = 






AR = 




Specialist 


110 
(mini- 
mum 
VE = 

60) 




Specialist 
(CG 

Reserves 
Only) 


101 






106 
(mini- 
mum 
VE = 
52) 



YN 



Yeoman 



VE + 

AR 

106 



Notes 









J 



Notes 









J 



Notes 









J 



Notes 









J 



Notes 









J 



Notes 









J 



Index 



Symbols 



~ (approximation symbol), 

circle operations, 87 
n (pi), 87 
V (square root symbol), 77 



absolute zero, Kelvin scale, 109 
AC (alternating current), 

163-164, 169 
AC power supply, 165 
accelerator, combustion 

engines, 129 
actions, Newton's third law of 

motion, 142 
acute angle, 85 
addition 
algebra, 80 
Arithmetic Reasoning (AR) 

clue word, 93 
decimals, 73 
fractions, 70-71 
order of operations, 68, 354 
AFQT. See Armed Forces 

Qualification Test (AFQT) 
AFQT For Dummies (Rod 

Powers), 353 
agriculture, biology subdisci- 

pline, 111 
air density, temperature 

element, 123 
Air Force 
ASVAB retest policy, 15 
Delayed Entry Program (DEP) 

retest policy, 15 
guaranteed job offers, 18 
line score computations, 21 
MAGE (mechanical, adminis- 
trative, general, and 
electronics), 21 
Military Occupation 

Specialties (MOS) 362-365 
strength ceilings, 30 
Word Knowledge line 
scores, 34 
air masses, characteristics, 123 
air pressure, barometer, 143 
air-injection systems, emissions- 
control, 131 



algebra 

addition, 80 

common factors, 82 

division, 80 

equations, 79-81 

exponents, 81 

greatest common factor, 82 

inequalities, 84 

Mathematics Knowledge, 79-84 

multiplication, 80 

multistep equations, 81 

powers, 81 

quadratic equation, 83 

solving for x, 79 

subtraction, 80 

three-term equation, 82 

variables, 79 
Algebra For Dummies 

(Sterling), 88 
Algebra II For Dummies 

(Sterling), 222 
Allen wrench, 133-134 
all-wheel drive, automotive, 130 
alternating current (AC), 

163-164, 169 
alternators, electrical 

systems, 130 
alto-, cloud prefix, 124 
altostratus clouds, 124 
AM radio stations, 163 
ammeters, 159 
amperes, electrical measure- 
ment, 158, 159, 168 
amplifier, 166, 168 
angles, geometry, 85 
Animal Kingdom, organism 

types, 113-114 
answers 

avoiding blank, 347 

changing after marking, 347 

double-checking, 347 

understanding the problem 
before answering, 348, 349 
antenna, electronics, 166, 168 
antifreeze, cooling systems, 129 
antilock brake system (ABS), 138 
antonyms, word meaning, 39 
approximation symbol (=), 

circle operations, 87 
archeology, social science 

discipline, 111 
arcs, 85 



areas, 86-88 

Arithmetic Reasoning (AR) 
AFQT Practice Exam, 323-326, 

337-339 
AFQT subtest, 12 
clue words, 93 
fact recognition, 93 
guessing strategies, 95-96 
interpreting word problems, 

92-93 
number sequences, 68-69 
plugging in possible 

answers, 96 
Practice Exam 1, 196-199, 

221-224 
Practice Exam 2, 238-241, 

266-268 
Practice Exam 3, 282-285, 

309-313 
practice questions, 97-101 
process of elimination, 95 
reading the problem, 92 
reasonable answers, 351 
reviewing answers, 94 
solution process, 93-94 
Armed Forces Qualification 

Test (AFQT) 
Arithmetic Reasoning (AR), 

91-96,351 
calculation process, 12 
Department of Defense, 16 
enlistment requirements, 12-14 
math operations, 351 
Mathematics Knowledge, 351 
Practice Exam answer 

sheet, 322 
Practice Exam answers and 

explanations, 337-342 
Practice Exam questions, 

321-336 
scratch paper uses, 349 
subtests, 12 
understand the problem 

before answering, 349 
Word Knowledge, 350 
Armed Services Vocational 

Aptitude Battery (ASVAB) 
Arithmetic Reasoning (AR) 

subtest, 68-69, 91-96 
Assembling Objects subtest, 

171-177 



382 ASVAB For Dummies, 3rd Edition 



Armed Services Vocational 

Aptitude Battery (ASVAB) 

(continued) 
Auto & Shop Information (AS), 

127-140 
blank answers, 347 
changing answers, 347 
composite scores, 11, 17 
confirmation tests, 26 
Department of Defense devel- 
opment, 16 
double-checking answers 

before marking the test, 347 
dummy scores, 19 
Electronic Information subtest, 

157-169 
General Science subtest, 

107-126 
line score computations, 18-21 
line scores, 11, 17, 22 
maintaining focus, 346 
Mathematics Knowledge 

subtest, 65-90 
Mechanical Comprehension 

(MC) subtest, 141-156 
paper versus computerized 

tests, 23-25 
Paragraph Comprehension 

subtest, 43-53, 350 
percentile scores, 1 1 
preparation, lack of, 345-348 
raw score, 10 
retest policies, 14-16 
sample score cards, 10-11 
standard scores, 11, 22 
subtest score importance, 345 
subtests, 8-9 
versions, 7-8 
Word Knowledge subtest, 

33-42 
subject range, 7 
Army 
Assembling Objects develop- 
ment, 172 
ASVAB retest policy, 15 
guaranteed job offers, 18 
line score computations, 18-19 
Military Occupation 

Specialties (MOS), 357-362 
special program AFQT score 

requirement, 14 
strength ceilings, 30 
Word Knowledge line 

scores, 34 
Army College Fund, minimum 

AFQT score, 14 
arrival time, test day, 29 



Assembling Objects 

comparing pieces, 176 

connectors, 172-174 

development history, 172 

jigsaw puzzles, 174-176 

mental rotation, 172-176 

mirroring, 173 

Practice Exam 1, 219-220, 232 

Practice Exam 2, 262-264, 275 

Practice Exam 3, 305-307, 320 

practice questions, 187-188 

practice techniques, 177 

time limits, 171 

visualizing spatial relation- 
ships, 171 
asteroid belt, 122 
asteroids, 121-122 
astronomy 

asteroids, 121-122 

comets, 121 

meteors, 121-122 

moons, 120-121 

planets, 120 

Pluto, 120 

scientific discipline, 110 

sun, 119-120 
Astronomy For Dummies 

(Maran), 107, 221 
atmosphere, layers, 122-123 
atmospheric pressure, 

barometer, 143 
atomic numbers, periodic table, 

118-119 
atoms, 118 
auger bits, drilling tools, 

132, 135 
Auto & Shop Information (AS) 

antilock brake system (ABS), 
138 

ASVAB subtest, 8 

brake systems, 131 

clamping tools, 136 

cooling systems, 129 

cutting tools, 134 

drilling tools, 135 

drive systems, 130-131 

electrical systems, 130 

emissions-control systems, 131 

engines, 128-129 

fasteners, 132, 137-139 

fastening tools, 133-134 

finishing tools, 136 

gouging tools, 135 

ignition systems, 130 

leveling tools, 137 

measuring tools, 136-137 

nuts, 132, 138 



Practice Exam 1, 211-213, 230 
Practice Exam 2, 254-256, 273 
Practice Exam 3, 297-299. 318 
practice questions, 181-183 
punching tools, 135 
rivets, 139 

solving guidelines, 139-140 
squaring tools, 137 
striking tools, 132 
time limits, 127 
washers, 132, 138 
Auto Repair For Dummies 

(Sclar), 139 
axles, Mechanical 

Comprehension (MC), 

152-153 



B 



balance, force concepts, 142-143 

band codes, resistors, 167 

Barnhart, Roy 
Home Improvement All-in-One 
For Dummies, 139 

barometer, atmospheric pres- 
sure measurement, 143 

base (math), 66 

Basic Math & Pre-Algebra For 
Dummies (Zegarelli), 222 

battery, 165-166 

bells, 165-166 

bench planes, 136 

bench vise, 136 

biology, 110-114 

Biology For Dummies (Siegfried), 
107,221 

bits, drilling, 132, 135 

blank answers, 347 

block and tackle systems, 
148-150 

block diagrams, 165-168 

body systems, human, 114-115 

bolts, 132, 138 

botany, biology subdiscipline, 
111 

box wrenches, 132, 133 

boxes, volume, 88, 355 

Boysen, Earl 
Electronics For Dummies, 169 

brads, nails, 137 

brake fluid, automotive, 131 

brake lines, 131 

brake shoes, 131 

brake systems, 131 

buzzers, 165-166 



Index 383 



calculators, not allowed, 353-354 
calipers 
automotive brake systems, 

131 
measuring tools, 132, 136 
Carey, James and Morris 
Home Improvement All-in-One 
For Dummies, 139 
cap nuts, 132, 138 
capacitive reactance, electrical 

concepts, 163, 169 
capacitors 
electrical concepts, 163-164 
electronic component sym- 
bols, 166-167 
carburetors, 129 
catalytic converter, 131 
CAT-ASVAB 
Assembling Objects limits, 171 
Auto & Shop Information (AS) 

limits, 127 
Electronic Information limits, 

157 
enlistment applicants, 8 
General Science limits, 107 
Mathematics Knowledge 

limits, 65 
Mechanical Comprehension 

(MC) limits, 141 
pros/cons, 24-25 
C-clamps, 132, 136 
cell membrane, 115-116 
cell walls, animal versus plant 

structures, 115-116 
cells, 115-116, 165-166 
cellular respiration, 116 
Celsius (Centigrade) scale, 
temperature conversions, 
109-110 
centrifugal force, 145 
centrioles (cell structures), 

115-116 
chemical effects, 162 
chemical reactions, General 

Science, 119 
chemistry, 111, 118-119 
Chemistry For Dummies 

(Moore), 107, 221 
chisels, gouging tools, 132, 135 
chlorophyll, 115-116 
chloroplasts, 115-116 
chromosomes, 117 
circles, 87-88, 354 
circuit breakers, 160 
circuit codes, 165-168 



circuits, electrical, 158-159, 161 
circumference (C), 87 
cirro-, cloud prefix, 124 
cirrocumulus clouds, 124 
clamping tools (AS), 136 
clamps, 132, 136 
class, classification system 

level, 113 
classification systems (biology), 

112-114 
closed circuit, electrical princi- 
ple, 169 
clouds, classifications, 123-124 
clue words, Arithmetic 

Reasoning (AR), 93 
clutch, automotive, 130-131 
Coast Guard 
ASVAB retest policy, 16 
guaranteed job offers, 18 
line score computations, 19-20 
Military Occupation 

Specialties (MOS) 373-374 
strength ceilings, 30 
Word Knowledge line 

scores, 34 
Coding Speed (CS) 
dummy scores, 19 
Naval ASVAB subtest, 8-9 
coils, electrical systems, 130 
cold chisels, 132, 135 
cold fronts, 123 
college-loan repayment pro- 
grams, AFQT score, 14 
comets, astronomy, 121 
common denominators, 

adding/subtracting 

fractions, 70-71 
common factor 
algebra, 82 
multiplying/reducing fractions, 

71-72 
common nails, 137 
common sense, Mechanical 

Comprehension (MC) 

concepts, 155-156 
comparing pieces, 176 
comparisons, Mathematics 

Knowledge, 76-77 
complementary angles, 85 
composite number, 66 
composite scores, ASVAB, 11,17 
compounds, chemical, 118 
compression stroke, engines, 

128 
computer games, spatial skill 

practice, 177 
computerized tests. See 

CAT-ASVAB 



computerized-adaptive testing 

ASVAB. See CAT-ASVAB 
condensers, ignition 

systems, 130 
conductors, electrical, 158 
confirmation tests, MEPCOM 

regulations, 26 
connectors, 172-174 
conventional current, 161 
cooling systems, 129 
core, Earth's geology layer, 122 
coulombs (C), electrical 

current, 159 
countersinks, drilling tools, 135 
Cox, John D. 

Weather For Dummies, 107, 221 
crossword puzzles, 

vocabulary, 41 
crust, Earth's geology layer, 122 
cube roots, 78-79 
current, electrical, 158-159, 

168-169 
curved-nose pliers, 134 
cutting pliers, 134 
cutting tools, 134 
cylinders 
combustion engines, 129 
volume determination, 88, 355 
cytoplasm (cell structure), 

115-116 



a 



DC (direct current), 163-164 
DC power supply, 165-166 
decimals 

adding/subtracting, 73 

changing percents, 350 

dividing, 74-75 

multiplying, 74 

percent expression, 73 

repeating, 73 
definitions, Word Knowledge 

question format, 34 
degrees, angle measurements, 85 
Delayed Entry Program (DEP), 
ASVAB retest policies, 15 
denominator, fractions, 69 
DEP Enrichment Program, 15 
Department of Defense, 10, 16 
depth gauges, 137 
diameter (d), 87 
dictionaries 

slow readers, 51 

synonym/antonym listings, 39 

vocabulary, 40-41 
Dictionary.com, 41 



38b 



ASVAB For Dummies, 3rd Edition 



difference (math), 67 
difference of potential, 158 
diodes, electronics, 166-167 
direct current (DC), 163-164 
disc brakes, automotive, 131 
distance 

gravity component, 144 

rate formula, 77 
distributors, ignition systems, 

130 
division 

algebra, 80 

Arithmetic Reasoning (AR) 
clue word, 93 

decimals, 74-75 

fractions, 72 

order of operations, 68, 354 
dominant gene, 117 
double-cut files, 136 
double-headed nails, 137 
drawing conclusions, Paragraph 

Comprehension, 50 
drill bits, 135 
drilling tools, 135 
drills, 135 

drive systems, 130-131 
drum brakes, automotive, 131 
dual on-off switch, 166 
dummy scores, ASVAB, 19 
dwarf planets, Pluto, 120 



£ 



earphones, 165-166 
Earth, 120, 122-124 
ecology, biology subdiscipline, 

111,112 
educated guesses 
Arithmetic Reasoning (AR), 

95-96 
Electronic Information, 169 
General Science, 124-125 
Mathematics Knowledge, 89-90 
Mechanical Comprehension 
(MC), 156 
elastic recoil, force component, 

145-146 
electrical current, 159 
electrical systems, 130 
electricity 
alternating current (AC), 

163-164 
amperes, 158, 159 
capacitors, 163-164 
chemical effects, 162 
circuit breakers, 160 



circuits, 158-159, 161 

conductors, 158 

conventional current, 161 

current, 158, 159 

direct current (DC), 163-164 

heat effects, 162 

impedance, 163 

inductors, 163-164 

insulators, 158 

kilowatt-hours (kWhs), 158, 161 

magnetic effects, 162 

magnetic lines of force, 162 

measurement units, 168 

ohms, 158 

Ohm's law, 160 

physiological effects, 162 

power measurements, 161 

rectifiers, 164 

resistance, 159-160 

rheostats, 160 

semiconductor diodes, 164 

short circuit, 161 

terminals, 158 

voltage measurement, 158-159 

volts, 158 

watt-hours, 158 

watts, 158, 161 
electromagnetic induction, 162 
electronic component symbols, 

165-168 
electronic fuel injection (EFI) 

computer, 129 
Electronics Information 

educated guesses, 169 

electrical measurement units, 
168 

electricity, 157-162 

electronic component 
symbols, 165-168 

Practice Exam 1, 209-210, 
229-230 

Practice Exam 2, 252-253, 
272-273 

Practice Exam 3, 295-296, 
317-318 

practice questions, 185-187 

principles, 169 

time limits, 157 
Electronics For Dummies 

(McComb and Boysen), 169 
electrons, 118 
electroplating, 162 
elements, periodic table, 

118-119 
ellipse, Earth's orbit, 120 
emissions-control systems, 131 



energy, electrical measurement 

units, 168 
engine knock, octane ratings, 

140 
engines, 128-129 
enlistment applicants 
AFQT score, 12-14 
CAT-ASVAB, 7-8 
guaranteed job offers, 18 
Military Occupation Specialties 
(MOS), 18, 357-374 
entomology, biology subdisci- 
pline, 111 
equals, Arithmetic Reasoning 

(AR) clue word, 93 
equations, algebra, 79-81 
equilateral triangles, 85-86 
equilibrium, force concepts, 

142-143 
exercise, test day, 29 
exhaust stroke, combustion 

engines, 128 
exhaust-gas-recirculation 

systems, 131 
experimentation, scientific 

problem solving, 108 
exponents 
algebra, 81 

mathematical operations, 68 
order of operations, 354 



fact recognition, 93 

factorial, 66 

factoring, algebra, 82 

factors, 66 

Fahrenheit scale, temperature 

conversions, 109-110 
family, classification system 

level, 113 
fasteners, 132, 137-139 
fastening tools, 133-134 
faults, Earth's geology, 122 
females, Navy minimum AFQT 

score, 14 
fifth roots, Mathematics 

Knowledge, 78-79 
files (tools), 136 
filters, emissions-control 

systems, 131 
finishing nails, 137 
finishing tools, 136 
flashcards, study technique, 

355-356 
flat files, 136 



Index 385 



flat washers, 132, 138 
focus, maintaining, 346 
FOIL, multiplying terms, 227 
folding rules, 136 
force 

centrifugal, 145 

elastic recoil, 145-146 

equilibrium, 142-143 

friction, 143-144 

gravity, 143-144 

magnetism, 143 

mathematical formula, 142 

mechanical advantage formu- 
las, 147-148, 153 

Newton's law of universal 
gravitation, 144 

Newton's third law of motion, 
142 

pressure formula, 143 

recoil, 143 

static electricity, 143 

static equilibrium, 143 

tension application, 145 
four-stroke, combustion 

engines, 129 
fourth roots, Mathematics 

Knowledge, 78-79 
four-wheel drive, automotive, 

130 
fraction bar character, order of 

operations, 354 
fractions 

adding/subtracting, 70-71 

common denominator, 70-71 

common factor, 71-72 

comparisons, 76-77 

denominator, 69 

dividing, 72 

expressing as a decimal, 73-75 

expressing as a percent, 75 

improper fraction conver- 
sions, 72-73 

Mathematics Knowledge, 69-77 

multiplying/reducing, 71-72 

numerator, 69 

ratios, 76-77 

repeating decimals, 73 
framing chisels, 135 
Free Vocabulary, online word 

lists, 41 
frequency, electrical concepts, 

163, 169 
friction (force), 143-144 
front-wheel drive, 130 
fuel injectors, 129 
Fungi Kingdom, organism, 114 
fuse, electronic component, 
165-166 



G 



Galilean satellites, Jupiter's 
moons, 121 

gears, Mechanical 

Comprehension (MC), 
150-151 

GED certificate, AFQT score, 13 

gender determinations, General 
Science, 117 

gene copying, 117 

genealogy, social science 
discipline, 111 

General Science 
air masses, 123 
astronomy, 119-122 
biology subdisciplines, 111 
cells, 115-116 
changing states (chemistry), 

118 
chemical reactions, 119 
chemistry, 117-119 
classification systems, 112-114 
cloud classifications, 123-124 
cold fronts, 123 
dominant gene, 117 
educated guesses, 124-125 
gender determinations, 117 
gene copying, 117 
genetics, 116-117 
geology, 122 
Greek word usage, 125 
human body systems, 114-115 
Latin word usage, 125-126 
measurement forms, 108-110 
meteorology, 122-124 
metric system, 108-109 
Practice Exam 1, 193-195, 221 
Practice Exam 2, 235-237, 265 
Practice Exam 3, 279-281, 309 
practice questions, 179-181 
recessive gene, 117 
scientific disciplines, 110-111 
scientific method, 107-108 
temperature conversions, 

109-110 
time limits, 107 

generators, electromagnetic 
induction, 162 

genetics 
biology subdiscipline, 111 
General Science, 116-117 

genus, classification system 
level, 113 

Geographic Area of Choice 
Program, minimum AFQT 
score, 14 



geology 
General Science, 122 
scientific discipline, 110 
geometry 
angles, 85 
circles, 87-88 
defined, 84 

equilateral triangles, 85-86 
formulas, 354-355 
isosceles triangles, 85-86 
Mathematics Knowledge, 

84-88 
quadrilaterals, 86 
Geometry For Dummies (Ryan), 

88,222 
gigahertz (GHz), frequency 

measurement, 163 
gouging tools, Auto & Shop 

Information (AS), 135 
gram (g), metric unit of 

mass, 108 
gravity, force component, 

143-144 
greatest common factor, 

algebra, 82 
Greek, word usage, 125, 355 
ground 
electrical concepts, 158-159 
electronic component 
symbols, 165-166 
grouping symbols, order of 

operations, 354 
guaranteed job offers, enlist- 
ment applicants, 18 
guessing, testing guidelines, 27 



H 



half-round files, 136 

Hamilton, Gene and Katie 
Home Improvement All-in-One 
For Dummies, 139 

hammers, 133 

handscrew vise, 132, 136 

Hatch, Scott and Lisa 
SAT II Math For Dummies, 
88, 222 

health, test day, 29 

heat effects, commercial appli- 
cations, 162 

heater, electronic component 
symbols, 165-166 

hertz (Hz), frequency measure- 
ment, 163 

high school diploma, AFQT 
score requirements, 13 



386 



ASVAB For Dummies, 3rd Edition 



Holzner, Steven, Ph.D. 
Physics I For Dummies, 

108, 221 
Physics II For Dummies, 108 
horsepower, work concepts, 

147 
human body systems, 114-115 
hydraulic cylinder, automotive 

brake systems, 131 
hypothesis, scientific method, 

108 



/ 



ichthyology, biology subdisci- 

pline, 111 
ignition systems, Auto & Shop 

Information (AS), 130 
impedance, electrical 

concepts, 163 
Imperial system, measurement 

form, 108 
implication questions, 

Paragraph Comprehension, 

46-47 
improper fractions, mixed 

number conversions, 72-73 
incentive programs, minimum 

AFQT score 14 
inclined planes, 148 
indicator lamp, 165-166 
inductive reactance, electrical 

concepts, 163, 169 
inductor 
electronic component 

symbols, 166 
electrical concepts, 163-164 
inequalities, algebra, 84 
inference questions, Paragraph 

Comprehension, 46-47 
inline cylinders, combustion 

engines, 129 
insulators, electrical, 158 
intake stroke, combustion 

engines, 128 
integer, 66 

interest rates, formula, 77 
International Astronomical 

Union (IAU), planet defini- 
tion, 120 
International System of Units 

(SI). See metric system 
inverse operations, math, 351 
irrational numbers, 

Mathematics Knowledge, 

77-78 
isosceles triangles, 85-86 



jacks, hydraulic, 154 

jigsaw puzzles 
Assembling Objects, 174-176 
spatial skill practice, 177 

job qualifications, scores, 345 

Jupiter, planet designation, 120 



a: 



Kelvin (K), temperature conver- 
sions, 109-110 

kilohertz (kHz), frequency 
measurement, 163 

kilowatt-hours (kWhs), electri- 
cal measurement, 158, 161 

kinetic energy, changing states, 
118 

kingdoms (biology classifica- 
tion), 112-114 



•£• 



lag screws, 138 

Latin, word usage, 125-126, 355 

leads, voltmeter connections, 

159 
leveling tools, Auto & Shop 

Information (AS), 137 
levers, mechanical advantage, 

147-148 
lighting lamp, 165-166 
line scores 
ASVAB, 11, 17, 22 
Paragraph Comprehension, 44 
service branch computations 

18-21 
Word Knowledge, 34 
liquids, test day, 29 
liter (L), metric unit of volume, 

108 
long-nosed pliers, 134 
lubricants 
combustion engines, 129 
friction reduction, 144 
lunar eclipse, astronomy, 

120-121 
lysosome (cell structure), 
115-116 



M 



machine screws, 

machines 
axles, 152-153 
gears, 150-152 



138 



hydraulic jacks, 154 
inclined planes, 148 
levers, 147-148 
mechanical advantage formu- 
las, 147-148 
pulley and belt systems, 152 
pulleys, 148-150 
screw jacks, 148 
vises, 153-154 
wedges, 148 
wheels, 152-153 
MAGE (mechanical, administra- 
tive, general, and 
electronics), Air Force, 21 
magma, molten rock, 122 
magnetic effects, 162 
magnetic lines of force, electri- 
cal concepts, 162 
magnetism, force, 143 
main idea questions, Paragraph 

Comprehension, 45, 48-49 
main points 
Paragraph Comprehension, 

48-49 
recognition techniques, 356 
mallets, striking tool, 133 
mantle, Earth's geology layer, 

122 
map reading, spatial skill 

practice, 177 
Maran, Stephen P. 
Astronomy For Dummies, 
107, 221 
Marine Corps College Fund, 

minimum AFQT score, 14 
Marines 
ASVAB retest policy, 16 
guaranteed job offers, 18 
line score computations, 20-21 
Military Occupation 

Specialties (MOS) 369-373 
special program AFQT 

score, 14 
strength ceilings, 30 
Word Knowledge line 
scores, 34 
Mars, planet designation, 120 
mass, gravity component, 144 
mathematical principles, 

Mechanical Comprehension 
(MC), 156 
Mathematics Knowledge (MK) 
AFQT Practice Exam, 334-336, 

341-342 
AFQT subtest, 12 
algebra, 79-84 
comparisons, 76-77 



Index $87 



cube roots, 78-79 
educated guesses, 89-90 
fifth roots, 78-79 
fourth roots, 78-79 
fractions, 69-77 
geometry, 84-88 
irrational numbers, 77-78 
number sequences, 68-69 
order of operations, 67-68 
paper tests limits, 65 
Practice Exam 1, 206-208, 

226-229 
Practice Exam 2, 249-251, 

270-272 
Practice Exam 3, 292-294, 

315-317 
practice questions, 101-104 
process of elimination, 90 
rate formulas, 77 
ratios, 76-77 
reasonable answers, 351 
roots, 77-79 
scientific notation, 77 
solving for an unknown, 89 
terminology, 66-67 
test-taking tips, 88-90 
understanding the questions, 

88-89 
McComb, Gordon 

Electronics For Dummies, 169 
meals, test preparation guide- 
lines, 29 
measurement, units of 
electrical, 158-161, 163 
force, 145 
metric, 108-109 
power, 147, 158 
pressure, 143 
temperature, 109-110 
in word problems, 94 
work, 146 
measuring tools, Auto & Shop 

Information (AS), 136-137 
mechanical advantage 
force concepts, 147-148, 153 
wheel and axle formula, 153 
Mechanical Comprehension 

(MC) 
axles, 152-153 
common sense, 155-156 
educated guesses, 156 
force concepts, 141-145 
force formula, 142 
gears, 150-151 
horsepower formula, 147 
hydraulic jacks, 154 
inclined planes, 148 



levers, 147-148 
mathematical principles, 156 
mechanical advantage formu- 
las, 147-148, 153 
observation concepts, 155-156 
power formula, 146-147 
Practice Exam 1,214-218, 

230-232 
Practice Exam 2, 257-261, 

273-275 
Practice Exam 3, 300-304, 

318-320 
practice questions, 183-185 
pressure formula, 143 
pulley and belt systems, 152 
pulleys, 148-150 
time limits, 141 
vises, 153-154 
wheels, 152-153 
work equation, 146 
megahertz (MHz), frequency 

measurement, 163 
membranes, cell structure, 

115-116 
mental rotation, Assembling 

Objects, 172-176 
MEPCOM, confirmation tests, 26 
MEPS Enlistment Test (MET) 
sites, paper and pencil test 
versions, 8 
Mercury, planet designation, 

120 
Merriam-Webster online, online 

word list resource, 41 
metabolism, cell process, 116 
metal-cutting chisels, 132, 135 
meteorology, scientific disci- 
pline, 110, 122-124 
meteors, astronomy, 121-122 
meter (m), metric unit of 

length, 108 
metric system, General Science, 

108-109 
Military Entrance Processing 
Station (MEPS) 
CAT-ASVAB, 7-8, 25 
confirmation tests, 26 
Military Occupation Specialties 
(MOS) 
Air Force, 362-365 
Army, 357-362 
Coast Guard, 373-374 
Marine Corps, 369-373 
Navy, 365-369 
subjobs, 18 
milliampere, electrical current 
measurement, 159 



minor planets, asteroids, 122 

mirroring, Assembling Objects, 
173 

mixed numbers, improper- 
fraction conversions, 72-73 

Mobile Examining Team (MET) 
sites, paper test format, 24 

molecules, 118 

Moneran Kingdom, organism 
types, 114 

monetary enlistment bonus, 
AFQT score, 14 

moons (satellites), 120-121 

Moore, John T. 
Chemistry For Dummies, 
107, 221 

mortising chisels, 132, 135 

motor, electronic component 
symbols, 165-166 

multimeter, voltage measure- 
ment tool, 159 

multiple fractions, common 
denominator, 71 

multiple operations, number 
sequences, 69 

multiple-choice questions, test- 
ing guidelines, 25-26 

multiplication 
algebra, 80 
Arithmetic Reasoning (AR) 

clue word, 93 
decimals, 74 
fractions, 71-72 
order of operations, 68, 354 

multistep equations, algebra, 81 



N 



nails, 137 
Navy 

Assembling Objects job quali- 
fications, 171 

ASVAB retest policy, 15 

ASVAB versions, 7 

Coding Speed subtest, 8-9 

Delayed Entry Program (DEP) 
retest policy, 15 

DEP Enrichment Program, 15 

guaranteed job offers, 18 

line score computations, 19-20 

Military Occupation 

Specialties (MOS) 365-369 

special program AFQT 
score, 14 

strength ceilings, 30 

Word Knowledge line 
scores, 34 



$88 ASVAB For Dummies, 3rd Edition 



Navy College Fund, AFQT 

score, 14 
needle-nosed pliers, 134 
Neptune, planet designation, 120 
neutrons, 118 

Newton's law of universal gravi- 
tation, 144 
Newton's third law of motion, 

142 
nimbo-, cloud prefix, 124 
nimbus, cloud suffix, 124 
nonvariable resistors, elec- 
tronic component, 166-167 
notes, scratch paper use, 349 
nucleus, cell structure, 115-116 
number sequences, 

Mathematics Knowledge, 
68-69 
numerator, fraction element, 69 
Numerical Operations (NO), 

dummy scores, 19 
nuts, fasteners, 132, 138 



organisms, biology, 112-113 
osmosis, cell process, 116 



P 







observation 
Mechanical Comprehension 

(MC) concepts, 155-156 
scientific method, 108 
obtuse angle, 85 
Ockham's razor, scientific 

method, 108 
octane ratings, combustion 

engines, 140 
offset screwdriver, 134 
Ohm's law, electrical concepts, 

160, 169 
ohms, electrical measurement, 

158 
oil pumps, combustion 

engines, 129 
omega (Greek letter), ohm 

symbol, 160 
online dictionaries, slow 

readers, 51 
online puzzles, spatial skill 

practice, 177 
on-off switch, electronic com- 
ponent, 166 
open, electrical concepts, 158 
open-end wrenches, 132, 133 
order, classification system 

level, 113 
order of operations 
Mathematics Knowledge, 

67-68 
memorizing, 354 



paleontology, scientific disci- 
pline, 110 
paper tests 
Assembling Objects limits, 171 
Auto & Shop Information (AS) 

limits, 127 
Electronic Information, 157 
General Science limits, 107 
Mathematics Knowledge, 65 
Mechanical Comprehension 

(MC) limits, 141 
pros/cons, 24 
Paragraph Comprehension (PC) 
AFQT Practice Exam, 330-333, 

340-341 
AFQT subtest, 12 
drawing conclusions, 50 
implication questions, 46-47 
job qualification, 43-44 
line scores, 44 
main idea, 45, 48-49 
paraphrasing, 49 
passage analyzing, 50 
passage formats, 44 
Practice Exam 1, 203-205, 

225-226 
Practice Exam 2, 245-248, 

268-269 
Practice Exam 3, 289—291, 

313-315 
practice questions, 58-62 
question forms, 44-47 
slow readers, 51-52 
specific information ques- 
tions, 45, 350 
subpoints, 49-50 
test-taking tips, 52-53 
topic sentences, 48 
word meaning questions, 46 
parallelograms, quadrilaterals, 

86 
paraphrasing, Paragraph 
Comprehension main 
points, 49 
parentheses, order of opera- 
tions, 68, 354 
passage analyzing, Paragraph 

Comprehension element, 50 
patterns, number sequences, 69 
percentile scores, ASVAB, 11 
percents 
decimal expression, 73, 350 
fraction conversion, 75 



perfect squares, Mathematics 
Knowledge, 77-78 

perimeters, geometry, 86 

periodic table, chemistry, 
118-119 

personal supplies, test day, 29 

Phillips screwdrivers, 133 

photosynthesis, cell process, 116 

phylum, classification system 
level, 113 

physics 
force formula, 142 
mechanical advantage formu- 
las, 147-148, 153 
Newton's law of universal 

gravitation, 144 
Newton's third law of motion, 

142 
power formula, 146-147 
pressure formula, 143 
scientific discipline, 111 
work equation, 146 

Physics I For Dummies 
(Holzner), 108, 221 

Physics II For Dummies 
(Holzner), 108 

physiological effects, 162 

pi (n), circle operations, 87 

pipe vise, 136 

pipe wrenches, 132, 133 

planes, 136 

planets, solar system, 120 

Plant Kingdom, organism types, 
113-114 

plasma membrane, cell struc- 
ture, 115-116 

pliers, 132, 134, 136 

plugging in possible answers, 
Arithmetic Reasoning 
(AR), 96 

plumb bob, 137 

Pluto, dwarf planet 
designation, 120 

poles, magnetic lines of 
force, 162 

polygon, perimeter formula, 354 

positive-crankcase, emissions- 
control systems, 131 

potentiometers, variable resis- 
tors, 167 

power stroke, combustion 
engines, 128 

power 
electrical measurements, 

161, 168 
electrical principle, 169 
rate of work formula, 146-147 

powers, algebra, 81 



Index 35^ 



Powers, Rod 

AFQT For Dummies, 353 
Practice Exam 1 
answer sheet, 192 
answers and explanations, 

221-232 
questions, 191-220 
Practice Exam 2 
answer sheet, 234 
answers and explanations, 

265-275 
questions, 233-264 
Practice Exam 3 
answer sheet, 278 
answers and explanations, 

309-320 
questions, 277-307 
practice questions 
Arithmetic Reasoning (AR), 

97-101 
Assembling Objects, 187-188 
Auto & Shop Information, 

181-183 
don't try to memorize, 347, 356 
Electronics Information, 

185-187 
Mathematics Knowledge, 

101-104 
Mechanical Comprehension, 

183-185 
Paragraph Comprehension, 

58-62 
Word Knowledge, 55-58 
practice tests, study aids, 28-29 
predictions, scientific 

method, 108 
prefixes 
cloud classifications, 124 
Latin/Greek roots, 355 
metric system, 109 
word definition strategies, 
35-36 
preset variable resistors, 167 
pressure formula, 143 
Priestly, Donald R. 
Home Improvement All-in-One 
For Dummies, 139 
prime number, 66 
process of elimination 
Arithmetic Reasoning (AR), 95 
Mathematics Knowledge, 90 
product (math), 67 
products, chemical reaction 

substance, 119 
Project A, Assembling Objects 
development, 172 



pronunciation, vocabulary 
improvement, 41-42 

Protist Kingdom, organism 
types, 114 

protons, 118 

pulley and belt systems, 

Mechanical Comprehension 
(MC), 152 

pulleys, 148-150 

punches, 135 

punching tools, Auto & Shop 
Information (AS), 135 

push switch, electronic compo- 
nent, 166 

push-to-break switch, elec- 
tronic component, 166 

•Q • 

quadratic equations, algebra, 83 
quadrilateral 
defined, 85 
geometry, 86 
question determination in word 

problems, 92-93 
question formats 
Paragraph Comprehension 

subtest, 44-47 
Word Knowledge subtest 
34-35 
quotient (math), 67 



/? 



radar, broadcast frequency 

range, 163 
radical sign character (V), 

square root indicator, 77 
radius (r), circle distance, 87 
rate formulas, Mathematics 

Knowledge, 77 
rates 
defined, 76, 351 
distance, 77 
interest, 77 
ratios 
described, 351 

Mathematics Knowledge, 76-77 
raw score, ASVAB, 10 
reactants, chemical reaction 

substance, 119 
reactions, Newton's third law of 

motion, 142 
reading the passage, Arithmetic 

Reasoning (AR), 92 
reading speed, 51-52 



reading, study technique, 356 
rear-wheel drive, automotive 

drive systems, 130 
recessive gene, 117 
reciprocal, 66 

recoil, force component, 143 
rectangles 
area formula, 354 
quadrilaterals, 86 
rectifiers, electrical concepts, 

164, 169 
related words, word meaning 

determination, 38-39 
relay switch, electronic compo- 
nent, 166-167 
repeating decimals, fraction 

conversions, 73 
rephrasing passages, Paragraph 

Comprehension main 

points, 49 
resistance 
electrical concepts, 159-160 
electrical measurement 

units, 168 
work component, 146 
resistors, electronic compo- 
nent, 166-167 
retest policies, ASVAB, 14-16 
review questions, Arithmetic 

Reasoning (AR) element, 94 
revolutions per minute (rpm) 

measurement, tachometer, 

129 
rheostats 
electrical concepts, 160 
variable resistors, 167 
rhombuses, quadrilaterals, 86 
right angle, degree angle, 85 
right triangles, 86 
rivets, fasteners, 139 
root words 
synonym/antonym study 

aid, 39 
word meaning determination 

strategies, 37-38 
roots, Mathematics Knowledge, 

77-79 
round chisels, 132, 135 
round files, 136 
rounding, 67 
Rozakis, Laurie E. 
Vocabulary For Dummies, 

41,225 
Ryan, Mark 
Geometry For Dummies, 88, 222 



3 90 ASVAB For Dummies, 3rd Edition 



SAT II Math For Dummies 
(Hatch), 88, 222 

SAT Math For Dummies 
(Zegarelli), 222 

SAT Vocabulary For Dummies 
(Vlk), 41,225 

satellites. See moons 

Saturn, planet designation, 120 

scale drawings, ratios, 76 

scale, 351 

scientific methods, General 
Science problem solving, 
107-108 

scientific notation, Mathematics 
Knowledge, 77 

Sclar, Deanna 
Auto Repair For Dummies, 139 

score cards, ASVAB samples, 
10-11 

scratch paper, provided at test 
sites, 349 

screw jacks, inclined plane 
form, 148 

screwdrivers, 133-134 

screws, 138 

semiconductor diodes, electri- 
cal concepts, 164 

service branches 
line score computations, 

18-21,44 
strength ceilings, 30 

shake-proof washers, 132, 138 

shapes, Assembling Objects, 
174-176 

short circuit, electrical con- 
cepts, 161 

short, electrical concepts, 158 

Siegfried, Donna Rae 
Biology For Dummies, 107, 221 

silent letters, pronunciation 
rules, 41 

simple machines. See machines. 

single-cut files, 136 

sketching, spatial skill practice, 
177 

sledges, striking tool, 133 

slide calipers, 132, 136 

slip-joint pliers, 134 

slow readers, Paragraph 

Comprehension tips, 51-52 

social sciences, scientific disci- 
plines, 111 

socket chisels, 132, 135 

socket wrenches, 132, 133 

solar eclipse, astronomy, 121 



solar system, planet designa- 
tion, 120 
solution process, Arithmetic 

Reasoning (AR), 93-94 
solving for x, algebra, 79 
sounding out words, vocabu- 
lary improvement method, 
41-42 
spatial relationships, 
Assembling Objects, 
171-177 
speakers, electronic compo- 
nent, 165-166 
species, classification system 

level, 113 
specific information ques- 
tions, Paragraph 
Comprehension, 45 
split-lock washers, 132, 138 
square files, 136 
square root 
defined, 66 

Mathematics Knowledge, 77-79 
order of operations, 354 
squares, quadrilaterals, 86 
squaring tools, Auto & Shop 

Information (AS), 137 
standard scores, ASVAB, 11, 22 
standard screwdrivers, 133 
staplers, 133 
starters, 130 

static electricity, force compo- 
nent, 143 
static equilibrium, force con- 
cepts, 143 
steel rules, 136 
Sterling, Mary Jane 
Algebra For Dummies, 88 
Algebra II For Dummies, 222 
straight line, degrees, 85 
strength ceilings, service 

branches, 30 
striking tools, Auto & Shop 

Information (AS), 133 
Strong, Jeff 
Home Improvement All-in-One 

For Dummies, 139 
Woodworking For Dumm ies, 1 39 
study habits 
double-check answer before 

marking the test, 347 
failure to study, 345 
flashcards, 355-356 
focus, 346 

Greek and Latin words, 355 
main point recognition, 356 
math problem practice, 353 



reading, 356 
scratch paper, 349 
testing guidelines, 28-29 
time limits, 346-347 
understand the problem 

before answering, 348, 349 
unnecessary subtest study, 

346 
word lists, 355 
subjobs, Military Occupation 

Specialties (MOS), 

18, 357-374 
subpoints, Paragraph 

Comprehension, 49-50 
subtests 
Arithmetic Reasoning (AR), 

68-69, 91-96 
Armed Forces Qualification 

Test (AFQT), 12 
Assembling Objects, 171-177 
ASVAB, 8-9 
Auto & Shop Information (AS), 

127-140 
avoiding blank answers, 347 
changing answers after mark- 
ing, 347 
double-checking answers, 347 
Electronic Information, 

157-169 
General Science, 107-126 
guessing guidelines, 27 
line score computations, 18-21 
maintaining focus, 346 
Mathematics Knowledge, 

65-90 
Mechanical Comprehension 

(MC), 141-156 
multiple-choice questions, 

25-26 
paper versus computerized, 

23-25 
Paragraph Comprehension, 

43-53 
practice tests as study aids, 

28-29 
pre-test day preparations, 29 
score importance, 345 
study habits, 28-29 
unnecessary study, 346 
Word Knowledge, 33-42 
subtraction 
algebra, 80 
Arithmetic Reasoning (AR) 

clue word, 93 
decimals, 73 
fractions, 70-71 
order of operations, 68, 354 



Index 



391 



suffixes 

cloud classifications, 124 

Latin/Greek roots, 355 

word meaning, 35-37 
sum, 67 

sun, General Science, 119-120 
supplementary angles, 85 
supplies, test day, 29 
switches, electronic compo- 
nent, 166 
synonyms, word meaning 39 
systems, auto 

brakes, 131 

cooling, 129 

drive, 130-131 

electrical, 130 

emissions-control, 131 

ignition, 130 



tachometer, revolutions per 
minute (rpm) measure- 
ment, 129 
tape rules, 136 

taxonomy (biology classifica- 
tion), 112-114 
television stations, 163 
temperature conversions, 

109-110 
temperatures, 123 
tension, force component, 145 
terminals, electrical, 158 
terminology, Mathematics 

Knowledge, 66-67 
terrestrial, earthlike planets, 

120 
test versions, ASVAB, 7-8 
thesaurus, study aid, 39 
third law of motion, Sir Isaac 

Newton, 142 
thread gauges, 137 
three-term equation, algebra, 82 
throttle, combustion engines, 

129 
time limits, 346-347 
timepieces, test day, 29 
tools 
air pressure, 143 
clamping, 136 
cutting, 134 
drilling, 135 
electricity meters, 159 
fastening, 134-135 
finishing, 136 
gouging, 135 



leveling, 137 

measuring, 136-137 

punching, 135 

squaring, 137 

striking, 132 
topic sentences, 48 
torque converters, automotive 

drive systems, 130 
torque wrenches, 133 
transducers, electronics, 

165-166 
transformer, 165-166 
transistor radios, 164 
transistors, 166, 168-169 
transmissions, automotive, 130 
trapezoids, quadrilaterals, 86 
triangles 

area formula, 354 

geometry, 85-86 
twist drills, 135 
two-way switch, 166 



U 



U.S. Air Force. See Air Force 

U.S. Army. See Army 

U.S. Coast Guard. See Coast 

Guard 
U.S. Marine Corps. See Marines 
U.S. Navy. See Navy 
underlined words, Word 

Knowledge question 

format, 34 
United States Congress, 

strength ceilings, 30 
units of measurement. See 
measurement, units of 
Uranus, planet designation, 120 



V 



V arrangement, 129 
vacuoles, cell structures, 

115-116 
variable resistors, 166-167 
variables, algebra, 79 
vectors, force quantities, 

142-143 
Venus, planet designation, 120 
vise-grip pliers, 132, 134 
vises 
clamping tools, 132, 136 
Mechanical Comprehension 
(MC), 153-154 
visualizing spatial relation- 
ships, 171-177 



Vlk, Suzee 
SAT Vocabulary For Dummies, 
41,225 
vocabulary 
improvement methods, 40-42 
Word Knowledge element, 33 
Vocabulary For Dummies 

(Rozakis), 41, 225 
voltage, 158-159, 168 
voltmeter, voltage measure- 
ment tool, 159 
volts, 158, 168 
volume, formulas, 88 
vowels, pronunciation rules, 42 



W 



washers, 132, 138 

watch (timepiece), test day 

preparations, 29 
water jackets, 129 
water pumps, 129 
watt-hour, 158, 168 
watts, 158, 161, 168 
Weather For Dummies (Cox), 

107, 221 
weather, meteorology, 122-124 
Web sites 
Dictionary.com, 41, 51 
Free Vocabulary, 41 
Merriam-Webster online, 41, 51 
online dictionaries, 51 
usmilitary.about.com, 14 
yourdictionary.com, 51 
wedges, inclined plane form, 

148 
wheel and axle formula, 153 
wheels, Mechanical 

Comprehension (MC), 
152-153 
wild guesses, 347 
wing nuts, 132, 138 
wire gauges, 137 
wires, electronic component 

symbols, 165-166 
wood screws, fasteners, 138 
wood-cutting chisels, 132, 135 
Woodworking For Dummies 

(Strong), 139 
word families, 38-39 
Word Knowledge 
AFQT Practice Exam, 

327-329, 340 
antonyms, 39 
closest in meaning, 350 
line scores, 34 



3 92 ASVAB For Dummies, 3rd Edition 



Word Knowledge (continued) 
Practice Exam 1, 200-202, 

224-225 
Practice Exam 2, 242-244, 268 
Practice Exam 3, 286-288, 313 
practice questions, 55-58 
prefixes, 35-36 
question formats, 34-35 
root words, 37-38 
suffixes, 35-37 
synonyms, 39 
vocabulary, 33, 40-42 
word families, 38-39 
word meaning strategies, 

35-39 



word lists 
study technique, 355 
synonym/antonym study 

aid, 39 
vocabulary, 40-41 

word meaning questions, 46 

word meanings, determination 
strategies, 35-39 

word sounds, 41-42 

work 
overcoming resistance, 146 
power formula, 146-147 

World Knowledge (WK), AFQT 
subtest, 12 

wrench pliers, 134 

wrenches, 132, 133 



X chromosomes, 117 
Y chromosomes, 117 

• Z» 

Zegarelli, Mark 
Basic Math & Pre-Algebra For 

Dummies, 222 
SAT Math For Dummies, 222 



Business/Accounting 
& Bookkeeping 

Bookkeeping For Dummies 
978-0-7645-9848-7 

eBay Business 
ALL-in-One For Dummies, 
2nd Edition 
978-0-470-38536-4 

Job Interviews 
For Dummies, 
3rd Edition 
978-0-470-17748-8 

Resumes For Dummies, 
5th Edition 
978-0-470-08037-5 

Stock Investing 
For Dummies, 
3rd Edition 
978-0-470-40114-9 

Successful Time 
Management 
For Dummies 
978-0-470-29034-7 



Computer Hardware 

BlackBerry For Dummies, 
3rd Edition 
978-0-470-45762-7 

Computers For Seniors 
For Dummies 
978-0-470-24055-7 

iPhone For Dummies, 
2nd Edition 
978-0-470-42342-4 



Laptops For Dummies, 
3rd Edition 
978-0-470-27759-1 

Macs For Dummies, 
10th Edition 
978-0-470-27817-8 



Cooking & Entertaining 

Cooking Basics 
For Dummies, 
3rd Edition 
978-0-7645-7206-7 

Wine For Dummies, 
4th Edition 
978-0-470-04579-4 



Diet & Nutrition 

Dieting For Dummies, 
2nd Edition 
978-0-7645-4149-0 

Nutrition For Dummies, 
4th Edition 
978-0-471-79868-2 

Weight Training 
For Dummies, 
3rd Edition 
978-0-471-76845-6 



Digital Photography 

Digital Photography 
For Dummies, 
6th Edition 
978-0-470-25074-7 

Photoshop Elements 7 
For Dummies 
978-0-470-39700-8 



Gardening 

Gardening Basics 
For Dummies 
978-0-470-03749-2 

Organic Gardening 
For Dummies, 
2nd Edition 
978-0-470-43067-5 



Green/Sustainable 

Green Building 
& Remodeling 
For Dummies 
978-0-4710-17559-0 

Green Cleaning 
For Dummies 
978-0-470-39106-8 

Green IT For Dummies 
978-0-470-38688-0 



Health 

Diabetes For Dummies, 
3rd Edition 
978-0-470-27086-8 

Food Allergies 
For Dummies 
978-0-470-09584-3 

Living Gluten-Free 
For Dummies 
978-0-471-77383-2 



Hobbies/General 

Chess For Dummies, 
2nd Edition 
978-0-7645-8404-6 

Drawing For Dummies 
978-0-7645-5476-6 

Knitting For Dummies, 
2nd Edition 
978-0-470-28747-7 

Organizing For Dummies 
978-0-7645-5300-4 

SuDoku For Dummies 
978-0-470-01892-7 



Home Improvement 

Energy Efficient Homes 
For Dummies 
978-0-470-37602-7 

Home Theater 
For Dummies, 
3rd Edition 
978-0-470-41189-6 

Living the Country Lifestyle 
All-in-One For Dummies 
978-0-470-43061-3 

Solar Power Your Home 
For Dummies 
978-0-470-17569-9 



Practices 




Available wherever books are sold. For more information or to order direct: U.S. customers visit www.dummies.com or call 1 -877-762-2974. 
U.K. customers visit www.wileyeurope.com or call (0] 1243 843291. Canadian customers visit www.wiley.ca or call 1-800-567-4797. 



Internet 

BLogging For Dummies, 
2nd Edition 
978-0-470-23017-6 



Macintosh 

Mac OS X Snow Leopard 
For Dummies 
978-0-470-43543-4 



Piano Exercises 
For Dummies 
978-0-470-38765-8 



Self-Help & Relationship 

Anger Management 
For Dummies 
978-0-470-03715-7 



eBay For Dummies, 
6th Edition 
978-0-470-49741-8 

Facebook For Dummies 
978-0-470-26273-3 

Google Blogger 
For Dummies 
978-0-470-40742-4 

Web Marketing 
For Dummies, 
2nd Edition 
978-0-470-37181-7 

WordPress For Dummies, 
2nd Edition 
978-0-470-40296-2 



Language & Foreign 
Language 

French For Dummies 
978-0-7645-5193-2 

Italian Phrases 
For Dummies 
978-0-7645-7203-6 

Spanish For Dummies 
978-0-7645-5194-9 

Spanish For Dummies, 
Audio Set 
978-0-470-09585-0 



Math & Science 

Algebra I For Dummies 
978-0-7645-5325-7 



Biology For Dummies 
978-0-7645-5326-4 

Calculus For Dummies 
978-0-7645-2498-1 

Chemistry For Dummies 
978-0-7645-5430-8 



Microsoft Office 

Excel 2007 For Dummies 
978-0-470-03737-9 

Office 2007 All-in-One 
Desk Reference 
For Dummies 
978-0-471-78279-7 



Music 

Guitar For Dummies, 
2nd Edition 
978-0-7645-9904-0 

iPod & iTunes 
For Dummies, 
6th Edition 
978-0-470-39062-7 



Parenting & Education 

Parenting For Dummies, 
2nd Edition 
978-0-7645-5418-6 

Type 1 Diabetes 
For Dummies 
978-0-470-17811-9 



Pets 

Cats For Dummies, 
2nd Edition 
978-0-7645-5275-5 

Dog Training For Dummies, 
2nd Edition 
978-0-7645-8418-3 

Puppies For Dummies, 
2nd Edition 
978-0-470-03717-1 



Religion & Inspiration 

The Bible For Dummies 
978-0-7645-5296-0 

Catholicism For Dummies 
978-0-7645-5391-2 

Women in the Bible 
For Dummies 
978-0-7645-8475-6 



Overcoming Anxiety 
For Dummies 
978-0-7645-5447-6 



Sports 

Baseball For Dummies, 
3rd Edition 
978-0-7645-7537-2 

Basketball For Dummies, 
2nd Edition 
978-0-7645-5248-9 

Golf For Dummies, 
3rd Edition 
978-0-471-76871-5 



Web Development 

Web Design All-in-One 
For Dummies 
978-0-470-41796-6 



Windows Vista 

Windows Vista 
For Dummies 
978-0-471-75421-3 




Macs 

DUMMIES 



wt 



Available wherever books are sold. For more information or to order direct: U.S. customers visit www.dummies.com or call 1 -877-762-2974. 
U.K. customers visit www.wileyeurope.com or call (0) 1 243 843291 . Canadian customers visit www.wiley.ca or call 1 -800-567-4797. 



SKwS 



How-to? 
How Easy. 





V, S* ■:-.: 



m\ whi 






From hooking up a modem to cooking up a 



:,-. _ casserole, knitting a scarf to navigating an iPod, 



you can trust Dummies.com to show you how 
to get things done the easy way. 

Visit us at Dummies.com 



Dummies products 
make life easier! 



DVDs • Music • Games • 
DIY • Consumer Electronics • 
Software • Crafts • Hobbies • 
Cookware • and more! 



For more information, go to 
Dummies.com® and search 
the store by category. 



Astronomy^j-f ractor JeleSCOpe 

DUNtffcS dummies _r 






There's a Dummies App for This and That 



With more than 200 million books in print and over 1,600 unique 
titles, Dummies is a global leader in how-to information. Now 
you can get the same great Dummies information in an App. With 
topics such as Wine, Spanish, Digital Photography, Certification, 
and more, you'll have instant access to the topics you need to 
know in a format you can trust. 

To get information on all our Dummies apps, visit the following: 
www.Dummies.com/go/mobile from your computer. 
www.Dummies.com/go/iphone/appsfrom your phone. 




Study Aids/Test Prep/ASVAB 



Ace the ASVAB by sharpening your 
test-taking skills with this updated 
and expanded bestselling guide 

Are you ready to tackle the ASVAB and begin your military career? 
If so, this essential guide provides a comprehensive review of 
all nine subtest subjects covered on the paper enlistment and 
the CAT-ASVAB (computer adaptive test). You'll get hands-on, 
easy-to-follow guidance on what to expect on all the subtests, 
cutting-edge strategies and tactics for studying, and test-taking tips 
and advice. With help from ASVAB For Dummies, you'll perform well 
on the test and be on your way to beginning (or advancing) your 
military career. 

• Decipher the ASVAB — getthe4-1-1 on every aspect of the ASVAB, from 
making sense of the subtests to understanding how it's scored 

• Be prepared — get ready for test day challenges 

• Take your best shot — discover strategies for making the best guesstimate 
when in doubt 

• Multiply your math skills — grasp the math operations needed to score 
your best — from working with fractions to solving algebra problems 

• Get technical — get a handle on the scientific, technical, electrical, and 
mechanical knowledge covered on specific subtests 

• Land your dream job — find out which subtests are most important to 
your specific military career goals 

• Practice makes perfect — get acquainted with the different types of 
problems before taking the actual practice tests 



SOpen the book and find: 

■ Tips for improving your 
vocabulary 

■ How to assemble objects 

• Math terminology and 
operations you need to know 

■ Algebra and geometry reviews 

> The "real world" of math word 
problems 

> Critical concepts in general 



• Auto & shop information 

■ Ten tips for doing well on test 
day 

> Three sample tests and one 
AFQT 

■ Questions throughout to help 
reinforce learning 



Go to Dummies.com 9 

for videos, step-by-step photos, 
how-to articles, or to shop! 



For Dummies® 

A Branded Imprint of 

©WILEY 



Rod Powers is a recognized expert in all U.S. military matters and serves 
as a military guide for About.com, where his page receives approximately 
1.5 million views per week. He is the author of ASVAB AFQT For Dummies 
and Veterans Benefits For Dummies. 



$18.00 US / $22.00 CN / £13.99 UK 

ISBN 17fl-D-M7D-h37hD-T 
5 1800 



780470"637609l